You are on page 1of 75

1.

APPLICATIONS OF MATRICES AND DETERMINANTS THEOREM 1: For every square matrix A of order n, 𝑨(𝒂𝒅𝒋 𝑨) = (𝒂𝒅𝒋 𝑨)𝑨 =
 If |𝐴| ≠ 0, then the square matrix A is called a non-singular matrix. |𝑨|𝑰𝒏
 If |𝐴| = 0, then the square matrix A is called a singular matrix. Proof:
 ADJOINT OF A SQUARE MATRIX: Let A be the square matrix of order n, then For simplicity, we prove the theorem for n=3 only.
the adjoint matrix of A is defined as the transpose of the matrix of cofactors 𝑎11 𝑎12 𝑎13 𝐴11 𝐴12 𝐴13
𝑻 𝑻 Consider 𝐴 = [𝑎21 𝑎22 𝑎23 ] 𝑎𝑛𝑑 𝑎𝑑𝑗 𝐴 = [𝐴21 𝐴22 𝐴23 ]
of A. It is denoted by adj A. 𝒂𝒅𝒋 𝑨 = [𝑨𝒊𝒋 ] = [(−𝟏)𝒊+𝒋 𝑴𝒊𝒋 ]
𝑎31 𝑎32 𝑎33 𝐴31 𝐴32 𝐴33
 𝐴(𝑎𝑑𝑗𝐴) = (𝑎𝑑𝑗𝐴)𝐴 = |𝐴|𝐼𝑛 𝑎11 𝑎12 𝑎13 𝐴11 𝐴12 𝐴13 |𝐴| 0 0
 INVERSE MATRIX: Let A be a square matrix of order n. If there exists a 𝐴(𝑎𝑑𝑗 𝐴) = [𝑎21 𝑎22 𝑎23 ] [𝐴21 𝐴22 𝐴23 ] = [ 0 |𝐴| 0 ]
square matrix B of order n such that 𝐴𝐵 = 𝐵𝐴 = 𝐼𝑛 , then the matrix B is 𝑎31 𝑎32 𝑎33 𝐴31 𝐴32 𝐴33 0 0 |𝐴|
called an inverse of A. 1 0 0
 𝐴𝐴−1 = 𝐴−1 𝐴 = 𝐼𝑛 = |𝐴| [0 1 0] = |𝐴|𝐼3 → (1)
 A singular matrix has no inverse. 0 0 1
 If A is non-singular, then 𝐴11 𝐴12 𝐴13 𝑎11 𝑎12 𝑎13 |𝐴| 0 0
(𝑖)|𝐴−1 | =
1 1
(𝑖𝑖) (𝐴𝑇 )−1 = (𝐴−1 )𝑇 (𝑖𝑖𝑖)(𝜆𝐴)−1 = 𝐴−1 (𝑎𝑑𝑗 𝐴)𝐴 = [𝐴21 𝐴22 𝐴23 ] [𝑎21 𝑎22 𝑎23 ] = [ 0 |𝐴| 0 ]
|𝐴|
1
𝜆 𝐴31 𝐴32 𝐴33 𝑎31 𝑎32 𝑎33 0 0 |𝐴|
 If A is non-singular, then (λA)−1 −1
= A , where 𝛌 is a non-zero scalar. 1 0 0
λ
 (AB)−1 = B −1 A−1 = |𝐴| [0 1 0] = |𝐴|𝐼3 → (2)
 If A is non-singular, then A−1 is also non-singular and (A−1 )−1 = A 0 0 1
From (1) & (2), we get A(adj A) = (adj A)A = |A|In
 If A is non-singular square matrix of order n, then
1 THEOREM 2: If a square matrix has an inverse, then it is unique.
(𝑖)(𝑎𝑑𝑗𝐴)−1 = 𝑎𝑑𝑗(𝐴−1 ) = 𝐴 ⟹ 𝑎𝑑𝑗 𝐴 = |𝐴|𝐴−1
|𝐴| Proof:
(𝑖𝑖)|𝑎𝑑𝑗𝐴| = |𝐴|𝑛−1 (𝑖𝑖𝑖)𝑎𝑑𝑗(𝑎𝑑𝑗𝐴) = |𝐴|𝑛−2 𝐴 Let A be a square matrix of order n such that an inverse of A exists. If possible, let
2
(𝑖𝑣)𝑎𝑑𝑗(𝜆𝐴) = 𝜆𝑛−1 𝑎𝑑𝑗(𝐴) (𝑣)|𝑎𝑑𝑗(𝑎𝑑𝑗𝐴)| = |𝐴|(𝑛−1) there be two inverse B and C of A.
(𝑣𝑖)(𝑎𝑑𝑗𝐴)𝑇 = 𝑎𝑑𝑗(𝐴𝑇 ) (𝑣𝑖𝑖)𝑎𝑑𝑗(𝐴𝐵) = (𝑎𝑑𝑗𝐵)(𝑎𝑑𝑗𝐴) By definition, 𝐴𝐵 = 𝐵𝐴 = 𝐼𝑛 𝑎𝑛𝑑 𝐴𝐶 = 𝐶𝐴 = 𝐼𝑛
 |𝑎𝑑𝑗 𝐴| = |𝐴|2 ⟹ |𝐴| = ±√|𝑎𝑑𝑗 𝐴| 𝐶 = 𝐶𝐼𝑛 = 𝐶(𝐴𝐵) = (𝐶𝐴)𝐵 = 𝐼𝑛 𝐵 = 𝐵 ⟹ 𝐵 = 𝐶
 A−1 = ±
1
adjA Hence it is proved.
√|adjA| THEOREM 3: Let A be a square matrix of order n. Then 𝑨−𝟏 exists if and only if A
1
 A=± adj(adjA) is non-singular.
√|adjA|
 If A and B are any two non-singular square matrices of order n, then Proof:
𝑎𝑑𝑗(𝐴𝐵) = (𝑎𝑑𝑗 𝐵)(𝑎𝑑𝑗 𝐴) Case (i) : Suppose that A−1 exists, then 𝐴𝐴−1 = 𝐴−1 𝐴 = 𝐼𝑛 .
 A square matrix A is called orthogonal if 𝐴𝐴𝑇 = 𝐴𝑇 𝐴 = 𝐼 By the product rule for determinants,
 A is orthogonal if and only if A is non-singular and A−1 = 𝐴𝑇 𝑑𝑒𝑡(𝐴𝐴−1 ) = 𝑑𝑒𝑡(𝐴−1 )𝑑𝑒𝑡(𝐴) = 𝑑𝑒𝑡(𝐼𝑛 ) = 1 ⟹ |𝐴| ≠ 0.
 Application of matrices is in computer graphic and cryptography. Hence A is non-singular.

V.GNANAMURUGAN, P,G,T, G.H.S.S, S.S.KOTTAI, SIVAGANGAI DT – 94874 43870 ; 82489 56766 , www.tnppgta.com, www.ednnet.in Page 1
Case (ii) : Conversely, suppose A is non-singular, then |𝐴| ≠ 0. Proof:
W.k.T. A(adj A) = (adj A)A = |A|In Since A is non-singular, 𝐴−1 exists and 𝐴𝐴−1 = 𝐴−1 𝐴 = 𝐼𝑛 .
1 1
÷ 𝑏𝑦 |𝐴| ⟹ 𝐴 ( 𝑎𝑑𝑗 𝐴) = ( 𝑎𝑑𝑗 𝐴) 𝐴 = In Given AB=AC. Pre multiply by 𝐴−1 on both sides,
|𝐴| |𝐴|
1 𝐴−1 (AB) = 𝐴−1 (AC) ⟹ (𝐴−1 𝐴)𝐵 = (𝐴−1 𝐴)𝐶 ⟹ 𝐼𝑛 𝐵 = 𝐼𝑛 𝐶 ⟹ 𝐵 = 𝐶
⟹ 𝐴𝐵 = 𝐵𝐴 = 𝐼𝑛 ; 𝐵 = |𝐴| 𝑎𝑑𝑗 𝐴 --------------------------------------------------------------------------------------------------------------
1 THEOREM 8: RIGHT CANCELLATION LAW
Hence the inverse of A exists and the inverse is 𝐴−1 = |𝐴| 𝑎𝑑𝑗 𝐴
Let A,B and C be square matrices of order n. If A is non-singular and BA=CA then
PROPERTIES OF INVERSES OF MATRICES
𝟏 B=C
THEOREM 4: If A is non-singular, then |𝐀−𝟏 | = |𝐀| Proof:
Proof: Since A is non-singular, 𝐴−1 exists and 𝐴𝐴−1 = 𝐴−1 𝐴 = 𝐼𝑛 .
Let A be non-singular then |𝐴| ≠ 0 and 𝐴−1 exists. Given BA=CA. Post multiply by 𝐴−1 on both sides,
By definition, 𝐴𝐴−1 = 𝐴−1 𝐴 = 𝐼𝑛 ⟹ |𝐴𝐴−1 | = |𝐴−1 𝐴| = |𝐼𝑛 | (BA)𝐴−1 = (CA)𝐴−1 ⟹ 𝐵(𝐴𝐴−1 ) = 𝐶(𝐴𝐴−1 ) ⟹ 𝐵𝐼𝑛 = 𝐶𝐼𝑛 ⟹ 𝐵 = 𝐶
By the product rule for determinants, --------------------------------------------------------------------------------------------------------------
1 THEOREM 9: REVERSAL LAW FOR INVERSES
|𝐴𝐴−1 | = |𝐼𝑛 | ⟹ |𝐴||𝐴−1 | = 1 ⟹ |A−1 | =
|A|
If A and B are non-singular matrices of the same order, then the product AB is
----------------------------------------------------------------------------------------------------------------------
also non-singular and (𝐀𝐁)−𝟏 = 𝐁 −𝟏 𝐀−𝟏
THEOREM 5: If A is non-singular, then (𝐀𝐓 )−𝟏 = (𝐀−𝟏 )𝐓
Proof:
Proof:
Since A and B are non-singular matrices of the same order n, then |𝐴| ≠ 0 , |𝐵| ≠
By definition, 𝐴𝐴−1 = 𝐴−1 𝐴 = 𝐼𝑛
0 ⟹ 𝐴−1 𝑎𝑛𝑑 𝐵−1 of order n exists. The product of AB and B −1 A−1 can also
Taking transpose ,(𝐴𝐴−1 )𝑇 = (𝐴−1 𝐴)𝑇 = (𝐼𝑛 )𝑇
found.
By the reversal law of transpose, (A−1 )T 𝐴𝑇 = 𝐴𝑇 (A−1 )T = |𝐼𝑛 |
|𝐴𝐵| = |𝐴||𝐵| ≠ 0 ⟹ 𝐴𝐵 is also non-singular and (AB)−1 exists.
Hence (AT )−1 = (A−1 )T
(AB)( B −1 A−1 ) = 𝐴(𝐵𝐵−1 )𝐴−1 = 𝐴(𝐼𝑛 )𝐴−1 = 𝐴𝐴−1 = 𝐼𝑛
--------------------------------------------------------------------------------------------------------------
𝟏
( B −1 A−1 )(AB) = B −1 (𝐴−1 𝐴)𝐵 = B −1 (𝐼𝑛 )𝐵 = 𝐵−1 𝐵 = 𝐼𝑛
THEOREM 6: If A is non-singular, then (𝛌𝐀)−𝟏 = 𝐀−𝟏 , where 𝛌 is a non-zero Hence (AB)−1 = B −1 A−1
𝛌
scalar. --------------------------------------------------------------------------------------------------------------
Proof: THEOREM 10: LAW OF DOUBLE INVERSE
By definition, 𝐴𝐴−1 = 𝐴−1 𝐴 = 𝐼𝑛 If A is non-singular, then 𝐀−𝟏 is also non-singular and (𝐀−𝟏 )−𝟏 = 𝐀
1 1 1
Muliply & divide by 𝛌, (𝜆𝐴) ( 𝐴−1 ) = ( 𝐴−1 ) (𝜆𝐴) = 𝐼𝑛 ⟹ (λA)−1 = A−1 Proof:
𝜆 𝜆 λ
-------------------------------------------------------------------------------------------------------------- Since A is non-singular,then |𝐴| ≠ 0 and 𝐴−1 exists.
1
THEOREM 7: LEFT CANCELLATION LAW Now |A−1 | = |A| ≠ 0 ⟹ A−1 is also non-singular and 𝐴𝐴−1 = 𝐴−1 𝐴 = 𝐼𝑛 .
Let A,B and C be square matrices of order n. If A is non-singular and AB=AC then Now 𝐴𝐴−1 = 𝐼𝑛 ⟹ (𝐴𝐴−1 )−1 = 𝐼 ⟹ (𝐴−1 )−1 𝐴−1 = 𝐼
B=C Post- multiply by A on both sides, (𝐴−1 )−1 = 𝐴

V.GNANAMURUGAN, P,G,T, G.H.S.S, S.S.KOTTAI, SIVAGANGAI DT – 94874 43870 ; 82489 56766 , www.tnppgta.com, www.ednnet.in Page 2
1
THEOREM 11: If A is non-singular square matrix of order n, then (𝐚𝐝𝐣𝐀)−𝟏 = Replace A by λA ⟹ 𝑎𝑑𝑗 (λA) = |λA|(λA)−1 = 𝜆𝑛 |𝐴| 𝐴−1 = 𝜆𝑛−1 |𝐴|𝐴−1 =
𝟏 𝜆
𝐚𝐝𝐣(𝐀−𝟏 ) = |𝐀| 𝐀 𝜆𝑛−1 adj(A)
Proof: THEOREM 15: If A is non-singular square matrix of order n, then |𝐚𝐝𝐣(𝐚𝐝𝐣𝐀)| =
1 𝟐
𝐴−1 = |𝐴| (𝑎𝑑𝑗 𝐴) ⟹ 𝑎𝑑𝑗 𝐴 = |𝐴|𝐴−1 |𝐀|(𝐧−𝟏)
1 Proof:
⟹ (𝑎𝑑𝑗 𝐴)−1 = (|𝐴|𝐴−1 )−1 = (𝐴−1 )−1 |𝐴|−1 = |A| A → (1)
W.K.T, adj (𝑎𝑑𝑗𝐴) = |A|n−2 𝐴
1 2 2
𝐴−1 = |𝐴| (𝑎𝑑𝑗 𝐴) ⟹ 𝑎𝑑𝑗 𝐴 = |𝐴|𝐴−1 ⟹ |adj (𝑎𝑑𝑗𝐴)| = ||A|n−2 𝐴| = (|A|n−2 )𝑛 |𝐴| = |𝐴|𝑛 −2𝑛+1 = |A|(n−1)
Replacing A by 𝐴−1 , 𝑎𝑑𝑗 (𝐴−1 ) = |𝐴−1 |(𝐴−1 )−1 = |A| A → (2)
1 --------------------------------------------------------------------------------------------------------------
1
THEOREM 16: If A is non-singular square matrix of order n, then(𝐚𝐝𝐣𝐀)𝐓 =
From (1) and (2), (adjA)−1 = adj(A−1 ) = |A| A 𝐚𝐝𝐣(𝐀𝐓 )
-------------------------------------------------------------------------------------------------------------- Proof:
1
THEOREM 12: If A is non-singular square matrix of order n, then |𝐚𝐝𝐣𝐀| = W.K.T, 𝐴−1 = |𝐴| (𝑎𝑑𝑗 𝐴)
|𝐀|𝐧−𝟏 1
Proof: Replace A by 𝐴𝑇 ⟹ (𝐴𝑇 )−1 = |𝐴𝑇 | (𝑎𝑑𝑗 (𝐴𝑇 ))
A(adj A) = (adj A)A = |A|In ⟹ 𝑑𝑒𝑡[A(adj A)] = det(𝐴)det(𝑎𝑑𝑗𝐴) = det[|A|In ] ⟹ 𝑎𝑑𝑗 (𝐴𝑇 ) = |𝐴𝑇 |(𝐴𝑇 )−1 = (|𝐴|𝐴−1 )𝑇 = (adjA)T
⟹ |𝐴||𝑎𝑑𝑗 𝐴| = |𝐴|𝑛 ⟹ |adjA| = |A|n−1 --------------------------------------------------------------------------------------------------------------
-------------------------------------------------------------------------------------------------------------- THEOREM 17: If A and B are any two non-singular matrices of order n, then
THEOREM 13: If A is non-singular square matrix of order n, then 𝐚𝐝𝐣(𝐚𝐝𝐣𝐀) = 𝐚𝐝𝐣(𝐀𝐁) = (𝐚𝐝𝐣𝐁)(𝐚𝐝𝐣𝐀)
|𝐀|𝐧−𝟐 𝐀 Proof:
Proof: W.K.T, 𝑎𝑑𝑗 𝐴 = |𝐴|𝐴−1
For any non-singular matrix B of order n, B(adj B) = (adj B)B = |B|In Replace A by AB ⟹ 𝑎𝑑𝑗 (𝐴𝐵) = |𝐴𝐵|(𝐴𝐵)−1 = |𝐴||𝐵|𝐴−1 𝐵−1
Put 𝐵 = 𝑎𝑑𝑗𝐴 ⟹ 𝑎𝑑𝑗𝐴(adj (𝑎𝑑𝑗𝐴)) = |𝑎𝑑𝑗𝐴|In = (|𝐵|𝐵−1 )(|𝐴|𝐴−1 ) = (adjB)(adjA)
⟹ 𝑎𝑑𝑗𝐴(adj (𝑎𝑑𝑗𝐴)) = |A|n−1 In EXERCISE 1.1
Pre-multiply by A on both sides, 1.Find the adjoint of the following matrices.
⟹ [𝐴(𝑎𝑑𝑗𝐴)](adj (𝑎𝑑𝑗𝐴)) = 𝐴|A|n−1 In ⟹ |A|In (adj (𝑎𝑑𝑗𝐴)) = 𝐴|A|n−1 In −3 4
(i) [ ]
6 2
⟹ adj (𝑎𝑑𝑗𝐴) = |A|n−2 𝐴 −3 4 2 −4
-------------------------------------------------------------------------------------------------------------- Let 𝐴 = [ ] ⟹ 𝑎𝑑𝑗 𝐴 = [ ]
6 2 −6 −3
THEOREM 14: If A is non-singular square matrix of order n, then 𝐚𝐝𝐣(𝛌𝐀) = --------------------------------------------------------------------------------------------------------------
𝛌𝐧−𝟏 𝐚𝐝𝐣(𝐀) 2 3 1
Proof: (ii) [3 4 1]
1
W.K.T,(adjA)−1 = A ⟹ 𝑎𝑑𝑗 𝐴 = |𝐴|𝐴−1 3 7 2
|A|

V.GNANAMURUGAN, P,G,T, G.H.S.S, S.S.KOTTAI, SIVAGANGAI DT – 94874 43870 ; 82489 56766 , www.tnppgta.com, www.ednnet.in Page 3
2 3 1 5 1 1 5 25 − 1 1 − 5 1−5 24 −4 −4
Let 𝐴 = [3 4 1] 1 5 1 1 ⟹ 𝑎𝑑𝑗 𝐴 = [ 1 − 5 25 − 1 1 − 5 ] = [−4 24 −4]
3 7 2 1 1 5 1
4 7 3 4 1−5 1 − 5 25 − 1 −4 −4 24
8−7 7−6 3−4 1 1 −1 5 1 1 5
1 2 1 1 ⟹ 𝑎𝑑𝑗 𝐴 = [ 3 − 6 24 −4 −4 6 −1 −1
3 4 − 3 3 − 2] = [−3 1 1] 1
𝐴−1 = |𝐴| 𝑎𝑑𝑗 𝐴 ⟹ 𝐴−1 =
1
=
1
3 3 2 [−4 24 −4 ] [ −1 6 −1]
4 21 − 12 9 − 14 8 − 9 9 −5 −1 112 28
4 7 3 −4 −4 24 −1 −1 6
-------------------------------------------------------------------------------------------------------------- --------------------------------------------------------------------------------------------------------------
2 2 1 2 3 1
1
(iii) [−2 1 2] 𝑎𝑑𝑗(𝜆𝐴) = 𝜆𝑛−1 𝑎𝑑𝑗(𝐴) (iii) [3 4 1]
3
1 −2 2 3 7 2
1 −2 2 1 2 + 4 −2 − 4 4 − 1 2 3 1
2 2 1 2 ⟹ 𝑎𝑑𝑗 𝐴 = (1)3−1 [2 + 4 4 − 1 −2 − 4] Let 𝐴 = [3 4 1]
−2 1 2 −2 3 3 7 2
4−1 2+4 2+4 2 3 1
1 −2 2 1
6 −6 3 2 −2 1 |𝐴| = |3 4 1| = 2(8 − 7) − 3(6 − 3) + 1(21 − 12) = 2 − 9 + 9 = 2 ≠ 0
1 1
= [6 3 −6] = [2 1 −2] 3 7 2
9 3
3 6 6 1 2 2 4 7 3 4 8−7 7−6 3−4 1 1 −1
2. Find the inverse of the following 1 2 1 1 ⟹ 𝑎𝑑𝑗 𝐴 = [ 3 − 6
3 4 − 3 3 − 2] = [−3 1 1]
−2 4 3 3 2
(i) [ ] 21 − 12 9 − 14 8 − 9 9 −5 −1
1 −3 4 7 3 4
−2 4 −2 4 −3 −4 1 1 −1
Let 𝐴 = [ ] ; |𝐴| = | | = 6 − 4 = 2 ≠ 0 ; 𝑎𝑑𝑗 𝐴 = [ ] −1 1 −1 1
1 −3 1 −3 −1 −2 𝐴 = |𝐴| 𝑎𝑑𝑗 𝐴 ⟹ 𝐴 = [−3 1 1]
1 1 −3 −4 2
𝐴−1 = |𝐴| 𝑎𝑑𝑗 𝐴 ⟹ 𝐴−1 = [ ] 9 −5 −1
2 −1 −2 𝑎 𝑏
-------------------------------------------------------------------------------------------------------------- 𝐄𝐗𝐀𝐌𝐏𝐋𝐄 𝟏. 𝟐 Find the inverse of[ ]
𝑐 𝑑
5 1 1 2 −1 3
(ii) [1 5 1] 𝐄𝐗𝐀𝐌𝐏𝐋𝐄 𝟏. 𝟑 Find the inverse of [−5 3 1]
1 1 5 −3 2 3
5 1 1 cos 𝛼 0 sin 𝛼
Let 𝐴 = [1 5 1] 3. 𝐼𝑓 𝐹(𝛼) = [ 0 1 0 ] , 𝑠ℎ𝑜𝑤 𝑡ℎ𝑎𝑡 [𝐹(𝛼)]−1 = 𝐹(−𝛼)
1 1 5 − sin 𝛼 0 cos 𝛼
5 1 1 cos(−𝛼) 0 sin(−𝛼) cos 𝛼 0 − sin 𝛼
|𝐴| = |1 5 1| = 5(25 − 1) − 1(5 − 1) + 1(1 − 5) 𝐹(−𝛼) = [ 0 1 0 ] =[ 0 1 0 ] → (1)
1 1 5 − sin(−𝛼) 0 cos(−𝛼) sin 𝛼 0 cos 𝛼
= 120 − 4 − 4 cos(−𝛼) = cos 𝛼
= 112 ≠ 0 (∵ )
sin(−𝛼) = − sin 𝛼

V.GNANAMURUGAN, P,G,T, G.H.S.S, S.S.KOTTAI, SIVAGANGAI DT – 94874 43870 ; 82489 56766 , www.tnppgta.com, www.ednnet.in Page 4
cos 𝛼 0 sin 𝛼 4 3 4𝑥 3𝑥 1 0 𝑦 0
| 𝐹(𝛼)| = | 0 𝑥𝐴 = 𝑥 [ ]=[ ] ; 𝑦𝐼2 = 𝑦 [ ]=[ ]
1 0 | = cos 𝛼 (cos 𝛼) − 0 + sin 𝛼 (sin 𝛼) 2 5 2𝑥 5𝑥 0 1 0 𝑦
− sin 𝛼 0 cos 𝛼 22 27 4𝑥 3𝑥 𝑦 0 0 0
= 𝑐𝑜𝑠 2 𝛼 + 𝑠𝑖𝑛2 𝛼 = 1 𝐴2 + 𝑥𝐴 + 𝑦𝐼2 = 02 ⟹ [ ]+[ ]+[ ]=[ ]
18 31 2𝑥 5𝑥 0 𝑦 0 0
1 0 0 1 22 + 4𝑥 + 𝑦 27 + 3𝑥 0 0
0 cos 𝛼 sin 𝛼 0⟹ ⟹[ ]=[ ]
18 + 2𝑥 31 + 5𝑥 + 𝑦 0 0
0 − sin 𝛼 cos 𝛼 0 27 + 3𝑥 = 0 ⟹ 3𝑥 = −27 ⟹ 𝑥 = −9
1 0 0 1 ⟹{
cos 𝛼 − 0 0−0 0 − sin 𝛼 cos 𝛼 0 − sin 𝛼 22 + 4𝑥 + 𝑦 = 0 ⟹ 22 + 4(−9) + 𝑦 = 0 ⟹ 22 − 36 + 𝑦 = 0 ⟹ 𝑦 = 14
𝑎𝑑𝑗𝐹(𝛼) = [ 0 − 0 2 2
𝑐𝑜𝑠 𝛼 + 𝑠𝑖𝑛 𝛼 0−0 ]=[ 0 1 0 ] 𝐴2 + 𝑥𝐴 + 𝑦𝐼2 = 02 ⟹ 𝐴2 − 9𝐴 + 14𝐼2 = 02
0 + sin 𝛼 0−0 cos 𝛼 − 0 sin 𝛼 0 cos 𝛼 pre-multiply by 𝐴−1 on both sides, we get
cos 𝛼 0 − sin 𝛼 1
[𝐹(𝛼)]−1 =
1
𝑎𝑑𝑗 𝐹(𝛼) ⟹ [𝐹(𝛼)]−1 = [ 0
1
1 0 ] 𝐴 − 9𝐼2 + 14𝐴−1 = 02 ⟹ 14𝐴−1 = 9𝐼2 − 𝐴 ⟹ 𝐴−1 = [9𝐼2 − 𝐴]
|𝐹(𝛼)| 14
1
sin 𝛼 0 cos 𝛼 1 1 0 4 3 1 9 0 4 3 1 5 −3
𝐴−1 = {9 [ ]−[ ]} = {[ ]−[ ]} = [ ]
cos 𝛼 0 − sin 𝛼 14 0 1 2 5 14 0 9 2 5 14 −2 4
[𝐹(𝛼)]−1 = [ 0 1 0 ] → (2) −8 1 4
1
sin 𝛼 0 cos 𝛼 5.If 𝐴 = [ 4
9
4 7], prove that 𝐴−1 = 𝐴𝑇
From (1) & (2), [𝐹(𝛼)]−1 = 𝐹(−𝛼) 1 −8 4
5 3 −8 4 1
4. 𝐼𝑓 𝐴 = [ ], show that 𝐴2 − 3𝐴 − 7𝐼2 = 02 . Hence find 𝐴−1 1
𝐴𝑇 = [ 1 4 8] → (1)
−1 −2 9
2 5 3 5 3 25 − 3 15 − 6 22 9 4 7 4
𝐴 =[ ][ ]=[ ]=[ ] 1 3
−1 −2 −1 −2 −5 + 2 −3 + 4 −3 1 |𝐴| = ( ) [−8(16 − 56) − 1(16 − 7) + 4(7 − 16)] (∵ |𝐾𝐴| = 𝐾 𝑛 |𝐴|)
5 3 −15 −9 9
−3𝐴 = −3 [ ]=[ ] 1 1
−1 −2 3 6 = [−576 − 9 − 144] = [−729] = −1
1 0 −7 0 729 729
−7𝐼2 = −7 [ ]=[ ] 4 −8 1 4
0 1 0 −7 7 4 4 7 ⟹
𝐴2 − 3𝐴 − 7𝐼2 = [
22 9
]+[
−15 −9
]+[
−7 0
]=[
0 0
] = 02 𝑎𝑑𝑗(𝜆𝐴) = 𝜆𝑛−1 𝑎𝑑𝑗(𝐴)
4 1 −8 4
−3 1 3 6 0 −7 0 0
𝐴2 − 3𝐴 − 7𝐼2 = 02 pre-multiply by 𝐴−1 on both sides, we get 4 −8 1 4
16 + 56 −32 − 4 7 − 16 72 −36 −9
1 1 3−1 1
𝐴 − 3𝐼2 − 7𝐴−1 = 02 ⟹ 7𝐴−1 = 𝐴 − 3𝐼2 ⟹ 𝐴−1 = [𝐴 − 3𝐼2 ] 𝑎𝑑𝑗 𝐴 = ( ) [ 7 − 16 −32 − 4 16 + 56 ] = 81 [ −9 −36 72 ]
7 9
1 5 3 1 0 1 5 3 −3 0 1 2 3 −32 − 4 1 − 64 −32 − 4 −36 −63 −36
𝐴−1 = {[ ] − 3[ ]} = {[ ]+[ ]} = [ ] 8 −4 −1
7 −1 −2 0 1 7 −1 −2 0 −3 7 −1 −5
1
4 3 = [− 1 −4 8 ]
𝐄𝐗𝐀𝐌𝐏𝐋𝐄 𝟏. 𝟏𝟎 If 𝐴 = [ ], find x and y such that 𝐴2 + 𝑥𝐴 + 𝑦𝐼2 = 02 . 9
2 5 −4 −7 −4
Hence find 𝐴−1 1 1 1
8 −4 −1
1
−8 4 1
−1 −1
4 3 4 3 16 + 6 12 + 15 22 27 𝐴 = |𝐴| 𝑎𝑑𝑗 𝐴 ⟹ 𝐴 = . [− 1 −4 8 ] = [ 1 4 8] → (2)
𝐴2 = [ ][ ]=[ ]=[ ] −1 9 9
2 5 2 5 8 + 10 6 + 25 18 31 −4 −7 −4 4 7 4

V.GNANAMURUGAN, P,G,T, G.H.S.S, S.S.KOTTAI, SIVAGANGAI DT – 94874 43870 ; 82489 56766 , www.tnppgta.com, www.ednnet.in Page 5
From (1) & (2), 𝐴−1 = 𝐴𝑇 |𝐴𝐵| = −77 + 90 = 13 ; 𝑎𝑑𝑗(𝐴𝐵) = [−11 5]
8 −4 −18 7
6. If 𝐴 = [ ], verify that 𝐴(𝑎𝑑𝑗𝐴) = (𝑎𝑑𝑗𝐴)𝐴 = |𝐴|𝐼2 −1 1 1 −11 5
−5 3 (AB) = |𝐴𝐵| 𝑎𝑑𝑗(𝐴𝐵) = [ ] → (1)
|𝐴| = 24 − 20 = 4 13 −18 7
1 0 4 0 2 −2 1 1 2 3
|𝐴|𝐼2 = 4 [ ]=[ ] → (1) |𝐵| = −2 + 15 = 13 ; 𝑎𝑑𝑗 𝐵 = [ ] ; B −1 = |𝐵| 𝑎𝑑𝑗 𝐵 = [ ]
0 1 0 4 −7 −1 13 −5 −1

𝑎𝑑𝑗𝐴 = [
3 4
] |𝐴| = 15 − 14 = 1 ; 𝑎𝑑𝑗 𝐴 = [ 5 −2]
5 8 −7 3
8 −4 3 4 4 0 1 1 5 −2 5 −2
𝐴(𝑎𝑑𝑗𝐴) = [ ][ ]=[ ] → (2) A−1 = |𝐴| 𝑎𝑑𝑗 𝐴 = [ ]=[ ]
1 −7 3 −7 3
−5 3 5 8 0 4 2 3 1 10 − 21 −4 + 9
3 4 8 −4 4 0 1 5 −2
(𝑎𝑑𝑗𝐴)𝐴 = [ ][ ]=[ ] → (3) B −1 A−1 = [ ][ ]= [ ]
13 −5 −1 −7 3 13 −25 + 7 10 − 3
5 8 −5 3 0 4 1 −11 5
From (1),(2) & (3) , 𝐴(𝑎𝑑𝑗𝐴) = (𝑎𝑑𝑗𝐴)𝐴 = |𝐴|𝐼2 = [ ] → (2)
13 −18 7
8 −6 2
From (1) & (2), (AB)−1 = B −1 A−1
𝐄𝐗𝐀𝐌𝐏𝐋𝐄 𝟏. 𝟏 If 𝐴 = [−6 7 −4], verify that 𝐴(𝑎𝑑𝑗𝐴) = (𝑎𝑑𝑗𝐴)𝐴 = |𝐴|𝐼3
0 −3 −2 −3
2 −4 3 𝐄𝐗𝐀𝐌𝐏𝐋𝐄 𝟏. 𝟗 If 𝐴 = [ ] 𝑎𝑛𝑑 𝐵 = [ ] verify that (AB)−1 =
|𝐴| = 8(21 − 16) − (−6)(−18 + 8) + 2(24 − 14) = 40 − 60 + 20 = 0 1 4 0 −1
1 0 0 0 0 0 B −1 A−1
|𝐴|𝐼3 = 0 [0 1 0] = [0 0 0] → (1) 2 −4 2
8. If 𝑎𝑑𝑗 𝐴 = [−3 12 −7], find A.
0 0 1 0 0 0
7 −4 −6 7 −2 0 2
1
−4 3 2 −4 ⟹ A=± adj(adjA)
√|adjA|
−6 2 8 −6
7 −4 −6 7 |𝑎𝑑𝑗𝐴| = 2(24 − 0) − (−4)(−6 − 14) + 2(0 + 24) = 48 − 80 + 48 = 16
21 − 16 −8 + 18 24 − 14 5 10 10 √|adjA| = √16 = 4
𝑎𝑑𝑗 𝐴 = [−8 + 18 24 − 4 −12 + 32] = [10 20 20] 12 0 −4 12
24 − 14 −12 + 32 56 − 36 10 20 20 24 − 0 0 + 8 28 − 24
−7 2 2 −7 ⟹ adj(adjA) = [14 + 6 4 + 4 −6 + 14]
8 −6 2 5 10 10 0 0 0 −3
−3 −2 2 0 + 24 8 − 0 24 − 12
𝐴(𝑎𝑑𝑗𝐴) = [−6 7 −4] [10 20 20] = [0 0 0] → (2) 12 0 −4 12
2 −4 3 10 20 20 0 0 0 24 8 4
5 10 10 8 −6 2 0 0 0 = [20 8 8 ]
(𝑎𝑑𝑗𝐴)𝐴 = [10 20 20] [−6 7 −4] = [0 0 0] → (3) 24 8 12
10 20 20 2 −4 3 0 0 0 24 8 4 6 2 1
1 1
From (1),(2) & (3) , 𝐴(𝑎𝑑𝑗𝐴) = (𝑎𝑑𝑗𝐴)𝐴 = |𝐴|𝐼3 A=± adj(adjA) = ± [20 8 8 ] = ± [5 2 2]
√|adjA| 4
3 2 −1 −3 24 8 12 6 2 3
7.If 𝐴 = [ ] 𝑎𝑛𝑑 𝐵 = [ ] verify that (AB)−1 = B −1 A−1
7 5 5 2 7 7 −7
3 2 −1 −3 −3 + 10 −9 + 4 7 −5 𝐄𝐗𝐀𝐌𝐏𝐋𝐄𝟏. 𝟓 Find a matrix A if 𝑎𝑑𝑗 𝐴 = [−1 11 7 ]
AB = [ ][ ]=[ ]=[ ]
7 5 5 2 −7 + 25 −21 + 10 18 −11 11 5 7
V.GNANAMURUGAN, P,G,T, G.H.S.S, S.S.KOTTAI, SIVAGANGAI DT – 94874 43870 ; 82489 56766 , www.tnppgta.com, www.ednnet.in Page 6
0 −2 0 5 3 14 7
12.Find the matrix A for which 𝐴 [ ]=[ ]
9. If 𝑎𝑑𝑗 𝐴 = [ 6 2 −6], find A−1 . −1 −2 7 7
−3 0 6 5 3 14 7 14 7 14 7 −1
𝐴[ ]=[ ] ⟹ 𝐴𝐵 = [ ]⟹𝐴=[ ] 𝐵 → (1)
1 −1 −2 7 7 7 7 7 7
A−1 = ± adjA 𝐵=[
5 3
] ⟹ |𝐵| = −10 + 3 = −7 ; 𝑎𝑑𝑗 𝐵 = [
−2 −3
]
√|adjA| −1 −2 1 5
1 1 −2 −3
|𝑎𝑑𝑗𝐴| = 0(12 − 0) − (−2)(36 − 18) + 0(0 + 6) = 0 + 36 + 0 = 36 𝐵−1 = |𝐵| 𝑎𝑑𝑗𝐵 ⟹ 𝐵−1 = [ ]
−7 1 5
√|adjA| = √36 = 6 14 7 1 −2 −3 1 −28 + 7 −42 + 35
0 −2 0 (1) ⟹ 𝐴 = [ ] [ ]=− [ ]
1 1 7 7 −7 1 5 7 −14 + 7 −21 + 35
−1 1 −21 −7 3 1 3 1
A =± adjA = ± [ 6 2 −6] =− [ ]=[ ] ∴𝐴=[ ]
√|adjA| 6
−3 0 6 7 −7 14 1 −2 1 −2
−1 2 2 1 −1 3 −2 1 1
13. Given 𝐴 = [ ],𝐵 = [ ] and 𝐶 = [ ], find a matrix X such that
𝐄𝐗𝐀𝐌𝐏𝐋𝐄 𝟏. 𝟔 If 𝑎𝑑𝑗 𝐴 = [ 1 1 2], find A−1 . 2 0 1 1 2 2
2 2 1 𝐴𝑋𝐵 = 𝐶.
1 0 1 𝐴𝑋𝐵 = 𝐶
10.Find adj(adjA) if 𝑎𝑑𝑗 𝐴 = [ 0 2 0] Pre multiplying by 𝐴 and post multiplying by 𝐵 ⟹𝑋 = 𝐴−1 𝐶𝐵−1 → (1)
−1 −1

−1 0 1 |𝐴| = 0 + 2 = 2 ; 𝑎𝑑𝑗 𝐴 = [ 0 1] ; A−1 = 𝑎𝑑𝑗 𝐴 = [ 0 1]


1 1
2 0 0 2 2−0 0−0 0−2 −2 1 |𝐴| 2 −2 1
2 0 −2
0 1 1 0 ⟹ adj(adjA) = [0 − 0 1 + 1 0 − 0] = [0 2 0 ] 1 2 −1 1 1 1 2
|𝐵| = 3 + 2 = 5 ; 𝑎𝑑𝑗 𝐵 = [ ] ; A = |𝐴| 𝑎𝑑𝑗 𝐴 = [ ]
0 −1 1 0 −1 3 5 −1 3
0+2 0−0 2−0 2 0 2 1 1 2 0+2 0+2 1 2
(1) ⟹ 𝑋 = [ 0 1] [1 1] [
2 0 0 2 1 1
]= [ ][ ]
1 tan 𝑥 cos 2𝑥 − sin 2𝑥 2 −2 1 2 2 5 −1 3 10 −2 + 2 −2 + 2 −1 3
11.𝐴 = [ ], show that 𝐴𝑇 𝐴−1 = [ ] 1 2 2 1 2 1 2−2 4+6 1 0 10 0 1
− tan 𝑥 1 sin 2𝑥 cos 2𝑥 = [ ][ ]= [ ]= [ ]=[ ]
1 − tan 𝑥 10 0 0 −1 3 10 0 0 10 0 0 0 0
𝐴𝑇 = [ ] 0 1 1
tan 𝑥 1 1
|𝐴| = 1 + 𝑡𝑎𝑛2 𝑥 = 𝑠𝑒𝑐 2 𝑥 = 2
1 14.If 𝐴 = [1 0 1], show that 𝐴−1 = (𝐴2 − 3𝐼)
𝑐𝑜𝑠 𝑥 2
1 − tan 𝑥 1 1 0
adjA = [ ] |𝐴| = 0 − 1(0 − 1) + 1(1 − 0) = 1 + 1 = 2
tan 𝑥 1 0 1 1 0
1 1 1 − tan 𝑥 1 − tan 𝑥 0−1 1−0 1−0 −1 1 1
A−1 = |𝐴| 𝑎𝑑𝑗 𝐴 = 1 [ ] = 𝑐𝑜𝑠 2 𝑥 [ ] 1 0 1 1 ⟹𝑎𝑑𝑗 𝐴 = [1 − 0 0 − 1 1 − 0] = [ 1 −1 1 ]
2 tan 𝑥 1 tan 𝑥 1
𝑐𝑜𝑠 𝑥
2 1 1 0 1
= [ 𝑐𝑜𝑠 𝑥 −𝑠𝑖𝑛𝑥𝑐𝑜𝑠𝑥 ]
0 1 1 0 1−0 1−0 0−1 1 1 −1
𝑠𝑖𝑛𝑥𝑐𝑜𝑠𝑥 𝑐𝑜𝑠 2 𝑥 −1 1 1
1 − tan 𝑥 2 1 1
] [ 𝑐𝑜𝑠 𝑥 −𝑠𝑖𝑛𝑥𝑐𝑜𝑠𝑥 ] −1
𝐴𝑇 A−1 = [ A = |𝐴| 𝑎𝑑𝑗 𝐴 = [ 1 −1 1 ] → (1)
tan 𝑥 1 𝑠𝑖𝑛𝑥𝑐𝑜𝑠𝑥 𝑐𝑜𝑠 2 𝑥 2
1 1 −1
2 2
= [ 𝑐𝑜𝑠 𝑥 − 𝑠𝑖𝑛 𝑥 −𝑠𝑖𝑛𝑥𝑐𝑜𝑠𝑥 − 𝑠𝑖𝑛𝑥𝑐𝑜𝑠𝑥 ] = [cos 2𝑥 − sin 2𝑥 ]
𝑠𝑖𝑛𝑥𝑐𝑜𝑠𝑥 + 𝑠𝑖𝑛𝑥𝑐𝑜𝑠𝑥 𝑐𝑜𝑠 2 𝑥 − 𝑠𝑖𝑛2 𝑥 sin 2𝑥 cos 2𝑥

V.GNANAMURUGAN, P,G,T, G.H.S.S, S.S.KOTTAI, SIVAGANGAI DT – 94874 43870 ; 82489 56766 , www.tnppgta.com, www.ednnet.in Page 7
0 1 1 0 1 1 0+1+1 0+0+1 0+1+0 2 1 1 cos 𝜃 − sin 𝜃
2 𝐄𝐗𝐀𝐌𝐏𝐋𝐄 𝟏. 𝟏𝟏 Prove that [ ] is orthogonal.
𝐴 = [1 0 1] [1 0 1] = [0 + 0 + 1 1 + 0 + 1 1 + 0 + 0] = [1 2 1] sin 𝜃 cos 𝜃
1 1 0 1 1 0 0+1+0 1+0+0 1+1+0 1 1 2 A square matrix A is called orthogonal if 𝐴𝐴𝑇 = 𝐴𝑇 𝐴 = 𝐼𝑛
2 1 1 1 0 0 cos 𝜃 − sin 𝜃 cos 𝜃 sin 𝜃
1 1
(𝐴2 − 3𝐼) = ([1 2 1] − 3 [0 1 0]) 𝐴=[ ] ⟹ 𝐴𝑇 = [ ]
2 2 sin 𝜃 cos 𝜃 −sin 𝜃 cos 𝜃
1 1 2 0 0 1 cos 𝜃 − sin 𝜃 cos 𝜃 sin 𝜃
2 1 1 −3 0 0 −1 1 1 𝐴𝐴𝑇 = [ ][ ]
1 1 sin 𝜃 cos 𝜃 −sin 𝜃 cos 𝜃
= ([1 2 1] + [ 0 −3 0 ]) = [ 1 −1 1 ] → (2) 𝑐𝑜𝑠 2 𝜃 + 𝑠𝑖𝑛2 𝜃 sin 𝜃 cos 𝜃 − sin 𝜃 cos 𝜃] = [1 0] = 𝐼 → (1)
2 2 =[
1 1 2 0 0 −3 1 1 −1 sin 𝜃 cos 𝜃 − sin 𝜃 cos 𝜃 𝑠𝑖𝑛2 𝜃 + 𝑐𝑜𝑠 2 𝜃 0 1 2
−1 1 2
From (1) & (2), 𝐴 = (𝐴 − 3𝐼) cos 𝜃 sin 𝜃 cos 𝜃 − sin 𝜃
2 𝐴𝑇 𝐴 = [ ][ ]
15. Decrypt the received encoded message [2 −3], [20 4] with the encryption −sin 𝜃 cos 𝜃 sin 𝜃 cos 𝜃
−1 −1 =[ 𝑐𝑜𝑠 2 𝜃 + 𝑠𝑖𝑛2 𝜃 sin 𝜃 cos 𝜃 − sin 𝜃 cos 𝜃] = [1 0] = 𝐼 → (2)
matrix [ ] and the decryption matrix as its inverse, where the system of 2
2 1 sin 𝜃 cos 𝜃 − sin 𝜃 cos 𝜃 𝑠𝑖𝑛2 𝜃 + 𝑐𝑜𝑠 2 𝜃 0 1
code are described by the numbers 1-26 to the letters A-Z respectively, and the From (1) & (2), 𝐴𝐴𝑇 = 𝐴𝑇 𝐴 = 𝐼2 , hence A is orthogonal
6 −3 𝑎
number 0 to a blank space. 1
𝐄𝐗𝐀𝐌𝐏𝐋𝐄 𝟏. 𝟏𝟐 If 𝐴 = [𝑏 −2 6] is orthogonal, find a, b and c and hence
𝐴 = [2 −3], 𝐵 = [20 4] 7
−1 −1 2 𝑐 3
−1
Encoded matrix C= [ ] ⟹ Decoded matrix = C −1 A
2 1
A square matrix A is called orthogonal if 𝐴𝐴𝑇 = 𝐴𝑇 𝐴 = 𝐼𝑛
|𝐶| = −1 + 2 = 1 ; 𝑎𝑑𝑗 𝐶 = [ 1 1 1 1
1
] ; C −1 = |𝐶| 𝑎𝑑𝑗 𝐶 = [ ]
−2 −1 −2 −1 6 −3 𝑎 6 𝑏 2
1 1
Received message= (𝐴C −1 )(𝐵C −1 ) 𝐴 = [𝑏 −2 6] ⟹ 𝐴 = [−3 −2 𝑐 ]𝑇
7 7
1 1 1 1 2 𝑐 3 𝑎 6 3
= {[2 −3] [ ]} {[20 4] [ ]}
−2 −1 −2 −1 6 −3 𝑎 6 𝑏 2 1 0 0
1 1
= [2 + 6 2 + 3][20 − 8 20 − 4] = [8 5][12 16] = 𝐻𝐸𝐿𝑃 𝐴𝐴𝑇 = 𝐼3 ⟹ [𝑏 −2 6] [−3 −2 𝑐 ] = [0 1 0]
7 7
( ∵ In alphabetical order, 8=H, 5=E, 12=L, 16=P ) 2 𝑐 3 𝑎 6 3 0 0 1
2 9 2 1 0 0
𝐄𝐗𝐀𝐌𝐏𝐋𝐄 𝟏. 𝟖 Verify the property (𝐴𝑇 )−1 = (𝐴−1 )𝑇 with 𝐴 = [ ] 1
36 + 9 + 𝑎 6𝑏 + 6 + 6𝑎 12 − 3𝑐 + 3𝑎
1 7 ⟹ [ 6𝑏 + 6 + 6𝑎 2
49
𝑏 + 4 + 36 2𝑏 − 2𝑐 + 18] = [0 1 0]
2 1 7 −1 4 + 𝑐2 + 9 0 0 1
𝐴𝑇 = [ ] ; |𝐴𝑇 | = 14 − 9 = 5 ; 𝑎𝑑𝑗(𝐴𝑇 ) = [ ] 12 − 3𝑐 + 3𝑎 2𝑏 − 2𝑐 + 18
9 7 −9 2 2
𝑎 + 45 6𝑎 + 6𝑏 + 6 3𝑎 − 3𝑐 + 12 49 0 0
1 1 7 −1
(𝐴𝑇 )−1 = 𝑇 𝑎𝑑𝑗(𝐴𝑇 ) = [ ] → (1) ⟹ [ 6𝑎 + 6𝑏 + 6 2
𝑏 + 40 2𝑏 − 2𝑐 + 18] = [ 0 49 0 ]
|𝐴 | 5 −9 2
3𝑎 − 3𝑐 + 12 2𝑏 − 2𝑐 + 18 𝑐 2 + 13 0 0 49
|𝐴| = 14 − 9 = 5 ; 𝑎𝑑𝑗 𝐴 = [ 7 −9] ; 𝐴−1 = 𝑎𝑑𝑗 𝐴 = [ 7 −9]
1 1
−1 2 |𝐴| 5 −1 2
−1 𝑇 1 7 −1
(𝐴 ) = [ ] → (2)
5 −9 2
From (1) & (2), (𝐴𝑇 )−1 = (𝐴−1 )𝑇

V.GNANAMURUGAN, P,G,T, G.H.S.S, S.S.KOTTAI, SIVAGANGAI DT – 94874 43870 ; 82489 56766 , www.tnppgta.com, www.ednnet.in Page 8
𝑎2 + 45 = 49 ⟹ 𝑎2 = 4 ⟹ 𝑎 = ±2 RANK OF A MATRIX IN MINOR METHOD
𝑏 2 + 40 = 49 ⟹ 𝑏 2 = 9 ⟹ 𝑏 = ±3 EXERCISE 1.2
2 2 1.Find the rank of the following matrices by minor method:
⟹ 𝑐 + 13 = 49 ⟹ 𝑐 = 36 ⟹ 𝑐 = ±6 ⟹ 𝑎 = 2, 𝑏 = −3, 𝑐 = 6
6𝑎 + 6𝑏 + 6 = 0 ⟹ 𝑎 + 𝑏 = −1 2 −4
(i)[ ]
3𝑎 − 3𝑐 + 12 = 0 ⟹ 𝑎 − 𝑐 = −4 −1 2
{ 2𝑏 − 2𝑐 + 18 = 0 ⟹ 𝑏 − 𝑐 = −9 } 2 −4
Let 𝐴 = [ ] . Order of A is 2 × 2. So 𝜌(𝐴) ≤ 𝑚𝑖𝑛{2,2} = 2
6 −3 2 −1 2
1
−1 𝑇 −1
Since A is orthogonal, 𝐴 = 𝐴 ⟹ 𝐴 = [−3 −2 6] |𝐴| = 4 − 4 = 0 ⟹ 𝜌(𝐴) < 2
7
2 6 3 Since A contains at-least one non-Zero element, 𝜌(𝐴) = 1.
RANK OF A MATRIX −1 3
 Rank of a matrix in minor method: The rank of a matrix A is defined as the (ii) [ 4 −7]
order of a highest order non-vanishing minor of the matrix A. It is denoted 3 −4
−1 3
by 𝜌(𝐴).
Let 𝐴 = [ 4 −7]. Order of A is 3 × 2. So 𝜌(𝐴) ≤ 𝑚𝑖𝑛{3,2} = 2
 The rank of zero matrix is 0.
3 −4
 The rank of the identity matrix 𝐼𝑛 is n. 2 × 2 Minor:
 If a matrix contains at-least one non-zero element then 𝜌(𝐴) ≥ 1. −1 3
 If A is an 𝑚 × 𝑛 matrix then 𝜌(𝐴) ≤ 𝑚𝑖𝑛{𝑚, 𝑛} | | = 7 − 12 = 5 ≠ 0 ⟹ 𝜌(𝐴) = 2
4 −7
 A square matrix A order n is invertible ⟺ 𝜌(𝐴) = 𝑛 (iii)[
1 −2 −1 0
]
 If all the entries below the leading diagonal are zero, use minor method for 3 −6 −3 1
1 −2 −1 0
finding the rank. Let 𝐴 = [ ]. Order of A is 2 × 4. So 𝜌(𝐴) ≤ 𝑚𝑖𝑛{2,4} = 2
3 −6 −3 1
 Rank of a matrix in row echelon form: The rank of a matrix in row echelon 2 × 2 Minors:
form is the number of non-zero rows in it. 1 −2 1 −1 1 0
 The rank of the non-zero matrix is equal to the number of non-zero rows in | |=6−6= 0; | | = −3 + 3 = 0 ; | |=1−0=1≠0
3 −6 3 −3 3 1
a row-echelon form of the matrix. ∴ 𝜌(𝐴) = 2
 Elementary matrix: An elementary matrix is defined as a matrix which is 1 −2 3
obtained from an identity matrix by applying only one elementary (iv)[2 4 −6]
transformation. 5 1 −1
1 −2 3
 Gauss-Jordan Method: Transforming a non-singular matrix A to the form 𝐼𝑛
Let 𝐴 = [2 4 −6]. Order of A is 3 × 3. So 𝜌(𝐴) ≤ 𝑚𝑖𝑛{3,3} = 3
by applying elementary row operations is called Gauss-Jordan method.
5 1 −1
 Steps in finding 𝑨−𝟏 by Gauss-Jordan method: |𝐴| = 1(−4 + 6) − (−2)(−2 + 30) + 3(2 − 20) = 2 + 56 − 54 = 4 ≠ 0
 Write matrix of the form [𝐴|𝐼𝑛 ] ∴ 𝜌(𝐴) = 3
 Apply elementary row operations and convert [𝐴|𝐼𝑛 ] 𝑡𝑜 [𝐼𝑛 |𝐴−1 ] and find 0 1 2 1
𝐴−1 . (v)[0 2 4 3 ]
8 1 0 2
V.GNANAMURUGAN, P,G,T, G.H.S.S, S.S.KOTTAI, SIVAGANGAI DT – 94874 43870 ; 82489 56766 , www.tnppgta.com, www.ednnet.in Page 9
0 1 2 1 echelon form:
Let 𝐴 = [0 2 4 3 ]. Order of A is3 × 4. So 𝜌(𝐴) ≤ 𝑚𝑖𝑛{3,4} = 3 2 0 −7
8 1 0 2 (i) [0 3 1 ]
3 × 3 Minors: 0 0 1
0 1 2 0 1 1 2 0 −7
|0 2 4| = 0 − 0 + 8(4 − 4) = 0 ; |0 2 3| 0 − 0 + 8(3 − 2) = 8 ≠ 0 Let 𝐴 = [0 3 1 ]. Order of A is3 × 3. So 𝜌(𝐴) ≤ 𝑚𝑖𝑛{3,3} = 3
8 1 0 8 1 2 0 0 1
∴ 𝜌(𝐴) = 3 3 × 3 Minor:
3 2 5 2 0 −7
𝐄𝐗𝐀𝐌𝐏𝐋𝐄 𝟏. 𝟏𝟓 [1 1 2] |0 3 1 | = 2(3 − 0) − 0 + 0 = 6 ≠ 0. So 𝜌(𝐴) = 3
3 3 6 0 0 1
3 2 5 −2 2 −1
Let 𝐴 = [1 1 2]. Order of A is3 × 3. So 𝜌(𝐴) ≤ 𝑚𝑖𝑛{3,3} = 3 (ii) [ 0 5 1 ]
3 3 6 0 0 0
|𝐴| = 0 (∵ 𝑅2 : 𝑅3 ) −2 2 −1
∴ 𝜌(𝐴) < 3 Let 𝐴 = [ 0 5 1 ]. Order of A is3 × 3. So 𝜌(𝐴) ≤ 𝑚𝑖𝑛{3,3} = 3
2 × 2 𝑀𝑖𝑛𝑜𝑟: 0 0 0
3 2 3 × 3 Minor:
| | = 3 − 2 = 1 ≠ 0 . So 𝜌(𝐴) = 2 −2 2 −1
1 1
4 3 1 −2 | 0 5 1 | = 0. So 𝜌(𝐴) < 3
𝐄𝐗𝐀𝐌𝐏𝐋𝐄𝟏. 𝟏𝟓 [−3 −1 −2 4 ] 0 0 0
6 7 −1 2 2 × 2 Minor:
4 3 1 −2 −2 2
| | = −10 − 0 = −10 ≠ 0 . So 𝜌(𝐴) = 2
Let 𝐴 = [−3 −1 −2 4 ]. Order of A is 3 × 4. So 𝜌(𝐴) ≤ 𝑚𝑖𝑛{3,4} = 3 0 5
6 7 −1 2 6 0 −9
3 × 3 Minors: (iii)[0 2 0 ]
4 3 1 4 3 −2 0 0 0
|−3 −1 −2| = 0; |−3 −1 4 | = 0; 0 0 0
6 7 −1 6 7 2 6 0 −9
4 1 −2 3 1 −2 Let 𝐴 = [0 2 0 ]. Order of A is4 × 3. So 𝜌(𝐴) ≤ 𝑚𝑖𝑛{4,3} = 3
|−3 −2 4 | = 0; |−1 −2 4 | = 0 0 0 0
6 −1 2 6 −1 2 0 0 0
So 𝜌(𝐴) < 3 3 × 3 Minor:
2 × 2 Minor: All the 3 × 3 minors are equal to zero. So 𝜌(𝐴) < 3
4 3 2 × 2 Minor:
| | = −4 + 9 = 5 ≠ 0 . So 𝜌(𝐴) = 2 6 0
−3 −1 | | = 12 − 0 = 12 ≠ 0.
𝐄𝐗𝐀𝐌𝐏𝐋𝐄 𝟏. 𝟏𝟔 Find the rank of the following matrices which are in row- 0 2
V.GNANAMURUGAN, P,G,T, G.H.S.S, S.S.KOTTAI, SIVAGANGAI DT – 94874 43870 ; 82489 56766 , www.tnppgta.com, www.ednnet.in Page 10
So 𝜌(𝐴) = 2 1 2 −1
RANK OF A MATRIX IN ROW REDUCTION METHOD ~ [0 −7 5 ] 𝑅4 → 𝑅4 + 2𝑅3
2.Find the rank of the following matrices by row reduction ( row-echelon ) 0 0 8
method: 0 0 0
The equivalent matrix is in row-echelon form and the number of non-zero rows is
1 1 1 3
3. So 𝜌(𝐴) = 3
(i) [2 −1 3 4 ]
3 −8 5 2
5 −1 7 11
1 1 1 3 (iii)[ 2 −5 1 4 ]
Let 𝐴 = [2 −1 3 4 ] −1 2 3 −2
5 −1 7 11 3 −8 5 2
1 1 1 3 𝑅 → 𝑅 − 2𝑅 Let 𝐴 = [ 2 −5 1 4 ]
~ [0 −3 1 −2 ] 2 2 1
−1 2 3 −2
𝑅3 → 𝑅3 − 5𝑅1 −1 2 3 −2
0 −6 2 −4
1 1 1 3 ~ [ 2 −5 1 4 ] 𝑅1 → 𝑅3
~ [0 −3 1 −2 ] 𝑅3 → 𝑅3 − 2𝑅2 3 −8 5 2
0 0 0 0 −1 2 3 −2 𝑅 → 𝑅 + 2𝑅
2 2 1
The equivalent matrix is in row-echelon form and the number of non-zero rows is ~ [ 0 −1 7 0 ] 𝑅 → 𝑅 + 3𝑅
2. So 𝜌(𝐴) = 2 0 −2 14 −4 3 3 1
−1 2 3 −2
1 2 −1
~ [ 0 −1 7 0 ] 𝑅3 → 𝑅3 − 2𝑅2
(ii) [3 −1 2 ] 0 0 0 −4
1 −2 3
The equivalent matrix is in row-echelon form and the number of non-zero rows is
1 −1 1
1 2 −1 3. So 𝜌(𝐴) = 3
1 2 3
Let 𝐴 = [3 −1 2 ] 𝐄𝐗𝐀𝐌𝐏𝐋𝐄 𝟏. 𝟏𝟕 Find the rank of the matrix [2 1 4] by reducing it to a row -
1 −2 3
1 −1 1 3 0 5
1 2 −1 𝑅 → 𝑅 − 3𝑅 echelon form.
2 2 1
~ [0 −7 5 ] 𝑅3 → 𝑅3 − 𝑅1 1 2 3
0 −4 4 Let 𝐴 = [2 1 4]
𝑅4 → 𝑅4 − 𝑅1
0 −3 2 3 0 5
1 2 −1 1 2 3 𝑅 → 𝑅 − 2𝑅
𝑅 → 7𝑅3 − 4𝑅2 ~ [0 −3 −2] 2 2 1
~ [0 −7 5 ] 3 𝑅 → 𝑅3 − 3𝑅1
0 0 8 𝑅4 → 4𝑅4 − 3𝑅3 0 −6 −4 3
0 0 −4 1 2 3
~ [0 −3 −2] 𝑅3 → 𝑅3 − 2𝑅2
0 0 0
The equivalent matrix is in row-echelon form and the number of non-zero rows is
V.GNANAMURUGAN, P,G,T, G.H.S.S, S.S.KOTTAI, SIVAGANGAI DT – 94874 43870 ; 82489 56766 , www.tnppgta.com, www.ednnet.in Page 11
1 4
2. So 𝜌(𝐴) = 2 1
3 3
2 −2 4 3 2 11 𝑅2 → 𝑅2 − 2𝑅1
𝐄𝐗𝐀𝐌𝐏𝐋𝐄 𝟏. 𝟏𝟖 Find the rank of the matrix [−3 4 −2 −1] by reducing it ~ 0 − −
3 3 𝑅3 → 𝑅3 − 5𝑅1
6 2 −1 7 1 17
to a row -echelon form. [0 3
− ]
3
1 4
2 −2 4 3 1
3 3
𝐴 = [−3 4 −2 −1] 11 3
6 2 −1 7 ~ 0 1
2
𝑅2 → (− ) 𝑅2
2
2 −2 4 3 1 17
~ [−6 8 −4 −2] 𝑅2 → 2𝑅2 [0 3
− ]
3
1
6 2 −1 7 1 0 − 1
2
2 −2 4 3 𝑅2 → 𝑅2 + 3𝑅1 11 𝑅1 → 𝑅1 − 𝑅2
3
~ [0 2 8 7 ] 𝑅 → 𝑅 − 3𝑅 ~ 0 1
2 1
0 8 −13 −2 3 3 1 𝑅3 → 𝑅3 − 𝑅2
15 3
2 −2 4 3 [0 0 − ]
2
1
~ [0 2 8 7 ] 𝑅3 → 𝑅3 − 4𝑅2 1 0 −
2 2
0 0 −45 −30 ~ [0 1 11 ] 𝑅3 → (− ) 𝑅3
The equivalent matrix is in row-echelon form and the number of non-zero rows is 2
15
3. So 𝜌(𝐴) = 3 0 0 1
GAUSS-JORDAN METHOD FOR FINDING INVERSE OF THE MATRIX 1 0 0 𝑅1 → 𝑅1 − 1 𝑅3
2
3 1 4 ~ [0 1 0] 11
𝐄𝐗𝐀𝐌𝐏𝐋𝐄 𝟏. 𝟏𝟗 Show that the matrix [2 0 −1] is non-singular and reduce it 0 0 1 𝑅2 → 𝑅2 − 2 𝑅3
5 2 1 3.Find the inverse of each of the following by Gauss-Jordan method:
to the identity matrix by elementary row transformation. 2 −1
(i)[ ]
3 1 4 5 −2
Let 𝐴 = [2 0 −1] 2 −1 1 0
[𝐴|𝐼2 ] = ( | )
5 2 1 5 −2 0 1
1 1
|𝐴| = 3(0 + 2) − 1(2 + 5) + 4(4 − 0) = 6 − 7 + 16 = 15 ≠ 0 1 − 0 1
~( 2| 2 ) 𝑅2 → 2 𝑅2
∴ A is non-singular. 5 −2 0 1
1 4 1 1
3 1 4 1 1 − 2 0
3 3 1 2
[2 0 −1] ~ [2 0 −1] 𝑅1 → 3 𝑅1 ~( 1 | 5 ) 𝑅2 → 𝑅2 − 5𝑅1
5 2 1 0 − 1
5 2 1 2 2
1 1
1 − 2 0
~( 2| ) 𝑅2 → 2𝑅2
0 1 −5 2

V.GNANAMURUGAN, P,G,T, G.H.S.S, S.S.KOTTAI, SIVAGANGAI DT – 94874 43870 ; 82489 56766 , www.tnppgta.com, www.ednnet.in Page 12
1 0 −2 1 1 1 0 9 5 −2 0 𝑅 → 𝑅 − 2𝑅
~( | ) 𝑅1 → 𝑅1 + 2 𝑅2
0 1 −5 2 ~ (0 1 −3|−2 1 0) 1 1 2

= [𝐼2 |𝐴−1 ] 𝑅 → 𝑅3 + 2𝑅2


0 0 −1 −5 2 1 3
−2 1 1 0 0 −40 16 9 𝑅 → 𝑅 + 9𝑅
⟹ 𝐴−1 = [ ]
−5 2 ~ (0 1 0 | 14 −5 −3) 1 1 3
1 −1 0 𝑅2 → 𝑅2 − 3𝑅3
0 0 −1 −5 2 1
(ii)[1 0 −1] 1 0 0 −44 16 9
6 −2 −3 ~ (0 1 0| 13 −5 −3) 𝑅3 → (−)𝑅3
1 −1 0 1 0 0 0 0 1 5 −2 −1
[𝐴|𝐼3 ] = (1 0 −1|0 1 0) −1
= [𝐼3 |𝐴 ]
6 −2 −3 0 0 1 −40 16 9
1 −1 0 1 0 0 𝑅 → 𝑅 − 𝑅 −1
⟹ 𝐴 = [ 13 −5 −3]
~ (0 1 −1|−1 1 0) 2 2 1
5 −2 −1
𝑅3 → 𝑅3 − 6𝑅1
0 4 −3 −6 0 1 0 5
1 −1 0 1 0 0 𝐄𝐗𝐀𝐌𝐏𝐋𝐄 𝟏. 𝟐𝟎 Find the inverse of the non-singular matrix 𝐴 = [ ], by
−1 6
~ (0 1 −1|−1 1 0) 𝑅3 → 𝑅3 − 4𝑅2 Gauss-Jordan method.
0 0 1 −2 −4 1 1 0
1 −1 0 1 0 0 [𝐴|𝐼2 ] = ( 0 5| )
−1 6 0 1
~ (0 1 0|−3 −3 1) 𝑅2 → 𝑅2 + 𝑅3 −1 6 0 1
~( | ) 𝑅 ↔ 𝑅2
0 0 1 −2 −4 1 0 51 0 1
1 0 0 −2 −3 1 1 −6 0 −1
~ (0 1 0|−3 −3 1) 𝑅1 → 𝑅1 + 𝑅2 ~( | ) 𝑅1 → (−)𝑅1
0 5 1 0
0 0 1 −2 −4 1 1 −6 01 −1 1
= [𝐼3 |𝐴−1 ] ~( | ) 𝑅2 → 𝑅
0 1 0 5 2
5
−2 −3 1 6
−1
⟹ 𝐴 = [−3 −3 1] 1 05 −1
−2 −4 1 ~( | ) 𝑅1 → 𝑅1 + 6𝑅2
0 11 0
1 2 3 5
(iii) [2 5 3] 6
−1
1 0 8 1 6 −5
= [𝐼2 |𝐴−1 ] ⟹ 𝐴−1 = [51 ] = 5[ ]
1 2 31 0 0 0 1 0
[𝐴|𝐼3 ] = (2 5 3|0 1 0) 5
2 1 1
1 0 80 0 1
1 2 3 1 0 0 𝑅 → 𝑅 − 2𝑅 𝐄𝐗𝐀𝐌𝐏𝐋𝐄 𝟏. 𝟐𝟏 Find the inverse of 𝐴 = [3 2 1], by Gauss-Jordan method.
~ (0 1 −3|−2 1 0) 2 2 1 2 1 2
𝑅3 → 𝑅3 − 𝑅1 2 1 11 0 0
0 −2 5 −1 0 1
[𝐴|𝐼3 ] = (3 2 1|0 1 0)
2 1 20 0 1
V.GNANAMURUGAN, P,G,T, G.H.S.S, S.S.KOTTAI, SIVAGANGAI DT – 94874 43870 ; 82489 56766 , www.tnppgta.com, www.ednnet.in Page 13
1 1 1 2 1
1 0 0 |𝐴| = 4 + 3 = 7 ; 𝑎𝑑𝑗 𝐴 = [ ]
2 2 2 1 −3 2
~ (3 2 1|0 1 0) 𝑅1 → 𝑅1 1 2
2 1 1
2 1 20 0 1 𝐴−1 = |𝐴| 𝑎𝑑𝑗𝐴 = [ ]
7 −3 2
1
1 1 1
0 0 𝑥 1 2 1 8 1 16 − 2 1 14 2
2 2 2 𝑅 → 𝑅2 − 3𝑅1 𝑋 = 𝐴−1 𝐵 ⟹ [𝑦] = [ ][ ] = [ ]= [ ]=[ ]
7 −3 2 −2 7 −24 − 4 7 −28 −4
~ (0 1 − 1|− 3 1 0) 2 Solution: (𝑥, 𝑦) = (2, −4)
2 2 2
𝑅3 → 𝑅3 − 2𝑅1
0 0 1 −1 0 1 𝐄𝐗𝐀𝐌𝐏𝐋𝐄 𝟏. 𝟐𝟐 Solve the following system of linear equations by matrix
1 1 1
1 0 0 inversion method: 5𝑥 + 2𝑦 = 3, 3𝑥 + 2𝑦 = 5
2 2 2
~ (0 1 −1|−3 2 0) 𝑅2 → 2𝑅2 (𝑖𝑖𝑖)2𝑥 + 3𝑦 − 𝑧 = 9, 𝑥 + 𝑦 + 𝑧 = 9, 3𝑥 − 𝑦 − 𝑧 = −1
0 0 1 −1 0 1 2 3 −1 𝑥 9
1 0 1 2 −1 0 [1 1 1 ] [𝑦] = [ 9 ] ⟹ 𝐴𝑋 = 𝐵 ⟹ 𝑋 = 𝐴−1 𝐵
1
~ (0 1 −1|−3 2 0) 𝑅1 → 𝑅1 − 𝑅2 3 −1 −1 𝑧 −1
2
0 0 1 −1 0 1 |𝐴| = 2(−1 + 1) − 3(−1 − 3) − 1(−1 − 3) = 12 + 4 = 16
1 0 0 3 −1 −1 𝑅 → 𝑅 − 𝑅 1 −1 3 1 −1 + 1 1 + 3 3+1 0 4 4
~ (0 1 0|−4 2 1 ) 𝑅1 → 𝑅1 + 𝑅3 1 −1 −1 1 ⟹ adjA = [ 3 + 1 −2 + 3 −1 − 2] = [ 4 1 −3]
0 0 1 −1 0 1 2 2 3
1 3 2 1
3 −1 −1 −1 − 3 9 + 2 2−3 −4 11 −1
1 −1 3 1
−1
= [𝐼3 |𝐴 ] ⟹ 𝐴 = [−4 2 −1
1] 0 4 4
−1 1 1
−1 0 1 𝐴 = |𝐴| 𝑎𝑑𝑗𝐴 = [ 4 1 −3]
16
MATRIX INVERSION METHOD −4 11 −1
𝑥 0 4 4 9 0 + 36 − 4 32
EXERCISE 1.3 −1 1 1 1
𝑋 = 𝐴 𝐵 ⟹ [𝑦] = [ 4 1 −3] [ 9 ] = 16 [ 36 + 9 + 3 ] = 16 [48]
1.Solve the following system of linear equations by matrix inversion method: 16
𝑧 −4 11 −1 −1 −36 + 99 + 1 64
(𝑖)2𝑥 + 5𝑦 = −2, 𝑥 + 2𝑦 = −3 𝑥 2
2 5 𝑥 −2 [𝑦] = [3] Solution : (𝑥, 𝑦, 𝑧) = (2,3,4)
[ ] [𝑦] = [ ] ⟹ 𝐴𝑋 = 𝐵 ⟹ 𝑋 = 𝐴−1 𝐵
1 2 −3 𝑧 4
|𝐴| = 4 − 5 = −1 ; 𝑎𝑑𝑗 𝐴 = [ 2 −5] (𝑖𝑣)𝑥 + 𝑦 + 𝑧 − 2 = 0, 6𝑥 − 4𝑦 + 5𝑧 − 31 = 0, 5𝑥 + 2𝑦 + 2𝑧 = 13
−1 2 1 1 1 𝑥 2
1 1 2 −5 −2 5
𝐴−1 = |𝐴| 𝑎𝑑𝑗𝐴 = [ ]=[ ] [6 −4 5] [ ] = [31] ⟹ 𝐴𝑋 = 𝐵 ⟹ 𝑋 = 𝐴−1 𝐵
𝑦
−1 −1 2 1 −2
𝑥 −2 5 −2 4 − 15 −11 5 2 2 𝑧 13
𝑋 = 𝐴−1 𝐵 ⟹ [𝑦] = [ ][ ] = [ ]=[ ] |𝐴| = 1(−8 − 10) − 1(12 − 25) + 1(12 + 20) = −18 + 13 + 32 = 27
1 −2 −3 −2 + 6 4
Solution: (𝑥, 𝑦) = (−11,4) −4 2 1 −4 −8 − 10 2 − 2 5 + 4 −18 0 9
(𝑖𝑖)2𝑥 − 𝑦 = 8, 3𝑥 + 2𝑦 = −2 5 2 1 5 ⟹ adjA = [
6 25 − 12 2 − 5 6 − 5 ] = [ 13 −3 1 ]
2 −1 𝑥 8 6 5 1 32 3 −10
[ ] [𝑦] = [ ] ⟹ 𝐴𝑋 = 𝐵 ⟹ 𝑋 = 𝐴−1 𝐵 −4 2 1 −4 12 + 20 5 − 2 −4 − 6
3 2 −2

V.GNANAMURUGAN, P,G,T, G.H.S.S, S.S.KOTTAI, SIVAGANGAI DT – 94874 43870 ; 82489 56766 , www.tnppgta.com, www.ednnet.in Page 14
−18 0 9 4 0 0
−1 1 1
𝐴 = |𝐴| 𝑎𝑑𝑗𝐴 = [ 13 −3 1 ] = [0 4 0] = 4𝐼3
27
32 3 −10 0 0 4
𝑥 −18 0 9 2 −36 + 0 + 117 1 1 2 −5 1 3 4 0 0
−1 1 1
𝑋 = 𝐴 𝐵 ⟹ [𝑦] = [ 13 −3 1 ] [31] = 27 [ 26 − 93 + 13 ] 𝐵𝐴 = [3 2 1] [ 7 1 −5] = [0 4 0] = 4𝐼3
27
𝑧 32 3 −10 13 64 + 93 − 130 2 1 3 1 −1 1 0 0 4
81 𝑥 3 −1
𝐴𝐵 = 𝐵𝐴 = 4𝐼3 ⟹ 𝐵 = 𝐴 → (1)
1
1
= [−54] ⟹ [𝑦] = [−2] Solution : (𝑥, 𝑦, 𝑧) = (3, −2,1) 4
27 1 1 2 𝑥 1
27 𝑧 1 [3 2 1] [𝑦] = [7] ⟹ 𝐵𝑋 = 𝐶 ⟹ 𝑋 = 𝐵−1 𝐶 → (2)
𝐄𝐗𝐀𝐌𝐏𝐋𝐄 𝟏. 𝟐𝟑 Solve the following system of linear equations by matrix
2 1 3 𝑧 2
inversion method: 2𝑥1 + 3𝑥2 + 3𝑥3 = 5, 𝑥1 − 2𝑥2 + 𝑥3 = −4,3𝑥1 − 𝑥2 − 2𝑥3 = −5 1 3
−1 1 1
3 From (1) ⟹ 𝐵 = 𝐴 = [ 7 1 −5]
4 4
2 3 3 𝑥1 5 1 −1 1
[1 −2 1 ] [𝑥2 ] = [−4] ⟹ 𝐴𝑋 = 𝐵 ⟹ 𝑋 = 𝐴−1 𝐵 𝑥 −5 1 3 1 −5 + 7 + 6
−1 1 1
3 −1 −2 𝑥3 3 From (2) ⟹ 𝑋 = 𝐵 𝐶 ⟹ [𝑦] = [ 7
4
1 −5] [7] = 4 [ 7 + 7 − 10 ]
|𝐴| = 2(4 + 1) − 3(−2 − 3) + 3(−1 + 6) = 10 + 15 + 15 = 40 𝑧 1 −1 1 2 1−7+2
−2 −1 3 −2 8 2
4 + 1 −3 + 6 3 + 6 5 3 9 1
1 −2 3 1 ⟹ adjA = [ 3 + 2 −4 − 9 3 − 2 ] = [ = [ 4 ]=[ 1 ] Solution : (𝑥, 𝑦, 𝑧) = (2,1, −1)
1 5 −13 1 ] 4
1 3 2 −4 −1
−1 + 6 9 + 2 −4 − 3 5 11 −7 −4 4 4 1 −1 1
−2 −1 3 −2
5 3 9 𝐄𝐗𝐀𝐌𝐏𝐋𝐄 𝟏. 𝟐𝟒 If 𝐴 = [−7 1 3 ] and 𝐵 = [1 −2 −2], find the
−1 1 1
𝐴 = |𝐴| 𝑎𝑑𝑗𝐴 = [5 −13 1 ] 5 −3 −1 2 1 3
40
5 11 −7 products AB and BA and hence solve the system of equations 𝑥 − 𝑦 + 𝑧 = 4, 𝑥 −
𝑥 5 3 9 5 25 − 12 + 27 40 2𝑦 − 2𝑧 = 9, 2𝑥 + 𝑦 + 3𝑧 = 1
−1 1 1 1
𝑋 = 𝐴 𝐵 ⟹ [𝑦] = [5 −13 1 ] [−4] = [ 25 + 52 + 3 ] = [ 80 ] 3. A man is appointed in a job with a monthly salary of certain amount and a fixed
40 40 40
𝑧 5 11 −7 3 25 − 44 − 21 −40 amount of annual increment. If his salary was Rs.19800 per month at the end of
𝑥 1
[𝑦 ] = [ 2 ] Solution : (𝑥, 𝑦, 𝑧) = (1,2, −1) the first month after 3 years of service and Rs.23400 per month at the end of the
𝑧 −1 first month after 9 years of service, find his starting salary and his annual
−5 1 3 1 1 2 increment. ( Use matrix inversion method )
2.If 𝐴 = [ 7 1 −5] and 𝐵 = [3 2 1], find the products AB and BA and Let his starting salary be x and the annual increment be y.
1 −1 1 2 1 3 From the given data, 𝑥 + 3𝑦 = 19800 → (1) ; 𝑥 + 9𝑦 = 23400 → (2)
hence solve the system of equations 𝑥 + 𝑦 + 2𝑧 = 1, 3𝑥 + 2𝑦 + 𝑧 = 7, 2𝑥 + 𝑦 + 1 3 𝑥 19800
3𝑧 = 2 [ ] [𝑦] = [ ] ⟹ 𝐴𝑋 = 𝐵 ⟹ 𝑋 = 𝐴−1 𝐵
1 9 23400
−5 1 3 1 1 2 −5 + 3 + 6 −5 + 2 + 3 −10 + 1 + 9 |𝐴| = 9 − 3 = 6 ; 𝑎𝑑𝑗 𝐴 = [ 9 −3]
𝐴𝐵 = [ 7 1 −5] [3 2 1] = [ 7 + 3 − 10 7 + 2 − 5 14 + 1 − 15 ] −1 1
1 −1 1 2 1 3 1−3+2 1−2+1 2−1+3
V.GNANAMURUGAN, P,G,T, G.H.S.S, S.S.KOTTAI, SIVAGANGAI DT – 94874 43870 ; 82489 56766 , www.tnppgta.com, www.ednnet.in Page 15
1 1 9 −3 5. The prices of three commodities A, B and C are Rs.x, y and z per units
𝐴−1 = |𝐴| 𝑎𝑑𝑗𝐴 = [ ]
6 −1 1 respectively. A person P purchases 4 units of B and sells 2 units of A and 5 units of
𝑥 1 9 −3 19800 1 178200 − 70200 1 108000
𝑋 = 𝐴−1 𝐵 ⟹ [𝑦] = [ ][ ]= [ ]= [ ] C. Person Q purchase 2 unit of C and sells 3 units of A and one unit of B. Person R
6 −1 1 23400 6 −19800 + 23400 6 3600
𝑥 18000 purchases one unit of A and sells 3 unit of B and one unit of C. In the process P, Q
[𝑦 ] = [ ] and R earn Rs.15000, Rs.1000 and Rs.4000 respectively. Find the prices per unit of
600
Solution: His starting salary = Rs.18000 ; Annual increment = Rs.600 A, B and C. ( Use matrix inversion method )
4. 4 men and 4 women can finish a piece of work jointly in 3 days while 2 men and Let the prices of one unit of A, B and C are Rs. x, y and z respectively.
5 women can finish the same work jointly in 4 days. Find the time taken by one From the given data,
man alone and that of one woman alone to finish the same work by using matrix 2𝑥 − 4𝑦 + 5𝑧 = 15000 → (1)
inversion method. 3𝑥 + 𝑦 − 2𝑧 = 1000 → (2)
Let the number of days taken by a man and woman be a and y respectively. −𝑥 + 3𝑦 + 𝑧 = 4000 → (3)
Work finished by a man in one day =
1 2 −4 5 𝑥 15000
𝑥
1
[3 1 −2] [𝑦] = [ 1000 ] ⟹ 𝐴𝑋 = 𝐵 ⟹ 𝑋 = 𝐴−1 𝐵
Work finished by a woman in one day = −1 3 1 𝑧 4000
𝑦
4 4 1 2 5 1 |𝐴| = 2(1 + 6) − (−4)(3 − 2) + 5(9 + 1) = 14 + 4 + 50 = 68
From the given data, + = ; + = 1 3 −4 1
𝑥 𝑦 3 𝑥 𝑦 4
1 1 1 + 6 15 + 4 8 − 5 7 19 3
Let = 𝑎 , = 𝑏 −2 1 5 −2 ⟹ adjA = [2 − 3 2 + 5 15 + 4] = [−1 7 19]
𝑥 𝑦 3
3 −1 2 10 −2 14
4 4 1 1
+ = ⟹ 4𝑎 + 4𝑏 = ⟹ 12𝑎 + 12𝑏 = 1 → (1) 1 3 −4 1 9 + 1 4 − 6 2 + 12
𝑥 𝑦 3 3
2 5 1 1 7 19 3
1 1
+ = ⟹ 2𝑎 + 5𝑏 = ⟹ 8𝑎 + 20𝑏 = 1 → (2) −1
𝐴 = |𝐴| 𝑎𝑑𝑗𝐴 = [−1 7 19]
𝑥 𝑦 4 4 68
12 12 𝑎 1 10 −2 14
[ ] [ ] = [ ] ⟹ 𝐴𝑋 = 𝐵 ⟹ 𝑋 = 𝐴−1 𝐵 𝑥 7 19 3 15000
8 20 𝑏 1 −1 1
20 −12 𝑋 = 𝐴 𝐵 ⟹ [𝑦] = [−1 7 19] [ 1000 ]
|𝐴| = 240 − 96 = 144 ; 𝑎𝑑𝑗 𝐴 = [ ] 68
−8 12 𝑧 10 −2 14 4000
1 1 20 −12 7 19 3 15
𝐴−1 = |𝐴| 𝑎𝑑𝑗𝐴 = [ ] 1000
144 −8 12 = [−1 7 19] [ 1 ]
1 68
𝑎 10 −2 14 4
1 20 −12 1 20 − 12
1 1 8 105 + 19 + 12
𝑋 = 𝐴−1 𝐵 ⟹ [ ] = [ ][ ] = [ ]= [ ] = [18
1] 1000
𝑏 144 −8 12 1 144 −8 + 12 144 4 = [ −15 + 7 + 76 ]
36 68
1 150 − 2 + 56
𝑎 1 1 136
[ ] = [18 ] ⟹ 𝑥 = 𝑎 = 18 ; 𝑦 = 𝑏 = 36 =
1000
[ 68 ]
𝑏 1
68
36 204
Solution: A man can finish the work in 18 days and a woman can finish the work in
36 days.
V.GNANAMURUGAN, P,G,T, G.H.S.S, S.S.KOTTAI, SIVAGANGAI DT – 94874 43870 ; 82489 56766 , www.tnppgta.com, www.ednnet.in Page 16
𝑥 2000 ∆ −44 ∆𝑦 −66 ∆ −88
𝑥= 𝑥= =2 ;𝑦 = = = 3 ;𝑧 = 𝑧 = =4
𝑦
⟹ [ ] = [1000] ∆ −22 ∆ −22 ∆ −22
𝑧 3000 Solution : (𝑥, 𝑦, 𝑧) = (2,3,4)
3 4 2 1 2 1 2 5 4
Solution: The price of one unit of A, B and C are Rs.2000, 1000 and 3000 (𝑖𝑣) − − − 1 = 0, + + − 2 = 0, − − + 1 = 0
𝑥 𝑦 𝑧 𝑥 𝑦 𝑧 𝑥 𝑦 𝑧
respectively. 1 1 1
Let = 𝑎, = 𝑏, = 𝑐
CRAMER’S RULE 𝑥 𝑦 𝑧
EXERCISE 1.4 ⟹ 3𝑎 − 4𝑏 − 2𝑐 = 1 → (1), 𝑎 + 2𝑏 + 𝑐 = 2 → (2), 2𝑎 − 5𝑏 − 4𝑐 = −1 → (3)
3 −4 −2
1. Solve the following systems of linear equations by Cramer’s rule: ∆= |1 2 1 | = 3(−8 + 5) − (−4)(−4 − 2) − 2(−5 − 4) = −9 − 24 + 18 = −15
(𝑖)5𝑥 − 2𝑦 + 16 = 0, 𝑥 + 3𝑦 − 7 = 0 2 −5 −4
5 −2 −16 −2 1−4 −2
∆= | | = 15 + 2 = 17 ∆𝑥 = | | = −48 + 14 = −34 ∆𝑎 = | 2 2 1 | = 1(−8 + 5) − (−4)(−8 + 1) − 2(−10 + 2) = −3 − 28 + 16 = −15
1 3 7 3
5 −16 ∆ −34 ∆𝑦 51 −1 −5 −4
∆𝑦 = | | = 35 + 16 = 51 𝑥= 𝑥= = −2 ; 𝑦 = = = 3 3 1 −2
1 7 ∆ 17 ∆ 17
Solution: (𝑥, 𝑦) = (−2,3) ∆ 𝑏 = |1 2 1 | = 3(−8 + 1) − 1(−4 − 2) − 2(−1 − 4) = −21 + 6 + 10 = −5
3 2 2 −1 −4
(𝑖𝑖) + 2𝑦 = 12 , + 3𝑦 = 13 3 −4 1
𝑥 𝑥
1 ∆𝑐 = |1 2 2 | = 3(−2 + 10) − (−4)(−1 − 4) + 1(−5 − 4) = 24 − 20 − 9 = −5
Let = 𝑎, 𝑦 = 𝑏 ⟹ 3𝑎 + 2𝑏 = 12 ; 2𝑎 + 3𝑏 = 13 2 −5 −1
𝑥
3 2 12 2 ∆𝑎 −15 1
∆= | |=9−4=5 ∆𝑎 = | | = 36 − 26 = 10 𝑎= = =1⟹𝑥= =1
∆ −15 𝑎
2 3 13 3 ∆𝑏 −5 1 1
3 12 𝑏= = = ⟹𝑦= =3
∆𝑦 = | | = 39 − 24 = 15 ∆ −15 3 𝑏
2 13 ∆𝑐 −5 1 1
∆ 10 1 1 ∆ 15 𝑐= = = ⟹𝑧= =3
𝑎= 𝑎= =2⟹𝑥= ⟹𝑥= ; 𝑏= 𝑏= =3⟹𝑦=3 ∆ −15 3 𝑐
∆ 5 𝑎 2 ∆ 5 Solution : (𝑥, 𝑦, 𝑧) = (1,3,3)
1
Solution: (𝑥, 𝑦) = ( , 3) 𝐄𝐗𝐀𝐌𝐏𝐋𝐄 𝟏. 𝟐𝟓 Solve by Cramer’s rule, the system of equations 𝑥1 − 𝑥2 =
2
(𝑖𝑖𝑖)3𝑥 + 3𝑦 − 𝑧 = 11, 2𝑥 − 𝑦 + 2𝑧 = 9, 4𝑥 + 3𝑦 + 2𝑧 = 25 3, 2𝑥1 + 3𝑥2 + 4𝑥3 = 17, 𝑥2 + 2𝑥3 = 7
3 3 −1 1 −1 0
∆= |2 −1 2 | = 3(−2 − 6) − 3(4 − 8) − 1(6 + 4) = −24 + 12 − 10 = −22 ∆= |2 3 4| = 1(6 − 4) − (−1)(4 − 0) + 0 = 2 + 4 = 6
4 3 2 0 1 2
11 3 −1 3 −1 0
∆𝑥 = | 9 −1 2 | = 11(−2 − 6) − 3(18 − 50) − 1(27 + 25) = −88 + 96 − 52 = −44 ∆1 = |17 3 4| = 3(6 − 4) − (−1)(34 − 28) + 0 = 6 + 6 = 12
25 3 2 7 1 2
3 11 −1 1 3 0
∆ 𝑦 = |2 9 2 | = 3(18 − 50) − 11(4 − 8) − 1(50 − 36) = −96 + 44 − 14 = −66 ∆2 = |2 17 4| = 1(34 − 28) − 3(4 − 0) + 0 = 6 − 12 = −6
4 25 2 0 7 2
3 3 11 1 −1 3
∆𝑧 = |2 −1 9 | = 3(−25 − 27) − 3(50 − 36) + 11(6 + 4) = −156 − 42 + 110 = −88 ∆3 = |2 3 17| = 1(21 − 17) − (−1)(14 − 0) + 3(2 − 0) = 4 + 14 + 6 = 24
4 3 25 0 1 7

V.GNANAMURUGAN, P,G,T, G.H.S.S, S.S.KOTTAI, SIVAGANGAI DT – 94874 43870 ; 82489 56766 , www.tnppgta.com, www.ednnet.in Page 17
∆ 12 ∆ −6 ∆ 24 ∆ −150 ∆𝑦 −100
𝑥1 = 1 = = 2 ; 𝑥2 = 2 = = −1 ;𝑥3 = 3 = = 4 𝑥= 𝑥= =6 ; 𝑦= = =4
∆ 6 ∆ 6 ∆ 6 ∆ −25 ∆ −25
Solution : (𝑥1 , 𝑥2 , 𝑥3 ) = (2, −1,4) Solution:
2. In a competitive examination, one mark is awarded for every correct answer 6 litres of solution containing 50% acid and 4 litres of solution containing 25%
1
while mark is deducted for every wrong answer. A student answered 100 acid must be mixed to make 40% acid solution.
4
questions and get 80 marks. How many questions did he answer correctly? (Use 4. A fish tank can be filled in 10 minutes using both pumps A and B simultaneously.
Cramer’s rule ) However, pump B can pump water in or out at the same rate. If pump B is
Let the number questions answered correctly be x and y be the number of inadvertently run in reverse, then the tank will be filled in 30 minutes. How long
questions answered wrong. would it take each pump to fill the tank by itself? (Use Cramer’s rule )
1 Let x and y be the time taken by pumps A and B respectively.
From the given data, 𝑥 + 𝑦 = 100 → (1) ; 𝑥 − 𝑦 = 80 ⟹ 4𝑥 − 𝑦 = 320 → (2) 1
4 Amount of water filled by pump A in one minute=
1 1 𝑥
∆= | | = −1 − 4 = −5 1
4 −1 Amount of water filled by pump B in one minute=
100 1 𝑦
∆𝑥 = | | = −100 − 320 = −420 From the given data, + =
1 1 1
→ (1) ;
1 1
− =
1
→ (2)
320 −1 𝑥 𝑦 10 𝑥 𝑦 30
1 100 1 1
∆𝑦 = | | = 320 − 400 = −80 Let = 𝑎 , = 𝑏
4 320 𝑥 𝑦
∆ −420 ∆𝑦 −80 1
𝑥= 𝑥= = 84 ; 𝑦= = = 16 (1) ⟹ 𝑎 + 𝑏 = ⟹ 10𝑎 + 10𝑏 = 1 → (3)
∆ −5 ∆ −5 10
Solution: 1
(2) ⟹ 𝑎 − 𝑏 = ⟹ 30𝑎 − 30𝑏 = 1 → (4)
The number questions answered correctly= 84 30
Solving (3) & (4) ,
The number questions answered wrong= 16
10 10
3. A chemist has one solution which is 50% acid and another solution which is ∆= | | = −300 − 300 = −600
30 −30
25% acid. How much each should be mixed to make 10 litres of 40% acid 1 10
∆𝑎 = | | = −30 − 10 = −40
solution? (Use Cramer’s rule ) 1 −30
Let x and y be the amount of solution containing 50% and 25% acid respectively. 10 1
∆𝑏 = | | = 10 − 30 = −20
From the given data, 30 1
∆ −40 1 ∆ −20 1 1 1
𝑥 + 𝑦 = 10 → (1) 𝑎= 𝑎= = ;𝑏 = 𝑏 = = ⟹ 𝑥 = = 15 , 𝑦 = = 30
∆ −600 15 ∆ −600 30 𝑎 𝑏
50 25 40 Solution:
50% 𝑜𝑓 𝑥 + 25% 𝑜𝑓 𝑦 = 40% 𝑜𝑓 10 ⟹ 𝑥+ 𝑦= . 10
100 100 100 Time taken by pump A= 15 minutes ; Time taken by pump B= 30 minutes
⟹ 50𝑥 + 25𝑦 = 400 → (2)
1 1 5.A family of 3 people went out for dinner in a restaurant. The cost of two dosai,
∆= | | = 25 − 50 = −25 three idlies and two vadais is Rs. 150. The cost of two dosai, two idlies and four
50 25
10 1 vadais is Rs. 200. The cost of five dosai, four idlies and two vadais is Rs. 250. The
∆𝑥 = | | = 250 − 400 = −150
400 25 family has Rs. 350 in hand and they ate 3 dosai and six idlies and six vadais. Will
1 100 they be able to manage to pay the bill within the amount they had? (Use Cramer’s
∆𝑦 = | | = 400 − 500 = −100
50 400
V.GNANAMURUGAN, P,G,T, G.H.S.S, S.S.KOTTAI, SIVAGANGAI DT – 94874 43870 ; 82489 56766 , www.tnppgta.com, www.ednnet.in Page 18
rule ) (20,16) ⟹ 400𝑎 + 20𝑏 + 𝑐 = 16 → (2)
Let Rs.x, y and z be the cost of I dosai, 1 idly and I vadai respectively. (40,22) ⟹ 1600𝑎 + 40𝑏 + 𝑐 = 22 → (3)
From the given data, 100 10 1 1 1 1
∆= | 400 20 1| = 100 × 10 | 4 2 1|
2𝑥 + 3𝑦 + 2𝑧 = 150 → (1)
1600 40 1 16 4 1
2𝑥 + 2𝑦 + 4𝑧 = 200,÷ 2 ⟹ 𝑥 + 𝑦 + 2𝑧 = 100 → (2) = 1000[1(2 − 4) − 1(4 − 16) + 1(16 − 32)] = 1000[−2 + 12 − 16] = −6000
5𝑥 + 4𝑦 + 2𝑧 = 250 → (3) 8 10 1 4 1 1
2 3 2 ∆𝑎 = |16 20 1| = 2 × 10 | 8 2 1| = 20[4(2 − 4) − 1(8 − 11) + 1(32 − 22)]
∆= |1 1 2| = 2(2 − 8) − 3(2 − 10) + 2(4 − 5) = −12 + 24 − 2 = 10 22 40 1 11 4 1
5 4 2 = 20[−8 + 3 + 10] = 100
150 3 2 100 8 1 1 4 1
∆𝑥 = |100 1 2| = 150(2 − 8) − 3(200 − 500) + 2(400 − 250) ∆𝑏 = | 400 16 1| = 100 × 2 | 4 8 1|
250 4 2 1600 22 1 16 11 1
= −900 + 900 + 300 = 300 = 200[1(8 − 11) − 4(4 − 16) + 1(44 − 128)] = 200[−3 + 48 − 84] = −7800
2 150 2 100 10 8 1 1 4
∆𝑦 = |1 100 2| = 2(200 − 500) − 150(2 − 10) + 2(250 − 500) ∆𝑐 = | 400 20 16| = 100 × 10 × 2 | 4 2 8 |
5 250 2 1600 40 22 16 4 11
= −600 + 1200 − 500 = 100 = 2000[1(22 − 32) − 1(44 − 128) + 4(16 − 32)] = 2000[−10 + 84 − 64] = 20000
2 3 150 ∆𝑎 100 1 ∆𝑏 −7800 13 𝑐 20000 10
∆𝑧 = |1 1 100| = 2(250 − 400) − 3(250 − 500) + 150(4 − 5) 𝑎= = =− ;𝑏 = = = ;𝑐 = = =−
∆ −6000 60 ∆ −6000 10 ∆ −6000 3
5 4 250 1 2 13 10
∴ The equation of the path is 𝑦 = − 𝑥 + 𝑥−
−300 + 750 − 150 = 300 60 10 3
∆𝑦 1 2 13 10 4900 910 10

𝑥= 𝑥=
300
= 30 ; 𝑦 = =
100 ∆
= 10 ;𝑧 = 𝑧 =
300
= 30 Sub. 𝑥 = 70, we get 𝑦 = − (70) + (70) − = − + − =6
60 10 3 60 10 3
∆ 10 ∆ 10 ∆ 10
The cost of 3 dosai , 6 idlies and 6 vadais = 3𝑥 + 6𝑦 + 6𝑧 ⟹The ball went 6 m high over the boundary line and it is impossible for a fielder
= 3(30) + 6(10) + 6(30) = 90 + 60 + 180 = 330 < 350 to catch the ball. Hence the ball went for six and the Chennai Super Kings won the
Hence they can manage to pay the bill. match.
𝐄𝐗𝐀𝐌𝐏𝐋𝐄 𝟏. 𝟐𝟔 In a T20 match, Chennai Super Kings needed just 6 runs to win GAUSSIAN ELIMINATION METHOD
with 1 ball left to go in the last over. The last ball was bowled and the batsman at EXERCISE 1.5
the crease hit it high up. The ball travelled along the path in a vertical plane and 1.Solve the following systems of linear equations by Gaussian elimination method:
the equation of the path is 𝑦 = 𝑎𝑥 2 + 𝑏𝑥 + 𝑐 with respect to a xy-coordinate (𝑖)2𝑥 − 2𝑦 + 3𝑧 = 2, 𝑥 + 2𝑦 − 𝑧 = 3, 3𝑥 − 𝑦 + 2𝑧 = 1
2 −2 3 2
ystem in the vertical plane and the ball travelled through the points
[𝐴|𝐵] = (1 2 −1|3)
(10,8), (20,16),
3 −1 2 1
(40,22), can you conclude that , Chennai Super Kings won the match ? Justify your 1 2 −1 3
answer. ( All the distances are measured in metres and the meeting point of the ~ (2 −2 3 |2) 𝑅1 ↔ 𝑅2
plane of the path with the farthest boundary line is (70,0).) 3 −1 2 1
𝑦 = 𝑎𝑥 2 + 𝑏𝑥 + 𝑐
(10,8) ⟹ 100𝑎 + 10𝑏 + 𝑐 = 8 → (1)
V.GNANAMURUGAN, P,G,T, G.H.S.S, S.S.KOTTAI, SIVAGANGAI DT – 94874 43870 ; 82489 56766 , www.tnppgta.com, www.ednnet.in Page 19
1 2 −1 3 𝑅 → 𝑅 − 2𝑅 9 −3 1 21
~ (0 −6 5 |−4) 2 2 1
[𝐴|𝐵] = (25 5 1|61)
𝑅 → 𝑅3 − 3𝑅1
0 −7 5 −8 3 1 1 1 9
1 2 −1 3 1 1 1 9
~ (0 −6 5 | −4 ) 𝑅3 → 6𝑅3 − 7𝑅2 ~ (25 5 1|61) 𝑅1 ↔ 𝑅3
0 0 −5 −20 9 −3 1 21
⟹ 𝑥 + 2𝑦 − 𝑧 = 3 → (1) ; −6𝑦 + 5𝑧 = −4 → (2); −5𝑧 = −20 ⟹ 𝑧 = 4 1 1 1 9 𝑅 → 𝑅2 − 25𝑅1
Put 𝑧 = 4 in (2) ⟹ −6𝑦 + 5(4) = −4 ⟹ −6𝑦 + 20 = −4 ~ (0 −20 −24|−164) 2
𝑅3 → 𝑅3 − 9𝑅1
⟹ −6𝑦 = −24 ⟹ 𝑦 = 4 0 −12 −8 −60
Put 𝑦 = 16, 𝑧 = 4 in (1) ⟹ 𝑥 + 2(4) − 4 = 3 ⟹ 𝑥 + 8 − 4 = 3 ⟹ 𝑥 = −1 1 1 1 9 𝑅2 → (− 1) 𝑅2
4
Solution: (𝑥, 𝑦, 𝑧) = (−1,4,4) ~ (0 5 6| 41 ) 1
(𝑖𝑖)2𝑥 + 4𝑦 + 6𝑧 = 22, 3𝑥 + 8𝑦 + 5𝑧 = 27, −𝑥 + 𝑦 + 2𝑧 = 2 0 3 2 −15 𝑅3 → (− 4) 𝑅3
2𝑥 + 4𝑦 + 6𝑧 = 22 ,÷ 2 ⟹ 𝑥 + 2𝑦 + 3𝑧 = 11 1 1 1 9
1 2 3 11 ~ (0 5 6 | 41 ) 𝑅3 → 5𝑅3 − 3𝑅2
[𝐴|𝐵] = ( 3 8 5|27) 0 0 −8 −48
−1 1 2 2 ⟹ 𝑎 + 𝑏 + 𝑐 = 9 → (1) ; 5𝑏 + 6𝑐 = 41 → (2) ; −8𝑐 = −48 ⟹ 𝑐 = 6
1 2 3 11 𝑅 → 𝑅 − 3𝑅 Put 𝑐 = 6 𝑖𝑛 (2) ⟹ 5𝑏 + 6(6) = 41 ⟹ 5𝑏 = 41 − 36 ⟹ 5𝑏 = 5 ⟹ 𝑏 = 1
~ (0 2 −4|−6) 2 2 1
Put 𝑏 = 1, 𝑐 = 6 𝑖𝑛 (1) ⟹ 𝑎 + 1 + 6 = 9 ⟹ 𝑎 + 7 = 9 ⟹ 𝑎 = 2
𝑅3 → 𝑅3 + 𝑅1
0 3 5 13 Solution: (𝑎, 𝑏, 𝑐) = (2,1,6)
1 2 3 11 3. An amount of Rs.65,000 is invested in three bonds at the rates of 6%, 8% and
~ (0 2 −4|−6) 𝑅3 → 2𝑅3 − 3𝑅2 10% per annum respectively. The total annual income is Rs.5,000. The income
0 0 22 44 from the third bond is Rs.800 more than that from the second bond. Determine
⟹ 𝑥 + 2𝑦 + 3𝑧 = 11 → (1) ; 2𝑦 − 4𝑧 = −6 → (2) ; 22𝑧 = 44 ⟹ 𝑧 = 2
Put 𝑧 = 2 𝑖𝑛 (2) ⟹ 2𝑦 − 4(2) = −6 ⟹ 2𝑦 = −6 + 8 ⟹ 2𝑦 = 2 ⟹ 𝑦 = 1 the price of each bond. ( Use Gaussian elimination method )
Put 𝑦 = 1, 𝑧 = 2 𝑖𝑛 (1) ⟹ 𝑥 + 2(1) + 3(2) = 11 ⟹ 𝑥 + 8 = 11 ⟹ 𝑥 = 3 Let the price of three bond be x, y and z respectively.
Solution: (𝑥, 𝑦, 𝑧) = (3,1,2) From the given data,
𝑥 + 𝑦 + 𝑧 = 65000 → (1)
𝐄𝐗𝐀𝐌𝐏𝐋𝐄 𝟏. 𝟐𝟕 Solve the following systems of linear equations by Gaussian
Total annual income = Rs.5,000⟹ 6% 𝑜𝑓 𝑥 + 8% 𝑜𝑓 𝑦 + 10% 𝑜𝑓 𝑧 = 5000
elimination method: 4𝑥 + 3𝑦 + 6𝑧 = 25, 𝑥 + 5 + 7𝑧 = 13, 2𝑥 + 9𝑦 + 𝑧 = 1 6 8 10
2. If 𝑎𝑥 2 + 𝑏𝑥 + 𝑐 is divided by 𝑥 + 3, 𝑥 − 5 and 𝑥 − 1 the remainders are 21, 61 ⟹ 𝑥+ 𝑦+ 𝑧 = 5000 ⟹ 6𝑥 + 8𝑦 + 10𝑧 = 500000
100 100 100
and 9 respectively. Find 𝑎, 𝑏 and c. ( Use Gaussian elimination method ) ⟹÷ 2, 3𝑥 + 4𝑦 + 5𝑧 = 250000 → (2)
Let 𝑝(𝑥) = 𝑎𝑥 2 + 𝑏𝑥 + 𝑐 Income from third bond = income from second bond + Rs.800
10 8
𝑝(−3) = 21 ⟹= 𝑎(−3)2 + 𝑏(−3) + 𝑐 = 21 ⟹ 9𝑎 − 3𝑏 + 𝑐 = 21 → (1) ⟹ 10% 𝑜𝑓 𝑧 = 8% 𝑜𝑓 𝑦 + 800 ⟹
100
𝑧=
100
𝑦 + 800
𝑝(5) = 61 ⟹= 𝑎(5)2 + 𝑏(5) + 𝑐 = 61 ⟹ 25𝑎 + 5𝑏 + 𝑐 = 61 → (2) ⟹
10
𝑧=
8𝑦+80000
⟹ 10𝑧 = 8𝑦 + 80000 ⟹ −8𝑦 + 10𝑧 = 80000
𝑝(1) = 9 ⟹= 𝑎(1)2 + 𝑏(1) + 𝑐 = 9 ⟹ 𝑎 + 𝑏 + 𝑐 = 9 → (3) 100 100
⟹÷ 2, −4𝑦 + 5𝑧 = 40000 → (3)

V.GNANAMURUGAN, P,G,T, G.H.S.S, S.S.KOTTAI, SIVAGANGAI DT – 94874 43870 ; 82489 56766 , www.tnppgta.com, www.ednnet.in Page 20
1 1 1 65000 36 −6 1 8
[𝐴|𝐵] = (3 4 5|250000) ~ ( 0 −6 4|−58) 𝑅3 → 𝑅3 + 𝑅2
0 −4 5 40000 0 0 5 −50
1 1 1 65000 ⟹ 36𝑎 − 6𝑏 + 𝑐 = 8 → (1) ; −6𝑏 + 4𝑐 = −58 → (2) ; 5𝑐 = −50 ⟹ 𝑐 = −10
~ (0 1 2|55000) 𝑅2 → 𝑅2 − 3𝑅1 Put 𝑐 = −10 𝑖𝑛 (2) ⟹ −6𝑏 + 4(−10) = −58 ⟹ −6𝑏 − 40 = −58
0 −4 5 40000 ⟹ −6𝑏 = −58 + 40 ⟹ −6𝑏 = −18 ⟹ 𝑏 = 3
1 1 1 65000 Put 𝑏 = 3, 𝑐 = −10 𝑖𝑛 (1) ⟹ 36𝑎 − 6(3) − 10 = 8 ⟹ 36𝑎 − 18 − 10 = 8
~ (0 1 2 | 55000 ) 𝑅3 → 𝑅3 + 4𝑅2
⟹ 36𝑎 = 8 + 28 ⟹ 36𝑎 = 36 ⟹ 𝑎 = 1
0 0 13 260000
⟹ 𝑥 + 𝑦 + 𝑧 = 65000 → (1) ; 𝑦 + 2𝑧 = 55000 → (2) ; Solution: (𝑎, 𝑏, 𝑐) = (1,3, −10) ⟹ 𝑦 = 𝑥 2 + 3𝑥 − 10
13𝑧 = 260000 ⟹ 𝑧 = 20000 Put 𝑥 = 7 ⟹ 𝑦 = (7)2 + 3(7) − 10 = 49 + 21 − 10 = 60 ⟹ 𝑦 = 60
Put 𝑧 = 20000 𝑖𝑛 (2) ⟹ 𝑦 + 2(20000) = 55000 ⟹ 𝑦 = 55000 − 40000 ⟹ The point 𝑃(7,60) satisfies the equation 𝑦 = 𝑥 2 + 3𝑥 − 10, hence the boy will
𝑦 = 15000 meet friend at 𝑃(7,60).
Put 𝑦 = 15000, 𝑧 = 20000 𝑖𝑛 (1) ⟹ 𝑥 + 15000 + 20000 = 65000 𝐄𝐗𝐀𝐌𝐏𝐋𝐄 𝟏. 𝟐𝟖 The upward speed 𝑣(𝑡) a rocket at time t is approximated by
⟹ 𝑥 + 35000 = 65000 ⟹ 𝑥 = 30000 𝑣(𝑡) = 𝑎𝑡 2 + 𝑏𝑡 + 𝑐, 0 ≤ 𝑡 ≤ 100 where a, b and c are constant. It has been
Solution: found that the speed at times 𝑡 = 3, 𝑡 = 6 and 𝑡 = 9 seconds are respectively 64,
The price of 6%, 8% and 10% are Rs.30000, Rs.15000 and Rs.20000 respectively. 133 and 208 miles per second respectively. Find the speed at time t=15 seconds.
4. A boy is walking along the path 𝑦 = 𝑎𝑥 2 + 𝑏𝑥 + 𝑐 through the ( Use Gaussian elimination method )
points(−6,8), (−2, −12), (3,8). He wants to meet his friend at𝑃(7,60). Will he RANK METHOD – NON-HOMOGENEOUS LINEAR EQUATIONS
meet his friend? ( Use Gaussian elimination method )  If 𝜌(𝐴) = 𝜌([𝐴|𝐵]) = 3, then the system of equations is consistent and has
𝑦 = 𝑎𝑥 2 + 𝑏𝑥 + 𝑐 unique solution.
(−6,8) ⟹ 8 = 𝑎(−6)2 + 𝑏(−6) + 𝑐 ⟹ 36𝑎 − 6𝑏 + 𝑐 = 8 → (1)  If 𝜌(𝐴) = 𝜌([𝐴|𝐵]) = 2, then the system of equations is consistent and has
(−2, −12) ⟹ −12 = 𝑎(−2)2 + 𝑏(−2) + 𝑐 ⟹ 4𝑎 − 2𝑏 + 𝑐 = −12 → (2) infinitely many solutions.( Put z=k and find x, y )
(3,8) ⟹ 8 = 𝑎(3)2 + 𝑏(3) + 𝑐 ⟹ 9𝑎 + 3𝑏 + 𝑐 = 8 → (3)  If 𝜌(𝐴) = 𝜌([𝐴|𝐵]) = 1, then the system of equations is consistent and has
36 −6 1 8
[𝐴|𝐵] = ( 4 −2 1|−12) infinitely many solutions.( Put y=s and z=t and find x )
 If 𝜌(𝐴) ≠ 𝜌([𝐴|𝐵]), then the system of equations is inconsistent and has no
9 3 1 8
36 −6 1 8 solution.
𝑅 → 9𝑅2 − 𝑅1
~ ( 0 −12 8|−116) 2 EXERCISE 1.6
𝑅3 → 4𝑅3 − 𝑅1
0 18 3 24 1.Test for consistency and if possible, solve the following systems of equations by
36 −6 1 8 𝑅2 → (1) 𝑅2 rank method:
2
~ ( 0 −6 4|−58) 1 (𝑖)𝑥 − 𝑦 + 2𝑧 = 2, 2𝑥 + 𝑦 + 4𝑧 = 7, 4𝑥 − 𝑦 + 𝑧 = 4
0 6 1 8 𝑅3 → (3) 𝑅3 1 −1 2 𝑥 2
𝐴𝑋 = 𝐵 ⟹ [2 1 4] [𝑦] = [7]
4 −1 1 𝑧 4

V.GNANAMURUGAN, P,G,T, G.H.S.S, S.S.KOTTAI, SIVAGANGAI DT – 94874 43870 ; 82489 56766 , www.tnppgta.com, www.ednnet.in Page 21
1 −1 2 2 1 −3 2 1
Augmented matrix [𝐴|𝐵] = (2 1 4|7) ~ (0 10 −5|−1) 𝑅3 → 𝑅3 − 2𝑅2
4 −1 1 4 0 0 0 0
1 −1 2 2 𝑅 → 𝑅 − 2𝑅 The last equivalent matrix is in echelon form.
~ (0 3 0 | 3 ) 𝑅2 → 𝑅2 − 4𝑅1 𝜌([𝐴|𝐵]) = 2 = 𝜌(𝐴) ⟹ The system of equations is consistent and has unique
0 3 −7 −4 3 3 1
solution
1 −1 2 2 1 −3 2 𝑥 1
~ (0 3 0 | 3 ) 𝑅3 → 𝑅3 − 𝑅2 𝐴𝑋 = 𝐵 ⟹ (0 10 −5) ( ) = (−1) 𝑦
0 0 −7 −7 0 0 0 𝑧 0
The last equivalent matrix is in echelon form. 𝑥 − 3𝑦 + 2𝑧 = 1 → (1)
𝜌([𝐴|𝐵]) = 3 = 𝜌(𝐴) ⟹ The system of equations is consistent and has unique ⟹
10𝑦 − 5𝑧 = −1 → (2)
solution 1
1 −1 2 𝑥 2 Put 𝑧 = 𝑡 in (2) ⟹ 10𝑦 − 5𝑡 = −1 ⟹ 10𝑦 = 5𝑡 − 1 ⟹ 𝑦 = (5𝑡 − 1)
10
1 1
𝐴𝑋 = 𝐵 ⟹ (0 3 0 ) (𝑦) = ( 3 ) Put 𝑦 = (5𝑡 − 1), 𝑧 = 𝑡 in (1) ⟹ 𝑥 − 3 (5𝑡 − 1) + 2𝑡 = 1
10 10
0 0 −7 𝑧 −7 1 10−20𝑡+15𝑡−3 1
𝑥 − 𝑦 + 2𝑧 = 2 → (1) ⟹ 𝑥 = 1 − 2𝑡 + 3 (5𝑡 − 1) = = (7 − 5𝑡)
10 10 10
⟹ 3𝑦 = 3 ⟹ 𝑦 = 1 1
Solution: (𝑥, 𝑦, 𝑧) = ( (7 − 5𝑡),
1
(5𝑡 − 1), 𝑡) ; 𝑡 ∈ 𝑅
10 10
−7𝑧 = −7 ⟹ 𝑧 = 1
Put 𝑦 = 1, 𝑧 = 1 in (1) ⟹ 𝑥 − 1 + 2 = 2 ⟹ 𝑥 = 1 𝐄𝐗𝐀𝐌𝐏𝐋𝐄 𝟏. 𝟑𝟎 Test for consistency of the following systems of equations and if
Solution: (𝑥, 𝑦, 𝑧) = (1,1,1) possible solve:4𝑥 − 2𝑦 + 6𝑧 = 8, 𝑥 + 𝑦 − 3𝑧 = −1, 15𝑥 − 3𝑦 + 9𝑧 = 21
𝐄𝐗𝐀𝐌𝐏𝐋𝐄 𝟏. 𝟐𝟗 Test for consistency of the following systems of equations and if (𝑖𝑖𝑖)2𝑥 + 2𝑦 + 𝑧 = 5, 𝑥 − 𝑦 + 𝑧 = 1, 3𝑥 + 𝑦 + 2𝑧 = 4
2 2 1 𝑥 5
possible solve:𝑥 + 2𝑦 − 𝑧 = 3, 3𝑥 − 𝑦 + 2𝑧 = 1, 𝑥 − 2𝑦 + 3𝑧 = 3, 𝑥 − 𝑦 + 𝑧 +
𝐴𝑋 = 𝐵 ⟹ [1 −1 1] [𝑦] = [1]
1=0
3 1 2 𝑧 4
(𝑖𝑖)3𝑥 + 𝑦 + 𝑧 = 2, 𝑥 − 3𝑦 + 2𝑧 = 1, 7𝑥 − 𝑦 + 4𝑧 = 5 2 2 15
3 1 1 𝑥 2 Augmented matrix [𝐴|𝐵] = (1 −1 1|1)
𝐴𝑋 = 𝐵 ⟹ [1 −3 2] [𝑦] = [1] 3 1 24
7 −1 4 𝑧 5 1 −1 1 1
3 1 12 ~ (2 2 1|5) 𝑅1 ↔ 𝑅2
Augmented matrix [𝐴|𝐵] = (1 −3 2|1) 3 1 24
7 −1 4 5 1 −1 1 1 𝑅 → 𝑅 − 2𝑅
1 −3 2 1 ~ (0 4 −1|3) 2 2 1
~ (3 1 1|2) 𝑅1 ↔ 𝑅2 𝑅 → 𝑅3 − 3𝑅1
0 4 −1 1 3
7 −1 4 5 1 −1 1 1
1 −3 2 1 𝑅 → 𝑅 − 3𝑅 ~ (0 4 −1| 3 ) 𝑅3 → 𝑅3 − 𝑅2
~ (0 10 −5 |−1) 2 2 1
0 0 0 −2
𝑅 → 𝑅3 − 7𝑅1
0 20 −10 −2 3 The last equivalent matrix is in echelon form.
V.GNANAMURUGAN, P,G,T, G.H.S.S, S.S.KOTTAI, SIVAGANGAI DT – 94874 43870 ; 82489 56766 , www.tnppgta.com, www.ednnet.in Page 22
𝜌([𝐴|𝐵]) = 3, 𝜌(𝐴) = 2 ⟹ 𝜌([𝐴|𝐵]) ≠ 𝜌(𝐴), The system of equations is 𝑘 −2 1 1
inconsistent and has no solution Augmented matrix [𝐴|𝐵] = (1 −2𝑘 1|−2)
𝐄𝐗𝐀𝐌𝐏𝐋𝐄 𝟏. 𝟑𝟐 Test for consistency of the following systems of equations and if 1 −2 𝑘 1
1 −2 𝑘 1
possible solve:𝑥 − 𝑦 + 𝑧 = −9, 2𝑥 − 𝑦 + 𝑧 = 4, 3𝑥 − 𝑦 + 𝑧 = 6, 4𝑥 − 𝑦 + 2𝑧 = 7 ~ (1 −2𝑘 1|−2) 𝑅1 ↔ 𝑅3
(𝑖𝑣)2𝑥 − 𝑦 + 𝑧 = 2, 6𝑥 − 3𝑦 + 3𝑧 = 6, 4𝑥 − 2𝑦 + 2𝑧 = 4 𝑘 −2 1 1
2 −1 1 𝑥 2 1 −2 𝑘 1 𝑅 → 𝑅2 − 𝑅1
𝐴𝑋 = 𝐵 ⟹ [6 −3 3] [𝑦] = [6] ~ (0 2 − 2𝑘 1 − 𝑘 | −3 ) 2
4 −2 2 𝑧 𝑅 → 𝑅3 − 𝑘𝑅1
4 0 2𝑘 − 2 1 − 𝑘 2 1 − 𝑘 3
2 −1 1 2 1 −2 𝑘 1
Augmented matrix [𝐴|𝐵] = (6 −3 3|6) ~( 0 2(1 − 𝑘) 1 − 𝑘 | −3 ) 𝑅3 → 𝑅3 + 𝑅2
4 −2 2 4 0 0 2−𝑘−𝑘 2 −𝑘 − 2
2 −1 1 2 𝑅 → 𝑅 − 3𝑅 1 −2 𝑘 1
~ (0 0 0|0) 2 2 1
𝑅 → 𝑅3 − 2𝑅1 ~ (0 2(1 − 𝑘) 1−𝑘 | −3 )
0 0 00 3 0 0 (𝑘 + 2)(1 − 𝑘) −(𝑘 + 2)
The last equivalent matrix is in echelon form.
(i) When 𝑘 = 1, 𝑘 ≠ −2, 𝜌([𝐴|𝐵]) = 3, 𝜌(𝐴) = 1 ⟹ 𝜌([𝐴|𝐵]) ≠ 𝜌(𝐴), the
𝜌([𝐴|𝐵]) = 1, 𝜌(𝐴) = 1 ⟹ The system of equations is consistent and has
system of equations is inconsistent and has no solution.
infinitely many solutions.
2 −1 1 𝑥 2 (ii) When 𝑘 ≠ 1, 𝑘 ≠ −2, 𝜌([𝐴|𝐵]) = 3 = 𝜌(𝐴), the system of equations is
𝐴𝑋 = 𝐵 ⟹ (0 0 0) ( ) = (0) 𝑦 consistent and has unique solution.
0 0 0 𝑧 0 (iii) When 𝑘 = −2, 𝑘 ≠ 1 , 𝜌([𝐴|𝐵]) = 2 = 𝜌(𝐴), the system of equations is
⟹ 2𝑥 − 𝑦 + 𝑧 = 2 consistent and has infinitely many solutions.
Put y= and z=t in the above equation, 2𝑥 − 𝑠 + 𝑡 = 2 ⟹ 2𝑥 = 𝑠 − 𝑡 + 2 3.Investigate the values of 𝜆 𝑎𝑛𝑑 𝜇 the system of linear equations 2𝑥 + 3𝑦 + 5𝑧 =
1
⟹ 𝑥 = (𝑠 − 𝑡 + 2) 9, 7𝑥 + 3𝑦 − 5𝑧 = 8, 2𝑥 + 3𝑦 + 𝜆𝑧 = 𝜇, have (i) no solution (ii) unique solution
2
1 (iii) infinitely many solution
Solution: (𝑥, 𝑦, 𝑧) = ( (𝑠 − 𝑡 + 2), 𝑠, 𝑡) ; 𝑠, 𝑡 ∈ 𝑅 2 3 5 𝑥 9
2
𝐄𝐗𝐀𝐌𝐏𝐋𝐄 𝟏. 𝟑𝟏 Test for consistency of the following systems of equation and if 𝐴𝑋 = 𝐵 ⟹ [7 3 −5] [𝑦] = [8]
possible solve:𝑥 − 𝑦 + 𝑧 = −9, 2𝑥 − 2𝑦 + 2𝑧 = −18, 3𝑥 − 3𝑦 + 3𝑧 + 27 = 0 2 3 𝜆 𝑧 𝜇
2.Find the value of k for which the equations 𝑘𝑥 − 2𝑦 + 𝑧 = 1, 𝑥 − 2𝑘𝑦 + 𝑧 = 2 3 5 9
−2, 𝑥 − 2𝑦 + 𝑘𝑧 = 1 have (i) no solution (ii) unique solution (iii) infinitely many Augmented matrix [𝐴|𝐵] = (7 3 −5|8)
solution 2 3 𝜆 𝜇
2 3 5 9
𝑘 −2 1 𝑥 1 𝑅 → 2𝑅2 − 7𝑅1
~ (0 −15 −25 |− − 9) 2
𝐴𝑋 = 𝐵 ⟹ [1 −2𝑘 1] [𝑦] = [−2] 𝑅3 → 𝑅3 − 𝑅1
0 0 𝜆−5 𝜇−9
1 −2 𝑘 𝑧 1
(i) When 𝜆 = 5, 𝜇 ≠ 9, 𝜌([𝐴|𝐵]) = 3, 𝜌(𝐴) = 2 ⟹ 𝜌([𝐴|𝐵]) ≠ 𝜌(𝐴), the
system of equations is inconsistent and has no solution.

V.GNANAMURUGAN, P,G,T, G.H.S.S, S.S.KOTTAI, SIVAGANGAI DT – 94874 43870 ; 82489 56766 , www.tnppgta.com, www.ednnet.in Page 23
(ii) When 𝜆 ≠ 5, 𝜇 ≠ 9, 𝜌([𝐴|𝐵]) = 3 = 𝜌(𝐴), the system of equations is 1 1 1𝑎
consistent and has unique solution. Augmented matrix [𝐴|𝐵] = (1 2 3|𝑏 )
(iii) When 𝜆 = 5, 𝜇 = 9, 𝜌([𝐴|𝐵]) = 2 = 𝜌(𝐴), the system of equations is 3 5 7𝑐
1 1 1 𝑎 𝑅 → 𝑅2 − 𝑅1
consistent and has infinitely many solutions. ~ (0 1 2| 𝑏 − 𝑎 ) 2
𝐄𝐗𝐀𝐌𝐏𝐋𝐄 𝟏. 𝟑𝟒 Investigate for what values of 𝜆 𝑎𝑛𝑑 𝜇 the system of linear 𝑅 → 𝑅3 − 3𝑅1
0 2 4 𝑐 − 3𝑎 3
equations 𝑥 + 2𝑦 + 𝑧 = 7, 𝑥 + 𝑦 + 𝜆𝑧 = 𝜇, 𝑥 + 3𝑦 − 5𝑧 = 5, have (i) no solution 1 1 1 𝑎
(ii) unique solution (iii) infinitely many solution ~ (0 1 2| 𝑏−𝑎 ) 𝑅3 → 𝑅3 − 2𝑅2
1 2 1 𝑥 7 (𝑐
0 0 0 − 3𝑎) − 2(𝑏 − 𝑎)
𝐴𝑋 = 𝐵 ⟹ [1 1 𝜆 ] [𝑦] = [𝜇] 1 1 1 𝑎
1 3 −5 𝑧 5 ~ (0 1 2| 𝑏 − 𝑎 )
1 2 1 7 0 0 0 𝑐 − 2𝑏 − 𝑎
Augmented matrix [𝐴|𝐵] = (1 1 𝜆 |𝜇) Since given that the system have one parameter family of solutions,
1 3 −5 5 𝜌([𝐴|𝐵]) = 𝜌(𝐴) = 2. So the third row must be a zero row. So 𝑐 − 2𝑏 − 𝑎 = 0
1 2 1 7 ⟹ 𝑐 = 𝑎 + 2𝑏
~ (1 3 −5|5) 𝑅2 ↔ 𝑅3 RANK METHOD – HOMOGENEOUS LINEAR EQUATIONS
1 1 𝜆 𝜇  If 𝜌(𝐴) = 𝜌([𝐴|𝑂)]) = 3, then the system of equations is consistent
1 2 1 7 and has trivial solution.ie., (𝑥, 𝑦, 𝑧) = (0,0,0)
𝑅 →𝑅 −𝑅
~ (0 1 −6 | −2 ) 𝑅2 → 𝑅2 − 𝑅1  If 𝜌(𝐴) = 𝜌([𝐴|𝑂]) = 2 𝑜𝑟 1 < 𝑛, then the system of equations is
0 −1 𝜆 − 1 𝜇 − 7 3 3 1
consistent and has non-trivial solutions.
1 2 1 7
~ (0 1 −6 | −2 ) 𝑅3 → 𝑅3 + 𝑅2  If the system of equations has non-trivial solution, 𝜌([𝐴|𝑂)]) < 𝑛, so
0 0 𝜆−7 𝜇−9 the determinant of the coefficient matrix is 0.
(i) When 𝜆 = 7, 𝜇 ≠ 9, 𝜌([𝐴|𝐵]) = 3, 𝜌(𝐴) = 2 ⟹ 𝜌([𝐴|𝐵]) ≠ 𝜌(𝐴), EXERCISE 1.7
the system of equations is inconsistent and has no solution. 1. Solve the following system of homogenous equations:
(ii) When 𝜆 ≠ 7, 𝜇 ≠ 9, 𝜌([𝐴|𝐵]) = 3 = 𝜌(𝐴), the system of equations is (𝑖)3𝑥 + 2𝑦 + 7𝑧 = 0, 4𝑥 − 3𝑦 − 2𝑧 = 0, 5𝑥 + 9𝑦 + 23𝑧 = 0
consistent and has unique solution. 3 2 7 𝑥 0
(iii) When 𝜆 = 7, 𝜇 = 9, 𝜌([𝐴|𝐵]) = 2 = 𝜌(𝐴), the system of 𝐴𝑋 = 𝑂 ⟹ [4 −3 −2] [𝑦] = [0]
equations is consistent and has infinitely many solutions. 5 9 23 𝑧 0
3 2 7 0
𝐄𝐗𝐀𝐌𝐏𝐋𝐄 𝟏. 𝟑𝟑 Find the condition on a, b and c so that the following system of Augmented matrix [𝐴|𝑂] = (4 −3 −2|0)
linear equations has one parameter family of solutions: 𝑥 + 𝑦 + 𝑧 = 𝑎, 𝑥 + 2𝑦 + 5 9 23 0
3𝑧 = 𝑏, 3𝑥 + 5𝑦 + 7𝑧 = 𝑐 3 2 7 0 𝑅 → 3𝑅 − 4𝑅
1 1 1 𝑥 𝑎 ~ (0 −17 −34|0) 2 2 1
𝑅 → 3𝑅3 − 5𝑅1
𝐴𝑋 = 𝐵 ⟹ [1 2 3] [𝑦] = [𝑏 ] 0 17 34 0 3
3 5 7 𝑧 𝑐

V.GNANAMURUGAN, P,G,T, G.H.S.S, S.S.KOTTAI, SIVAGANGAI DT – 94874 43870 ; 82489 56766 , www.tnppgta.com, www.ednnet.in Page 24
3 2 7 0 2.Determine the values of 𝛌 for which the following system of equations
~ (0 −17 −34|0) 𝑅3 → 𝑅3 + 𝑅2 𝑥 + 𝑦 + 3𝑧 = 0, 4𝑥 + 3𝑦 + 𝜆𝑧 = 0, 2𝑥 + 𝑦 + 2𝑧 = 0 has (i) a unique solution (ii)
0 0 0 0 a non- trivial solution
⟹ 𝜌([𝐴|𝐵]) = 2 = 𝜌(𝐴) , the system of equations is consistent and has non- 1 1 3 𝑥 0
trivial solution. 𝐴𝑋 = 𝑂 ⟹ [4 3 𝜆] [𝑦] = [0]
3 2 7 𝑥 0 2 1 2 𝑧 0
3𝑥 + 2𝑦 + 7𝑧 = 0 → (1)
𝐴𝑋 = 𝑂 ⟹ (0 −17 −34) (𝑦) = (0) ⟹ 1 1 30
𝑧 −17𝑦 − 34𝑧 = 0 → (2)
0 0 0 0 Augmented matrix [𝐴|𝑂] = (4 3 𝜆|0)
Put z=t in (2) ⟹ −17𝑦 − 34𝑡 = 0 ⟹ 𝑦 = −2𝑡 2 1 20
Put 𝑦 = −2𝑡, 𝑧 = 𝑡 𝑖𝑛 (1) ⟹ 3𝑥 + 2(−2𝑡) + 7𝑡 = 0 ⟹ 3𝑥 = −3𝑡 ⟹ 𝑥 = −𝑡 1 1 30
Solution : (𝑥, 𝑦, 𝑧) = (−𝑡, −2𝑡, 𝑡); 𝑡 ∈ 𝑅 ~ (2 1 2|0) 𝑅2 ↔ 𝑅3
𝐄𝐗𝐀𝐌𝐏𝐋𝐄 𝟏. 𝟑𝟔 Solve the system : 𝑥 + 3𝑦 − 2𝑧 = 0, 2𝑥 − 𝑦 + 4𝑧 = 0, 𝑥 − 4 3 𝜆0
1 1 3 0 𝑅 → 𝑅 − 2𝑅
11𝑦 + 14𝑧 = 0 ~ (0 −1 −4 |0) 𝑅2 → 𝑅2 − 4𝑅1
1(𝑖𝑖)2𝑥 + 3𝑦 − 𝑧 = 0, 𝑥 − 𝑦 − 2𝑧 = 0, 3𝑥 + 𝑦 + 3𝑧 = 0 0 −1 𝜆 − 12 0 3 3 1
2 3 −1 𝑥 0 1 1 3 0
𝐴𝑋 = 𝑂 ⟹ [1 −1 −2] [𝑦] = [0] ~ (0 −1 −4 |0) 𝑅3 → 𝑅3 − 𝑅2
3 1 3 𝑧 0 0 0 𝜆−8 0
2 3 −1 0 (i)When 𝜆 ≠ 8 ⟹ 𝜌(𝐴) = 𝜌([𝐴|𝑂)]) = 3, then the system of equations is
Augmented matrix [𝐴|𝑂] = (1 −1 −2|0) consistent and has trivial solution. (𝑥, 𝑦, 𝑧) = (0,0,0)
3 1 3 0 (ii)When 𝜆 = 8 ⟹ 𝜌(𝐴) = 𝜌([𝐴|𝑂)]) = 2, then the system of equations is
1 −1 −2 0
~ (2 3 −1|0) 𝑅1 ↔ 𝑅2 consistent and has non- trivial solution.
3 1 3 0 𝐄𝐗𝐀𝐌𝐏𝐋𝐄 𝟏. 𝟑𝟖 Determine the values of 𝛌 for which the following system of
1 −1 −2 0 𝑅 → 𝑅 − 2𝑅 equations (3𝜆 − 8)𝑥 + 3𝑦 + 3𝑧 = 0, 3𝑥 + (3𝜆 − 8)𝑦 + 3𝑧 = 0, 3𝑥 + 3𝑦 +
~ (0 5 3 |0) 𝑅2 → 𝑅2 − 3𝑅1 (3𝜆 − 8)𝑧 = 0 has a non- trivial solution.
0 4 9 0 3 3 1
Here the number of unknowns is 3.
1 −1 −2 0
~ (0 5 3 |0) 𝑅3 → 5𝑅3 − 4𝑅2 Since the system of equations has non-trivial solution, 𝜌([𝐴|𝑂)]) < 3, so the
0 0 33 0 determinant of the coefficient matrix is 0.
⟹ 𝜌([𝐴|𝐵]) = 3 = 𝜌(𝐴) , the system of equations is consistent and has trivial 3𝜆 − 8 3 3
solution. (𝑥, 𝑦, 𝑧) = (0,0,0) | 3 3𝜆 − 8 3 |=0
3 3 3𝜆 − 8
𝐄𝐗𝐀𝐌𝐏𝐋𝐄 𝟏. 𝟑𝟓 Solve the system : 𝑥 + 2𝑦 + 3𝑧 = 0, 3𝑥 + 4𝑦 + 4𝑧 = 0, 7𝑥 + 3𝜆 − 2 3𝜆 − 2 3𝜆 − 2
10𝑦 + 12𝑧 = 0 ⟹| 3 3𝜆 − 8 3 | = 0 (𝑅1 → 𝑅1 + 𝑅2 + 𝑅3 )
𝐄𝐗𝐀𝐌𝐏𝐋𝐄 𝟏. 𝟑𝟕 Solve the system : 𝑥 + 𝑦 − 2𝑧 = 0, 2𝑥 − 3𝑦 + 𝑧 = 0, 3𝑥 − 3 3 3𝜆 − 8
7𝑦 + 10𝑧 = 0, 6𝑥 − 9𝑦 + 10𝑧 = 0

V.GNANAMURUGAN, P,G,T, G.H.S.S, S.S.KOTTAI, SIVAGANGAI DT – 94874 43870 ; 82489 56766 , www.tnppgta.com, www.ednnet.in Page 25
1 1 1 𝑥1 𝑥2 𝑥3 𝑥4
⟹ (3𝜆 − 2) |3 3𝜆 − 8 3 |=0 C H O C H O → C H O C H O
3 3 3𝜆 − 8 2 6 0 0 0 2 0 2 1 1 0 2
1 1 1 𝑅 → 𝑅 − 3𝑅
⟹ (3𝜆 − 2) |0 3𝜆 − 11 0 | = 0 (𝑅2 → 𝑅2 − 3𝑅1 ) Equating the number of atom of C on both sides with𝑥1 , 𝑥2 , 𝑥3 , 𝑥4 , we get
0 0 3𝜆 − 11 3 3 1 2𝑥1 + 0𝑥2 = 0𝑥3 + 1𝑥4 ⟹ 2𝑥1 + 0𝑥2 − 0𝑥3 − 1𝑥4 = 0 → (2)
Expanding through 𝐶1 , Equating the number of atom of H on both sides with𝑥1 , 𝑥2 , 𝑥3 , 𝑥4 , we get
2
⟹ (3𝜆 − 2)(3𝜆 − 11)2 = 0 ⟹ λ = , λ =
11 6𝑥1 + 0𝑥2 = 2𝑥3 + 0𝑥4 ⟹ 6𝑥1 + 0𝑥2 − 2𝑥3 − 0𝑥4 = 0
3 3 ÷ 2 ⟹ 3𝑥1 + 0𝑥2 − 𝑥3 − 0𝑥4 = 0 → (3)
𝐄𝐗𝐀𝐌𝐏𝐋𝐄 𝟏. 𝟒𝟎 If the system of equations 𝑝𝑥 + 𝑏𝑦 + 𝑐𝑧 = 0, 𝑎𝑥 + 𝑞𝑦 + 𝑐𝑧 = Equating the number of atom of O on both sides with𝑥1 , 𝑥2 , 𝑥3 , 𝑥4 , we get
0, 𝑎𝑥 + 𝑏𝑦 + 𝑟𝑧 = 0 has a non-trivial solution and 𝑝 ≠ 𝑎, 𝑞 ≠ 𝑏, 𝑟 ≠ 𝑐, prove that 0𝑥1 + 2𝑥2 = 1𝑥3 + 2𝑥4 ⟹ 0𝑥1 + 2𝑥2 − 1𝑥3 − 2𝑥4 = 0 → (4)
𝑝 𝑞 𝑟
+ + =2 2 0 0 −1 0
𝑝−𝑎 𝑞−𝑏 𝑟−𝑐
Since the system of equations has non-trivial solution, 𝜌([𝐴|𝑂)]) < 3, so the From (2), (3), (4) ⟹Augmented matrix [𝐴|𝑂] = (3 0 −1 0 |0)
determinant of the coefficient matrix is 0. 0 2 −1 −2 0
2 0 0 −1 0
𝑝 𝑏 𝑐 ~ (0 0 −2 3 |0) ~𝑅2 → 2𝑅2 − 3𝑅1
|𝑎 𝑞 𝑐 | = 0 0 2 −1 −2 0
𝑎 𝑏 𝑟 2 0 0 −1 0
𝑝 𝑏 𝑐 ~ (0 2 −1 −2|0) 𝑅2 ↔ 𝑅3
𝑅 → 𝑅2 − 𝑅1
⟹ |𝑎 − 𝑝 𝑞 − 𝑏 0 |=0 ( 2 ) 0 0 −2 3 0
𝑅3 → 𝑅3 − 𝑅1
𝑎−𝑝 0 𝑟−𝑐 𝜌(𝐴) = 𝜌([𝐴|𝑂)]) = 3 < 4,the systems of equations is consistent and has infinite
𝑝 𝑏 𝑐 solution.
⟹ |−(𝑝 − 𝑎) 𝑞 − 𝑏 0 |=0 2𝑥1 − 1𝑥4 = 0 → (5) ; 2𝑥2 − 𝑥3 − 2𝑥4 = 0 → (6) ; −2𝑥3 + 3𝑥4 = 0 → (7)
𝑡
−(𝑝 − 𝑎) 0 𝑟−𝑐 Put 𝑥4 = 𝑡 in (5) ⟹ 2𝑥1 − 𝑡 = 0 ⟹ 𝑥1 =
𝑝 𝑞 𝑟 𝑝 𝑞 𝑟 2
3𝑡
𝑝−𝑎 𝑞−𝑏 𝑟−𝑐 𝑝−𝑎 𝑞−𝑏 𝑟−𝑐 Put 𝑥4 = 𝑡 in (7) ⟹ −2𝑥3 + 3𝑥4 = 0 ⟹ −2𝑥3 + 3𝑡 = 0 ⟹ 𝑥3 =
⟹ (𝑝 − 𝑎)(𝑞 − 𝑏)(𝑟 − 𝑐) | −1 2
1 0 |=0 ⟹ | −1 1 0 |=0 Put 𝑥3 =
3𝑡
, 𝑥4 = 𝑡 in (6) ⟹ 2𝑥2 −
3𝑡
− 2𝑡 = 0 ⟹ 𝑥2 =
7𝑡

𝑝 𝑞 𝑟
−1 0 1 −1 0 1 2
𝑡 7𝑡 3𝑡
2 4
⟹ + + =0 (𝑥1 , 𝑥2 , 𝑥3 , 𝑥4 ) = ( , , , 𝑡)
𝑝−𝑎 𝑞−𝑏 𝑟−𝑐 2 4 2

𝑝
+
𝑞−(𝑞−𝑏) 𝑟−(𝑟−𝑐)
+ =0 Put 𝑡 = 4 ⟹ 1 , 𝑥2 , 𝑥3 , 𝑥4 ) = (2,7,6,4)
(𝑥
𝑝−𝑎 𝑞−𝑏 𝑟−𝑐 Sub. The above values in (1) ⟹ 2𝐶2 𝐻6 + 7𝑂2 → 6𝐻2 𝑂 + 4𝐶𝑂2
𝑝 𝑞 𝑟 𝑝 𝑞 𝑟
⟹ + + −1−1 =0 ⟹ + + =2 𝐄𝐗𝐀𝐌𝐏𝐋𝐄 𝟏. 𝟑𝟖 By using Gaussian elimination method, balance the chemical
𝑝−𝑎 𝑞−𝑏 𝑟−𝑐 𝑝−𝑎 𝑞−𝑏 𝑟−𝑐
3.By using Gaussian elimination method, balance the chemical reaction equation : reaction equation : 𝐶5 𝐻8 + 𝑂2 → 𝐶𝑂2 + 𝐻2 𝑂
𝐶2 𝐻6 + 𝑂2 → 𝐻2 𝑂 + 𝐶𝑂2
𝐶2 𝐻6 + 𝑂2 → 𝐻2 𝑂 + 𝐶𝑂2 ⟹ 𝑥1 𝐶2 𝐻6 + 𝑥2 𝑂2 → 𝑥3 𝐻2 𝑂 + 𝑥4 𝐶𝑂2 → (1)

V.GNANAMURUGAN, P,G,T, G.H.S.S, S.S.KOTTAI, SIVAGANGAI DT – 94874 43870 ; 82489 56766 , www.tnppgta.com, www.ednnet.in Page 26
2.COMPLEX NUMBERS = 1.1. 𝑖
POWERS OF IMAGINARY UNIT =𝑖
2 3
 𝑖 = −1, 𝑖 = −𝑖, 𝑖 = 1 4 6.∑10𝑛=1 𝑖
𝑛+50

 Division algorithm : 𝑛 = 4(𝑞) + 𝑟 ∑10


𝑛=1 𝑖
𝑛+50
= 𝑖 51 + 𝑖 52 + 𝑖 53 + ⋯ + 𝑖 60
 𝑖 𝑛 = (𝑖)4𝑞+𝑟 = (𝑖)4𝑞 (𝑖)𝑟 = (𝑖 4 )𝑞 (𝑖)𝑟 ⟹ 𝑖𝑓 𝑟 = 0, 𝑖 𝑛 = 1 ; 𝑖𝑓 𝑟 = 1 , 𝑖 𝑛 = 𝑖; = 𝑖 50 (𝑖 1 + 𝑖 2 + 𝑖 3 + 𝑖 4 + 𝑖 5 + 𝑖 6 + 𝑖 7 + 𝑖 8 + 𝑖 9 + 𝑖 10 )
𝑖𝑓 𝑟 = 2, 𝑖 𝑛 = −1; 𝑖𝑓 𝑟 = 3, 𝑖 𝑛 = −𝑖 = (𝑖 4 )12 𝑖 2 (𝑖 − 1 − 𝑖 + 1 + 𝑖 − 1 − 𝑖 + 1 + 𝑖 − 1)
 √𝑎𝑏 = √𝑎√𝑏 is valid only if at least one of a, b is non-negative. = −1(𝑖 − 1)
 For 𝑦 ∈ 𝑅, 𝑦 2 > 0 =1−𝑖
EXERCISE 2.1 𝐄𝐗𝐀𝐌𝐏𝐋𝐄 𝟐. 𝟏 Simplify the following:
Simplify the following: (i) i7 (ii)i1729 (iii)i−1924 + i2018 (iv) ∑102 n 2 3 4
n=1 i (v)i. i . i . i … . . i
40

1.𝑖 1947 + 𝑖 1950 COMPLEX NUMBERS


1947 ÷ 4 = 4(486) + 3 ; 1950 = 4(487) + 2  Rectangular Form Of a Complex Number: A complex number is of the form
𝑖 1947 + 𝑖 1950 = (𝑖 4 )486 (𝑖)3 + (𝑖 4 )487 (𝑖)2 = −𝑖 − 1 = −1 − 𝑖 𝑧 = 𝑥 + 𝑖𝑦, 𝑥, 𝑦 ∈ 𝑅. 𝑥 is called the real part and 𝑦 is called the imaginary
2.𝑖 1948 − 𝑖 −1869 part of the complex number.
1948 ÷ 4 = 4(487) + 0 ; 1869 ÷ 4 = 4(467) + 1  Two complex numbers 𝑧1 = 𝑎 + 𝑖𝑏, 𝑧2 = 𝑐 + 𝑖𝑑 are said to be equal if and
1 1 1
𝑖 1948 − 𝑖 −1869 = 𝑖 1948 − 1869 = (𝑖 4 )487 (𝑖)0 − (𝑖 4)467 1 = 1 − = 1 − ( × )
1 −𝑖 only if 𝑅𝑒(𝑧1 ) = 𝑅𝑒(𝑧2 ) and 𝐼𝑚(𝑧1 ) = 𝐼𝑚(𝑧2 ). ie. 𝑎 = 𝑐 𝑎𝑛𝑑 𝑏 = 𝑑
𝑖 𝑖 𝑖 𝑖 −𝑖  Scalar multiplication of complex numbers: 𝐼𝑓 𝑧 = 𝑥 + 𝑖𝑦 𝑎𝑛𝑑 𝑘 ∈
−𝑖
= 1 − ( 2 ) = 1 + 𝑖 (∵ −𝑖 2 = 1) 𝑅 𝑡ℎ𝑒𝑛 𝑘𝑧 = 𝑘(𝑥 + 𝑖𝑦) = 𝑘𝑥 + 𝑘𝑦𝑖
−𝑖  Addition of complex numbers: If 𝑧1 = 𝑎 + 𝑖𝑏, 𝑧2 = 𝑐 + 𝑖𝑑, 𝑡ℎ𝑒𝑛 𝑧1 + 𝑧2 =
12 𝑛
3.∑𝑛=1 𝑖 (𝑎 + 𝑐) + 𝑖(𝑏 + 𝑑)
∑12 𝑛 1 2 3 4 5 6
𝑛=1 𝑖 = 𝑖 + 𝑖 + 𝑖 + 𝑖 + 𝑖 + 𝑖 + 𝑖 + +𝑖 + 𝑖 + 𝑖
7 8 9 10
+ 𝑖 11 + 𝑖 12  Subtraction of complex numbers: If 𝑧1 = 𝑎 + 𝑖𝑏, 𝑧2 = 𝑐 + 𝑖𝑑, 𝑡ℎ𝑒𝑛 𝑧1 −
= 𝑖 1 + 𝑖 2 + 𝑖 3 + 𝑖 4 + 𝑖 4 𝑖 + (𝑖 2 )3 + (𝑖 2 )3 𝑖 + (𝑖 2 )4 + (𝑖 3 )3 + (𝑖 2 )5 + (𝑖 2 )5 𝑖 + (𝑖 2 )6 𝑧2 = (𝑎 − 𝑐) + 𝑖(𝑏 − 𝑑)
=𝑖−1−𝑖+1+𝑖−1−𝑖+1+𝑖−1−𝑖+1  Multiplication of complex numbers: If 𝑧1 = 𝑎 + 𝑖𝑏, 𝑧2 = 𝑐 + 𝑖𝑑, 𝑡ℎ𝑒𝑛
=0 𝑧1 𝑧2 = (𝑎𝑐 − 𝑏𝑑) + 𝑖(𝑎𝑑 + 𝑏𝑐)
1
4.𝑖 59 + 59  Multiplication of a complex z by i successively gives a 900 counter clockwise
𝑖
59 ÷ 4 = 4(14) + 3 rotation successively about the origin.
1 1
𝑖 59 + 59 = (𝑖 4 )14 𝑖 3 + (𝑖 4)14 3 EXERCISE 2.2
𝑖 𝑖
1 1 𝑖 𝑖 1.Evaluate the following if 𝑧 = 5 − 2𝑖 and 𝑤 = −1 + 3𝑖
= −𝑖 + =𝑖+ = −𝑖 + = 𝑖 − 𝑖 = 0 (𝑖)𝑧 + 𝑤 (𝑖𝑖)𝑧 − 𝑖𝑤 (𝑖𝑖𝑖)2𝑧 + 3𝑤 (𝑖𝑣)𝑧𝑤 (𝑣)𝑧 2 + 2𝑧𝑤 + 𝑤 2 (𝑣𝑖)(𝑧 + 𝑤)2
−𝑖 −𝑖 𝑖 1
5.𝑖. 𝑖 2 . 𝑖 3 . 𝑖 4 … . . 𝑖 2000 (𝑖)𝑧 + 𝑤 = 5 − 2𝑖 − 1 + 3𝑖 = 4 + 𝑖
= 𝑖 (1+2+3…..+2000) (𝑖𝑖)𝑧 − 𝑖𝑤 = 5 − 2𝑖 − 𝑖(−1 + 3𝑖) = 5 − 2𝑖 + 𝑖 − 3𝑖 2 = 5 − 2𝑖 + 𝑖 + 3 = 8 − 𝑖
=𝑖 2
2000×2001
= 𝑖 1000×2001 = 𝑖 1000 . 𝑖 2001 ∵ 𝑖 2 = −1
= (𝑖 4 )250 (𝑖 4 )500 𝑖 1 (∵ 2001 ÷ 4 = 4(500) + 1) (𝑖𝑖𝑖)2𝑧 + 3𝑤 = 2(5 − 2𝑖 ) + 3(−1 + 3𝑖) = 10 − 4𝑖 − 3 + 9𝑖 = 7 + 5𝑖

V.GNANAMURUGAN, P,G,T, G.H.S.S, S.S.KOTTAI, SIVAGANGAI DT – 94874 43870 ; 82489 56766 , www.tnppgta.com, www.ednnet.in Page 27
(𝑖𝑣) 𝑧1 𝑧2 = (𝑎𝑐 − 𝑏𝑑) + 𝑖(𝑎𝑑 + 𝑏𝑐); 𝑎 = 5, 𝑏 = −2, 𝑐 = −1, 𝑑 = 3 3. Find the values of the real numbers 𝑥 𝑎𝑛𝑑 𝑦, if the complex numbers (3 − 𝑖)𝑥 −
𝑧𝑤 = (5 − 2𝑖)(−1 + 3𝑖) = [(5)(−1) − (−2)(3)] + 𝑖[(5)(3) + (−2)(−1)] (2 − 𝑖)𝑦 + 2𝑖 + 5 and 2𝑥 + (−1 + 2𝑖)𝑦 + 3 + 2𝑖 are equal.
= [−5 + 6] + 𝑖[15 + 2] = 1 + 17𝑖 𝑧1 = (3 − 𝑖)𝑥 − (2 − 𝑖)𝑦 + 2𝑖 + 5
(𝑣)𝑧 = (5 − 2𝑖) = 25 − 20𝑖 + 4𝑖 2 = 25 − 20𝑖 − 4 = 21 − 20𝑖
2 2
= 3𝑥 − 𝑥𝑖 − 2𝑦 + 𝑦𝑖 + 2𝑖 + 5
2𝑧𝑤 = 2(5 − 2𝑖 )(−1 + 3𝑖) = 2[1 + 17𝑖] from (𝑖𝑣) = (3𝑥 − 2𝑦 + 5) + 𝑖(−𝑥 + 𝑦 + 2)
2𝑧𝑤 = 2 + 34𝑖 𝑧2 = 2𝑥 + (−1 + 2𝑖)𝑦 + 3 + 2𝑖
𝑤 2 = (−1 + 3𝑖)2 = 1 − 6𝑖 + 9𝑖 2 = 1 − 6𝑖 − 9 = −8 − 6𝑖 = 2𝑥 − 𝑦 + 2𝑦𝑖 + 3 + 2𝑖
𝑧 2 + 2𝑧𝑤 + 𝑤 2 = 21 − 20𝑖 + 2 + 34𝑖 − 8 − 6𝑖 = 15 + 8𝑖 = (2𝑥 − 𝑦 + 3) + 𝑖(2𝑦 + 2)
(𝑣𝑖)(𝑧 + 𝑤)2 = (4 + 𝑖)2 from(𝑖) Given : 𝑧1 = 𝑧2 ⟹ (3𝑥 − 2𝑦 + 5) + 𝑖(−𝑥 + 𝑦 + 2) = (2𝑥 − 𝑦 + 3) + 𝑖(2𝑦 + 2)
= 16 + 8𝑖 + 𝑖 2 = 16 + 8𝑖 − 1 = 15 + 8𝑖 Equating the real and imaginary parts on both sides,
2. Given the complex number 𝑧 = 2 + 3𝑖, represent the complex numbers in 3𝑥 − 2𝑦 + 5 = 2𝑥 − 𝑦 + 3 −𝑥 + 𝑦 + 2 = 2𝑦 + 2
Argand diagram. 3𝑥 − 2𝑦 + 5 − 2𝑥 + 𝑦 − 3 = 0 −𝑥 + 𝑦 + 2 − 2𝑦 − 2 = 0
(𝑖)𝑧, 𝑖𝑧 𝑎𝑛𝑑 𝑧 + 𝑖𝑧 (𝑖𝑖)𝑧, −𝑖𝑧 𝑎𝑛𝑑 𝑧 − 𝑖𝑧 𝑥 − 𝑦 = −2 → (1) −𝑥 − 𝑦 = 0 → (2)
(𝑖)𝑧 = 2 + 3𝑖 = (2,3) (1) + (2) ⟹ −2𝑦 = −2 ⟹ 𝑦 = 1
𝑖𝑧 = 𝑖(2 + 3𝑖) = 2𝑖 + 3𝑖 2 = −3 + 2𝑖 = (−3,2) Sub. 𝑦 = −1 in (1) ⟹ 𝑥 − 1 = −2 ⟹ 𝑥 = −1
𝑧 + 𝑖𝑧 = 2 + 3𝑖 + 𝑖(2 + 3𝑖) = 2 + 3𝑖 − 3 + 2𝑖 = −1 + 5𝑖 = (−1,5)
𝐄𝐗𝐀𝐌𝐏𝐋𝐄 𝟐. 𝟐 Find the values of the real numbers 𝑥 𝑎𝑛𝑑 𝑦, if the complex
numbers (2 + 𝑖)𝑥 + (1 − 𝑖)𝑦 + 2𝑖 − 3 and 𝑥 + (−1 + 2𝑖)𝑦 + 1 + 𝑖 are equal.
[𝐴𝑛𝑠: 𝑥 = 2 𝑎𝑛𝑑 𝑦 = 1]
PROPERTIES OF COMPLEX NUMBERS
Properties of complex numbers under addition:
I. Closure property: ∀ 𝑧1 , 𝑧2 ∈ ℂ , (𝑧1 + 𝑧2 ) ∈ ℂ
II. Commutative property: ∀ 𝑧1 , 𝑧2 ∈ ℂ, 𝑧1 + 𝑧2 = 𝑧2 + 𝑧1
III. Associative property: ∀ 𝑧1 , 𝑧2 , 𝑧3 ∈ ℂ, (𝑧1 + 𝑧2 ) + 𝑧3 = 𝑧1 + (𝑧2 + 𝑧3 )
(𝑖)𝑧 = 2 + 3𝑖 = (2,3) IV. Additive identity: ∀ 𝑧 ∈ ℂ, 𝑧 + 0 = 0 + 𝑧 = 𝑧 ⟹ 0 = 0 + 0𝑖 is an additive
−𝑖𝑧 = −𝑖(2 + 3𝑖) = −2𝑖 − 3𝑖 2 = 3 − 2𝑖 = (3, −2) identity.
𝑧 − 𝑖𝑧 = 2 + 3𝑖 − 𝑖(2 + 3𝑖) = 2 + 3𝑖 + 3 − 2𝑖 = 5 + 𝑖 = (5,1) V. Additive inverse: ∀ 𝑧 ∈ ℂ , − 𝑧 ∈ ℂ ⋺ 𝑧 + (−𝑧) = (−𝑧) + 𝑧 = 0 ∈ ℂ , 𝑠𝑜 − 𝑧
is an additive identity.
Properties of complex numbers under multiplication:
I. Closure property: ∀ 𝑧1 , 𝑧2 ∈ ℂ , (𝑧1 . 𝑧2 ) ∈ ℂ
II. Commutative property: ∀ 𝑧1 , 𝑧2 ∈ ℂ, 𝑧1 . 𝑧2 = 𝑧2 . 𝑧1
III. Associative property: ∀ 𝑧1 , 𝑧2 , 𝑧3 ∈ ℂ, (𝑧1 . 𝑧2 ). 𝑧3 = 𝑧1 . (𝑧2 . 𝑧3 )
IV. Multiplicative identity: ∀ 𝑧 ∈ ℂ, 𝑧. 1 = 1. 𝑧 = 𝑧 ⟹ 1 = 1 + 0𝑖 is a
multiplicative identity.
V.GNANAMURUGAN, P,G,T, G.H.S.S, S.S.KOTTAI, SIVAGANGAI DT – 94874 43870 ; 82489 56766 , www.tnppgta.com, www.ednnet.in Page 28
1 1 1 1
V. Multiplicative inverse: ∀ 𝑧 ∈ ℂ ∃ 𝑧 ∈ ℂ ⋺ 𝑧. = . 𝑧 = 1 + 𝑖0 ∈ ℂ , 𝑠𝑜 is So, Additive inverse of 𝑧1 = 2 + 5𝑖 is −𝑧1 = −2 − 5𝑖,
𝑧 𝑧 𝑧 𝑧
multiplicative identity. Additive inverse of 𝑧2 = −3 − 4𝑖 is- 𝑧2 = 3 + 4𝑖
𝑥 𝑦 Additive inverse of 𝑧3 = 1 + 𝑖 is −𝑧3 = −1 − 𝑖
 If 𝑧 = 𝑥 + 𝑖𝑦 then its multiplicative inverse is 𝑧 −1 = ( ) + 𝑖 (− 𝑥 2+𝑦2 ) x y
𝑥 2 +𝑦 2 Multiplicative inverse of z = x + iy is z −1 = ( ) + i (− x2 +y2)
 Distributive property: ∀ 𝑧1 , 𝑧2 , 𝑧3 ∈ ℂ, (𝑧1 + 𝑧2 ). 𝑧3 = 𝑧1 𝑧3 + 𝑧2 𝑧3 (or) x2 +y2
2 5 2 5
o ∀ 𝑧1 , 𝑧2 , 𝑧3 ∈ ℂ , 𝑧1 . (𝑧2 + 𝑧3 ) = 𝑧1 𝑧2 + 𝑧1 𝑧3 Multiplicative inverse of 𝑧1 = 2 + 5𝑖 is z1 −1 = ( ) + i (− 22+52) = 29 − 29 𝑖
22 +52
 Complex numbers obey the laws of indices: Multiplicative inverse of 𝑧2 = −3 − 4𝑖 is𝑧2 −1 = ((−3)2
−3 −4
) + i (− (−3)2+(−4)2)
𝑧𝑚 +(−4)2
(𝑖)𝑧 𝑚 𝑧 𝑛 = 𝑧 𝑚+𝑛 (𝑖𝑖) 𝑛 = 𝑧 𝑚−𝑛 3 4
𝑧 =− + 𝑖
(𝑖𝑖𝑖)(𝑧 𝑚 )𝑛 = 𝑧 𝑚 𝑧 𝑛 (𝑖𝑣)(𝑧1 𝑧2 )𝑚 = 𝑧1 𝑚 𝑧2 𝑚 1
25 25
1 1 1
EXERCISE 2.3 Multiplicative inverse of 𝑧3 = 1 + 𝑖 is z −1 = ( ) + i (− 12+12 ) = 2 − 2 𝑖
12 +12
1.If 𝑧1 = 1 − 3𝑖, 𝑧2 = −4𝑖 and 𝑧3 = 5, show that CONJUGATE OF A COMPLEX NUMBER
(𝑖)(𝑧1 + 𝑧2 ) + 𝑧3 = 𝑧1 + (𝑧2 + 𝑧3 ) (𝑖𝑖)(𝑧1 𝑧2 )𝑧3 = 𝑧1 (𝑧2 𝑧2 )  The conjugate of the complex number z = x + iy is z̅ = x − iy.
(𝑖)(𝑧1 + 𝑧2 ) + 𝑧3 = (1 − 3𝑖 − 4𝑖) + 5 = 1 − 7𝑖 + 5 = 6 − 7𝑖 → (1)  𝑧𝑧̅ = 𝑥 2 + 𝑦 2
𝑧1 + (𝑧2 + 𝑧3 ) = 1 − 3𝑖 + (−4𝑖 + 5) = 1 − 3𝑖 − 4𝑖 + 5 = 6 − 7𝑖 → (2)  The conjugate is useful in division of complex numbers.
From (1) & (2), (𝑧1 + 𝑧2 ) + 𝑧3 = 𝑧1 + (𝑧2 + 𝑧3 ) z1 𝑎+𝑖𝑏 𝑎𝑐+𝑏𝑑 𝑏𝑐−𝑎𝑑
 = =[ ]+𝑖[ ]
(𝑖𝑖)(𝑧1 𝑧2 )𝑧3 = [(1 − 3𝑖)(−4𝑖)]5 = (−4𝑖 + 12𝑖 2 )5 z2 𝑐+𝑖𝑑 𝑐 2 +𝑑 2 𝑐 2 +𝑑 2
= (−4𝑖 − 12)5 = −60 − 20𝑖 → (1) PROPERTIES OF COMPLEX CONJUGATES:
𝑧1 (𝑧2 𝑧2 ) = (1 − 3𝑖)[(−4𝑖)(5)] = (1 − 3𝑖)[−20𝑖]  ̅̅̅̅̅̅̅̅̅
z1 + z2 = z̅1 + z̅2  Im(z) =
z−z̅
2i
= −20𝑖 + 60𝑖 2 = −60 − 20𝑖 → (2)  ̅̅̅̅̅̅̅̅̅
z1 − z2 = z̅1 − z̅2 ̅̅̅̅̅̅
 (z ̅)n
n ) = (z
From (1) & (2), (𝑧1 𝑧2 )𝑧3 = 𝑧1 (𝑧2 𝑧2 )  ̅̅̅̅̅̅̅
z1 . z2 = z̅1 . z̅2
̅̅̅̅̅
z1 ̅̅̅
z1
 z is real if and only if z = z̅
2.If 𝑧1 = 3, 𝑧2 = −7𝑖 and 𝑧3 = 5 + 4𝑖, show that  ( ) = ̅̅̅ , z2 ≠ 0  z is purely imaginary if and only if z = −z̅
z2 z2
(𝑖)𝑧1 . (𝑧2 + 𝑧3 ) = 𝑧1 𝑧2 + 𝑧1 𝑧3 (𝑖𝑖)(𝑧1 + 𝑧2 ). 𝑧3 = 𝑧1 𝑧3 + 𝑧2 𝑧3 z+z̅  z̿ = z
(𝑖)𝑧1 . (𝑧2 + 𝑧3 ) = 3(−7𝑖 + 5 + 4𝑖) = 3(5 − 3𝑖) = 15 − 9𝑖 → (1)  Re(z) =
2
𝑧1 𝑧2 + 𝑧1 𝑧3 = (3)(−7𝑖) + (3)(5 + 4𝑖) = −21𝑖 + 15 + 12𝑖 = 15 − 9𝑖 → (2) EXERCISE 2.4
From (1) & (2), 𝑧1 . (𝑧2 + 𝑧3 ) = 𝑧1 𝑧2 + 𝑧1 𝑧3 1.Write the following in the rectangular form:
(𝑖𝑖)(𝑧1 + 𝑧2 ). 𝑧3 = (3 − 7𝑖)(5 + 4𝑖) ̅̅̅̅̅̅̅̅̅̅̅̅̅̅̅̅̅̅̅̅̅̅̅̅̅ 10−5𝑖
̅+ 1
(𝑖)(5 + 9𝑖) + (2 − 4𝑖) (𝑖𝑖) (𝑖𝑖𝑖)3𝑖
= 15 + 12𝑖 − 35𝑖 − 28𝑖 2 = 15 + 12𝑖 − 35𝑖 + 28 = 43 − 23𝑖 → (1) 6+2𝑖 2−𝑖
̅̅̅̅̅̅̅̅̅̅̅̅̅̅̅̅̅̅̅̅̅̅̅̅̅ ̅̅̅̅̅̅̅̅ ̅̅̅̅̅̅̅̅
(𝑖)(5 + 9𝑖) + (2 − 4𝑖) = 5 + 9𝑖 + 2 − 4𝑖 = 5 − 9𝑖 + 2 + 4𝑖 = 7 − 5𝑖
𝑧1 𝑧3 + 𝑧2 𝑧3 = (3)(5 + 4𝑖) + (−7𝑖)(5 + 4𝑖)
10−5𝑖 10−5𝑖 6−2𝑖 60−20𝑖−30𝑖+10𝑖 2
= 15 + 12𝑖 − 35𝑖 − 28𝑖 2 = 15 + 12𝑖 − 35𝑖 + 28 = 43 − 23𝑖 → (2) (𝑖𝑖) = × =
6+2𝑖 6+2𝑖 6−2𝑖 62 +22
From (1) & (2), (𝑧1 + 𝑧2 ). 𝑧3 = 𝑧1 𝑧3 + 𝑧2 𝑧3 60−50𝑖−10 50−50𝑖 5 5
= = = − 𝑖
3.If 𝑧1 = 2 + 5𝑖, 𝑧2 = −3 − 4𝑖 and 𝑧3 = 1 + 𝑖, find the additive and multiplicative 40 40 4 4
1 1 2+𝑖 2+𝑖 2 1
inverse of 𝑧1 , 𝑧2 and 𝑧3 ̅+
(𝑖𝑖𝑖)3𝑖 = −3𝑖 + × = −3𝑖 + = −3𝑖 + + 𝑖
2−𝑖 2−𝑖 2+𝑖 22 +12 5 5
Additive inverse of 𝑧 is −𝑧.

V.GNANAMURUGAN, P,G,T, G.H.S.S, S.S.KOTTAI, SIVAGANGAI DT – 94874 43870 ; 82489 56766 , www.tnppgta.com, www.ednnet.in Page 29
2 −15𝑖+𝑖 2 14 3+4𝑖 4−3𝑖
= + = − 𝑖 = +
5 5 5 5 32 +4 2 4 2 +32
1
2.If z = x + iy, find the following in rectangular form: = (3 + 4𝑖 + 4 − 3𝑖)
1 25
(𝑖)𝑅𝑒 ( ) (𝑖𝑖)𝑅𝑒(𝑖𝑧̅) (𝑖𝑖𝑖)𝐼𝑚(3𝑧 + 4𝑧̅ − 4𝑖) ⟹
1
=
7+𝑖
𝑧
1 1 x−iy x−iy 𝑥 𝑦 1 𝑥 𝑢 25
(𝑖) = × = = − 𝑖 ⟹ 𝑅𝑒 ( ) = 2 2 (or) 𝑢=
25
=
25
×
7−𝑖
=
25(7−𝑖)
=
25(7−𝑖)
=
7−𝑖 7
= − 𝑖
1
𝑧 x+iy x−iy 𝑥 2 +𝑦 2 𝑥 2 +𝑦 2 𝑥 2 +𝑦 2 𝑧 𝑥 +𝑦
7+𝑖 7+𝑖 7−𝑖 72 +12 50 2 2 2
1 −1 ) 𝑥 𝑦 𝑥
∴ 𝑅𝑒 ( ) = 𝑅𝑒(𝑧 = 𝑅𝑒 ( 2 2 − 2 2 𝑖 ) = 2 2 5.Prove the following properties:
𝑧 𝑥 +𝑦 𝑥 +𝑦 𝑥 +𝑦 z+z̅ z−z̅
(𝑖𝑖)𝑖𝑧̅ = 𝑖(x̅̅̅̅̅̅̅̅ 2
+ iy) = 𝑖(𝑥 − 𝑖𝑦) = 𝑖𝑥 − 𝑖 𝑦 = 𝑦 + 𝑖𝑥 (i)z is real if and only if z = z̅ (𝑖𝑖)Re(z) = (𝑖𝑖𝑖)Im(z) =
2 2i
∴ 𝑅𝑒(𝑖𝑧̅) = 𝑅𝑒(−𝑦 + 𝑖𝑥) = 𝑦 (i)z is real ⟹ z = x + 0i
(𝑖𝑖𝑖)3𝑧 + 4𝑧̅ − 4𝑖 = 3(x + iy) + 4x̅̅̅̅̅̅̅̅
+ iy − 4𝑖 = 3𝑥 + 3𝑦𝑖 + 4(𝑥 − 𝑖𝑦) − 4𝑖 z̅ = ̅̅̅̅̅̅̅̅
𝑥 + 0𝑖 = 𝑥 − 0𝑖 = 𝑥 ⟹ z is real if and only if z = z̅
= 3𝑥 + 3𝑦𝑖 + 4𝑥 − 4𝑦𝑖 − 4𝑖 = 7𝑥 − 𝑦𝑖 − 4𝑖 = 7𝑥 + 𝑖(−𝑦 − 4) (𝑖𝑖) 𝐿𝑒𝑡 𝑧 = 𝑥 + 𝑖𝑦 ; 𝑅𝑒(𝑧) = 𝑥 → (1)
∴ 𝐼𝑚(3𝑧 + 4𝑧̅ − 4𝑖) = 𝐼𝑚(7𝑥 + 𝑖(−𝑦 − 4)) = −𝑦 − 4 ⟹ 𝑧̅ = 𝑥 − 𝑖𝑦
z 𝑧+𝑧̅
3. If 𝑧1 = 2 − 𝑖 and 𝑧2 = −4 + 3𝑖, find the inverse of 𝑧1 𝑧2 and 1 𝑧 + 𝑧̅ = 𝑥 + 𝑖𝑦 + 𝑥 − 𝑖𝑦 = 2𝑥 ⟹ = 𝑥 → (2)
z2 2
z+z̅
𝑧1 𝑧2 = (2 − 𝑖 )(−4 + 3𝑖) = (−8 + 3) + 𝑖(6 + 4) = −5 + 10𝑖 From (1) & (2), Re(z) =
2
(∵ 𝑧1 𝑧2 = (𝑎𝑐 − 𝑏𝑑) + 𝑖(𝑎𝑑 + 𝑏𝑐)) (𝑖𝑖𝑖) 𝐿𝑒𝑡 𝑧 = 𝑥 + 𝑖𝑦 ; 𝐼𝑚(𝑧) = 𝑦 → (1)
x y −5 10
Inverse of 𝑧1 𝑧2 = ( 2 2) + i (− 2 2) = ((−5)2 2 ) + i (− (−5)2 2 ) ⟹ 𝑧̅ = 𝑥 − 𝑖𝑦
x +y x +y +10 +10
𝑧−𝑧̅
=
−5

10
𝑖=−
1

2
𝑖 𝑧 − 𝑧̅ = 𝑥 + 𝑖𝑦 − 𝑥 + 𝑖𝑦 = 2𝑦𝑖 ⟹ = 𝑦 → (2)
125 125 25 25 2𝑖
z−z̅
z1
=
2−𝑖
= [(−4)2
−8−3
] + 𝑖 [(−4)2
6−4
]=−
11 2
+ 𝑖 From (1) & (2), Im(z) =
2i
z2 −4+3𝑖 +32 +32 25 25 𝑛
z1 𝑎+𝑖𝑏 𝑎𝑐+𝑏𝑑 𝑏𝑐−𝑎𝑑 6.Find the least value of the positive integer n for which (√3 + 𝑖)
(∵ = =[ ]+𝑖[ ])
z2 𝑐+𝑖𝑑 𝑐 2 +𝑑 2 𝑐 2 +𝑑 2 (𝑖)𝑟𝑒𝑎𝑙 (𝑖𝑖)𝑝𝑢𝑟𝑒𝑙𝑦 𝑖𝑚𝑎𝑔𝑖𝑛𝑎𝑟𝑦
z1 x y 3
Inverse of =( ) + i (− x2 +y2) (√3 + 𝑖) = 3√3 + 9𝑖 − 3√3 − 𝑖 = 8𝑖
z2 x2 +y2
−11⁄25 −2⁄25 6 3 2
=( 2 2 ) + i (− 2 2 ) (√3 + 𝑖) = ((√3 + 𝑖) ) = (8𝑖)2 = −64
(−11⁄25) +(−2⁄25) (−11⁄25) +(−2⁄25) 𝑛
11 2 11 2 (𝑖)𝑊ℎ𝑒𝑛 𝑛 = 6, (√3 + 𝑖) 𝑖𝑠 𝑟𝑒𝑎𝑙
=− 125 + 125 𝑖=− + 𝑖 𝑛
25× 25× 5 5
625 625 (𝑖𝑖)𝑊ℎ𝑒𝑛 𝑛 = 3, (√3 + 𝑖) 𝑖𝑠 𝑝𝑢𝑟𝑒𝑙𝑦 𝑖𝑚𝑎𝑔𝑖𝑛𝑎𝑟𝑦
1 1 1
4. The complex numbers 𝑢, 𝑣 and 𝑤 are related by = + . If 𝑣 = 3 − 4𝑖 and 10 10
𝑢 𝑣 𝑤 7.Show that (𝑖)(2 + 𝑖 √3) − (2 − 𝑖 √3) is purely imaginary.
𝑤 = 4 + 3𝑖, find 𝑢 in the rectangular form. 19−7𝑖 12 20−5𝑖 12
1 1 1 1 1 1 (𝑖𝑖) ( ) +( ) is real.
= + ⟹ = + 9+𝑖 7−6𝑖
𝑢 𝑣 𝑤 𝑢 3−4𝑖 4+3𝑖 10 10
1 3+4𝑖 1 4−3𝑖 (𝑖) 𝑧 = (2 + 𝑖√3) − (2 − 𝑖√3)
=( × ) + (4+3𝑖 × 4−3𝑖 )
3−4𝑖 3+4𝑖 ̅̅̅̅̅̅̅̅̅̅̅̅̅̅̅̅̅̅̅̅̅̅̅̅̅̅̅̅̅̅̅̅̅̅
10 10 ̅̅̅̅̅̅̅̅̅̅̅̅̅̅̅
10 ̅̅̅̅̅̅̅̅̅̅̅̅̅̅̅
10
𝑧̅ = (2 + 𝑖√3) − (2 − 𝑖√3) = (2 + 𝑖√3) − (2 − 𝑖√3)
V.GNANAMURUGAN, P,G,T, G.H.S.S, S.S.KOTTAI, SIVAGANGAI DT – 94874 43870 ; 82489 56766 , www.tnppgta.com, www.ednnet.in Page 30
̅̅̅̅̅̅̅̅̅̅ 10
̅̅̅̅̅̅̅̅̅̅ 10 10 10 ⟹ 2𝑧 + 6 = 𝑧 − 5𝑖 + 4𝑧𝑖 − 20𝑖 2
= (2 + 𝑖√3) − (2 − 𝑖√3) = (2 − 𝑖 √3) − (2 + 𝑖 √3)
10 10
⟹ 2𝑧 − 𝑧 − 4𝑧𝑖 = −5𝑖 + 20 − 6
= − {(2 + 𝑖 √3) − (2 − 𝑖 √3) } = −𝑧 ⟹ 𝑧 − 4𝑧𝑖 = 14 − 5𝑖
10 10 ⟹ 𝑧(1 − 4𝑖) = 14 − 5𝑖
∴ (2 + 𝑖 √3) − (2 − 𝑖 √3) is purely imaginary. 14−5𝑖 z1 𝑎+𝑖𝑏 𝑎𝑐+𝑏𝑑 𝑏𝑐−𝑎𝑑
19−7𝑖 19−7𝑖 9−𝑖 171−19𝑖−63𝑖+7𝑖 2 164−82𝑖 82(2−𝑖) ⟹𝑧= = =[ ]+𝑖[ ]
(𝑖𝑖) = × = = = =2−𝑖 1−4𝑖 z2 𝑐+𝑖𝑑 𝑐 2 +𝑑 2 𝑐 2 +𝑑 2
9+𝑖 9+𝑖 9−𝑖 92 +12 82 82
14+20 −5+56 34 51
20−5𝑖 20−5𝑖 7+6𝑖 140+120𝑖−35𝑖−30𝑖 2 170+85𝑖 =[ ]+𝑖[ ]= +𝑖 = 2 + 3𝑖
= × = = =2+𝑖 12 +(−4)2 12 +(−4)2 17 17
7−6𝑖 7−6𝑖 7+6𝑖 72 +62 85 z
19−7𝑖 12 20−5𝑖 12 12 12 𝐄𝐗𝐀𝐌𝐏𝐋𝐄 𝟐. 𝟔 If 𝑧1 = 3 − 2𝑖 and 𝑧2 = 6 + 4𝑖, find 1.
𝑧=(
9+𝑖
) + ( 7−6𝑖 ) = (2 − 𝑖) − (2 + 𝑖) z2
z1 3−2𝑖 3−2𝑖 6−4𝑖 18−12𝑖−12𝑖−8 10−24𝑖 2(5−12𝑖) 5−12𝑖
̅̅̅̅̅̅̅̅̅̅̅̅̅̅̅̅̅̅̅̅̅̅̅̅̅̅
𝑧̅ = (2 − 𝑖)12 − (2 + 𝑖)12 = (2̅̅̅̅̅̅̅̅̅̅̅
− 𝑖)12 − ̅̅̅̅̅̅̅̅̅̅̅
(2 + 𝑖)12 = (2 + 𝑖)12 − (2 − 𝑖)12 = 𝑧 = = × = = = =
z2 6+4𝑖 6+4𝑖 6−4𝑖 62 +4 2 52 52 26
19−7𝑖 12 20−5𝑖 12 −1
𝐄𝐗𝐀𝐌𝐏𝐋𝐄 𝟐. 𝟕 Find 𝑧 , if 𝑧 = (2 + 3𝑖)(1 − 𝑖).
∴( ) +( ) is real.
9+𝑖 7−6𝑖
10 10 𝑧 = (2 + 3𝑖)(1 − 𝑖) = 2 − 2𝑖 + 3𝑖 − 3𝑖 2 = 2 + 𝑖 + 3 = 5 + 𝑖
𝐄𝐗𝐀𝐌𝐏𝐋𝐄 𝟐. 𝟖 Show that (𝑖)(2 + 𝑖 √3) + (2 − 𝑖 √3) is real and 𝒙 𝒚 𝟓 1 5 1
𝑧 −1 = ( 𝟐 𝟐) + 𝒊 (− 𝟐 𝟐) = ( 𝟐 𝟐) + i (− 2 2) = − i
19+9𝑖𝑖 15 8+𝑖 15 𝒙 +𝒚 𝒙 +𝒚 𝟓 +𝟏 5 +1 26 26
(𝑖𝑖) ( ) −( ) is purely imaginary. MODULUS OF A COMPLEX NUMBER
5−3𝑖 1+2𝑖
3+4𝑖
𝐄𝐗𝐀𝐌𝐏𝐋𝐄 𝟐. 𝟑 Write
5−12𝑖
in the x + iy form , hence find its real and imaginary  If 𝑧 = x + iy , then the modulus of z is |𝑧| = √𝑥 2 + 𝑦 2
parts.  𝑧𝑧̅ = |𝑧|2
z1 𝑎+𝑖𝑏 𝑎𝑐+𝑏𝑑 𝑏𝑐−𝑎𝑑  PROPERTIES OF MODULUS OF A COMPLEX NUMBER:
= =[ ]+𝑖[ ]  |𝑍| = |𝑍̅|
z2 𝑐+𝑖𝑑 𝑐 2 +𝑑 2 𝑐 2 +𝑑 2
3+4𝑖
=[
15−48
]+𝑖[
20+36
]=
−33+56𝑖
=−
33
+
56
𝑖  |𝑧1 + 𝑧2 | ≤ |𝑧1 | + |𝑧2 | ( Triangle inequality )
5−12𝑖 52 +(−12)2 52 +(−12)2 169 169 169  |𝑧1 − 𝑧2 | ≥ |𝑧1 | − |𝑧2 |
33 56
Real part = −
169
and imaginary part=
169
 |𝑧1 𝑧2 | = |𝑧1 ||𝑧2 |
𝑧 |𝑧 |
1+𝑖 3 1−𝑖 3  | 1| = |𝑧1 | , 𝑧2 ≠ 0
𝐄𝐗𝐀𝐌𝐏𝐋𝐄 𝟐. 𝟒 Simplify ( ) − (1+𝑖 ) 𝑧2 2
1−𝑖
1+𝑖 1−1 1+1 2𝑖  |𝑧 𝑛 | = |𝑧|𝑛 , 𝑛 is an integer.
=[ ]+𝑖[ ] == =𝑖
1−𝑖 12 +(−1)2 12 +(−1)2 2  𝑅𝑒(𝑧) ≤ |𝑧|
1−𝑖 1+𝑖 −1  𝐼𝑚(𝑧) ≤ |𝑧|
=( ) = −𝑖
1+𝑖 1−𝑖
 ||𝑧1 | − |𝑧2 || ≤ |𝑧1 + 𝑧2 | ≤ |𝑧1 | + |𝑧2 |
1+𝑖 3 1−𝑖 3
(1−𝑖) − (1+𝑖 ) = (𝑖)3 − (−𝑖)3 = −𝑖 − 𝑖 = −2𝑖  The distance between the two points 𝑧1 and 𝑧2 in the complex plane is
𝑧+3 1+4𝑖 |𝑧1 − 𝑧2 | (or) √(𝑥1 − 𝑥2 )2 + (𝑦1 − 𝑦2 )2
𝐄𝐗𝐀𝐌𝐏𝐋𝐄 𝟐. 𝟓 If = , find the complex number z.
𝑧−5𝑖 2
𝑧+3 1+4𝑖  Formula for finding the square root of the complex number 𝑧 = 𝑎 + 𝑖𝑏 is
=
𝑧−5𝑖 2 |𝑧|+𝑎 𝑏 |𝑧|−𝑎
⟹ 2(𝑧 + 3) = (1 + 4𝑖)(𝑧 − 5𝑖) √𝑎 + 𝑖𝑏 = ± (√ 2
+ 𝑖 |𝑏| √
2
),

V.GNANAMURUGAN, P,G,T, G.H.S.S, S.S.KOTTAI, SIVAGANGAI DT – 94874 43870 ; 82489 56766 , www.tnppgta.com, www.ednnet.in Page 31
𝑏 3.Which one of the points 10 − 8𝑖, 11 + 6𝑖 is closest to 1 + 𝑖.
 If b is negative |𝑏|
= −1, x and y have different signs.
𝑏 Distance between the two points 𝑧1 and 𝑧2 = √(𝑥1 − 𝑥2 )2 + (𝑦1 − 𝑦2 )2
 If b is positive = 1, x and y have same signs.
|𝑏| 𝑧 = 1 + 𝑖, 𝑧1 = 10 − 8𝑖
Exercise 2.5 Distance between the two points 𝑧 and 𝑧1 = √(1 − 10)2 + (1 + 8)2
1.Find the modulus of the following complex numbers: = √81 + 81 = √162 = √2 × 81 = 9√2
2𝑖 2−𝑖 1−2𝑖
(𝑖) (𝑖𝑖) + (𝑖𝑖𝑖)(1 − 𝑖)10 (𝑖𝑣)2𝑖(3 − 4𝑖)(4 − 3𝑖) 𝑧 = 1 + 𝑖, 𝑧2 = 11 + 6𝑖
3+4𝑖 1+𝑖 1−𝑖
|𝑧| = √𝑥 2 + 𝑦 2 Distance between the two points 𝑧 and 𝑧2 = √(1 − 11)2 + (1 − 6)2
2𝑖 |2𝑖| √02 +22 √4 2 = √100 + 125 = √125 = √5 × 25 = 5√5
(𝑖) | | = |3+4𝑖| = = = ∵ 5√5 < 9√2, 11 + 6𝑖 is the closest point to 1 + 𝑖
3+4𝑖 √32 +4 2 √25 5
𝐄𝐗𝐀𝐌𝐏𝐋𝐄 𝟐. 𝟏𝟏 Which one of the points 𝑖, −2 + 𝑖 𝑎𝑛𝑑 3 is farthest from the
2−𝑖 1−2𝑖 (1−𝑖)(2−𝑖)+(1+𝑖)(1−2𝑖) 2−𝑖−2𝑖+𝑖 2 +1−2𝑖+𝑖−2𝑖 2
(𝑖𝑖) | + |=| |=| | origin?
1+𝑖 1−𝑖 (1+𝑖)(1−𝑖) 12 +12
2−𝑖−2𝑖−1+1−2𝑖+2 4−4𝑖 4.If |𝑧| = 3, show that 7 ≤ |𝑧 + 6 − 8𝑖| ≤ 13.
=| |=| | = |2 − 2𝑖| = √22 + 22 = √8 = 2√2 Let 𝑧1 = 𝑧 𝑎𝑛𝑑 𝑧2 = 6 − 8𝑖
2 2
10 10
(𝑖𝑖𝑖)|(1 − 𝑖)10 | = |1 − 𝑖| = (√12 + 12 ) = √2 = 25 = 32
10 W.K.T, ||𝑧1 | − |𝑧2 || ≤ |𝑧1 + 𝑧2 | ≤ |𝑧1 | + |𝑧2 |
(𝑖𝑣)|2𝑖(3 − 4𝑖)(4 − 3𝑖)| = |2𝑖||3 − 4𝑖||4 − 3𝑖| = √02 + 22 √32 + 42 √42 + 32 ⟹ |3 − √62 + 82 | ≤ |𝑧 + 6 − 8𝑖| ≤ 3 + √62 + 82
= √4√25√25 = 2.5.5 = 50 ⟹ |3 − √100| ≤ |𝑧 + 6 − 8𝑖| ≤ 3 + √100
𝐄𝐗𝐀𝐌𝐏𝐋𝐄 𝟐. 𝟗 If 𝑧1 = 3 + 4𝑖 , 𝑧2 = 5 − 12𝑖 and 𝑧3 = 6 + 8𝑖, find ⟹ |3 − 10| ≤ |𝑧 + 6 − 8𝑖| ≤ 3 + 10
|𝑧1 |, |𝑧2 |, |𝑧3 |, |𝑧1 + 𝑧2 |, |𝑧2 − 𝑧3 | and |𝑧1 + 𝑧3 |. ⟹ |−7| ≤ |𝑧 + 6 − 8𝑖| ≤ 13
2+𝑖 ⟹ 7 ≤ |𝑧 + 6 − 8𝑖| ≤ 13
𝐄𝐗𝐀𝐌𝐏𝐋𝐄 𝟐. 𝟏𝟎 Find the following : (𝑖) | | (𝑖𝑖)|̅̅̅̅̅̅̅̅̅
(1 + 𝑖)(2 + 3𝑖)(4𝑖 − 3)|
−1+2𝑖 𝐄𝐗𝐀𝐌𝐏𝐋𝐄 𝟐. 𝟏𝟑 If |𝑧| = 2, show that 3 ≤ |𝑧 + 3 + 4𝑖| ≤ 7.
𝑖(2+𝑖)3
(𝑖𝑖𝑖) | (1+𝑖)2 | 5. If |𝑧| = 1, show that 2 ≤ |𝑧 2 − 3| ≤ 4.
2.For any two complex numbers 𝑧1 and 𝑧2 , such that |𝑧1 | = |𝑧2 | = 1 and Let 𝑧1 = 𝑧 2 𝑎𝑛𝑑 𝑧2 = −3
𝑧 +𝑧
𝑧1 𝑧2 ≠ −1 then show that 1 2 is a real number. W.K.T, ||𝑧1 | − |𝑧2 || ≤ |𝑧1 + 𝑧2 | ≤ |𝑧1 | + |𝑧2 |
1+𝑧1 𝑧2
⟹ ||𝑧|2 − |−3|| ≤ |𝑧 2 − 3| ≤ |𝑧|2 + |−3|
Given |𝑧1 | = |𝑧2 | = 1
1 ⟹ |12 − 3| ≤ |𝑧 2 − 3| ≤ 12 + 3
|𝑧1 | = 1 ⟹ |𝑧1 |2 = 1 ⟹ 𝑧1 𝑧̅1 = 1 ⟹ 𝑧̅1 = ⟹ |−2| ≤ |𝑧 2 − 3| ≤ 4
𝑧1
𝑙𝑙𝑙 𝑙𝑦 𝑧̅2 =
1 ⟹ 2 ≤ |𝑧 2 − 3| ≤ 4
𝑧2 2
1
+
1 𝑧1 +𝑧2 6.If |𝑧 − | = 2, show that the greatest and least value of |𝑧| are √3 + 1 and
𝑧1 +𝑧2 ̅̅̅̅̅̅̅̅̅̅
𝑧 +𝑧 ̅̅̅+𝑧
𝑧 ̅̅̅ ̅̅̅+𝑧
𝑧 ̅̅̅ 𝑧1 𝑧2 𝑧1 𝑧2 𝑧1 +𝑧2 𝑧
Let 𝑤 = ̅ = ( 1 2 ) = ̅ 1̅̅̅̅̅̅
⟹𝑤 2
= 1̅̅̅𝑧̅̅̅
2
= = = =𝑤
1+𝑧1 𝑧2 1+𝑧 𝑧 1+𝑧 𝑧
1 2 1+𝑧 1 2 1 2 1+
1 1
𝑧1 𝑧2
1+𝑧1 𝑧2
𝑧1 𝑧2
1+𝑧1 𝑧2 √3 − 1 respectively.
2 2 2
Since 𝑤
̅ = 𝑤, 𝑤 is purely real. |𝑧 − | = 2 ⟹ 2 = |𝑧 − | ≥ ||𝑧| − | ||
𝑧 𝑧 𝑧

V.GNANAMURUGAN, P,G,T, G.H.S.S, S.S.KOTTAI, SIVAGANGAI DT – 94874 43870 ; 82489 56766 , www.tnppgta.com, www.ednnet.in Page 32
2 2 |9𝑧1 𝑧2 +4𝑧1 𝑧3 +𝑧2 𝑧3 |
⟹ ||𝑧| − | || ≤ 2 ⟹ −2 ≤ |𝑧| − | | ≤ 2 ∵ |𝑥 − 𝑎| ≤ 𝑟 ⟹ −𝑟 ≤ 𝑥 − 𝑎 ≤ 𝑟 ⟹ |𝑧1 + 𝑧2 + 𝑧3 | = |𝑧1 ||𝑧2 ||𝑧3 |
∵ |𝑧1 𝑧2 | = |𝑧1 ||𝑧2 |
𝑧 𝑧
|𝑧|2 −2 |9𝑧 𝑧 +4𝑧 𝑧 +𝑧 𝑧 |
2
⟹ −2 ≤ |𝑧| − |𝑧| ≤ 2 ⟹ −2 ≤ ≤ 2 ⟹ −2|𝑧| ≤ |𝑧|2 − 2 ≤ 2|𝑧| → (1) ⟹1= 1 2 1 3 2 3
|𝑧| 1.2.3
From (1) ⟹ −2|𝑧| ≤ |𝑧|2 − 2 ⟹ |9𝑧1 𝑧2 + 4𝑧1 𝑧3 + 𝑧2 𝑧3 | = 6
⟹ 2 ≤ |𝑧|2 + 2|𝑧| (OR)
⟹ 3 ≤ |𝑧|2 + 2|𝑧| + 1 ( Adding 1 on both sides ) |𝑧1 | = 1 ⟹ |𝑧1 |2 = 1 ⟹ 𝑧1 𝑧̅1 = 1 ∵ 𝑧𝑧̅ = |𝑧|2
⟹ 3 ≤ (|𝑧| + 1)2 |𝑧2 | = 2 ⟹ |𝑧2 |2 = 4 ⟹ 𝑧2 𝑧̅2 = 4
⟹ √3 ≤ |𝑧| + 1 ( Taking square root on both sides) |𝑧3 | = 3 ⟹ |𝑧3 |2 = 9 ⟹ 𝑧3 𝑧̅3 = 9
|9𝑧1 𝑧2 + 4𝑧1 𝑧3 + 𝑧2 𝑧3 | = |𝑧3 𝑧̅3 𝑧1 𝑧2 + 𝑧2 𝑧̅2 𝑧1 𝑧3 + 𝑧1 𝑧̅1 𝑧2 𝑧3 |
⟹ (√3 − 1) ≤ |𝑧| → (2)
= |(𝑧1 𝑧2 𝑧3 )(𝑧̅1 + 𝑧̅2 + 𝑧̅3 )|
From (1) ⟹ |𝑧|2 − 2 ≤ 2|𝑧| = |𝑧1 𝑧2 𝑧3 ||𝑧̅1 + 𝑧̅2 + 𝑧̅3 |
⟹ |𝑧|2 − 2|𝑧| ≤ 2 ̅̅̅̅̅̅̅̅̅̅̅̅̅̅̅̅̅
= |𝑧1 𝑧2 𝑧3 ||𝑧 1 + 𝑧2 + 𝑧3 |
⟹ |𝑧|2 − 2|𝑧| + 1 ≤ 3 ( Adding 1 on both sides ) = |𝑧1 ||𝑧2 ||𝑧3 ||𝑧1 + 𝑧2 + 𝑧3 |
⟹ (|𝑧| − 1)2 ≤ 3 = 1.2.3.1
⟹ |𝑧| − 1 ≤ √3 ( Taking square root on both sides) =6
⟹ |𝑧| ≤ (√3 + 1) → (3) 𝐄𝐗𝐀𝐌𝐏𝐋𝐄 𝟐. 𝟏𝟐 If 𝑧1 , 𝑧2 𝑎𝑛𝑑 𝑧3 are three complex numbers such that |𝑧1 | =
1 1 1
From (2) & (3), √3 − 1 ≤ |𝑧| ≤ √3 + 1 |𝑧2 | = |𝑧3 | = |𝑧1 + 𝑧2 + 𝑧3 | = 1, find the value of | + + |.
𝑧1 𝑧2 𝑧3
Hence the greatest and least value of |𝑧| are √3 + 1 and √3 − 1 respectively. 𝐄𝐗𝐀𝐌𝐏𝐋𝐄 𝟐. 𝟏𝟓 If 𝑧1 , 𝑧2 𝑎𝑛𝑑 𝑧3 are three complex numbers such that |𝑧1 | =
7.If 𝑧1 , 𝑧2 𝑎𝑛𝑑 𝑧3 are three complex numbers such that |𝑧1 | = 1, |𝑧2 | = 2, |𝑧3 | = 3 𝑧 𝑧 +𝑧 𝑧 +𝑧 𝑧
|𝑧2 | = |𝑧3 | = 𝑟 > 0 and 𝑧1 + 𝑧2 + 𝑧3 ≠ 0. Prove that | 1 2 2 3 3 1| = 𝑟
𝑧1 +𝑧2 +𝑧3
and |𝑧1 + 𝑧2 + 𝑧3 | = 1, show that |9𝑧1 𝑧2 + 4𝑧1 𝑧3 + 𝑧2 𝑧3 | = 6. 𝑟2
|𝑧1 | = 1 ⟹ |𝑧1 |2 = 1 ⟹ 𝑧1 𝑧̅1 = 1 ⟹ 𝑧1 =
1
∵ 𝑧𝑧̅ = |𝑧|2 |𝑧1 | = 𝑟 ⟹ |𝑧1 |2 2 2
= 𝑟 ⟹ 𝑧1 𝑧̅1 = 𝑟 ⟹ 𝑧1 = ̅̅̅ ∵ 𝑧𝑧̅ = |𝑧|2
𝑧1
̅̅̅
𝑧 1
𝑟2
|𝑧2 | = 2 ⟹ |𝑧2 |2 = 4 ⟹ 𝑧2 𝑧̅2 = 4 ⟹ 𝑧2 = ̅̅̅
4 |𝑧2 | = 𝑟 ⟹ |𝑧2 |2 = 𝑟 2 ⟹ 𝑧2 𝑧̅2 = 𝑟 2 ⟹ 𝑧2 =
𝑧2 ̅̅̅
𝑧2
9 𝑟2
|𝑧3 | = 3 ⟹ |𝑧3 |2 = 9 ⟹ 𝑧3 𝑧̅3 = 9 ⟹ 𝑧3 = |𝑧3 | = 𝑟 ⟹ |𝑧3 |2 = 𝑟 2 ⟹ 𝑧3 𝑧̅3 = 𝑟 2 ⟹ 𝑧3 =
̅̅̅ 𝑧3 ̅̅̅ 𝑧3
1 4 9 𝑟2 𝑟2 𝑟2 ̅̅̅𝑧
𝑧 ̅̅̅+𝑧 ̅̅̅𝑧
1 ̅̅̅+𝑧
3 ̅̅̅𝑧
1 ̅̅̅
𝑧1 + 𝑧2 + 𝑧3 = ̅̅̅ + ̅̅̅ + ̅̅̅
𝑧1 𝑧2 𝑧3
𝑧1 + 𝑧2 + 𝑧3 = ̅̅̅ + ̅̅̅ + ̅̅̅ = 𝑟 2 ( 2 3 ̅̅̅̅̅̅̅̅̅ 2
)
𝑧1 𝑧2 𝑧3 𝑧1 𝑧2 𝑧3
̅̅̅𝑧
𝑧 2 ̅̅̅+4𝑧
3 ̅̅̅𝑧
1 ̅̅̅+9𝑧
3 ̅̅̅𝑧
1 ̅̅̅
2 ̅̅̅𝑧
𝑧 ̅̅̅+𝑧̅̅̅𝑧
1 ̅̅̅+𝑧
3 ̅̅̅𝑧
1 ̅̅̅ ̅̅̅̅̅̅̅̅̅̅̅̅̅̅̅̅̅̅̅̅̅̅̅
|𝑧
̅̅̅𝑧̅̅̅+𝑧̅̅̅𝑧
1 ̅̅̅+𝑧
3 ̅̅̅𝑧 2|
1 ̅̅̅ |𝑧1 𝑧2 +𝑧2 𝑧3 +𝑧3 𝑧1 |
⟹ 𝑧1 + 𝑧2 + 𝑧3 = ̅̅̅̅̅̅̅̅̅ ⟹ |𝑧1 + 𝑧2 + 𝑧3 | = |𝑟 2 | | 2 3 ̅̅̅̅̅̅̅̅̅ 2
| = 𝑟 2 2 3|𝑧̅̅̅̅̅̅̅̅̅ = 𝑟2
𝑧1 𝑧2 𝑧3 𝑧1 𝑧2 𝑧3 1 𝑧2 𝑧3 | |𝑧1 ||𝑧2 ||𝑧3 |
̅̅̅̅̅̅̅̅̅̅̅̅̅̅̅̅̅̅̅̅̅
̅̅̅𝑧
𝑧 ̅̅̅+4𝑧
̅̅̅𝑧
1 ̅̅̅+9𝑧
̅̅̅𝑧
1 ̅̅̅
⟹ |𝑧1 + 𝑧2 + 𝑧3 | = | 2 3 ̅̅̅̅̅̅̅̅̅
3 2
|
𝑧1 𝑧2 𝑧3 |𝑧1 𝑧2 +𝑧2 𝑧3 +𝑧3 𝑧1 | |𝑧1 𝑧2 +𝑧2 𝑧3 +𝑧3 𝑧1 |
̅̅̅̅̅̅̅̅̅̅̅̅̅̅̅̅̅̅̅̅̅̅̅̅̅̅
|𝑧
̅̅̅𝑧
2 ̅̅̅+4𝑧
3 ̅̅̅𝑧̅
1 3 +9𝑧 ̅̅̅𝑧 2|
1 ̅̅̅ ⟹ |𝑧1 + 𝑧2 + 𝑧3 | = 𝑟 2 ⟹ |𝑧1 + 𝑧2 + 𝑧3 | =
⟹ |𝑧1 + 𝑧2 + 𝑧3 | = ̅̅̅̅̅̅̅̅̅
|𝑧
𝑟.𝑟.𝑟
𝑧1 𝑧2 +𝑧2 𝑧3 +𝑧3 𝑧1
𝑟
̅̅̅𝑧
1 ̅̅̅𝑧 3|
2 ̅̅̅
|9𝑧1 𝑧2 +4𝑧1 𝑧3 +𝑧2 𝑧3 | ⟹| |=𝑟
𝑧1 +𝑧2 +𝑧3
⟹ |𝑧1 + 𝑧2 + 𝑧3 | = |𝑧1 𝑧2 𝑧3 |
∵ z̿ = z
V.GNANAMURUGAN, P,G,T, G.H.S.S, S.S.KOTTAI, SIVAGANGAI DT – 94874 43870 ; 82489 56766 , www.tnppgta.com, www.ednnet.in Page 33
8.If the area of the triangle formed by the vertices 𝑧, 𝑖𝑧 𝑎𝑛𝑑 𝑧 + 𝑖𝑧 is 50 square 𝐴𝐵 = √(𝑥1 − 𝑥2 )2 + (𝑦1 − 𝑦2 )2 = √(3 + 2)2 + (2 − 3)2 = √25 + 1 = √26
units, find the value of |𝑧|. 𝐵𝐶 = √(𝑥1 − 𝑥2 )2 + (𝑦1 − 𝑦2 )2 = √(−2 − 1)2 + (3 − 5)2 = √9 + 4 = √13
Let 𝑧 = 𝑎 + 𝑖𝑏 = (𝑎, 𝑏) ⟹ |𝑧| = √𝑎2 + 𝑏 2
𝐶𝐴 = √(𝑥1 − 𝑥2 )2 + (𝑦1 − 𝑦2 )2 = √(1 − 3)2 + (5 − 2)2 = √4 + 9 = √13
𝑖𝑧 = 𝑖(𝑎 + 𝑖𝑏) = 𝑖𝑎 + 𝑖 2 𝑏 = −𝑏 + 𝑖𝑎 ⟹ |𝑖𝑧| = √𝑎2 + 𝑏 2 𝐴𝐵2 = 26 , 𝐵𝐶 2 = 13, 𝐶𝐴2 = 13
𝑧 + 𝑖𝑧 = 𝑎 + 𝑖𝑏 − 𝑏 + 𝑖𝑎 = (𝑎 − 𝑏) + 𝑖(𝑎 + 𝑏)
Since (𝑖)𝐵𝐶 = 𝐶𝐴 = √13 and (𝑖𝑖)𝐵𝐶 2 + 𝐶𝐴2 = 13 + 13 = 26 = 𝐴𝐵2 , So the
|𝑧 + 𝑖𝑧| = √(𝑎 − 𝑏)2 + (𝑎 + 𝑏)2 = √2(𝑎2 + 𝑏 2 ) given vertices form an isosceles right triangle.
|𝑧|𝟐 + |𝑖𝑧|𝟐 = 𝟐(𝑎2 + 𝑏 2 ) = |𝑧 + 𝑖𝑧|𝟐 9.Show that the equation 𝑧 3 + 2𝑧̅ = 0 has five solutions.
∴ The given vertices form an isosceles right triangle. 𝑧 3 + 2𝑧̅ = 0
Area of the triangle = 50 sq. units ⟹ 𝑧 3 = −2𝑧̅
1
⟹ √𝑎2 + 𝑏 2 . √𝑎2 + 𝑏 2 = 50 ⟹ |𝑧 3 | = |−2||𝑧̅|
2
⟹ 𝑎2 + 𝑏 2 = 100 ⟹ |𝑧 3 | = 2|𝑧|
⟹ √𝑎2 + 𝑏 2 = 10 ⟹ |𝑧|3 − 2|𝑧| = 0
⟹ |𝑧| = 10 |𝑧| = 0 ⟹ 𝑧 = 0 𝑖𝑠 𝑜𝑛𝑒 𝑜𝑓 𝑡ℎ𝑒 𝑠𝑜𝑙𝑢𝑡𝑖𝑜𝑛
2
1 √3 1 √3 ⟹ |𝑧|(|𝑧| − 2) = 0 ⟹ { 2
𝐄𝐗𝐀𝐌𝐏𝐋𝐄 𝟐. 𝟏𝟒 Show that the points 1, − + 𝑖 and − − 𝑖 are the vertices |𝑧|2 − 2 = 0 ⟹ 𝑧𝑧̅ − 2 = 0 ⟹ 𝑧̅ =
2 2 2 2 𝑧
of an equilateral triangle. 2 2
Put 𝑧̅ = in 𝑧 3 + 2𝑧̅ = 0 ⟹ 𝑧 3 + 2 ( ) = 0 ⟹ 𝑧 4 + 4 = 0, which provides four
1 √3 1 √3 𝑧 𝑧
Let 𝑧1 = 1, 𝑧2 = − + 𝑖 and 𝑧3 = − − 𝑖 solution.
2 2 2 2
The length of the sides of the triangle are Hence 𝑧 3 + 2𝑧̅ = 0 has five solutions.
1 √3 3 √3 3 2 √3
2
9 3 12 𝐄𝐗𝐀𝐌𝐏𝐋𝐄 𝟐. 𝟏𝟔 Show that the equation 𝑧 2 = 𝑧̅ has four solutions.
|𝑧1 − 𝑧2 | = |1 − (− + 𝑖 )| =| −𝑖 | = √ ( ) + ( ) = √ + = √ = √3 10.Find the square root of (𝑖)4 + 3𝑖 (𝑖𝑖) − 6 + 8𝑖 (𝑖𝑖𝑖) − 5 − 12𝑖.
2 2 2 2 2 2 4 4 4
2
1 √3 1 √3 1 √3 1 √3 2√3
|𝑧2 − 𝑧3 | = |(− + 𝑖 ) − (− − 𝑖 )| = |− + 𝑖 + +𝑖 | = √( ) = √3 |𝑧| + 𝑎 𝑏 |𝑧| − 𝑎
2 2 2 2 2 2 2 2 2 √𝑎 + 𝑖𝑏 = ± (√ +𝑖 √ )
2 2 |𝑏| 2
1 √3 3 √3 3 2 √3 9 3
|𝑧3 − 𝑧1 | = |(− − 𝑖 ) − 1| = |− − 𝑖 | = √ ( ) + ( ) = √ + = √3
2 2 2 2 2 2 4 4
(𝑖)4 + 3𝑖
Since all the three sides are equal, the given points form an equilateral triangle.
𝑎 = 4, 𝑏 = 3, |𝑧| = √42 + 32 = √25 = 5
𝐄𝐗𝐀𝐌𝐏𝐋𝐄 𝟐. 𝟏𝟖 Given the complex number = 3 + 2𝑖 , represent the complex
5+4 3 5−4 3 1
numbers 𝑧, 𝑖𝑧 and 𝑧 + 𝑖𝑧 in one Argand diagram. Show that these complex numbers √4 + 3𝑖 = ± (√ +𝑖 √ ) = ±( +𝑖 )
2 3 2 √2 √2
form the vertices of an isosceles right triangle.
(𝑖𝑖) − 6 + 8𝑖
Given : 𝑧 = 3 + 2𝑖 = 𝐴(3,2)
𝑖𝑧 = 𝑖(3 + 2𝑖) = −2 + 3𝑖 = 𝐵(−2,3) 𝑎 = −6, 𝑏 = 8, |𝑧| = √62 + 82 = √100 = 10
𝑧 + 𝑖𝑧 = 3 + 2𝑖 − 2 + 3𝑖 = 1 + 5𝑖 = 𝐶(1,5) 10−6 8 10+6
√−6 + 8𝑖 = ± (√ +𝑖 √ ) = ±(√2 + 𝑖√8) = ±(√2 + 𝑖2√2)
2 8 2
The length of the sides of the triangle are
V.GNANAMURUGAN, P,G,T, G.H.S.S, S.S.KOTTAI, SIVAGANGAI DT – 94874 43870 ; 82489 56766 , www.tnppgta.com, www.ednnet.in Page 34
(𝑖𝑖𝑖) − 5 − 12𝑖 4 ∴ The locus of z is real axis.
2𝑧+1
𝑎 = −5, 𝑏 = −12, |𝑧| = √52 + 122 = √169 = 13 2. If 𝑧 = 𝑥 + 𝑖𝑦 is a complex number such that 𝐼𝑚 ( ) = 0 , show that the locus
𝑖𝑧+1
√−5 − 12𝑖 = ± (√
13−5
+𝑖
−12

13+5
) = ±(√4 − 𝑖 √9) = ±(2 − 3𝑖) of z is 2𝑥 2 + 2𝑦 2 + 𝑥 − 2𝑦 = 0.
2 12 2
Given: 𝑧 = 𝑥 + 𝑖𝑦
𝐄𝐗𝐀𝐌𝐏𝐋𝐄 𝟐. 𝟏𝟕 Find the square root of 6 + 8𝑖 2𝑧+1 2(𝑥+𝑖𝑦)+1 (2𝑥+1)+𝑖2𝑦 𝑏𝑐−𝑎𝑑
= = Im(Z) =
GEOMETRY AND LOCUS OF COMPLEX NUMBERS 𝑖𝑧+1 𝑖(𝑥+𝑖𝑦)+1 (1−𝑦)+𝑖𝑥 𝑐 2 +𝑑 2
 A circle is defined as the locus of a point which moves in a plane such that its Here 𝑎 = 2𝑥 + 1 , 𝑏 = 2𝑦, 𝑐 = 1 − 𝑦, 𝑑 = 𝑥
2𝑧+1 2𝑦(1−𝑦)−𝑥(2𝑥+1)
distance from a fixed point in that plane is always a constant. The fixed point 𝐼𝑚 ( )=0⟹ (1−𝑦)2 +𝑥 2
= 0 ⟹ 2𝑦 − 2𝑦 2 − 2𝑥 2 − 𝑥 = 0
𝑖𝑧+1
is the centre and the constant distant is the radius of the circle. ⟹ 2𝑥 2 + 2𝑦 2 + 𝑥 − 2𝑦 = 0
 Equation of complex form of a circle is |𝑧 − 𝑧0 | = 𝑟. 3.Obtain the Cartesian form of the locus of 𝑧 = 𝑥 + 𝑖𝑦 in each of the following
 |𝑧 − 𝑧0 | < 𝑟 represents the points interior of the circle. cases:
 |𝑧 − 𝑧0 | > 𝑟 represents the points exterior of the circle. (𝑖)[𝑅𝑒(𝑖𝑧)]2 = 3 (𝑖𝑖)𝐼𝑚[(1 − 𝑖)𝑧 + 1] = 0 (𝑖𝑖𝑖)|𝑧 + 𝑖| = |𝑧 − 1| (𝑖𝑣)𝑧̅ = 𝑧 −1
 𝑥 2 + 𝑦 2 = 𝑟 2 represents a circle with centre at the origin and the radius r Given: 𝑧 = 𝑥 + 𝑖𝑦
units. (𝑖)𝑖𝑧 = 𝑖(𝑥 + 𝑖𝑦) = 𝑦 − 𝑖𝑥
 𝑟 = √𝑥 2 + 𝑦 2 [𝑅𝑒(𝑖𝑧)]2 = 3
a+ib 𝑎𝑐+𝑏𝑑
 If z = then Re(Z) = ⟹ [𝑅𝑒(𝑦 − 𝑖𝑥 )]2 = 3
𝑐+𝑖𝑑 𝑐 2 +𝑑 2
a+ib 𝑏𝑐−𝑎𝑑 ⟹ 𝑦2 = 3
 If z = then Im(Z) = (𝑖𝑖)(1 − 𝑖)𝑧 + 1 = (1 − 𝑖)(𝑥 + 𝑖𝑦) + 1 = 𝑥 + 𝑖𝑦 − 𝑖𝑥 + 𝑦 + 1
𝑐+𝑖𝑑 𝑐 2 +𝑑 2
EXERCISE 2.6 = (𝑥 + 𝑦 + 1) + 𝑖(𝑦 − 𝑥)
𝑧−4𝑖 𝐼𝑚[(1 − 𝑖)𝑧 + 1] = 0
1.If 𝑧 = 𝑥 + 𝑖𝑦 is a complex number such that | | = 1. Show that the locus of z is
𝑧+4𝑖
⟹ 𝐼𝑚[(𝑥 + 𝑦 + 1) + 𝑖(𝑦 − 𝑥)] = 0
real axis.
⟹ 𝑦 − 𝑥 = 0 (𝑜𝑟)𝑥 − 𝑦 = 0
Given: 𝑧 = 𝑥 + 𝑖𝑦
𝑧−4𝑖
(𝑖𝑖𝑖) |𝑧 + 𝑖| = |𝑧 − 1|
| | = 1 ⟹ |𝑧 − 4𝑖| = |𝑧 + 4𝑖| ⟹ |𝑥 + 𝑖𝑦 + 𝑖| = |𝑥 + 𝑖𝑦 − 1|
𝑧+4𝑖
⟹ |𝑥 + 𝑖𝑦 − 4𝑖| = |𝑥 + 𝑖𝑦 + 4𝑖| ⟹ |𝑥 + 𝑖(𝑦 + 1)| = |(𝑥 − 1) + 𝑖𝑦|
⟹ |𝑥 + 𝑖(𝑦 − 4)| = |𝑥 + 𝑖(𝑦 + 4)| ⟹ √(𝑥)2 + (𝑦 + 1)2 = √(𝑥 − 1)2 + (𝑦)2
⟹ √𝑥 2 + (𝑦 − 4)2 = √𝑥 2 + (𝑦 − 4)2 Squaring on both sides,
Squaring on both sides, ⟹ (𝑥)2 + (𝑦 + 1)2 = (𝑥 − 1)2 + (𝑦)2
⟹ 𝑥 2 + (𝑦 − 4)2 = 𝑥 2 + (𝑦 − 4)2 ⟹ 𝑥 2 + 𝑦 2 + 2𝑦 + 1 = 𝑥 2 − 2𝑥 + 1 + 𝑦 2
⟹ 𝑥 2 + 𝑦 2 − 8𝑦 + 16 = 𝑥 2 + 𝑦 2 + 8𝑦 + 16 ⟹ 2𝑥 + 2𝑦 = 0
⟹ 8𝑦 = 0 ⟹𝑥+𝑦 =0
1
⟹𝑦=0 (𝑖𝑣)𝑧̅ = 𝑧 −1 ⟹ 𝑧̅ = ⟹ 𝑧𝑧̅ = 1 ⟹ (𝑥 + 𝑖𝑦)(𝑥 − 𝑖𝑦) = 1 ⟹ 𝑥 2 + 𝑦 2 = 1
𝑧

V.GNANAMURUGAN, P,G,T, G.H.S.S, S.S.KOTTAI, SIVAGANGAI DT – 94874 43870 ; 82489 56766 , www.tnppgta.com, www.ednnet.in Page 35
4.Show that the following equations represent a circle, and find its centre and Find its centre and radius.
radius: (𝑖)|𝑧 − 2 − 𝑖| = 3 (𝑖𝑖)|2𝑧 + 2 − 4𝑖| = 2 (𝑖𝑖𝑖)|3𝑧 − 6 + 12𝑖| = 8 Consider |𝑧 + 2 − 𝑖| = 2 ⟹ |𝑧 − (−2 + 𝑖)| = 2.
𝐸𝑞𝑢𝑎𝑡𝑖𝑜𝑛 𝑜𝑓 𝑡ℎ𝑒 𝑐𝑖𝑟𝑐𝑙𝑒 𝑖𝑠 |𝑧 − 𝑧0 | = 𝑟 Centre = −2 + 𝑖 = (−2,1) ; Radius = 2 𝑢𝑛𝑖𝑡𝑠
(𝑖)|𝑧 − 2 − 𝑖| = 3 ⟹ |𝑧 − (2 + 𝑖)| = 3 ∴ |𝑧 + 2 − 𝑖| < 2 represents all points inside the circle.
Centre = 2 + 𝑖 = (2,1) ; Radius = 3 𝑢𝑛𝑖𝑡𝑠 POLAR AND EULER FORM OF A COMPLEX NUMBER
(𝑖𝑖)|2𝑧 + 2 − 4𝑖| = 2 ⟹ 2 |𝑧 + 1 − 2𝑖| = 2 ⟹ |𝑧 − (−1 + 2𝑖)| = 1  Polar form of 𝑧 = 𝑥 + 𝑖𝑦 is 𝑧 = 𝑟(cos 𝜃 + 𝑖 sin 𝜃) (𝑜𝑟) 𝑧 = 𝑟𝑐𝑖𝑠𝜃
Centre = −1 + 2𝑖 = (−1,2) ; Radius = 1 𝑢𝑛𝑖𝑡  𝑀𝑜𝑑𝑢𝑙𝑢𝑠 𝑟 = √𝑥 2 + 𝑦 2
8 𝑦
(𝑖𝑖𝑖)|3𝑧 − 6 + 12𝑖| = 8 ⟹ 3|𝑧 − 2 + 4𝑖| = 8 ⟹ |𝑧 − (2 − 4𝑖)| =  𝛼 = 𝑡𝑎𝑛−1 ( )
3 𝑥
8
Centre = 2 − 4𝑖 = (2, −4) ; Radius = 𝑢𝑛𝑖𝑡𝑠 𝜃 = 𝛼 ; 𝑖𝑓(+, +) 𝜃 𝑙𝑖𝑒𝑠 𝑖𝑛 𝑡ℎ𝑒 𝐼𝑠𝑡 𝑞𝑢𝑎𝑑𝑟𝑎𝑛𝑡
3
𝐄𝐗𝐀𝐌𝐏𝐋𝐄 𝟐. 𝟏𝟗 Show that |3𝑧 − 5 + 𝑖| = 4 represent a circle, and find its centre 𝜃 = 𝜋 − 𝛼 ; 𝑖𝑓(−, +) 𝜃 𝑙𝑖𝑒𝑠 𝑖𝑛 𝑡ℎ𝑒 2𝑛𝑑 𝑞𝑢𝑎𝑑𝑟𝑎𝑛𝑡
 arg 𝑧 = 𝜃 ;
and radius. 𝜃 = −𝜋 + 𝛼 ; 𝑖𝑓(−, −) 𝜃 𝑙𝑖𝑒𝑠 𝑖𝑛 𝑡ℎ𝑒 3𝑟𝑑 𝑞𝑢𝑎𝑑𝑟𝑎𝑛𝑡
5. Obtain the Cartesian form of the locus of 𝑧 = 𝑥 + 𝑖𝑦 in each of the following { 𝜃 = −𝛼 ; 𝑖𝑓(+, −) 𝜃 𝑙𝑖𝑒𝑠 𝑖𝑛 𝑡ℎ𝑒 4𝑡ℎ 𝑞𝑢𝑎𝑑𝑟𝑎𝑛𝑡
cases: (𝑖)|𝑧 − 4| = 16 (𝑖𝑖)|𝑧 − 4|2 − |𝑧 − 1|2 = 16  In general arg 𝑧 = 𝐴𝑟𝑔 𝑧 + 2𝑛𝜋 , 𝑛 ∈ 𝑧.
Given: 𝑧 = 𝑥 + 𝑖𝑦  Properties of arguments:
(𝑖)|𝑧 − 4| = 16 ⟹ |𝑥 + 𝑖𝑦 − 4| = 16  𝑎𝑟𝑔(𝑧1 𝑧2 ) = 𝑎𝑟𝑔(𝑧1 ) + 𝑎𝑟𝑔(𝑧2 )
𝑧
⟹ |(𝑥 − 4) + 𝑖𝑦| = 16  𝑎𝑟𝑔 ( 1 ) = 𝑎𝑟𝑔(𝑧1 ) − 𝑎𝑟𝑔(𝑧2 )
𝑧2
⟹ √(𝑥 − 4)2 + 𝑦 2 = 16 𝑛)
 𝑎𝑟𝑔(𝑧 = 𝑛 𝑎𝑟𝑔𝑧
Squaring on both sides,  Some of the principal argument and argument:
⟹ (𝑥 − 4)2 + 𝑦 2 = 162 𝑧 1 𝑖 −1 −𝑖
⟹ 𝑥 2 − 8𝑥 + 16 + 𝑦 2 = 256
⟹ 𝑥 2 + 𝑦 2 − 8𝑥 − 240 = 0 is the locus of z. 𝜋 𝜋
𝐴𝑟𝑔 𝑧 0 𝜋 −
(𝑖𝑖)|𝑧 − 4|2 − |𝑧 − 1|2 = 16 ⟹ |𝑥 + 𝑖𝑦 − 4|2 − |𝑥 + 𝑖𝑦 − 1|2 = 16 2 2
⟹ |(𝑥 − 4) + 𝑖𝑦|2 − |(𝑥 − 1) + 𝑖𝑦|2 = 16
2 2 𝜋 𝜋
⟹ (√(𝑥 − 4)2 + 𝑦 2 ) − √(𝑥 − 1)2 + 𝑦 2 = 16 𝑎𝑟𝑔𝑧 2𝑛𝜋 2𝑛𝜋 + 2𝑛𝜋 + 𝜋 2𝑛𝜋 −
⟹ (𝑥 − 4)2 + 𝑦 2 − [(𝑥 − 1)2 + 𝑦 2 ] = 16 2 2
𝑖𝜃 𝑖𝜃
⟹ 𝑥 2 − 8𝑥 + 16 + 𝑦 2 − 𝑥 2 + 2𝑥 − 1 − 𝑦 2 = 16  Euler’s form of the complex number: 𝑧 = 𝑟𝑒 ; 𝑒 = cos 𝜃 + 𝑖 sin 𝜃
⟹ −6𝑥 − 1 = 0  Properties of polar form:
1
⟹ 6𝑥 + 1 = 0 is the locus of z.  𝑧 = 𝑟(cos 𝜃 + 𝑖 sin 𝜃) ⟹ 𝑧 −1 = (cos 𝜃 − 𝑖 sin 𝜃)
𝑟
𝐄𝐗𝐀𝐌𝐏𝐋𝐄 𝟐. 𝟐𝟏 Obtain the Cartesian form of the locus of 𝑧 in each of the  If 𝑧1 = 𝑟1 (cos 𝜃1 + 𝑖 sin 𝜃1 ), 𝑧2 = 𝑟2 (cos 𝜃2 + 𝑖 sin 𝜃2 ) 𝑡ℎ𝑒𝑛
following cases: (𝑖)|𝑧| = |𝑧 − 𝑖| (𝑖𝑖)|2𝑧 − 3 − 𝑖| = 3 𝑧1 𝑧2 = 𝑟1 𝑟2 [cos(𝜃1 + 𝜃2 ) + 𝑖 sin(𝜃1 + 𝜃2 )]
𝐄𝐗𝐀𝐌𝐏𝐋𝐄 𝟐. 𝟐𝟎 Show that |𝑧 + 2 − 𝑖| < 2 represents interior points of a circle.  If 𝑧1 = 𝑟1 (cos 𝜃1 + 𝑖 sin 𝜃1 ), 𝑧2 = 𝑟2 (cos 𝜃2 + 𝑖 sin 𝜃2 ) 𝑡ℎ𝑒𝑛

V.GNANAMURUGAN, P,G,T, G.H.S.S, S.S.KOTTAI, SIVAGANGAI DT – 94874 43870 ; 82489 56766 , www.tnppgta.com, www.ednnet.in Page 36
𝑧1 𝑟1 𝑦 −1 𝜋
= [cos(𝜃1 − 𝜃2 ) + 𝑖 sin(𝜃1 − 𝜃2 )] 𝛼 = 𝑡𝑎𝑛−1 ( ) = 𝑡𝑎𝑛−1 | | =
𝑧2 𝑟2 𝑥 3 √ 6
 Some of the useful polar forms : Since the complex number √3 − 𝑖 lies in the fourth quadrant, principal argument
𝜋 𝜋 𝜋
1 = 𝑐𝑖𝑠(0) ; −1 = 𝑐𝑖𝑠(𝜋) ; 𝑖 = 𝑐𝑖𝑠 ( ) ; −𝑖 = 𝑐𝑖𝑠 (− ) 𝜃 = −𝛼 ⟹ 𝜃 = −
2 2 6
𝑚1 −𝑚2 −2
 𝑡𝑎𝑛−1 (𝑚1 ) − 𝑡𝑎𝑛−1 (𝑚2 ) = 𝐄𝐗𝐀𝐌𝐏𝐋𝐄 𝟐. 𝟐𝟒 Find the principal argument 𝐴𝑟𝑔 𝑧, when 𝑧 =
1+𝑚1 𝑚2 1+𝑖 √3
𝐄𝐗𝐀𝐌𝐏𝐋𝐄 𝟐. 𝟐𝟐 Find the modulus and principal argument of the following −2
𝑎𝑟𝑔𝑧 = 𝑎𝑟𝑔 (
1+𝑖 √3
) = 𝑎𝑟𝑔(−2) − 𝑎𝑟𝑔(1 + 𝑖 √3) → (1)
complex numbers: (𝑖)√3 + 𝑖 (𝑖𝑖) − √3 + 𝑖 (𝑖𝑖𝑖) − √3 − 𝑖 (𝑖𝑣)√3 − 𝑖 𝑦 0
𝑎𝑟𝑔(−2) = 𝑡𝑎𝑛−1 | |
= 𝑡𝑎𝑛−1 | | = 0
(𝑖)√3 + 𝑖 𝑥 −2
Since the complex number −2 lies in the second quadrant, principal argument
2
𝑀𝑜𝑑𝑢𝑙𝑢𝑠 𝑟 = √𝑥 2 + 𝑦 2 = √√3 + 12 = √4 = 2 𝜃 =𝜋−𝛼 ⟹𝜃 =𝜋−0=𝜋
𝑦 √3 𝜋
𝑦
𝛼 = 𝑡𝑎𝑛−1 | | = 𝑡𝑎𝑛−1 | | =
1 𝜋
𝑎𝑟𝑔(1 + 𝑖 √3) = 𝑡𝑎𝑛−1 | | = 𝑡𝑎𝑛−1 | | =
𝑥 √3 6 𝑥 1 3
Since the complex number √3 + 𝑖 lies in the first quadrant, principal argument Since the complex number 1 + 𝑖 √3 lies in the first quadrant, principal argument
𝜋 𝜋
𝜃=𝛼⟹𝜃= 𝜃=𝛼⟹𝜃=
6 3
−2 𝜋 2𝜋
(𝑖𝑖) − √3 + 𝑖 (1) ⟹ 𝑎𝑟𝑔 ( )=𝜋−3=
1+𝑖 √ 3 3
2 EXERCISE 2.7
𝑀𝑜𝑑𝑢𝑙𝑢𝑠 𝑟 = √𝑥 2 + 𝑦 2 = √(−√3) + 12 = √4 = 2
1.Write in polar form of the following complex numbers:
𝑦 1 𝜋
𝛼 = 𝑡𝑎𝑛−1 ( ) = 𝑡𝑎𝑛−1 | | = (𝑖)2 + 𝑖2√3 (𝑖𝑖)3 − 𝑖 √3 (𝑖𝑖𝑖) − 2 − 𝑖2 (𝑖𝑣) 𝜋
𝑖−1
𝑥 − 3 √ 6 cos +𝑖 sin
𝜋
3 3
Since the complex number −√3 + 𝑖 lies in the second quadrant, principal
𝜋 5𝜋 (𝑖)2 + 𝑖2√3 = 𝑟𝑐𝑖𝑠(𝜃) → (1)
argument 𝜃 = 𝜋 − 𝛼 ⟹ 𝜃 = 𝜋 − = 2
6 6
(𝑖𝑖𝑖) − √3 − 𝑖 𝑟 = √𝑥 2 + 𝑦 2 = √(2)2 + (2√3) = √4 + 12 = √16 = 4
𝑦 2√3 𝜋
𝑀𝑜𝑑𝑢𝑙𝑢𝑠 𝑟 = √𝑥 2 + 𝑦 2 = √(−√3) + (−1)2 = √4 = 2
2 𝛼 = 𝑡𝑎𝑛−1 | | = 𝑡𝑎𝑛−1 | | =
𝑥 2 3
𝜋
𝑦 −1 𝜋 Principal argument lies in the first quadrant 𝜃 = 𝛼 =
𝛼= 𝑡𝑎𝑛−1 ( ) = 𝑡𝑎𝑛−1 | |= 𝜋 𝜋
3
𝑥 −√3 6
(1) ⟹ 2 + 𝑖2√3 = 4𝑐𝑖𝑠 ( ) = 4𝑐𝑖𝑠 (2𝑘𝜋 + ) , 𝑘 ∈ 𝑧
Since the complex number −√3 − 𝑖 lies in the third quadrant, principal argument 3 3

𝜃 = −𝜋 + 𝛼 ⟹ 𝜃 = −𝜋 + = −
𝜋 5𝜋 (𝑖𝑖)3 − 𝑖 √3 = 𝑟𝑐𝑖𝑠(𝜃) → (1)
6 6 2
(𝑖𝑣)√3 − 𝑖 𝑟 = √𝑥 2 + 𝑦 2 = √(3)2 + (−√3) = √9 + 3 = √4 × 3 = 2√3
2 𝑦 −√3 1 𝜋
𝑀𝑜𝑑𝑢𝑙𝑢𝑠 𝑟 = √𝑥 2 + 𝑦 2 = √√3 + (−1)2 = √4 = 2 𝛼 = 𝑡𝑎𝑛−1 | | = 𝑡𝑎𝑛−1 | | = 𝑡𝑎𝑛−1 | | =
𝑥 3 √3 6
𝜋
Principal argument lies in the 4th quadrant 𝜃 = −𝛼 = −
6

V.GNANAMURUGAN, P,G,T, G.H.S.S, S.S.KOTTAI, SIVAGANGAI DT – 94874 43870 ; 82489 56766 , www.tnppgta.com, www.ednnet.in Page 37
𝜋 𝜋 1 𝜋 𝜋 1 3𝜋 1 𝜋 1 𝜋 𝜋
(1) ⟹ 3 − 𝑖 √3 = 2√3𝑐𝑖𝑠 (− ) = 2√3𝑐𝑖𝑠 (2𝑘𝜋 − ) , 𝑘 ∈ 𝑧 = 𝑐𝑖𝑠 (− − ) = 𝑐𝑖𝑠 (− ) = 2 𝑐𝑖𝑠 (− 2 ) = 2 [cos (− 2 ) + 𝑖 sin (− 2 )]
6 6 2 6 3 2 6
(𝑖𝑖𝑖) − 2 − 𝑖2 = 𝑟𝑐𝑖𝑠(𝜃) → (1) 1 𝜋 𝜋
= (cos − 𝑖 sin )
2 2 2
𝑟 = √𝑥 2 + 𝑦 2 = √(−2)2 + (−2)2 = √4 + 4 = √2 × 4 = 2√2 1
𝑦 −2 𝜋 = (0 − 𝑖)
2
𝛼 = 𝑡𝑎𝑛−1 | | = 𝑡𝑎𝑛−1 |− |= 1
𝑥 2 4 =− 𝑖
𝜋 3𝜋 2
Principal argument lies in the 3rd quadrant 𝜃 = −𝜋 + 𝛼 = −𝜋 + = − 3 𝜋 𝜋 5𝜋 5𝜋
4 4 𝐄𝐗𝐀𝐌𝐏𝐋𝐄 𝟐. 𝟐𝟓 Find the product (cos + 𝑖 sin ) . 6 (cos + 𝑖 sin ) in
3𝜋 3𝜋 2 3 3 6 6
(1) ⟹ −2 − 𝑖2 = 2√2𝑐𝑖𝑠 (− ) = 2√2𝑐𝑖𝑠 (2𝑘𝜋 − ),𝑘 ∈ 𝑧
4 4 rectangular form.
𝑖−1
(𝑖𝑣) 𝜋 𝜋
9𝜋
2(cos
4
9𝜋
+𝑖 sin )
4
cos +𝑖 sin 𝐄𝐗𝐀𝐌𝐏𝐋𝐄 𝟐. 𝟐𝟔 Find the quotient in rectangular form.
3 3 −3𝜋 −3𝜋
4(cos( 2 )+𝑖 sin( 2 ))
𝑖 − 1 = −1 + 𝑖 = 𝑟𝑐𝑖𝑠(𝜃) → (1)
𝑟 = √𝑥 2 + 𝑦 2 = √(−1)2 + (1)2 = √2 3.If (𝑥1 + 𝑖𝑦1 )(𝑥2 + 𝑖𝑦2 ) … . . (𝑥𝑛 + 𝑖𝑦𝑛 ) = 𝑎 + 𝑖𝑏 , show that (i) (𝑥12 + 𝑦12 )(𝑥22 +
𝑏
𝑦 1 𝜋 𝑦22 ) … . . (𝑥𝑛2 + 𝑦𝑛2 ) = 𝑎2 + 𝑏 2 (ii) ∑𝑛𝑟=1 𝑡𝑎𝑛−1 (𝑎𝑏𝑟𝑟 ) = 𝑡𝑎𝑛−1 (𝑎) + 2𝑘𝜋, 𝑘𝜖𝑍.
𝛼= 𝑡𝑎𝑛−1 | | = 𝑡𝑎𝑛 −1
| |=
𝑥 −1 4 (𝑖)(𝑥1 + 𝑖𝑦1 )(𝑥2 + 𝑖𝑦2 ) … . . (𝑥𝑛 + 𝑖𝑦𝑛 ) = 𝑎 + 𝑖𝑏
nd 𝜋 3𝜋
Principal argument lies in the 2 quadrant 𝜃 = 𝜋 − 𝛼 ⟹ 𝜃 = 𝜋 − = Taking modulus on both sides,
4 4
(1) ⟹ 𝑖 − 1 = √2𝑐𝑖𝑠 ( )
3𝜋 ⟹ |(𝑥1 + 𝑖𝑦1 )(𝑥2 + 𝑖𝑦2 ) … . . (𝑥𝑛 + 𝑖𝑦𝑛 )| = |𝑎 + 𝑖𝑏|
4 ⟹ |𝑥1 + 𝑖𝑦1 ||𝑥2 + 𝑖𝑦2 | … … . . |𝑥𝑛 + 𝑖𝑦𝑛 | = |𝑎 + 𝑖𝑏|
3𝜋
𝑖−1 √2𝑐𝑖𝑠( 4 ) 3𝜋 𝜋 5𝜋 5𝜋
𝜋 𝜋 = 𝜋 = √2𝑐𝑖𝑠 ( − ) = √2𝑐𝑖𝑠 ( ) = √2𝑐𝑖𝑠 (2𝑘𝜋 + ) , 𝑘 ∈ 𝑧 ⟹ √(𝑥21 + 𝑦21 )√(𝑥22 + 𝑦22 ) … . . √(𝑥2𝑛 + 𝑦2𝑛 ) = √𝑎2 + 𝑏2
cos +𝑖 sin 𝑐𝑖𝑠( 3 ) 4 3 12 12
3 3

𝐄𝐗𝐀𝐌𝐏𝐋𝐄 𝟐. 𝟐𝟑 Represent the complex number (𝑖) − 1 − 𝑖 (𝑖𝑖)1 + 𝑖√3 in polar Squaring on both sides,
form. ⟹ (𝑥12 + 𝑦12 )(𝑥22 + 𝑦22 ) … . . (𝑥𝑛2 + 𝑦𝑛2 ) = 𝑎2 + 𝑏 2
2.Find the rectangular form of the complex numbers: (𝑖)(𝑥1 + 𝑖𝑦1 )(𝑥2 + 𝑖𝑦2 ) … . . (𝑥𝑛 + 𝑖𝑦𝑛 ) = 𝑎 + 𝑖𝑏
𝜋 𝜋
𝜋 𝜋 𝜋 𝜋 cos −𝑖 sin
6 6
Taking argument on both sides,
(𝑖) (cos + 𝑖 sin ) (cos + 𝑖 sin ) (𝑖𝑖) 𝜋 𝜋 ⟹ 𝑎𝑟𝑔[(𝑥1 + 𝑖𝑦1 )(𝑥2 + 𝑖𝑦2 ) … . . (𝑥𝑛 + 𝑖𝑦𝑛 )] = 𝑎𝑟𝑔(𝑎 + 𝑖𝑏)
6 6 12 12 2(cos 3 +𝑖 sin 3 )
𝑏
𝜋 𝜋 𝜋
(𝑖) (cos + 𝑖 sin ) (cos + 𝑖 sin ) = cis ( ) + cis ( )
𝜋 𝜋 𝜋 ⟹ 𝑡𝑎𝑛−1 (𝑦𝑥11 ) + 𝑡𝑎𝑛−1 (𝑦𝑥22 ) + ⋯ . +𝑡𝑎𝑛−1 (𝑦𝑥𝑛𝑛 ) = 𝑡𝑎𝑛−1 ( )
𝑎
6 6 12 12 6 12 𝑏
𝜋 𝜋 ⟹ ∑𝑛𝑟=1 𝑡𝑎𝑛−1 (𝑎𝑏𝑟𝑟 ) = 𝑡𝑎𝑛−1 (𝑎) + 2𝑘𝜋, 𝑘𝜖𝑍
= cis ( + ) 1+𝑧
6 12
3𝜋 𝜋 4.If = cos 2𝜃 + 𝑖 sin 2𝜃 , show that 𝑧 = 𝑖 tan 𝜃.
= cis ( ) = 𝑐𝑖𝑠 ( ) 1−𝑧
1+𝑧
12 4
𝜋 𝜋 = cos 2𝜃 + 𝑖 sin 2𝜃
= cos + 𝑖 sin 1−𝑧
1+𝑧
4 4
1 1 Add 1 on both sides, + 1 = (1 + cos 2𝜃) + 𝑖 sin 2𝜃
= +𝑖 1−𝑧
1+𝑧+1−𝑧
√2 √2
𝜋
cos −𝑖 sin
𝜋 ⟹ = 2𝑐𝑜𝑠 2 𝜃 + 𝑖2 sin 𝜃 cos 𝜃
6 6 1−𝑧
(𝑖𝑖) 2
𝜋 𝜋
2(cos 3 +𝑖 sin 3 ) ⟹ = 2 cos 𝜃 [cos 𝜃 + 𝑖 sin 𝜃]
1−𝑧

V.GNANAMURUGAN, P,G,T, G.H.S.S, S.S.KOTTAI, SIVAGANGAI DT – 94874 43870 ; 82489 56766 , www.tnppgta.com, www.ednnet.in Page 38
1 𝜋
÷ 𝑏𝑦 2 ⟹ = cos 𝜃 [cos 𝜃 + 𝑖 sin 𝜃] ⇒ 𝑎𝑟𝑔(𝑥 + 𝑖(𝑦 − 1)) − 𝑎𝑟𝑔(𝑥 + 2 + 𝑖𝑦) =
1−𝑧 4
1
⟹ =1−𝑧 𝑦 − 1 𝑦 𝜋
cos 𝜃[cos 𝜃+𝑖 sin 𝜃] ⇒ 𝑡𝑎𝑛−1 − 𝑡𝑎𝑛−1 =
⟹𝑧 =1−
1 𝑥 𝑥+2 4
cos 𝜃[cos 𝜃+𝑖 sin 𝜃] 𝑦−1 𝑦
1 cos 𝜃−𝑖 sin 𝜃 − 𝜋
⟹𝑧 =1− × ⇒ 𝑡𝑎𝑛−1 𝑥 𝑥+2 =
cos 𝜃[cos 𝜃+𝑖 sin 𝜃] cos 𝜃−𝑖 sin 𝜃 𝑦−1 𝑦
cos 𝜃−𝑖 sin 𝜃 1+( × ) 4
⟹𝑧 =1− 𝑥 𝑥+2
cos 𝜃(𝑐𝑜𝑠 2 𝜃+𝑠𝑖𝑛2 𝜃)
cos 𝜃 sin 𝜃 ((𝑦 − 1))(𝑥 + 2) − 𝑥𝑦
⟹𝑧 =1−[ −𝑖 ] 𝑥(𝑥 + 2) 𝜋
cos 𝜃 cos 𝜃
⟹ 𝑧 = 1 − 1 + 𝑖 tan 𝜃 ⇒ = tan
𝑦2 − 𝑦 4
⟹ 𝑧 = 𝑖 tan 𝜃 1+
𝑥(𝑥 + 2)
5.If 𝑐𝑜𝑠𝛼 + 𝑐𝑜𝑠𝛽 + 𝑐𝑜𝑠𝛾 = 𝑠𝑖𝑛𝛼 + 𝑠𝑖𝑛𝛽 + 𝑠𝑖𝑛𝛾 = 0 , then show that 𝑥𝑦 + 2𝑦 − 𝑥 − 2 − 𝑥𝑦
(i) 𝑐𝑜𝑠3𝛼 + 𝑐𝑜𝑠3𝛽 + 𝑐𝑜𝑠3𝛾 = 3 cos(𝛼 + 𝛽 + 𝛾) (𝑥 + 2)
⟹ =1
(ii)𝑠𝑖𝑛3𝛼 + 𝑠𝑖𝑛3𝛽 + 𝑠𝑖𝑛3𝛾 = 3sin(𝛼 + 𝛽 + 𝛾) 𝑥 + 2𝑥 + 𝑦 2 − 𝑦
2

Let 𝑎 = 𝑐𝑖𝑠𝛼, 𝑏 = 𝑐𝑖𝑠𝛽, 𝑐 = 𝑐𝑖𝑠𝛾 (𝑥 + 2)


W.K.T, If 𝑎 + 𝑏 + 𝑐 = 0 𝑡ℎ𝑒𝑛 𝑎3 + 𝑏 3 + 𝑐 3 = 3𝑎𝑏𝑐 −𝑥 + 2𝑦 − 2
⟹ 2 =1
𝑎 + 𝑏 + 𝑐 = (𝑐𝑜𝑠𝛼 + 𝑐𝑜𝑠𝛽 + 𝑐𝑜𝑠𝛾) + 𝑖(𝑠𝑖𝑛𝛼 + 𝑠𝑖𝑛𝛽 + 𝑠𝑖𝑛𝛾) = 0 𝑥 + 𝑦 2 + 2𝑥 − 𝑦
𝑎3 + 𝑏 3 + 𝑐 3 = 3𝑎𝑏𝑐 ⟹ 𝑥 2 + 𝑦 2 + 2𝑥 − 𝑦 + 𝑥 − 2𝑦 + 2 = 0
⟹ (𝑐𝑖𝑠𝛼)3 + (𝑐𝑖𝑠𝛽)3 + (𝑐𝑖𝑠𝛾)3 = 3𝑐𝑖𝑠𝛼. 𝑐𝑖𝑠𝛽. 𝑐𝑖𝑠𝛾 ⟹ 𝑥 2 + 𝑦 2 + 3𝑥 − 3𝑦 + 2 = 0
𝑧−1 𝜋
⟹ 𝑐𝑖𝑠(3𝛼) + 𝑐𝑖𝑠(3𝛽) + 𝑐𝑖𝑠(3𝛾) = 3𝑐𝑖𝑠(𝛼 + 𝛽 + 𝛾) 𝐄𝐗𝐀𝐌𝐏𝐋𝐄 𝟐. 𝟐𝟕 If 𝑧 = 𝑥 + 𝑖𝑦 and 𝑎𝑟𝑔 ( ) = 2 , then show that 𝑥 2 + 𝑦 2 = 1.
𝑧+1
⟹ (𝑐𝑜𝑠3𝛼 + 𝑐𝑜𝑠3𝛽 + 𝑐𝑜𝑠3𝛾) + 𝑖(𝑠𝑖𝑛3𝛼 + 𝑠𝑖𝑛3𝛽 + 𝑠𝑖𝑛3𝛾) = 3(cos(𝛼 + 𝛽 +
De MOIVRE’S THEOREM AND ITS APPLICATIONS
𝛾)) + 𝑖3(sin(𝛼 + 𝛽 + 𝛾))
 De Moivre’s theorem: Given any complex number cos 𝜃 + 𝑖 sin 𝜃 and any
Equating the real and imaginary parts on both sides,
integer n, (cos 𝜃 + 𝑖 sin 𝜃)𝑛 = cos 𝑛𝜃 + 𝑖 sin 𝑛𝜃
𝑐𝑜𝑠3𝛼 + 𝑐𝑜𝑠3𝛽 + 𝑐𝑜𝑠3𝛾 = 3 cos(𝛼 + 𝛽 + 𝛾)
𝑠𝑖𝑛3𝛼 + 𝑠𝑖𝑛3𝛽 + 𝑠𝑖𝑛3𝛾 = 3sin(𝛼 + 𝛽 + 𝛾)  Some results:
𝑧−𝑖 𝜋  (cos 𝜃 − 𝑖 sin 𝜃)𝑛 = cos 𝑛𝜃 − 𝑖 sin 𝑛𝜃
6.If 𝑧 = 𝑥 + 𝑖𝑦 and 𝑎𝑟𝑔 ( ) = , then show that 𝑥 2 + 𝑦 2 + 3𝑥 − 3𝑦 + 2 = 0.  (cos 𝜃 + 𝑖 sin 𝜃)−𝑛 = cos 𝑛𝜃 − 𝑖 sin 𝑛𝜃
𝑧+2 4
Given: 𝑍 = 𝑥 + 𝑖𝑦  (cos 𝜃 − 𝑖 sin 𝜃)−𝑛 = cos 𝑛𝜃 + 𝑖 sin 𝑛𝜃
𝑎𝑟𝑔 (
𝑧−𝑖
)=
𝜋  sin 𝜃 + 𝑖 cos 𝜃 = 𝑖(cos 𝜃 − 𝑖 sin 𝜃)
𝑧+2 4
𝜋  Formula for finding 𝑛𝑡ℎ root of a complex number:
⇒ 𝑎𝑟𝑔(𝑧 − 𝑖) − 𝑎𝑟𝑔(𝑧 + 2) = 1 1 𝜃+2𝑘𝜋
4 𝑧 ⁄𝑛 = 𝑟 ⁄𝑛 𝑐𝑖𝑠 ( ) , 𝑘 = 0,1, … . , (𝑛 − 1)
𝜋 𝑛
⇒ 𝑎𝑟𝑔(𝑥 + 𝑖𝑦 − 𝑖) − 𝑎𝑟𝑔(𝑥 + 𝑖𝑦 + 2) =  𝜔3 = 1 ⟹ 1 + 𝜔 + 𝜔2 = 0
4
 The sum of all the 𝑛𝑡ℎ roots of unity is 0.(𝑖𝑒. , 1 + 𝜔 + 𝜔2 + ⋯ . +𝜔𝑛−1 = 0)

V.GNANAMURUGAN, P,G,T, G.H.S.S, S.S.KOTTAI, SIVAGANGAI DT – 94874 43870 ; 82489 56766 , www.tnppgta.com, www.ednnet.in Page 39
 The product of all the 𝑛𝑡ℎ roots of unity is (−1)𝑛−1 . (𝑖𝑒. , 1 + 𝜔 + 𝜔2 + 5𝜋
= 2 cos ( ) = 2 cos (
5×180
) = 2 cos 150 = −2 cos 30 = −2 ×
√3
= −√3
⋯ . +𝜔𝑛−1 = (−1)𝑛−1 ) 6 6
31
2

 All the n roots of 𝑛𝑡ℎ roots of unity are in geometrical progression. 𝐄𝐗𝐀𝐌𝐏𝐋𝐄 𝟐. 𝟑𝟏 Simplify (𝑖)(1 + 𝑖)18 (𝑖𝑖)(−√3 + 3𝑖)
10
 All the n roots of 𝑛𝑡ℎ roots of unity lie on the circumference of a circle whose 𝜋
1+sin +𝑖 cos
10
𝜋
10
centre is at the origin and radius equal to 1 and these roots divide the circle 3.Find the value of ( 𝜋 𝜋 )
1+sin −𝑖 cos
10 10
into n equal parts and form a polygon of n sides. Let 𝑧 = sin
𝜋
+ 𝑖 cos
𝜋
⟹ = sin
1 𝜋
− 𝑖 cos
𝜋

EXERCISE 2.8 10 10 𝑧 10 10
𝜋 𝜋 10 10
𝑎+𝑏𝜔+𝑐𝜔2 𝑎+𝑏𝜔+𝑐𝜔2 1+sin +𝑖 cos 1+𝑧 1+𝑧 10
1.If 𝜔 ≠ 1 is a cube root of unity, then show that 2
+ = −1. ( 10
𝜋
10
𝜋 ) =( 1 ) =( × 𝑧) = 𝑧10
𝑏+𝑐𝜔+𝑎𝜔 𝑐+𝑎𝜔+𝑏𝜔2 1+sin −𝑖 cos
10 10
1+ 1+𝑧
𝑧
𝑎+𝑏𝜔+𝑐𝜔2 𝑎+𝑏𝜔+𝑐𝜔2
+ 𝜋 𝜋 10 𝜋 𝜋 10
𝑏+𝑐𝜔+𝑎𝜔2 𝑐+𝑎𝜔+𝑏𝜔2 𝑧 10 = (sin + 𝑖 cos ) = [𝑖 (cos − 𝑖 sin )]
𝑎+𝑏𝜔+𝑐𝜔 2 𝜔 𝑎+𝑏𝜔+𝑐𝜔2 𝜔2 10 10 10 10
=( × )+( × 2) 𝜋 𝜋
𝑏+𝑐𝜔+𝑎𝜔2 𝜔 𝑐+𝑎𝜔+𝑏𝜔2 𝜔 = 𝑖 10 [cos (10 × ) − 𝑖 sin (10 × )]
𝜔(𝑎+𝑏𝜔+𝑐𝜔2 ) 𝜔2 (𝑎+𝑏𝜔+𝑐𝜔2 ) 10 10
= + 𝜔3 = 1 ; 1 + 𝜔 + 𝜔2 = 0 = −1[cos 𝜋 − 𝑖 sin 𝜋]
𝑎𝜔3 +𝑐𝜔2 +𝑏𝜔 𝑎𝜔3 +𝑏𝜔4 +𝑐𝜔2
𝜔(𝑎+𝑏𝜔+𝑐𝜔2 ) 𝜔2 (𝑎+𝑏𝜔+𝑐𝜔2 ) = −1(−1 − 0)
= +
𝑎+𝑏𝜔+𝑐𝜔2 𝑎+𝑏𝜔+𝑐𝜔2 =1
= 𝜔 + 𝜔2 𝜋 𝜋 18
= −1 𝐄𝐗𝐀𝐌𝐏𝐋𝐄 𝟐. 𝟐𝟗 Simplify (sin + 𝑖 cos )
6 6
√3 𝑖
5
√3 𝑖
5 1+cos 2𝜃+𝑖 sin 2𝜃 30
2.Show that (2 + ) + (2 − ) = −√3 𝐄𝐗𝐀𝐌𝐏𝐋𝐄 𝟐. 𝟑𝟎 Simplify ( )
2 2 1+cos 2𝜃−𝑖 sin 2𝜃
1 1
√3 𝑖 4.If 2 cos 𝛼 = 𝑥 + and 2 cos 𝛽 = 𝑦 + , show that
+ = 𝑟𝑐𝑖𝑠(𝜃) → (1) 𝑥 𝑦
2 2
𝑥 𝑦 1
√3 1
2
3 1 2 (𝑖) + = 2 cos(𝛼 − 𝛽) (𝑖𝑖)𝑥𝑦 − = 2𝑖 sin(𝛼 + 𝛽)
𝑟= √𝑥 2 + 𝑦2 = √( ) + ( ) = √ + = √1 = 1 𝑦
𝑥𝑚
𝑥
𝑦 𝑛
𝑥𝑦
1
2 2 4 4
(𝑖𝑖𝑖) − = 2𝑖 sin(𝑚𝛼 − 𝑛𝛽) (𝑖𝑣)𝑥 𝑚 𝑦 𝑛 + = 2 cos(𝑚𝛼 + 𝑛𝛽)
𝑦 1⁄ 𝜋 𝑦𝑛 𝑥𝑚 𝑥𝑚𝑦𝑛
𝛼= 𝑡𝑎𝑛−1 | | = 𝑡𝑎𝑛 | 3 2 |
−1
= 1
𝑥 + = 2 cos 𝛼 ⟹ 𝑥 = 𝑥 = cos 𝛼 + 𝑖 sin 𝛼
𝑥 √ ⁄ 6
2 𝑥
𝜋 1
Principal argument lies in the first quadrant 𝜃 = 𝛼 = 𝑦 + = 2 cos 𝛽 ⟹ 𝑦 = cos 𝛽 + 𝑖 sin 𝛽
6 𝑦
√3 𝑖 𝜋 𝑥 cos 𝛼+𝑖 sin 𝛼
(1) ⟹ + = 𝑐𝑖𝑠 ( ) (𝑖) = = cos(𝛼 − 𝛽) + 𝑖 sin(𝛼 − 𝛽)
2 2 6 𝑦 cos 𝛽+𝑖 sin 𝛽
5 5 𝑦 𝑥 −1
√3 𝑖 𝜋 5𝜋 5𝜋 5𝜋
⟹( + ) = [𝑐𝑖𝑠 ( )] = 𝑐𝑖𝑠 ( ) = cos ( ) + 𝑖 sin ( ) =( ) = cos(𝛼 − 𝛽) − 𝑖 sin(𝛼 − 𝛽)
2 2 6 6 6 6 𝑥 𝑦
5 𝑥 𝑦
√3 𝑖 5𝜋 5𝜋 + = 2 cos(𝛼 − 𝛽)
Similarly, (2 − ) = cos ( ) − 𝑖 sin ( ) 𝑦 𝑥
2 6 6
5 5 (𝑖𝑖) 𝑥𝑦 = (cos 𝛼 + 𝑖 sin 𝛼)(cos 𝛽 + 𝑖 sin 𝛽) = cos(𝛼 + 𝛽) + 𝑖 sin(𝛼 + 𝛽)
√3 𝑖 √3 𝑖 5𝜋 5𝜋 5𝜋 5𝜋 1
(2 + ) + (2 − ) = cos ( ) + 𝑖 sin ( ) + cos ( ) − 𝑖 sin ( ) = (𝑥𝑦)−1 = cos(𝛼 + 𝛽) − 𝑖 sin(𝛼 + 𝛽)
2 2 6 6 6 6 𝑥𝑦

V.GNANAMURUGAN, P,G,T, G.H.S.S, S.S.KOTTAI, SIVAGANGAI DT – 94874 43870 ; 82489 56766 , www.tnppgta.com, www.ednnet.in Page 40
1 1
𝑥𝑦 −
𝑥𝑦
= 2𝑖 sin(𝛼 + 𝛽) ⟹ 𝑧 − 1 = −2(1) ⁄3 ⟹ 𝑧 − 1 = −2[1 (𝑜𝑟) 𝜔 (𝑜𝑟) 𝜔2 ]
(𝑖𝑖𝑖) 𝑥 = (cos 𝛼 + 𝑖 sin 𝛼)𝑚 = cos 𝑚𝛼 + 𝑖 sin 𝑚𝛼
𝑚 𝑧 − 1 = −2(1) ⟹ 𝑧 − 1 = −2 ⟹ 𝑧 = −1
⟹ {𝑧 − 1 = −2𝜔 ⟹ 𝑧 = 1 − 2𝜔
𝑦 𝑛 = (cos 𝛽 + 𝑖 sin 𝛽)𝑛 = cos 𝑛𝛽 + 𝑖 sin 𝑛𝛽
𝑥𝑚 cos 𝑚𝛼+𝑖 sin 𝑚𝛼 𝑧 − 1 = −2𝜔2 ⟹ 𝑧 = 1 − 2𝜔2
𝑛
= = cos(𝑚𝛼 − 𝑛𝛽) + 𝑖 sin(𝑚𝛼 − 𝑛𝛽) Thus the roots of the equation (𝑧 − 1)3 + 8 = 0 are −1, 1 − 2𝜔, 1 − 2𝜔2 .
𝑦 cos 𝑛𝛽+𝑖 sin 𝑛𝛽
𝑦 𝑛 𝑥 𝑚 −1 𝐄𝐗𝐀𝐌𝐏𝐋𝐄 𝟐. 𝟑𝟒 Solve the equation 𝑧 3 + 8𝑖 = 0, where 𝑧 ∈ ℂ.
=( ) = cos(𝑚𝛼 − 𝑛𝛽) − 𝑖 sin(𝑚𝛼 − 𝑛𝛽) 1 1
𝑥𝑚 𝑦𝑛
𝑥𝑚 𝑦 𝑛
Note: 𝑧 3 + 8𝑖 = 0 ⟹ 𝑧 = (−8𝑖) ⁄3 ⟹ 𝑧 = −2(𝑖) ⁄3
− = 2𝑖 sin(𝑚𝛼 − 𝑛𝛽) 𝑓𝑖𝑛𝑑 𝑝𝑜𝑙𝑎𝑟 𝑓𝑜𝑟𝑚 𝑓𝑜𝑟 𝑖 𝑎𝑛𝑑 𝑢𝑠𝑒 𝑑𝑒 𝑀𝑜𝑖𝑣𝑟𝑒 ′ 𝑡ℎ𝑒𝑜𝑟𝑒𝑚
𝑦𝑛 𝑥𝑚
(𝑖𝑣)𝑥 𝑚 = (cos 𝛼 + 𝑖 sin 𝛼)𝑚 = cos 𝑚𝛼 + 𝑖 sin 𝑚𝛼 𝐄𝐗𝐀𝐌𝐏𝐋𝐄 𝟐. 𝟑𝟓 Find all cube roots of √3 + 𝑖.
1⁄
𝑦 𝑛 = (cos 𝛽 + 𝑖 sin 𝛽)𝑛 = cos 𝑛𝛽 + 𝑖 sin 𝑛𝛽 3
Note: (√3 + 𝑖) ; 𝑓𝑖𝑛𝑑 𝑝𝑜𝑙𝑎𝑟 𝑓𝑜𝑟𝑚 𝑓𝑜𝑟 √3 + 𝑖 𝑎𝑛𝑑 𝑢𝑠𝑒 𝑑𝑒 𝑀𝑜𝑖𝑣𝑟𝑒 ′ 𝑡ℎ𝑒𝑜𝑟𝑒𝑚
𝑥 𝑚 𝑦 𝑛 = (cos 𝑚𝛼 + 𝑖 sin 𝑚𝛼)(cos 𝑛𝛽 + 𝑖 sin 𝑛𝛽) 2𝑘𝜋 2𝑘𝜋
= cos(𝑚𝛼 + 𝑛𝛽) + 𝑖 sin(𝑚𝛼 + 𝑛𝛽) 7.Find the value of ∑8𝑘=1 (cos + 𝑖 sin )
9 9
1
𝑚 𝑛
= cos(𝑚𝛼 + 𝑛𝛽) − 𝑖 sin(𝑚𝛼 + 𝑛𝛽) W.K.T,the sum of all the roots Is equal to zero.
𝑥 𝑦 2𝑘𝜋 2𝑘𝜋
𝑚 𝑛
𝑥 𝑦 +
1
= 2 cos(𝑚𝛼 + 𝑛𝛽) ∑8𝑘=0 (cos + 𝑖 sin ) = 0
9 9
𝑥 𝑚𝑦 𝑛 2𝑘𝜋 2𝑘𝜋
8
𝐄𝐗𝐀𝐌𝐏𝐋𝐄 𝟐. 𝟐𝟖 If 𝑧 = cos 𝜃 + 𝑖 sin 𝜃, show that 𝑧 𝑛 +
1
= 2 cos 𝑛𝜃 and ⟹ 𝑐𝑖𝑠(0) + ∑𝑘=1 (cos + 𝑖 sin ) =0
9 9
𝑧𝑛 2𝑘𝜋 2𝑘𝜋
8
1
𝑧 𝑛 − 𝑛 = 2 sin 𝑛𝜃. ⟹ 1 + ∑𝑘=1 (cos + 𝑖 sin ) = 0
𝑧 9 9
2𝑘𝜋
5.Solve the equation 𝑧 3 + 27 = 0 ⟹ ∑8𝑘=1 𝑐𝑖𝑠 ( ) = −1
9
1 1
𝑧 3 + 27 = 0 ⟹ 𝑧 3 = −27 ⟹ 𝑧 = (−27) ⁄3 = (27 × −1) ⁄3 ⟹ 8
∑𝑘=1 (cos
2𝑘𝜋 2𝑘𝜋
+ 𝑖 sin ) = −1
1 1 1 9 9
𝑧 = 3(−1) ⁄3 = 3[𝑐𝑖𝑠(𝜋)] ⁄3 = 3[𝑐𝑖𝑠(2𝑘𝜋 + 𝜋)] ⁄3 , 𝑘 = 0,1,2 8. If 𝜔 ≠ 1 is a cube root of unity, show that
𝜋
= 3𝑐𝑖𝑠(2𝑘 + 1) , 𝑘 = 0,1,2 (𝑖)(1 − 𝜔 + 𝜔2 )6 + (1 + 𝜔 − 𝜔2 )6 = 128
3
𝜋 5𝜋 11
= 3𝑐𝑖𝑠 ( ) , 3𝑐𝑖𝑠(𝜋), 3𝑐𝑖𝑠 ( ) (𝑖𝑖)(1 + 𝜔)(1 + 𝜔2 )(1 + 𝜔4 )(1 + 𝜔8 ) … . (1 + 𝜔2 ) = 1
3 3
6. If 𝜔 ≠ 1 is a cube root of unity, show that the roots of the equation (𝑧 − 1)3 + W.K.T, 𝜔3 = 1 ; 1 + 𝜔 + 𝜔2 = 0
8 = 0 are −1, 1 − 2𝜔, 1 − 2𝜔2 . (𝑖)(1 − 𝜔 + 𝜔2 )6 + (1 + 𝜔 − 𝜔2 )6
(𝑧 − 1)3 + 8 = 0 = (−𝜔 − 𝜔)6 + (−𝜔2 − 𝜔2 )6
⟹ (𝑧 − 1)3 = −8 = (−2𝜔)6 + (−2𝜔2 )6 = 26 𝜔6 + 26 𝜔12 = 64(𝜔3 )2 + 64(𝜔3 )4 = 64 + 64 = 128
11
(𝑧−1)3 (𝑖𝑖)(1 + 𝜔)(1 + 𝜔2 )(1 + 𝜔4 )(1 + 𝜔8 ) … . (1 + 𝜔2 )
⟹ (𝑧 − 1)3 = (−2)3 ⟹ (−2)3
=1 2 3 11
𝑧−1 3
= (1 + 𝜔)(1 + 𝜔2 )(1 + 𝜔2 )(1 + 𝜔2 ) … . (1 + 𝜔2 )
⟹( ) =1 Clearly the above expression contains 12 terms,
−2
1⁄

𝑧−1
= (1) 3 = (1 + 𝜔)(1 + 𝜔2 )(1 + 𝜔4 )(1 + 𝜔8 ) … . .12 𝑡𝑒𝑟𝑚𝑠
−2

V.GNANAMURUGAN, P,G,T, G.H.S.S, S.S.KOTTAI, SIVAGANGAI DT – 94874 43870 ; 82489 56766 , www.tnppgta.com, www.ednnet.in Page 41
= [(1 + 𝜔)(1 + 𝜔2 )][(1 + 𝜔)(1 + 𝜔2 )] … .6 𝑡𝑒𝑟𝑚𝑠 inscribed in the circle |𝑧| = 2. If 𝑧1 = 1 + 𝑖√3, then find 𝑧2 and 𝑧3 .
= [(1 + 𝜔)(1 + 𝜔2 )]6 |𝑧| = 2 represents the circle with centre (0,0) and radius 2.
= (1 + 𝜔 + 𝜔2 + 𝜔3 )6 Let 𝐴, 𝐵 𝑎𝑛𝑑 𝐶 be the vertices of the given triangle. Since the vertices 𝑧1 , 𝑧2 and 𝑧3
= (0 + 1)6 form an equilateral triangle inscribed in the circle |𝑧| = 2, the sides of this triangle
=1 2𝜋
𝐴𝐵, 𝐵𝐶 𝑎𝑛𝑑 𝐶𝐴 subtend radian at the origin.
9.If 𝑧 = 2 − 2𝑖, find the rotation of 𝑧 by 𝜃 radians in the counter clockwise 3
2𝜋 4𝜋
direction about the origin when (𝑖)𝜃 =
𝜋
(𝑖𝑖)𝜃 =
2𝜋
(𝑖𝑖𝑖)𝜃 =
3𝜋 We obtain 𝑧2 and 𝑧3 by the rotation of 𝑧1 by and respectively.
3 3
3 3 2
𝑧 = 2 − 2𝑖 = 𝑟𝑐𝑖𝑠(𝜃) → (1) Given: 𝑧1 = 1 + 𝑖√3
2𝜋 2𝜋 2𝜋
𝑟 = √𝑥 2 + 𝑦 2 = √(2)2 + (−2)2 = √4 + 4 = √4 × 2 = 2√2 𝑧2 = 𝑧1 𝑐𝑖𝑠 ( ) = (1 + 𝑖 √3) [cos ( ) + 𝑖 sin ( )]
3 3 3
𝑦 −2 𝜋
𝛼 = 𝑡𝑎𝑛−1 | | = 𝑡𝑎𝑛−1 | | = = (1 + 𝑖 √3)[cos(180 − 60) + 𝑖 sin(180 − 60)]
𝑥 2 4
𝜋 = (1 + 𝑖 √3)(− cos 60 + 𝑖 sin 60)
Principal argument lies in the 4th quadrant 𝜃 = −𝛼 = −
4 1 √3
𝜋 = (1 + 𝑖 √3) (− + 𝑖 )
(1) ⟹ 𝑧 = 2 − 2𝑖 = 2√2𝑐𝑖𝑠 (− ) 2 2
4
𝜋 𝜋 𝜋 = −2
(𝑖)𝑤ℎ𝑒𝑛 𝜃 = ⟹ 𝑧 = 2√2𝑐𝑖𝑠 (− ) 𝑐𝑖𝑠 ( ) 2𝜋 2𝜋 2𝜋
3 4 3 𝑧3 = 𝑧2 𝑐𝑖𝑠 ( ) = −2 [cos ( ) + 𝑖 sin ( )]
𝜋 𝜋 3 3 3
= 2√2𝑐𝑖𝑠 (− + )
4 3 = −2[cos(180 − 60) + 𝑖 sin(180 − 60)]
−3𝜋+4𝜋
= 2√2𝑐𝑖𝑠 ( ) = −2(− cos 60 + 𝑖 sin 60)
12
𝜋 1 √3
= 2√2𝑐𝑖𝑠 ( ) = −2 (− + 𝑖 )
12 2 2

(𝑖𝑖)𝑤ℎ𝑒𝑛 𝜃 =
2𝜋 𝜋
⟹ 𝑧 = 2√2𝑐𝑖𝑠 (− ) 𝑐𝑖𝑠 ( )
2𝜋 = 1 − 𝑖 √3
3 4 3 4 1
𝜋 2𝜋 10.Prove that the values of √−1 are ± (1 ± 𝑖)
= 2√2𝑐𝑖𝑠 (− + ) √2
4 3 4
−3𝜋+8𝜋 Let 𝑥 = √−1
= 2√2𝑐𝑖𝑠 ( ) 1
12 = (−1) ⁄4
5𝜋 1
= 2√2𝑐𝑖𝑠 ( ) = (𝑐𝑖𝑠𝜋)4
12
3𝜋 𝜋 3𝜋 1
(𝑖𝑖)𝑤ℎ𝑒𝑛 𝜃 = ⟹ 𝑧 = 2√2𝑐𝑖𝑠 (− ) 𝑐𝑖𝑠 ( ) = [𝑐𝑖𝑠(2𝑘𝜋 + 𝜋)]4 , 𝑘 = 0,1,2,3
2 4 2 𝜋
𝜋 3𝜋 = 𝑐𝑖𝑠(2𝑘 + 1) , 𝑘 = 0,1,2,3
= 2√2𝑐𝑖𝑠 (− + ) 4
4 2 𝜋 3𝜋 5𝜋 7𝜋
= 2√2𝑐𝑖𝑠 (
−𝜋+6𝜋
) = 𝑐𝑖𝑠 ( ) , 𝑐𝑖𝑠 ( ) , 𝑐𝑖𝑠 ( ) , 𝑐𝑖𝑠 ( )
4 4 4 4
4
5𝜋 = cos 45 + 𝑖 sin 45 , cos 135 + 𝑖 sin 135 , cos 225 + 𝑖 sin 225 , cos 315 + 𝑖 sin 315
= 2√2𝑐𝑖𝑠 ( ) = cos 45 + 𝑖 sin 45 , cos(90 + 45) + 𝑖 sin(90 + 45) , cos(180 + 45) +
4
𝐄𝐗𝐀𝐌𝐏𝐋𝐄 𝟐. 𝟑𝟔 Suppose 𝑧1 , 𝑧2 and 𝑧3 are the vertices of an equilateral triangle 𝑖 sin(180 + 45) , cos(360 − 45) + 𝑖 sin(360 − 45)

V.GNANAMURUGAN, P,G,T, G.H.S.S, S.S.KOTTAI, SIVAGANGAI DT – 94874 43870 ; 82489 56766 , www.tnppgta.com, www.ednnet.in Page 42
= cos 45 + 𝑖 sin 45 , − cos 45 + 𝑖 sin 45 , − cos 45 − 𝑖 sin 45 , cos 45 − 𝑖 sin 45 Let 𝑧 −1 = 𝑢 + 𝑖𝑣 be the inverse of 𝑧 = 𝑥 + 𝑖𝑦
1 1 1 1 1 1 1 1
= + 𝑖, − + 𝑖, − − 𝑖, − 𝑖 W.K.T, 𝑧𝑧 −1 = 1 ⟹ (𝑥 + 𝑖𝑦)(𝑢 + 𝑖𝑣) = 1
√2 √2 √2 √2 √2 √2 √2 √2
1 ⟹ (𝑥𝑢 − 𝑦𝑣) + 𝑖(𝑥𝑣 + 𝑦𝑢) = 1 + 𝑖0
∴𝑥=± (1 ± 𝑖)
√2 Equating the Real and Imaginary parts on both sides,
𝐄𝐗𝐀𝐌𝐏𝐋𝐄 𝟐. 𝟑𝟐 Find the cube roots of unity. 𝑥𝑢 − 𝑦𝑣 = 1 → (1) ; 𝑥𝑣 + 𝑦𝑢 = 0 → (2)
1 1 𝑥 −𝑦 1 −𝑦 𝑥 1
Let 𝑧 3 = 1 ⟹ 𝑧 = (1) ⁄3 = [𝑐𝑖𝑠(0)] ⁄3 ∆= |𝑦 𝑥 | = 𝑥 2 + 𝑦 2 ; ∆𝑢 = | | = 𝑥; ∆𝑣 = | | = −𝑦
1 0 𝑥 𝑦 0
⟹ 𝑧 = [𝑐𝑖𝑠(2𝑘𝜋 + 0)] ⁄3 , 𝑘 = 0,1,2 ∆𝑢 𝑥 ∆𝑣 −𝑦
2𝑘𝜋 𝑢 = = 2 2 ;𝑣 = = 2 2
⟹ 𝑧 = 𝑐𝑖𝑠 ( ) , 𝑘 = 0,1,2 ∆ 𝑥 +𝑦 ∆ 𝑥 +𝑦
3 −1 −1 𝑥 𝑦
2𝜋 4𝜋 𝑧 = 𝑢 + 𝑖𝑣 ⟹ 𝑧 =( ) + 𝑖 (− 𝑥 2+𝑦2)
⟹ 𝑧 = 𝑐𝑖𝑠(0), 𝑐𝑖𝑠 ( ) , 𝑐𝑖𝑠 ( ) 𝑥 2 +𝑦 2
3 3
𝜋 𝜋 2.For any two complex numbers 𝒛𝟏 𝒂𝒏𝒅 𝒛𝟐 prove that 𝐳̅̅̅̅̅̅̅̅̅̅
𝟏 + 𝐳𝟐 = ̅̅̅
𝐳𝟏 + ̅̅̅
𝐳𝟐
⟹ 𝑧 = 𝑐𝑖𝑠(0), 𝑐𝑖𝑠 (𝜋 − ) , 𝑐𝑖𝑠 (𝜋 + ) PROOF:
3 3
𝜋 𝜋 𝜋 𝜋
⟹ 𝑧 = cos 0 + 𝑖 sin 0 , cos (𝜋 − ) + 𝑖 sin (𝜋 − ) , cos (𝜋 + ) + 𝑖 sin (𝜋 + ) Let 𝑧1 = 𝑎 + 𝑖𝑏 and 𝑧2 = 𝑐 + 𝑖𝑑 , 𝑎, 𝑏, 𝑐, 𝑑 ∈ 𝑅
3 3 3 3
𝜋 𝜋
⟹ 𝑧 = 1, − cos + 𝑖 sin , − cos − 𝑖 sin
𝜋 𝜋 z1 + z2 = ̅̅̅̅̅̅̅̅̅̅̅̅̅̅̅̅̅̅̅̅̅̅̅̅̅
̅̅̅̅̅̅̅̅̅ (𝑎 + 𝑖𝑏) + (𝑐 + 𝑖𝑑)
3 3 3 3 = ̅̅̅̅̅̅̅̅̅̅̅̅̅̅̅̅̅̅̅̅̅̅̅̅
(𝑎 + 𝑐) + 𝑖(𝑏 + 𝑑)
1 √3 1 √3 −1+𝑖 √3 −1−𝑖 √3
⟹ 𝑧 = 1, − + 𝑖 ,− + 𝑖 ⟹ 𝑧 = 1, , = (𝑎 + 𝑐) − 𝑖(𝑏 + 𝑑)
2 2 2 2 2 2
2 −1+𝑖 √3 −1−𝑖 √3 = (𝑎 − 𝑖𝑏) + (𝑐 − 𝑖𝑑)
∴ The cube roots of unity 1, 𝜔, 𝜔 are1, , = z̅1 + z̅2
2 2
𝐄𝐗𝐀𝐌𝐏𝐋𝐄 𝟐. 𝟑𝟑 Find the fourth roots of unity. 3.Prove that ̅̅̅̅̅̅̅
𝐳𝟏 . 𝐳𝟐 = ̅̅̅.
𝐳𝟏 ̅̅̅
𝐳𝟐 𝒘𝒉𝒆𝒓𝒆 𝒂, 𝒃, 𝒄, 𝒅 ∈ 𝑹.
1 1
Let 𝑧 4 = 1 ⟹ 𝑧 = (1) ⁄4 = [𝑐𝑖𝑠(0)] ⁄4 PROOF:
1 Let 𝑧1 = 𝑎 + 𝑖𝑏 and 𝑧2 = 𝑐 + 𝑖𝑑 , 𝑎, 𝑏, 𝑐, 𝑑 ∈ 𝑅
⟹ 𝑧 = [𝑐𝑖𝑠(2𝑘𝜋 + 0)] ⁄4 , 𝑘 = 0,1,2,3
2𝑘𝜋 z1 . z2 = (𝑎
̅̅̅̅̅̅̅ ̅̅̅̅̅̅̅̅̅̅̅̅̅̅̅̅̅̅̅̅̅
+ 𝑖𝑏)(𝑐 + 𝑖𝑑)
⟹ 𝑧 = 𝑐𝑖𝑠 ( ) , 𝑘 = 0,1,2,3 ̅̅̅̅̅̅̅̅̅̅̅̅̅̅̅̅̅̅̅̅̅̅̅̅̅̅̅̅̅̅
4
2𝜋 4𝜋 6𝜋
= (𝑎𝑐 − 𝑏𝑑) + 𝑖(𝑎𝑑 + 𝑏𝑐)
⟹ 𝑧 = 𝑐𝑖𝑠(0), 𝑐𝑖𝑠 ( ) , 𝑐𝑖𝑠 ( ) , 𝑐𝑖𝑠 ( ) = (𝑎𝑐 − 𝑏𝑑) − 𝑖(𝑎𝑑 + 𝑏𝑐) → (1)
4 4 4
𝜋
⟹ 𝑧 = 𝑐𝑖𝑠(0), 𝑐𝑖𝑠 ( ) , 𝑐𝑖𝑠(𝜋), 𝑐𝑖𝑠 ( )
3𝜋 z̅1 . z̅2 = ̅̅̅̅̅̅̅̅
𝑎 + 𝑖𝑏. ̅̅̅̅̅̅̅̅
𝑐 + 𝑖𝑑
2 2 (𝑎
= − 𝑖𝑏)(𝑐 − 𝑖𝑑)
𝜋 𝜋 3𝜋 3𝜋
⟹ 𝑧 = cos 0 + 𝑖 sin 0 , cos ( ) + 𝑖 sin ( ) , cos(𝜋) + 𝑖 sin(𝜋) , cos ( ) + 𝑖 sin ( ) = (𝑎𝑐 − 𝑏𝑑) − 𝑖(𝑎𝑑 + 𝑏𝑐) → (2)
2 2 2 2
⟹ 𝑧 = 1, 𝑖, −1, −𝑖 From (1) and (2), ̅̅̅̅̅̅̅z1 . z2 = z̅1 . z̅2
∴ The fourth roots of unity 1, 𝜔, 𝜔2 , 𝜔3 are 1, 𝑖, −1, −𝑖. 4.Prove that 𝒛 is purely imaginary if and only if 𝒛 = −𝒛̅
1.Prove that the multiplicative inverse of a non- zero complex number 𝒛 = 𝒙 + 𝒊𝒚 PROOF:
is 𝒛−𝟏 = (
𝒙
) + 𝒊 (− 𝒙𝟐 +𝒚𝟐)
𝒚 Let 𝑧 = 𝑥 + 𝑖𝑦 ⟹ 𝑧̅ = 𝑥 − 𝑖𝑦
𝒙𝟐 +𝒚𝟐 𝑧 = −𝑧̅ ⟺ 𝑥 + 𝑖𝑦 = −(𝑥 − 𝑖𝑦)
PROOF: ⟺ 𝑥 + 𝑖𝑦 = −𝑥 + 𝑖𝑦

V.GNANAMURUGAN, P,G,T, G.H.S.S, S.S.KOTTAI, SIVAGANGAI DT – 94874 43870 ; 82489 56766 , www.tnppgta.com, www.ednnet.in Page 43
⟺ 2𝑥 = 0 8. If 𝒛𝟏 = 𝒓𝟏 (𝐜𝐨𝐬 𝜽𝟏 + 𝒊 𝐬𝐢𝐧 𝜽𝟏 ), 𝒛𝟐 = 𝒓𝟐 (𝐜𝐨𝐬 𝜽𝟐 + 𝒊 𝐬𝐢𝐧 𝜽𝟐 ) 𝒕𝒉𝒆𝒏
⟺𝑥=0 𝒛𝟏 𝒛𝟐 = 𝒓𝟏 𝒓𝟐 [𝐜𝐨𝐬(𝜽𝟏 + 𝜽𝟐 ) + 𝒊 𝐬𝐢𝐧(𝜽𝟏 + 𝜽𝟐 )]
⟺ 𝑧 𝑖𝑠 𝑝𝑢𝑟𝑒𝑙𝑦 𝑖𝑚𝑎𝑔𝑖𝑛𝑎𝑟𝑦 PROOF:
5.State and prove triangle inequality of two complex numbers (or) z1 z2 = [𝑟1 (cos 𝜃1 + 𝑖 sin 𝜃1 )][𝑟2 (cos 𝜃2 + 𝑖 sin 𝜃2 )]
For any two complex numbers 𝒛𝟏 𝒂𝒏𝒅 𝒛𝟐 prove that |𝒛𝟏 + 𝒛𝟐 | ≤ |𝒛𝟏 | + |𝒛𝟐 | = 𝑟1 𝑟2 [(cos 𝜃1 cos 𝜃2 − sin 𝜃1 sin 𝜃2 ) + 𝑖(sin 𝜃1 cos 𝜃2 + cos 𝜃1 sin 𝜃2 )]
PROOF: = 𝑟1 𝑟2 [cos(𝜃1 + 𝜃2 ) + 𝑖 sin(𝜃1 + 𝜃2 )]
|𝑧1 + 𝑧2 |2 = (𝑧1 + 𝑧2 )(𝑧 ̅̅̅̅̅̅̅̅̅̅̅̅
1 + 𝑧2 ) (∵ 𝑧𝑧̅ = |𝑧|2 ) 9. If 𝒛𝟏 = 𝒓𝟏 (𝐜𝐨𝐬 𝜽𝟏 + 𝒊 𝐬𝐢𝐧 𝜽𝟏 ), 𝒛𝟐 = 𝒓𝟐 (𝐜𝐨𝐬 𝜽𝟐 + 𝒊 𝐬𝐢𝐧 𝜽𝟐 ) 𝒕𝒉𝒆𝒏
𝒛𝟏 𝒓
= (𝑧1 + 𝑧2 )(𝑧̅1 + 𝑧̅2 ) (∵ ̅̅̅̅̅̅̅̅̅
z1 + z2 = z̅1 + z̅2 ) = 𝟏 [𝐜𝐨𝐬(𝜽𝟏 − 𝜽𝟐 ) + 𝒊 𝐬𝐢𝐧(𝜽𝟏 − 𝜽𝟐 )]
𝒛𝟐 𝒓𝟐
= 𝑧1 𝑧̅1 + (𝑧1 𝑧̅2 + 𝑧̅1 𝑧2 ) + 𝑧2 𝑧̅2 PROOF:
= 𝑧1 𝑧̅1 + (𝑧1 𝑧̅2 + ̅̅̅̅̅̅
𝑧1 𝑧̅2 ) + 𝑧2 𝑧̅2 (∵ 𝑧̅̅ = 𝑧) 𝒛𝟏 𝑟1 (cos 𝜃1 +𝑖 sin 𝜃1)
2
= |𝑧1 | + 2𝑅𝑒(𝑧1 𝑧̅2 ) + |𝑧2 | 2 (∵ 𝑧 + 𝑧̅ = 2𝑅𝑒(𝑧)) =
𝒛𝟐 𝑟2 (cos 𝜃2 +𝑖 sin 𝜃2)
2 2
≤ |𝑧1 | + 2|𝑧1 𝑧̅2 | + |𝑧2 | (∵ 𝑅𝑒(𝑧) ≤ |𝑧|) =
𝑟1 cos 𝜃1 +𝑖 sin 𝜃1
. ×
cos 𝜃2 −𝑖 sin 𝜃2
2 2 𝑟2 cos 𝜃2 +𝑖 sin 𝜃2 cos 𝜃2 −𝑖 sin 𝜃2
= |𝑧1 | + 2|𝑧1 ||𝑧2 | + |𝑧2 | (∵ ̅̅̅̅̅̅̅
z1 . z2 = z̅1 . z̅2 &|𝑧| = |𝑧̅|) 𝑟1 (cos 𝜃1 cos 𝜃2+sin 𝜃1 sin 𝜃2 )+𝒊(sin 𝜃1 cos 𝜃2 −cos 𝜃1 sin 𝜃2 )
2 = .
= (|𝑧1 | + |𝑧2 |) 𝑟2 cos2 (θ2 )+sin2 (θ2 )
Taking square root on both sides, =
𝑟1
[cos(𝜃1 − 𝜃2 ) + 𝑖 sin(𝜃1 − 𝜃2 )]
|𝑧1 + 𝑧2 |2 ≤ |z1 | + |z2 | 𝑟2

6.For any two complex numbers 𝒛𝟏 𝒂𝒏𝒅 𝒛𝟐 prove that |𝐳𝟏 𝐳𝟐 | = |𝐳𝟏 ||𝐳𝟐 | 3. THEORY OF EQUATIONS
PROOF: BASICS OF POLYNOMIAL EQUATIONS
𝑛 𝑛−1
|z1 z2 |2 = (z1 z2 )(z̅̅̅̅̅̅̅̅ (∵ 𝑧𝑧̅ = |𝑧|2 )  𝑎𝑛 𝑥 + 𝑎𝑛−1 𝑥 + ⋯ + 𝑎1 𝑥 + 𝑎0 = 0 is called a polynomial equation.
1 z2 )
= (z1 )(z2 )(z̅1 )(z̅2 ) (∵ ̅̅̅̅̅̅̅
z1 . z2 = z̅1 . z̅2 )  A polynomial with the leading coefficient 1 is called a monic polynomial.
= (z1 𝑧̅1 )(z2 𝑧̅2 )  Polynomial functions are defined for all values of x.
= |𝑧1 |2 |𝑧2 |2  Every non-zero constant is a polynomial of degree 0.
Taking square root on both sides,  The constant 0 is also a polynomial called zero polynomial.
|z1 z2 | = |z1 ||z2 |  The degree of a polynomial is a non- negative integer.
7.If 𝒛 = 𝒓(𝐜𝐨𝐬 𝜽 + 𝒊 𝐬𝐢𝐧 𝜽) then prove that 𝒛−𝟏 = (𝐜𝐨𝐬 𝜽 − 𝒊 𝐬𝐢𝐧 𝜽)
𝟏  The zero polynomial is the only polynomial with leading coefficient 0.
𝒓  Polynomials of degree two are called quadratic polynomials.
PROOF:
 Polynomials of degree three are called cubic polynomials.
𝑧 = 𝑟(cos 𝜃 + 𝑖 sin 𝜃)
1 1  Polynomials of degree four are called quartic polynomials.
𝑧 −1 = =  Formula for finding roots of the quadratic equation 𝑎𝑥 2 + 𝑏𝑥 + 𝑐 = 0 is
𝑧 𝑟(cos 𝜃+𝑖 sin 𝜃)
1 1 cos 𝜃−𝑖 sin 𝜃 −𝑏±√𝑏2 −4𝑎𝑐
= × 𝑥= .
𝑟 cos 𝜃+𝑖 sin 𝜃 cos 𝜃−𝑖 sin 𝜃 2𝑎
1 cos 𝜃−𝑖 sin 𝜃
= .  Discriminant ∆= 𝑏 2 − 4𝑎𝑐
𝑟 𝑐𝑜𝑠 2 𝜃+𝑠𝑖𝑛2 𝜃
1  Nature of the roots of the quadratic equation:
= (cos 𝜃 − 𝑖 sin 𝜃)
𝑟  If ∆> 0, then the roots are real and distinct.
V.GNANAMURUGAN, P,G,T, G.H.S.S, S.S.KOTTAI, SIVAGANGAI DT – 94874 43870 ; 82489 56766 , www.tnppgta.com, www.ednnet.in Page 44
 If ∆< 0, then the roots are not real. (𝑥 + 1), (𝑥 + 2) and (𝑥 + 3) are the length, breadth and height of the cuboid.
 If ∆= 0, then the roots are real and equal. Volume of the cuboid = volume of the cube + 52 cubic units.
 Vieta’s formula for polynomial equation of degree 2: ⟹ (𝑥 + 1)(𝑥 + 2) (𝑥 + 3) = 𝑥 3 + 52
𝑥 2 − (𝛼 + 𝛽)𝑥 + 𝛼𝛽 = 0. (or) ⟹ (𝑥 2 + 3𝑥 + 2)(𝑥 + 3) = 𝑥 3 + 52 [∵ (𝑥 + 𝑎)(𝑥 + 𝑏) = 𝑥 2 + (𝑎 + 𝑏)𝑥 + 𝑎𝑏]
𝑥 2 − (Sum of the roots )𝑥 + (Product of the roots) = 0 ⟹ 𝑥 3 + 3𝑥 2 + 3𝑥 2 + 9𝑥 + 2𝑥 + 6 = 𝑥 3 + 52
 Sum of the roots 𝛼 + 𝛽 = −
𝑏 ⟹ 𝑥 3 + 3𝑥 2 + 3𝑥 2 + 9𝑥 + 2𝑥 + 6 − 𝑥 3 − 52 = 0
𝑐
𝑎 ⟹ 6𝑥 2 + 11𝑥 − 46 = 0
 Product of the roots 𝛼𝛽 = ⟹ (𝑥 − 2)(6𝑥 + 23) = 0
𝑎
 If 𝑃(𝑥) = 0 is a quadratic equation whose roots are 𝛼 𝑎𝑛𝑑 𝛽, then 𝑐𝑃(𝑥) is ⟹ 𝑥 = 2 (𝑜𝑟) 𝑥 = −23 ( 𝑁𝑜𝑡 𝑎𝑑𝑚𝑖𝑠𝑖𝑏𝑙𝑒)
also a quadratic equation with roots 𝛼 𝑎𝑛𝑑 𝛽 for any non-zero constant c. Volume of the cuboid= 𝑥 3 + 52 = 23 + 52 = 8 + 52 = 60 cubic units.
 Fundamental Theorem of Algebra: A polynomial equation of degree n has 2. Construct a cubic equation with roots (𝑖)1,2 𝑎𝑛𝑑 3 𝑖𝑖) 1,1 𝑎𝑛𝑑 − 2
exactly n root in ℂ when the roots are counted with their multiplicities. (𝑖𝑖𝑖)2, −2 𝑎𝑛𝑑4
 Vieta’s formula for polynomial equation of degree 3: (𝑖)1,2 𝑎𝑛𝑑 3
𝑥 3 − (∑ 1)𝑥 2 + (∑ 2)𝑥 − (∑ 3) = 0, where Here 𝛼 = 1, 𝛽 = 2, 𝛾 = 3
𝑏 𝑐 𝑑 ∑1 = 𝛼 + 𝛽 + 𝛾 = 1 + 2 + 3 = 6
∑1 = 𝛼 + 𝛽 + 𝛾 = − ; ∑2 = 𝛼𝛽 + 𝛽𝛾 + 𝛾𝛼 = ; ∑ 3 = 𝛼𝛽𝛾 = −
𝑎 𝑎 𝑎 ∑2 = 𝛼𝛽 + 𝛽𝛾 + 𝛾𝛼 = (1 × 2) + (2 × 3) + (3 × 1) = 2 + 6 + 3 = 11
 Vieta’s formula for polynomial equation of degree 4:
∑3 = 𝛼𝛽𝛾 = 1 × 2 × 3 = 6
𝑥 4 + (∑ 1)𝑥 3 + (∑ 2)𝑥 2 + (∑ 3)𝑥 + (∑ 4) = 0 , where Vieta’s formula for polynomial equation of degree 3:
𝑏
∑1 = 𝛼 + 𝛽 + 𝛾 + 𝛿 = − 𝑥 3 − (∑ 𝛼)𝑥 2 + (∑ 𝛼𝛽)𝑥 − (∑ 𝛼𝛽𝛾) = 0
𝑎
𝑐
∑ 2 = 𝛼𝛽 + 𝛽𝛾 + 𝛾𝛿 + 𝛿𝛼 = ⟹ 𝑥 3 − 6𝑥 2 + 11𝑥 − 6 = 0
𝑎
𝑑 𝑖𝑖) 1,1 𝑎𝑛𝑑 − 2
∑ 3 = 𝛼𝛽𝛾 + 𝛼𝛽𝛿 + 𝛼𝛾𝛿 + 𝛽𝛾𝛿 = −
𝑎 Here 𝛼 = 1, 𝛽 = 1, 𝛾 = −2
𝑒
∑ 4 = 𝛼𝛽𝛾𝛿 = ∑1 = 𝛼 + 𝛽 + 𝛾 = 1 + 1 − 2 = 0
𝑎
 Vieta’s formula for polynomial equation of degree 𝒏 > 𝟑: ∑2 = 𝛼𝛽 + 𝛽𝛾 + 𝛾𝛼 = (1 × 1) + (1 × −2) + (−2 × 1) = 1 − 2 − 2 = −3
A polynomial equation of degree n with roots 𝛼1 , 𝛼2 , … . , 𝛼𝑛 is ∑3 = 𝛼𝛽𝛾 = 1 × 1 × (−2) = −2
𝑥 𝑛 − (∑ 𝛼1 )𝑥 𝑛−1 + (∑ 𝛼1 𝛼2 )𝑥 𝑛−2 − (∑ 𝛼1 𝛼2 𝛼3 )𝑥 𝑛−3 + ⋯ + Vieta’s formula for polynomial equation of degree 3:
(−1)𝑛 𝛼1 𝛼2 … 𝛼𝑛 = 0 𝑥 3 − (∑ 1)𝑥 2 + (∑ 2)𝑥 − (∑ 3) = 0
EXERCISE 3.1 ⟹ 𝑥 3 − 0𝑥 2 − 3𝑥 − (−2) = 0
1. If the sides of a cubic box are increased by 1,2,3 units respectively to form a ⟹ 𝑥 3 − 3𝑥 + 2 = 0
cuboid, then the volume is increased by 52 cubic units. Find the volume of the (𝑖𝑖𝑖)2, −2 𝑎𝑛𝑑 4
cuboid. Here 𝛼 = 2, 𝛽 = −2, 𝛾 = 4
Let 𝑥 be the length of the side of a cube. ∑1 = 𝛼 + 𝛽 + 𝛾 = 2 − 2 + 4 = 4
∑2 = 𝛼𝛽 + 𝛽𝛾 + 𝛾𝛼 = (2 × −2) + (−2 × 4) + (4 × 2) = −4 − 8 + 8 = −4
From the given data,
∑3 = 𝛼𝛽𝛾 = 2 × (−2) × 4 = −16
V.GNANAMURUGAN, P,G,T, G.H.S.S, S.S.KOTTAI, SIVAGANGAI DT – 94874 43870 ; 82489 56766 , www.tnppgta.com, www.ednnet.in Page 45
Vieta’s formula for polynomial equation of degree 3: ∑3 = (−𝛼)(−𝛽)(−𝛾) = −(𝛼𝛽𝛾) = −(−4) = 4
𝑥 3 − (∑ 1)𝑥 2 + (∑ 2)𝑥 − (∑ 3) = 0 ∴ The required cubic polynomial equation is
⟹ 𝑥 3 − 4𝑥 2 − 4𝑥 − (−16) = 0 𝑥 3 − (∑ 1)𝑥 2 + (∑ 2)𝑥 − ∑ 3 = 0
⟹ 𝑥 3 − 4𝑥 2 − 4𝑥 + 16 = 0 ⟹ 𝑥 3 − 2𝑥 2 + 3𝑥 − 4 = 0
3. If 𝛼, 𝛽 𝑎𝑛𝑑 𝛾 are the roots of the cubic equation 𝑥 3 + 2𝑥 2 + 3𝑥 + 4 = 0, form 4. Solve the equation 3𝑥 3 − 16𝑥 2 + 23𝑥 − 6 = 0 if the product of two roots is 1.
1 1 1
a cubic equation whose roots are (𝑖)2𝛼, 2𝛽, 2𝛾 (𝑖𝑖) , , (𝑖𝑖𝑖) − 𝛼, −𝛽 , − 𝛾 Let 𝛼, 𝛽 𝑎𝑛𝑑 𝛾 are the roots of the equation 3𝑥 3 − 16𝑥 2 + 23𝑥 − 6 = 0.
𝛼 𝛽 𝛾 1
Let 𝛼, 𝛽 𝑎𝑛𝑑 𝛾 are the roots of the equation 𝑥 + 2𝑥 + 3𝑥 + 4 = 0. 3 2 The product of two roots is 1⟹ 𝛼𝛽 = 1 ⟹ 𝛽 = → (1)
𝛼
𝑏 2 𝑏 −16 16 16
∑1 = 𝛼 + 𝛽 + 𝛾 = − = − = −2 ∑1 = 𝛼 + 𝛽 + 𝛾 = − = − = ⟹𝛼+𝛽+𝛾 = → (2)
𝑎 1 𝑎 3 3 3
𝑐 3 𝑐 23 23
∑2 = 𝛼𝛽 + 𝛽𝛾 + 𝛾𝛼 = = =3 ∑2 = 𝛼𝛽 + 𝛽𝛾 + 𝛾𝛼 = = ⟹ 𝛼𝛽 + 𝛽𝛾 + 𝛾𝛼 = → (3)
𝑎 1 𝑎 3 3
𝑑 4 𝑑 −6
∑3 = 𝛼𝛽𝛾 = − = − = −4 ∑3 = 𝛼𝛽𝛾 = − = − = 2 ⟹ 𝛼𝛽𝛾 = 2 → (4)
𝑎 1 𝑎 3
(i)Roots are 2𝛼, 2𝛽, 2𝛾 From (1) & (4) ⟹ (1)𝛾 = 2 ⟹ 𝛾 = 2
∑1 = 2𝛼 + 2𝛽 + 2𝛾 = 2(𝛼 + 𝛽 + 𝛾) = 2(−2) = −4 (2) ⟹ 𝛼 + 𝛽 + 2 =
16
⟹𝛼+𝛽 =
10
∑2 = (2𝛼)(2𝛽) + (2𝛽)(2𝛾) + (2𝛾)(2𝛼) = 4(𝛼𝛽 + 𝛽𝛾 + 𝛾𝛼) = 4(3) = 12 3 3
1 10
∑3 = (2𝛼)(2𝛽)(2𝛾) = 8(𝛼𝛽𝛾) = 8 × (−4) = −32 ⟹𝛼+ = (𝐹𝑟𝑜𝑚 (1))
𝛼 3
∴ The required cubic polynomial equation is 𝛼 2 +1 10 1
⟹ = ⟹ 3𝛼 2 − 10𝛼 + 3 = 0 ⟹ (𝛼 − 3)(3𝛼 − 1) = 0 ⟹ 𝛼 = 3 (𝑜𝑟)
𝑥 3 − (∑ 1)𝑥 2 + (∑ 2)𝑥 − (∑ 3) = 0 𝛼 3 3
1 1
⟹ 𝑥 3 − (−4)𝑥 2 + 12𝑥 − (−32) = 0 If 𝛼 = 3 (𝑜𝑟) , (1) ⟹ 𝛽 = (𝑜𝑟)3
3 3
⟹ 𝑥 3 + 4𝑥 2 + 12𝑥 + 32 = 0 1
1 1 1
(ii) Roots are , , Thus the roots are 3, , 2
3
𝛼 𝛽 𝛾
1 1 1 𝛼𝛽+𝛽𝛾+𝛾𝛼 3 5.Find the sum of the squares of root of the equation 2𝑥 4 − 8𝑥 3 + 6𝑥 2 − 3 = 0.
∑1 = + + = =− Let 𝛼, 𝛽 , 𝛾, 𝛿 are the roots of the equation 2𝑥 4 − 8𝑥 3 + 6𝑥 2 − 3 = 0
𝛼 𝛽 𝛾 𝛼𝛽𝛾 4
1 1 1 1 1 1 1 1 1 𝛼+𝛽+𝛾 −2 1 𝑎 = 2, 𝑏 = −8, 𝑐 = 6, 𝑑 = 0, 𝑒 = −3
∑2 = ( × ) + ( × ) + ( × ) = + + = = =
𝛼 𝛽 𝛽 𝛾 𝛾 𝛼 𝛼𝛽 𝛽𝛾 𝛾𝛼 𝛼𝛽𝛾 −2 2 𝑏 −8
1 1 1 1 1 ∑1 = 𝛼 + 𝛽 + 𝛾 + 𝛿 = − = − = 4
∑3 = × × = =− 𝑎 2
𝑐 6
𝛼 𝛽 𝛾 𝛼𝛽𝛾 4
∑2 = 𝛼𝛽 + 𝛼𝛾 + 𝛼𝛿 + 𝛽𝛾 + 𝛽𝛿 + 𝛾𝛼 = = = 3
∴ The required cubic polynomial equation is 𝑎 2
𝑥 3 − (∑ 1)𝑥 2 + (∑ 2)𝑥 − (∑ 3) = 0 W.K.T,
3 1 1 (𝛼 + 𝛽 + 𝛾 + 𝛿)2 = 𝛼 2 + 𝛽 2 + 𝛾 2 + 𝛿 2 − 2 ∑ 𝛼𝛽
⟹ 𝑥 3 − (− ) 𝑥 2 + 𝑥 − (− ) = 0
4 2 4 ⟹ 𝛼 2 + 𝛽 2 + 𝛾 2 + 𝛿 2 = (𝛼 + 𝛽 + 𝛾 + 𝛿)2 − 2 ∑ 𝛼𝛽
⟹ 4𝑥 3 + 3𝑥 2 + 2𝑥 + 1 = 0 ⟹ 𝛼 2 + 𝛽 2 + 𝛾 2 + 𝛿 2 = (4)2 − 2(3) = 16 − 6 = 10
(iii) Roots are −𝛼, −𝛽 , − 𝛾 𝐸𝑋𝐴𝑀𝑃𝐿𝐸 3.4 Find the sum of the squares of roots of 𝑎𝑥 4 + 𝑏𝑥 3 + 𝑐𝑥 2 + 𝑑𝑥 +
∑1 = −(𝛼 + 𝛽 + 𝛾) = − (−2) = 2 𝑒 = 0.
∑2 = (−𝛼)(−𝛽) + (−𝛽)(−𝛾) + (−𝛾)(−𝛼) = 𝛼𝛽 + 𝛽𝛾 + 𝛾𝛼 = 3
V.GNANAMURUGAN, P,G,T, G.H.S.S, S.S.KOTTAI, SIVAGANGAI DT – 94874 43870 ; 82489 56766 , www.tnppgta.com, www.ednnet.in Page 46
6. Solve the equation 𝑥 3 − 9𝑥 2 + 14𝑥 + 24 = 0 if it is given that two of its roots 𝐸𝑋𝐴𝑀𝑃𝐿𝐸 3.3 If 𝛼, 𝛽 𝑎𝑛𝑑 𝛾 are the roots of the equation 𝑥 3 + 𝑝𝑥 2 + 𝑞𝑥 + 𝑟 =
are in the ratio 3: 2. 1
0, find the value of ∑ in terms of the coefficients.
𝛽𝛾
Let 𝛼, 𝛽 𝑎𝑛𝑑 𝛾 are the roots of the equation 𝑥 3 − 9𝑥 2 + 14𝑥 + 24 = 0.
Two of its roots are in the ratio 3: 2. So, the roots are 𝛼, 𝛽 = 3𝑝 , 𝛾 = 2𝑝 8. . If 𝛼, 𝛽, 𝛾 𝑎𝑛𝑑 𝛿 are the roots of the polynomial equation 2𝑥 4 + 5𝑥 3 − 7𝑥 2 +
𝑏 −9 +8 = 0 , find a quadratic equation with integer coefficient whose root are
∑1 = 𝛼 + 𝛽 + 𝛾 + 𝛿 = − = − = 9 ⟹ 𝛼 + 3𝑝 + 2𝑝 = 9
𝑎 1 𝛼 + 𝛽 + 𝛾 + 𝛿 and 𝛼𝛽𝛾𝛿
⟹ 𝛼 = 9 − 5𝑝 → (1) If 𝛼, 𝛽, 𝛾 𝑎𝑛𝑑 𝛿 are the roots of the equation 2𝑥 4 + 5𝑥 3 − 7𝑥 2 + +8 = 0
𝑐 14
∑2 = 𝛼𝛽 + 𝛽𝛾 + 𝛾𝛼 = = ⟹ 𝛼𝛽 + 𝛽𝛾 + 𝛾𝛼 = 14 𝑎 = 2, 𝑏 = 5, 𝑐 = −7, 𝑑 = 0, 𝑒 = 8
𝑎 1
𝑏 5
⟹ (9 − 5𝑝)3𝑝 + (3𝑝)(2𝑝) + (2𝑝)(9 − 5𝑝) = 14 ∑1 = 𝛼 + 𝛽 + 𝛾 + 𝛿 = − = −
𝑎 2
⟹ 27𝑝 − 15𝑝2 + 6𝑝2 + 18𝑝 − 10𝑝2 − 14 = 0 𝑒 8
∑4 = 𝛼𝛽𝛾𝛿 = = = 4
⟹ −19𝑝2 + 45𝑝 − 14 = 0 𝑎 2
5 3
⟹ 19𝑝2 − 45𝑝 + 14 = 0 Sum of the roots = − + 4 =
2 2
⟹ 19𝑝2 − 38𝑝 − 7𝑝 + 14 = 0 Product of the roots = (− ) 4 = −10
5
⟹ 19𝑝(𝑝 − 2) − 7(𝑝 − 2) = 0 2
⟹ (𝑝 − 2)(19𝑝 − 7) = 0 The required quadratic equation is
7 𝑥 2 − (Sum of the roots )𝑥 + (Product of the roots) = 0
⟹ 𝑝 = 2 (𝑜𝑟) (𝑁𝑜𝑡 𝑎𝑑𝑚𝑖𝑠𝑖𝑏𝑙𝑒) 3
19
⟹ 𝑥 2 − ( ) 𝑥 + (−10) = 0
Thus the roots are 9 − 5𝑝, 3𝑝, 2𝑝 = −1,6,4. 2
2
7. If 𝛼, 𝛽 𝑎𝑛𝑑 𝛾 are the roots of the polynomial equation 𝑎𝑥 3 + 𝑏𝑥 2 + 𝑐𝑥 + 𝑑 = ⟹ 2𝑥 − 3𝑥 − 20 = 0
𝛼 𝑝 𝑞 √𝑛
0, find the value of ∑ in terms of the coefficients. 9.If p and are roots of the equation 𝑙𝑥 2 + 𝑛𝑥 + 𝑛 = 0, show that √ + √ + =
𝛽𝛾 𝑞 𝑝 √𝑙
𝛼, 𝛽 𝑎𝑛𝑑 𝛾 are the roots of the equation 𝑎𝑥 3 + 𝑏𝑥 2 + 𝑐𝑥 + 𝑑 = 0 0.
𝑏 𝑛 𝑛
∑1 = 𝛼 + 𝛽 + 𝛾 = − 𝑝+𝑞 =− ; 𝑝𝑞 =
𝑎 𝑙 𝑙
𝑐
∑2 = 𝛼𝛽 + 𝛽𝛾 + 𝛾𝛼 = 𝑝 𝑞 √𝑝 √𝑞 𝑝+𝑞 −𝑛⁄𝑙 √𝑛
𝑑
𝑎 √𝑞 + √𝑝 = + = = = −√𝑛⁄𝑙 = −
∑3 = 𝛼𝛽𝛾 = − √𝑞 √𝑝 √𝑝𝑞 √𝑛⁄𝑙
√𝑙
𝑎
𝛼 𝛼 𝛽 𝛾 𝛼 2 +𝛽 2 +𝛾2 (𝛼+𝛽+𝛾)2 −2 ∑ 𝛼𝛽 𝑝 𝑞 √𝑛 √𝑛 √𝑛
∑1 = ∑ = + + = = √𝑞 + √𝑝 + =− + =0
𝛽𝛾 𝛽𝛾 𝛼𝛾 𝛼𝛽 𝛼𝛽𝛾 𝛼𝛽𝛾 √𝑙 √𝑙 √𝑙
𝑏 2 𝑐
(− ) −2( )
𝑎 𝑎 10.If the equations 𝑥 + 𝑝𝑥 + 𝑞 = 0 and 𝑥 2 + 𝑝′ 𝑥 + 𝑞 ′ = 0 have a common root,
2
= 𝑑 𝑝𝑞 ′ −𝑝′ 𝑞 𝑞−𝑞 ′

𝑎 show that it must be equal to or
𝑏2 2𝑐 𝑎 𝑞−𝑞 ′ 𝑝′ −𝑝
=( − ) (− 𝑑) 2 2 ′ ′
𝑥 + 𝑝𝑥 + 𝑞 = 0 and 𝑥 + 𝑝 𝑥 + 𝑞 = 0 have a common root. Let 𝛼 be the
𝑎2 𝑎
𝑏2 −2𝑎𝑐 𝑎 common root of the above equations. So the equations become 𝛼 2 + 𝑝𝛼 + 𝑞 =
=( ) (− 𝑑)
𝑎2
2𝑎𝑐−𝑏2 0 → (1) and 𝛼 2 + 𝑝′ 𝛼 + 𝑞 ′ = 0 → (2)
= Solving (1) and (2) by cross multiplication method,
𝑎𝑑

V.GNANAMURUGAN, P,G,T, G.H.S.S, S.S.KOTTAI, SIVAGANGAI DT – 94874 43870 ; 82489 56766 , www.tnppgta.com, www.ednnet.in Page 47
−73−86
α2 α 1 = +4
𝑝 𝑞 1 𝑝 17
−159+68
𝑝′ 𝑞 ′ 1 𝑝′ =
17
𝛼2 𝛼 1 91
⟹ = = =−
𝑝𝑞 ′ −𝑝′ 𝑞 𝑞−𝑞 ′ 𝑝′ −𝑝 17
𝛼2 𝛼 𝛼 1 The required quadratic equation is
⟹ = (or) = 𝑥 2 − (Sum of the roots )𝑥 + (Product of the roots) = 0
𝑝𝑞 ′ −𝑝′ 𝑞 𝑞−𝑞 ′ 𝑞−𝑞 ′ 𝑝′ −𝑝
′ ′
𝑝𝑞 −𝑝 𝑞 𝑞−𝑞 ′ 25 91
⟹𝛼 = (or) 𝛼 = ⟹ 𝑥 2 − ( ) 𝑥 + (− ) = 0 ⟹ 17𝑥 2 − 25𝑥 − 91 = 0
𝑞−𝑞 ′ 𝑝′ −𝑝 17 17
11. Formulate into a mathematical problem to find a number such that when it 𝐸𝑋𝐴𝑀𝑃𝐿𝐸 3.2 If 𝛼 𝑎𝑛𝑑 𝛽 are the roots of the quadratic equation 2𝑥 2 − 7𝑥 +
cube root is added to it, the result is 6. 13 = 0 , construct a quadratic equation whose roots are 𝛼 2 𝑎𝑛𝑑 𝛽 2.
7 13
Let 𝑥 be the required number. 𝛼+𝛽 = ; 𝛼𝛽 =
3 3 2 2
Given: 𝑥 + √𝑥 = 6 ⟹ √𝑥 = 6 − 𝑥 7 2 13 49 3
Raising the power to 3 on both sides, 𝑥 = (6 − 𝑥)3 Sum of the roots = 𝛼 + 𝛽 = (𝛼 + 𝛽)2 − 2𝛼𝛽 = ( ) − 2 ( ) =
2 2
− 13 = −
2 2 4 4
⟹ 𝑥 = (6)3 − 3(6)2 𝑥 + 3(6)𝑥 2 − 𝑥 3 13 2 169
Product of the roots= 𝛼 2 𝛽 2 = ( 𝛼𝛽)2 = ( ) =
⟹ 𝑥 = 216 − 108𝑥 + 18𝑥 2 − 𝑥 3 2 4
The required quadratic equation is
⟹ 𝑥 3 − 18𝑥 2 + 109𝑥 − 216 = 0
12. A 12 metre tall tree was broken into two parts. It was found that the height of 𝑥 2 − (Sum of the roots )𝑥 + (Product of the roots) = 0
3 169
the part which was left standing was the cube root of the length of the part that ⟹ 𝑥 2 − (− ) 𝑥 + ( ) = 0 ⟹ 4𝑥 2 + 3𝑥 + 169 = 0
4 4
was cut away. Formulate this into a mathematical problem to find the height of 𝐸𝑋𝐴𝑀𝑃𝐿𝐸 3.5 Find the condition that the roots of 𝑥 3 + 𝑎𝑥 2 + 𝑏𝑥 + 𝑐 = 0are in
the part which was cut away. the ratio 𝑝: 𝑞: 𝑟.
Let 𝑥 3 be the length of the part of the tree standing and 𝑥 be the length of the part Since the roots are in the ratio 𝑝: 𝑞: 𝑟, then the roots are 𝑝𝜆, 𝑞𝜆, 𝑟𝝀.
of the tree cut away. 𝑎 a
∑ 1 = 𝑝𝜆 + 𝑞𝜆 + 𝑟𝝀 = − ⟹ λ(p + q + r) = −a ⟹ λ = − → (1)
1 p+q+r
From the given data, 𝑥 3 + 𝑥 = 12 ⟹ 𝑥 3 + 𝑥 − 12 = 0 𝑐 3 3 𝑐
∑3 = (𝑝𝜆)(𝑞𝜆)(𝑟𝝀) = − ⟹ 𝝀 𝑝𝑞𝑟 = −𝑐 ⟹ 𝝀 = − → (2)
𝐸𝑋𝐴𝑀𝑃𝐿𝐸 3.1 If 𝛼 𝑎𝑛𝑑 𝛽 are the roots of the quadratic equation 17𝑥 2 + 43𝑥 − 1 𝑝𝑞𝑟
3
73 = 0 , construct a quadratic equation whose roots are 𝛼 + 2 𝑎𝑛𝑑 𝛽 + 2. Sub. (1) in (2) ⟹ (−
a
) = − 𝑝𝑞𝑟
𝑐
43 73 p+q+r
𝛼+𝛽 =− ; 𝛼𝛽 = − 𝑎3 𝑐
17 17
43 25 ⟹ − (p+q+r)3 = − ⟹ 𝑝𝑞𝑟𝑎3 = 𝑐(p + q + r)3
Sum of the roots = 𝛼 + 2 + 𝛽 + 2 = 𝛼 + 𝛽 + 4 = − + 4 = 𝑝𝑞𝑟
17 17
Product of the roots= (𝛼 + 2)(𝛽 + 2) 𝐸𝑋𝐴𝑀𝑃𝐿𝐸 3.6 Form the equation whose roots are the squares of the roots of
= 𝛼𝛽 + 2𝛼 + 2𝛽 + 4 the cubic equation 𝑥 3 + 𝑎𝑥 2 + 𝑏𝑥 + 𝑐 = 0.
= 𝛼𝛽 + 2(𝛼 + 𝛽) + 4 Let 𝛼, 𝛽 𝑎𝑛𝑑 𝛾 are the roots of the equation 𝑥 3 + 𝑎𝑥 2 + 𝑏𝑥 + 𝑐 = 0.
𝑎
73 43 ∑1 = 𝛼 + 𝛽 + 𝛾 = − = −𝑎
= − + 2 (− ) + 4 1
17 17 𝑏
∑2 = 𝛼𝛽 + 𝛽𝛾 + 𝛾𝛼 = = 𝑏
1

V.GNANAMURUGAN, P,G,T, G.H.S.S, S.S.KOTTAI, SIVAGANGAI DT – 94874 43870 ; 82489 56766 , www.tnppgta.com, www.ednnet.in Page 48
𝑐
∑3 = 𝛼𝛽𝛾 = −
1
= −𝑐  √∆ is rational ⟺ m and n are perfect squares.
Now the roots are 𝛼 2 , 𝛽 2 , 𝛾 2  √∆ is irrational ⟺ at least one of m or n is not a perfect square
∑1 = 𝛼 2 + 𝛽 2 + 𝛾 2 = (𝛼 + 𝛽 + 𝛾)2 − 2(𝛼𝛽 + 𝛽𝛾 + 𝛾𝛼) = 𝑎2 − 2𝑏  If p and q are rational numbers such that √𝑞 is irrational. If 𝑝 + √𝑞 is a root
∑2 = 𝛼 2 𝛽 2 + 𝛽 2 𝛾 2 + 𝛾 2 𝛼 2 = (𝛼𝛽 + 𝛽𝛾 + 𝛾𝛼)2 − 2𝛼𝛽𝛾(𝛼 + 𝛽 + 𝛾) of a quadratic equation with rational coefficients, then 𝑝 − √𝑞 is also a root
= 𝑏 2 − 2(−𝑐)(−𝑎) = 𝑏 2 − 2𝑐𝑎 of the same equation.
∑3 = 𝛼 2 𝛽 2 𝛾 2 = (𝛼𝛽𝛾)2 = (−𝑐)2 = 𝑐 2
 If p and q be rational numbers so that √𝑝 and √𝑞 are irrational numbers;
Vieta’s formula for polynomial equation of degree 3:
𝑥 3 − (∑ 1)𝑥 2 + (∑ 2)𝑥 − (∑ 3) = 0 further let one of √𝑝 and √𝑞 be not a rational multiple of the other. If
𝑥 3 − (𝑎2 − 2𝑏 )𝑥 2 + (𝑏2 − 2𝑐𝑎)𝑥 − 𝑐 2 = 0 √𝑝 + √𝑞 is a root of a polynomial equation with rational coefficients, then
𝐸𝑋𝐴𝑀𝑃𝐿𝐸 3.7 If p is real, discuss the nature of the roots of the equation √𝑝 − √𝑞, − √𝑝 + √𝑞 and −√𝑝 − √𝑞 are also roots of the same
4𝑥 2 + 4𝑝𝑥 + 𝑝 + 2 = 0, in terms of p. polynomial equation.
Discriminant ∆= 𝑏 2 − 4𝑎𝑐 EXERCISE 3.2
= (4𝑝)2 − 4(4)(𝑝 + 2) 1.If k is real, discuss the nature of the roots of the polynomial equation
= 16𝑝2 − 16𝑝 − 32 2𝑥 2 + 𝑘𝑥 + 𝑘 = 0, in terms of k.
= 16(𝑝2 − 𝑝 − 2) Here 𝑎 = 2, 𝑏 = 𝑘, 𝑐 = 𝑘
= 16(𝑝 + 1)(𝑝 − 2) ∆= 𝑏 2 − 4𝑎𝑐 = 𝑘 2 − 8𝑘 = 𝑘(𝑘 − 8)
∆< 0 𝑖𝑓 − 1 < 𝑝 < 2 If ∆= 0, then 𝑘(𝑘 − 8) = 0 ⟹ 𝑘 = 0 𝑜𝑟 𝑘 = 8
∆= 0 𝑖𝑓 𝑝 = −1 𝑜𝑟 𝑝 = 2 When 𝑘 <0 ⟹ ∆> 0, then the equation has real roots.
∆> 0 𝑖𝑓 − ∞ < 𝑝 < −1 𝑜𝑟 2 < 𝑝 < ∞ When 𝑘 = 0 , 𝑘 = 8 ⟹ ∆= 0, then the equation has equal roots.
Thus the given polynomial has When 0 < 𝑘 <8 ⟹ ∆< 0, then the equation has imaginary roots.
Imaginary roots 𝑖𝑓 − 1 < 𝑝 < 2 ; When 𝑘 >8 ⟹ ∆> 0, then the equation has real and distinct roots.
Equal and real roots 𝑖𝑓 𝑝 = −1 𝑜𝑟 𝑝 = 2; 2.Find the polynomial equation of minimum degree with rational coefficients,
Distinct real roots 𝑖𝑓 − ∞ < 𝑝 < −1 𝑜𝑟 2 < 𝑝 < ∞.
having 2 + √3𝑖 as a root.
NATURE OF ROOT AND NATURE OF COEFFICIENTS OF POLYNOMIAL EQUATIONS
Given: One root 𝛼 = 2 + √3𝑖
 COMPLEX CONJUGATE ROOT THEOREM: If a complex number 𝑧0 is a root of
a polynomial equation with real coefficients, then its complex conjugate 𝑧̅0 ∴ Another root 𝛽 = 2 − √3𝑖
is also a root. 𝛼 + 𝛽 = 2 + √3𝑖 + 2 − √3𝑖 = 4
 Non-real complex roots or imaginary roots occur as conjugate pairs. 𝛼𝛽 = (2 + √3𝑖)(2 − √3𝑖) = 4 + 3 = 7
 IRRATIONAL ROOTS: ∴ The required polynomial is 𝑥 2 − (𝛼 + 𝛽 )𝑥 + (𝛼𝛽) = 0 ⟹ 𝑥 2 − 4𝑥 + 7 = 0
 If √∆ is rational, then both the roots are rational. 𝐸𝑋𝐴𝑀𝑃𝐿𝐸 3.8 Find the monic polynomial equation of minimum degree with real
 If √∆ is irrational, then both the roots are irrational. coefficients having 2 − √3𝑖 as a root.
𝑚
 If ∆= for some positive integers m and n with G.C.D of m, n=1. 𝐸𝑋𝐴𝑀𝑃𝐿𝐸 3.9 Find the polynomial equation of minimum degree with rational
𝑛

V.GNANAMURUGAN, P,G,T, G.H.S.S, S.S.KOTTAI, SIVAGANGAI DT – 94874 43870 ; 82489 56766 , www.tnppgta.com, www.ednnet.in Page 49
coefficients, having 2 − √3 as a root. Sub. (2) in (1) ⟹ 𝑥 2 + (𝑚𝑥 + 𝑐)2 = 𝑟 2
3. Find the polynomial equation of minimum degree with rational coefficients, ⟹ 𝑥 2 + 𝑚2 𝑥 2 + 2𝑚𝑐𝑥 + 𝑐 2 = 𝑟 2
having 2𝑖 + 3 as a root. ⟹ (1 + 𝑚2 )𝑥 2 + 2𝑚𝑐𝑥 + (𝑐 2 − 𝑟 2 ) = 0
One root 𝛼 = 3 + 2𝑖 This is a quadratic equation which cannot have more than two solutions. Hence a
∴ Another root 𝛽 = 3 − 2𝑖 circle and a line cannot intersect at more than two points.
𝛼 + 𝛽 = 3 + 2𝑖 + 3 − 2𝑖 = 6 √2
𝐸𝑋𝐴𝑀𝑃𝐿𝐸 3.10 Form the polynomial equation with integer coefficients with √
𝛼𝛽 = (3 + 2𝑖)(3 − 2𝑖) = 9 + 4 = 13 √3
∴ The required polynomial is 𝑥 2 − (𝛼 + 𝛽 )𝑥 + (𝛼𝛽) = 0 ⟹ 𝑥 2 − 6𝑥 + 13 = 0 as a root.
4. Find the polynomial equation of minimum degree with rational coefficients, √2 √2 √2
Since √ is a root, (𝑥 − √ ) is a factor. So (𝑥 + √ ) is another factor.
having √5 − √3 as a root. √3 3√ 3 √
2
Since √5 − √3 is a root. 𝑥 = √5 − √3 2 2 √2 √2
Product of the factors = (𝑥 − √√ ) (𝑥 + √√ ) =𝑥 − 2
(√ 3) = 𝑥2 −
⟹ 𝑥 + √3 = √5 √3 √3 √ √3
2 2
Squaring on both sides, (𝑥 + √3) = √5 √2 √2
Now (𝑥 2 − ) is one factor. So (𝑥 2 + ) is another factor.
√3 √3
⟹ 𝑥 2 + 2√3𝑥 + 3 = 5 2
√2 √2 √2 2
⟹ 𝑥 2 − 2 = 2√3𝑥 Again product of the factors= (𝑥 2 − ) (𝑥 2 + ) = (𝑥 2 )2 − ( ) = 𝑥 4 −
√3 √3 √ 3 3
2 4
Again squaring on both sides, (𝑥 2 − 2)2 = (2√3𝑥) ⟹ 3𝑥 − 2 = 0 is the required polynomial equation with integer coefficients.
⟹ 𝑥 4 − 4𝑥 2 + 4 = 12𝑥 2 𝐸𝑋𝐴𝑀𝑃𝐿𝐸 3.11 Show that the equation 2𝑥 2 − 6𝑥 + 7 = 0 cannot be satisfied
⟹ 𝑥 4 − 16𝑥 2 + 4 = 0 by any real values of x.
5.Prove that a straight line and parabola cannot intersect at more than two points. Here 𝑎 = 2, 𝑏 = −6, 𝑐 = 7
Equation of the straight line is 𝑦 = 𝑚𝑥 + 𝑐 → (1) ∆= 𝑏 2 − 4𝑎𝑐 = (−6)2 − 4(2)(7) = 36 − 56 = −20 < 0
Equation of the parabola is 𝑦 2 = 4𝑎𝑥 → (2) ∴ The roots are imaginary numbers.
Sub. (1) in (2) ⟹ (𝑚𝑥 + 𝑐)2 = 4𝑎𝑥 𝐸𝑋𝐴𝑀𝑃𝐿𝐸 3.12 If 𝑥 2 + 2(𝑘 + 2)𝑥 + 9𝑘 = 0 has equal roots, find k.
⟹ 𝑚2 𝑥 2 + 2𝑚𝑐𝑥 + 𝑐 2 = 4𝑎𝑥 Here 𝑎 = 1, 𝑏 = 2𝑘 + 4, 𝑐 = 9𝑘
⟹ 𝑚2 𝑥 2 + 2𝑚𝑐𝑥 + 𝑐 2 − 4𝑎𝑥 = 0 ∆= 0 ⟹ 𝑏 2 − 4𝑎𝑐 = 0
⟹ 𝑚2 𝑥 2 + (2𝑚𝑐 − 4𝑎)𝑥 + 𝑐 2 = 0 ⟹ (2𝑘 + 4)2 − 4(1)(9𝑘) = 0
This is a quadratic equation which cannot have more than two solutions. Hence a ⟹ 4𝑘 2 + 16𝑘 + 16 − 36𝑘 = 0
line and a parabola cannot intersect at more than two points. ⟹ 4𝑘 2 − 20𝑘 + 16 = 0
𝐸𝑋𝐴𝑀𝑃𝐿𝐸 3.14 Prove that a line cannot intersect a circle at more than two ⟹ 𝑘 2 − 5𝑘 + 4 = 0 ⟹ (𝑘 − 1)(𝑘 − 4) = 0 ⟹ 𝑘 = 1 𝑜𝑟 4
points. 𝐸𝑋𝐴𝑀𝑃𝐿𝐸 3.13 Show that if p, q, r are rational, the roots of the equation
Equation of the circle is 𝑥 2 + 𝑦 2 = 𝑟 2 → (1) 𝑥 2 − 2𝑝𝑥 + 𝑝2 − 𝑞 2 + 2𝑞𝑟 − 𝑟 2 = 0 are rational.
Equation of the straight line is 𝑦 = 𝑚𝑥 + 𝑐 → (2) Here 𝑎 = 1, 𝑏 = −2𝑝, 𝑐 = 𝑝2 − 𝑞 2 + 2𝑞𝑟 − 𝑟 2
∆= 𝑏 2 − 4𝑎𝑐 = (−2𝑝)2 − 4(1)(𝑝2 − 𝑞 2 + 2𝑞𝑟 − 𝑟 2 )
V.GNANAMURUGAN, P,G,T, G.H.S.S, S.S.KOTTAI, SIVAGANGAI DT – 94874 43870 ; 82489 56766 , www.tnppgta.com, www.ednnet.in Page 50
= 4𝑝2 − 4𝑝2 + 4𝑞 2 − 8𝑞𝑟 + 4𝑟 2 ∑1 = 𝛼 − 𝛼 + 𝛽 = − = −
𝑏 −1 1
= ⟹𝛽=
1
𝑎 2 2 2
= 4𝑞 2 − 8𝑞𝑟 + 4𝑟 2 𝑐 18
= 4(𝑞2 − 2𝑞𝑟 + 𝑟 2 ) ∑2 = 𝛼(−𝛼) + (−𝛼)(𝛽) + (𝛽)(𝛼) = =−
𝑎 2
= [2(𝑞 − 𝑟)]2 is a perfect square. ⟹ −𝛼 2 − 𝛼𝛽 + 𝛼𝛽 = −9
Hence the roots are rational. ⟹ 𝛼2 = 9
ROOTS OF HIGHER DEGREE POLYNOMIAL EQUATIONS ⟹ 𝛼 = ±3
1
 IMAGINARY OR SURDS ROOTS: Thus the roots are 3, −3,
2
i. If 𝛼 + 𝑖𝛽 is an imaginary root of a quartic polynomial with real 2. Solve the equation: 9𝑥 3 − 36𝑥 2 + 44𝑥 − 16 = 0 if the roots form an arithmetic
coefficients, then 𝛼 − 𝑖𝛽 is also a root. progression.
ii. Thus[𝑥 − (𝛼 + 𝑖𝛽)] and [𝑥 − (𝛼 − 𝑖𝛽)] are the factors of the polynomial. Let (𝑎 − 𝑑), 𝑎, (𝑎 + 𝑑) are the roots of 9𝑥 3 − 36𝑥 2 + 44𝑥 − 16 = 0. (∵ 𝐴. 𝑃)
iii. Divide the polynomial with the product of the factors 𝑥 2 − 2𝛼𝛽𝑥 + 𝛼 2 + ∑1 = 𝑎 − 𝑑 + 𝑎 + 𝑎 + 𝑑 = − = −
𝑏 −36
= 4 ⟹ 3𝑎 = 4 ⟹ 𝑎 =
4
𝛽 2 and reduce the polynomial to quadratic form. 𝑎 9 3
9 −36 44 −16
iv. solving the quadratic equation find the other roots. 4
0 12 −32 16
 POLYNOMIAL EQUATIONS WITH EVEN POWERS ONLY: 3
9 −24 12 0
If 𝑃(𝑥) is a polynomial equation of degree 2n, having only even powers 1 2
Quotient = (9𝑥 − 24𝑥 + 12)
i. Replace 𝑥 2 by 𝑦 (𝑖𝑒. , 𝑥 2 = 𝑦) and solve the equation to find 𝑦. 3
= 3𝑥 2 − 8𝑥 + 4
ii. Substituting the values of 𝑦 in 𝑥 2 = 𝑦 and find the roots.
= 3𝑥 2 − 6𝑥 − 2𝑥 + 4
 If the sum of all coefficients is zero, then 1 is a root of 𝑃(𝑥).
= 3𝑥(𝑥 − 2) − 2(𝑥 − 2)
 If the sum of the coefficients of the even degree is equal to the sum of the = (𝑥 − 2)(3𝑥 − 2)
coefficients of the odd degree, then -1 is a root of 𝑃(𝑥). 2
(𝑥 − 2)(3𝑥 − 2) = 0 ⟹ 𝑥 = 2,
 If the roots of the polynomial equation are in A.P , then the roots are 3
2 4 4 2
… . , (𝑎 − 𝑑), 𝑎, (𝑎 + 𝑑), … .. Thus the roots = , , 2. (or) 2, , .
3 3 3 3
 If the roots of the polynomial equation are in G.P , then the roots are 𝐸𝑋𝐴𝑀𝑃𝐿𝐸 3.22 It is known that the roots of the equation 𝑥 3 − 6𝑥 2 − 4𝑥 +
𝑎
… . , ( ) , 𝑎, (𝑎𝑟), … .. 24 = 0 are in arithmetic progression. Find its roots.
𝑟
 If the roots of the polynomial equation are in H.P , then the roots are 3. Solve the equation: 3𝑥 3 − 26𝑥 2 + 52𝑥 − 24 = 0 if the roots form a geometric
….,(
1 1 1
) , , ( ) , … .. progression.
𝑎−𝑑 𝑎 𝑎+𝑑 𝑎
EXERCISE 3.3 Let ( ) , 𝑎, (𝑎𝑟) are the roots of 3𝑥 3 − 26𝑥 2 + 52𝑥 − 24 = 0. (∵ 𝐺. 𝑃)
𝑟
𝑎 𝑑 −24
1.Solve the cubic equation: 2𝑥 3 − 𝑥 2 − 18𝑥 + 9 = 0 if sum of two of its ∑3 = ( ) 𝑎(𝑎𝑟) = − = − = 8 ⟹ 𝑎 3 = 23 ⟹ 𝑎 = 2
𝑟 𝑎 3
roots vanishes. 3 −26 52 −24
Let 𝛼, −𝛼, 𝛽 are the root of 2𝑥 3 − 𝑥 2 − 18𝑥 + 9 = 0. (∵sum of two of its 2 0 6 −40 24
roots vanishes.) 3 −20 12 0
Quotient = 3𝑥 2 − 20𝑥 + 12

V.GNANAMURUGAN, P,G,T, G.H.S.S, S.S.KOTTAI, SIVAGANGAI DT – 94874 43870 ; 82489 56766 , www.tnppgta.com, www.ednnet.in Page 51
= 3𝑥 2 − 18𝑥 − 2𝑥 + 12 Equating the coefficient of x on both side, −39 = −54 − 15𝑝 ⟹ 𝑝 = −1
= 3𝑥(𝑥 − 6) − 2(𝑥 − 6) So,𝑥 2 + 𝑝𝑥 − 9 = 𝑥 2 − 𝑥 − 9 , solving 𝑥 2 − 𝑥 − 9 = 0.
= (𝑥 − 6)(3𝑥 − 2) −𝑏±√𝑏2 −4𝑎𝑐 −(−1)±√(−1)2 −4(1)(−9) 1±√1+36 1±√37 1+√37 1−√37
2 𝑥= = = = = ,
(𝑥 − 6)(3𝑥 − 2) = 0 ⟹ 𝑥 = 6, . 2𝑎 2(1) 2 2 2 2
3 1+√37 1−√37
2
Thus the roots = , 2, 6. (or) 6,2, ,
2 Thus the roots are 1 + 2𝑖, 1 − 2𝑖, √3, −√3, ,
2 2
3 3
4. Determine k and Solve the equation 2𝑥 3 − 6𝑥 2 + 3𝑥 + 𝑘 = 0 if one of its roots 𝐸𝑋𝐴𝑀𝑃𝐿𝐸 3.15 If 2 + 𝑖 and 3 − √2 are roots of the equation 𝑥 6 − 13𝑥 5 +
is twice the sum of the other two roots. 62𝑥 4 − 126𝑥 3 + 65𝑥 2 + 127𝑥 − 140 = 0, find all roots.
The polynomial equation is 2𝑥 3 − 6𝑥 2 + 3𝑥 + 𝑘 = 0 6. Solve the cubic equations:
Let 𝛼, 𝛽 𝑎𝑛𝑑 𝛾 are the roots. (𝑖) 2𝑥 3 − 9𝑥 2 + 10𝑥 = 3 (𝑖𝑖)8𝑥 3 − 2𝑥 2 − 7𝑥 + 3 = 0
Given: 𝛼 = 2(𝛽 + 𝛾) → (1) (𝑖) 2𝑥 3 − 9𝑥 2 + 10𝑥 = 3 ⟹ 2𝑥 3 − 9𝑥 2 + 10𝑥 − 3 = 0
𝑏 −6 Sum of all coefficients = 2 − 9 + 10 − 3 = 0
∑1 = 𝛼 + 𝛽 + 𝛾 = − = − = 3
𝑎 2 ∴ (𝑥 − 1) is a factor.
⟹𝛼+𝛽+𝛾 =3 2 −9 10 −3
⟹ 2𝛼 + 2(𝛽 + 𝛾) = 6 1 0 2 −7 3
⟹ 2𝛼 + 𝛼 = 6 ⟹ 3𝛼 = 6 ⟹ 𝛼 = 2 2 −7 3 0
2 −6 3 𝑘 Quotient = 2𝑥 2 − 7𝑥 + 3
2 0 4 −4 −2 = 2𝑥 2 − 𝑥 − 6𝑥 + 3
2 −2 −1 0 = 𝑥(2𝑥 − 1) − 3(2𝑥 − 1)
∵ 𝛼 = 2 is one of the root, so 𝑘 − 2 = 0 ⟹ 𝑘 = 2 = (𝑥 − 3)(2𝑥 − 1)
Quotient = 2𝑥 2 − 2𝑥 − 1 𝑥 = 3,
1
−𝑏±√𝑏2 −4𝑎𝑐 −(−2)±√(−2)2 −4(2)(−1) 2±√4+8 2±2√3 1±√3 1+√3 1−√3 2
𝑥= = = = = = , 1
2𝑎 2(2) 4 4 2 2 2 Thus the roots are 1,3, .
2
1+√3 1−√3 (𝑖𝑖)8𝑥 3 − 2𝑥 2 − 7𝑥 + 3 = 0
Thus the roots are 2, , .
2 2
Sum of the coefficient of odd degree = 8 − 7 = 1
5. Find all the zeros of the polynomial 𝑥 6 − 3𝑥 5 − 5𝑥 4 + 22𝑥 3 − 39𝑥 2 − 39𝑥 +
Sum of the coefficient of even degree = −2 + 3 = 1
135 , if it is known that 1 + 2𝑖 and √3 are two of its zeros.
∴ (𝑥 + 1) is a factor.
Given roots are 1 + 2𝑖 and √3. Therefore 1 − 2𝑖 and −√3 are also roots. 8 −2 −7 3
𝑥 = 1 + 2𝑖 ⟹ 𝑥 − 1 = 2𝑖 −1 0 −8 10 −3
⟹ (𝑥 − 1)2 = (2𝑖)2 8 −10 3 0
⟹ 𝑥 2 − 2𝑥 + 1 = −4 Quotient = 8𝑥 2 − 10𝑥 + 3
⟹ 𝑥 2 − 2𝑥 + 5 = 0 = 8𝑥 2 − 4𝑥 − 6𝑥 + 3
𝑥 = √3 ⟹ 𝑥 2 = 3 ⟹ 𝑥 2 − 3 = 0 = 4𝑥(2𝑥 − 1) − 3(2𝑥 − 1)
𝑥 6 − 3𝑥 5 − 5𝑥 4 + 22𝑥 3 − 39𝑥 2 − 39𝑥 + 135 ≡ (𝑥 2 − 2𝑥 + 5)(𝑥 2 − 3)(𝑥 2 + = (4𝑥 − 3)(2𝑥 − 1)
𝑝𝑥 − 9)
V.GNANAMURUGAN, P,G,T, G.H.S.S, S.S.KOTTAI, SIVAGANGAI DT – 94874 43870 ; 82489 56766 , www.tnppgta.com, www.ednnet.in Page 52
3 1
𝑥= , Applying the Vieta’s formula,
4 2 𝑎 𝑏 1 𝑏
1 3
Thus the roots are −1, , . ∑1 = + 𝑎 + 𝑎𝑟 = − ⟹ 𝑎 ( + 1 + 𝑟) = − → (1)
𝑟 𝑎 𝑟 𝑎
2 4 𝑎 𝑎 𝑐 1 𝑐
𝐸𝑋𝐴𝑀𝑃𝐿𝐸 3.17 Solve the equation: 𝑥 3 − 3𝑥 2 − 33𝑥 + 35 = 0 ∑2 = ( ) (𝑎) + 𝑎(𝑎𝑟) + (𝑎𝑟) ( ) = ⟹ 𝑎2 ( + 1 + 𝑟) = → (2)
𝑟 𝑟 𝑎 𝑟 𝑎
𝐸𝑋𝐴𝑀𝑃𝐿𝐸 3.18 Solve the equation: 2𝑥 3 + 11𝑥 2 − 9𝑥 − 18 = 0 𝑎 𝑑
∑3 = ( ) 𝑎(𝑎𝑟) = − ⟹ 𝑎3 = − → (3)
𝑑
𝑟 𝑎 𝑎
7. Solve the equation: 𝑥 4 − 14𝑥 2 + 45 = 0 𝑐 𝑎 𝑐
Let 𝑥 2 = 𝑦 → (1) Dividing (2) by (2) ⟹ 𝑎 = ( ) (− ) ⟹ 𝑎 = − → (4)
𝑎 𝑏 𝑏
𝑥 4 − 14𝑥 2 + 45 = 0 ⟹ 𝑦 2 − 14𝑦 + 45 = 0 𝑐 3 𝑑 𝑐3 𝑑
Sub. (4) in (3) ⟹ (− ) = − ⟹ − 3 = − ⟹ 𝑎𝑐 3 = 𝑑𝑏 3
⟹ 𝑦 2 − 9𝑦 − 5𝑦 + 45 = 0 𝑏 𝑎 𝑏 𝑎

⟹ 𝑦(𝑦 − 9 ) − 5(𝑦 − 9 ) = 0 𝐸𝑋𝐴𝑀𝑃𝐿𝐸 3.21 If the roots of 𝑥 3 + 𝑝𝑥 2 + 𝑞𝑥 + 𝑟 = 0 are in H.P., prove that
⟹ (𝑦 − 5)(𝑦 − 9 ) = 0 9𝑝𝑞𝑟 = 27𝑟 3 + 2𝑝.
⟹ 𝑦 = 5,9 Let the roots are in H.P. then their reciprocals are in A.P. and roots of the equation
1 3 1 2 1 1 𝑝 𝑞
Sub. 𝑦 = 5,9 𝑖𝑛 (1) ⟹ 𝑥 2 = 5 ; 𝑥 2 = 9 ⟹ 𝑥 = ±√5 ; 𝑥 = ±3 (𝑥) + 𝑝 (𝑥) + 𝑞 (𝑥) + 𝑟 = 0 ⟹ 𝑥 3 + 𝑥 2 + 𝑥 + 𝑟 = 0
Thus the roots are −3,3, −√5, √5 . (×)𝑥 3 ⟹ 𝑟𝑥 3 + 𝑞𝑥 2 + 𝑝𝑥 + 1 = 0
𝐸𝑋𝐴𝑀𝑃𝐿𝐸 3.16 Solve the equation: 𝑥 4 − 9𝑥 2 + 20 = 0 Let (𝑎 − 𝑑), 𝑎, (𝑎 + 𝑑) are the roots of 𝑟𝑥 3 + 𝑞𝑥 2 + 𝑝𝑥 + 1 = 0. (∵ 𝐴. 𝑃)
𝐸𝑋𝐴𝑀𝑃𝐿𝐸 3.19 Obtain the condition that the roots of 𝑥 3 + 𝑝𝑥 2 + 𝑞𝑥 + 𝑟 = 0 𝑏
∑𝛼 = 𝑎 − 𝑑 + 𝑎 + 𝑎 + 𝑑 = − = − ⟹ 3𝑎 =
−𝑞 −𝑞
⟹𝑎=
−𝑞
𝑎 𝑟 𝑟 3𝑟
are in A.P. −𝑞 3 2
Since 𝑎 = is the root of the equation 𝑟𝑥 + 𝑞𝑥 + 𝑝𝑥 + 1 = 0
Let (𝑎 − 𝑑), 𝑎, (𝑎 + 𝑑) are the roots of 𝑥 3 + 𝑝𝑥 2 + 𝑞𝑥 + 𝑟 = 0. (∵ 𝐴. 𝑃) 3𝑟
𝑏 −𝑞 3 −𝑞 2 −𝑞
Applying the Vieta’s formula, 𝑎 − 𝑑 + 𝑎 + 𝑎 + 𝑑 = − = −𝑝 ⟹ 𝑟( ) +𝑞( ) +𝑝( )+1 = 0
𝑎 3𝑟 3𝑟 3𝑟
𝑝 𝑟𝑞 3 𝑞3 𝑝𝑞
⟹ 3𝑎 = −𝑝 ⟹ 𝑎 = − ⟹− + − +1=0
3 27𝑟 3 9𝑟 2 3𝑟
𝑝 3 2
Since 𝑎 = − is the root of the equation 𝑥 + 𝑝𝑥 + 𝑞𝑥 + 𝑟 = 0 𝑞3 𝑞3 𝑝𝑞
3 ⟹− 2+ 2− +1
27𝑟 9𝑟 3𝑟
𝑝 3 𝑝 2 𝑝
⟹ (− ) + 𝑝 (− ) + 𝑞 (− ) + 𝑟 = 0 (×)27𝑟 2 ⟹ −𝑞 3 + 3𝑞 3 − 9𝑝𝑞𝑟 + 27𝑟 2 = 0
3 3 3
𝑝3 𝑝2 𝑝𝑞 ⟹ 9𝑝𝑞𝑟 = 2𝑞 3 + 27𝑟 2
⟹− +𝑝( )− +𝑟 =0 PARTLY FACTORED POLYNOMIALS
27 9 3
𝑝3 𝑝3 𝑝𝑞
⟹− + +𝑟 =  Quadratic polynomial equations of the form (𝑎𝑥 + 𝑏)(𝑐𝑥 + 𝑑)(𝑝𝑥 +
27 9 3
3 3
(×) 27 ⟹ −𝑝 + 3𝑝 + 27𝑟 = 9𝑝𝑞 𝑞)(𝑟𝑥 + 𝑠) + 𝑘 = 0, 𝑘 ≠ 0 can be rewritten in the form (𝛼𝑥 2 + 𝛽𝑥 +
⟹ 9𝑝𝑞 = 2𝑝3 + 27𝑟 𝜆)(𝛼𝑥 2 + 𝛽𝑥 + 𝜇) + 𝑘 = 0
 (𝑥 + 𝑎)(𝑥 + 𝑏) = 𝑥 2 + (𝑎 + 𝑏)𝑥 + 𝑎𝑏
𝐸𝑋𝐴𝑀𝑃𝐿𝐸 3.20 Find the condition that the roots of 𝑎𝑥 3 + 𝑏𝑥 2 + 𝑐𝑥 + 𝑑 = 0
EXERCISE 3.4
are in geometric progression. Assume 𝑎, 𝑏, 𝑐, 𝑑 ≠ 0.
𝑎 1.Solve : (𝑖)(𝑥 − 5)(𝑥 − 7)(𝑥 + 6)(𝑥 + 4) = 504
Let ( ) , 𝑎, (𝑎𝑟) are the roots of 𝑎𝑥 3 + 𝑏𝑥 2 + 𝑐𝑥 + 𝑑 = 0. (∵ 𝐺. 𝑃) (𝑖𝑖)(𝑥 − 4)(𝑥 − 7)(𝑥 − 2)(𝑥 + 1) = 16
𝑟

V.GNANAMURUGAN, P,G,T, G.H.S.S, S.S.KOTTAI, SIVAGANGAI DT – 94874 43870 ; 82489 56766 , www.tnppgta.com, www.ednnet.in Page 53
(𝑖)(𝑥 − 5)(𝑥 − 7)(𝑥 + 6)(𝑥 + 4) = 504 −(−6)±√(−6)2 −4(1)(−8)
⟹𝑥=
⟹ [(𝑥 − 5)(𝑥 + 4)][(𝑥 − 7)(𝑥 + 6)] = 504 2(1)
⟹ [𝑥 2 − 𝑥 − 20][𝑥 2 − 𝑥 − 42] = 504 ⟹𝑥=
6±√36+32
2
Let 𝑥 2 − 𝑥 = 𝑦 → (1) 6±√78
⟹ [𝑦 − 20][𝑦 − 42] = 504 ⟹𝑥=
2
⟹ 𝑦 2 − 62𝑦 + 840 − 504 = 0 ⟹𝑥=
6±√4×17
⟹ 𝑦 2 − 62𝑦 + 336 = 0 2
6±2√17
⟹ 𝑦 2 − 6𝑦 − 56𝑦 + 336 = 0 ⟹𝑥=
2
⟹ 𝑦(𝑦 − 6) − 56(𝑦 − 6) = 0 ⟹ 𝑥 = 3 ± √17
⟹ (𝑦 − 6)(𝑦 − 56) = 0
Thus the roots are 3,3, 3 + √17, 3 − √17.
⟹ 𝑦 = 6,56
2.Solve: (2𝑥 − 1)(𝑥 + 3)(𝑥 − 2)(2𝑥 + 3) + 20 = 0
Put 𝑦 = 6,56 in (1),
⟹ [(2𝑥 − 1)(2𝑥 + 3)][(𝑥 + 3)(𝑥 − 2)] + 20 = 0
⟹ 𝑥2 − 𝑥 = 6 ⟹ 𝑥 2 − 𝑥 = 56
⟹ [4𝑥 2 + 4𝑥 − 3][𝑥 2 + 𝑥 − 6] + 20 = 0
⟹ 𝑥2 − 𝑥 − 6 = 0 ⟹ 𝑥 2 − 𝑥 − 56 = 0
⟹ [4(𝑥 2 + 𝑥) − 3][𝑥 2 + 𝑥 − 6] + 20 = 0
⟹ (𝑥 + 2)(𝑥 − 3) = 0 ⟹ (𝑥 + 7)(𝑥 − 8) = 0
Let 𝑥 2 + 𝑥 = 𝑦 → (1)
⟹ 𝑥 = −2,3 ⟹ 𝑥 = −7,8
⟹ [4𝑦 − 3][𝑦 − 6] + 20 = 0
Thus the roots are −7, −2,3,8.
⟹ 4𝑦 2 − 24𝑦 − 3𝑦 + 18 + 20 = 0
(𝑖𝑖)(𝑥 − 4)(𝑥 − 7)(𝑥 − 2)(𝑥 + 1) = 16
⟹ 4𝑦 2 − 27𝑦 + 38 = 0
⟹ [(𝑥 − 4)(𝑥 − 2)][(𝑥 − 7)(𝑥 + 1)] = 16
⟹ 4𝑦 2 − 8𝑦 − 19𝑦 + 38 = 0
⟹ [𝑥 2 − 6𝑥 + 8][𝑥 2 − 6𝑥 − 7] = 16
⟹ 4𝑦(𝑦 − 2) − 19(𝑦 − 2) = 0
Let 𝑥 2 − 6𝑥 = 𝑦 → (1)
⟹ (𝑦 − 2)(4𝑦 − 19) = 0
⟹ [𝑦 + 8][𝑦 − 7] = 16 19
⟹ 𝑦 2 + 𝑦 − 56 − 16 = 0 ⟹ 𝑦 = 2,
4
19
⟹ 𝑦 2 + 𝑦 − 72 = 0 Put 𝑦 = 2, in (1),
4
⟹ 𝑦 2 − 8𝑦 + 9𝑦 − 72 = 0 2
⟹𝑥 +𝑥 =2 19
⟹ 𝑥2 + 𝑥 =
⟹ 𝑦(𝑦 − 8) + 9(𝑦 − 8) = 0 ⟹ 𝑥2 + 𝑥 − 2 = 0 4
2
⟹ (𝑦 − 8)(𝑦 +) = 0 ⟹ 4𝑥 + 4𝑥 − 19 = 0
⟹ (𝑥 + 2)(𝑥 − 1) = 0 −𝑏±√𝑏2 −4𝑎𝑐
⟹ 𝑦 = 8, −9 ⟹ 𝑥 = −2,1 ⟹𝑥=
2𝑎
Put 𝑦 = 8, −9 in (1), −(4)±√(4)2 −4(4)(−19)
⟹ 𝑥 2 − 6𝑥 = 8 ⟹ 𝑥 2 − 6𝑥 = −9 ⟹𝑥=
2(4)
⟹ 𝑥 2 − 6𝑥 − 8 = 0 ⟹ 𝑥 2 − 6𝑥 + 9 = 0 −4±√320
⟹𝑥=
⟹𝑥=
−𝑏±√𝑏2 −4𝑎𝑐 ⟹ (𝑥 − 3)(𝑥 − 3) = 0 8
−4±4√20
2𝑎 ⟹ 𝑥 = 3,3 ⟹𝑥=
8

V.GNANAMURUGAN, P,G,T, G.H.S.S, S.S.KOTTAI, SIVAGANGAI DT – 94874 43870 ; 82489 56766 , www.tnppgta.com, www.ednnet.in Page 54
⟹𝑥=
−1±√4×5  For an even degree reciprocal equation satisfying
2 𝑎𝑛 = −𝑎0 , 𝑎𝑛−1 = −𝑎1 , 𝑎𝑛−2 = −𝑎2 , … … .,and the middle term
−1±2√5
⟹𝑥= must be 0, then 𝑥 = 1 and 𝑥 = −1 are solutions.
2
1 1
Thus the roots are −2,1,
−1+2√5 −1−2√5
, .  For an even degree reciprocal equation by taking 𝑥 + or 𝑥 − as y,
𝑥 𝑥
2 2
we can obtain a polynomial equation of degree one half of the degree
𝐸𝑋𝐴𝑀𝑃𝐿𝐸 3.23 Solve the equation (𝑥 − 2)(𝑥 − 7)(𝑥 − 3)(𝑥 + 2) + 19 = 0
of the given equation.
HINT: [(𝑥 − 2)(𝑥 − 3)][(𝑥 − 7)(𝑥 + 2)] + 19 = 0
EXERCISE 3.5
𝐸𝑋𝐴𝑀𝑃𝐿𝐸 3.24 Solve the equation (2𝑥 − 3)(6𝑥 − 1)(3𝑥 − 2)(𝑥 − 12) − 7 = 0
1.Solve the following equations:
HINT: [(2𝑥 − 3)(3𝑥 − 2)][(6𝑥 − 1)(𝑥 − 12)] − 7 = 0
𝑖)𝑠𝑖𝑛2 𝑥 − 5 sin 𝑥 + 4 = 0 (𝑖𝑖)12𝑥 3 + 8𝑥 = 29𝑥 2 − 4
POLYNOMIAL EQUATION WITH NO ADDITIONAL INFORMATION
𝑖)𝑠𝑖𝑛2 𝑥 − 5 sin 𝑥 + 4 = 0
 RATIONAL ROOT THEOREM: Let 𝑎𝑛 𝑥 𝑛 + 𝑎𝑛−1 𝑥 𝑛−1 + ⋯ + 𝑎1 𝑥 + 𝑎0 with Let sin 𝑥 = 𝑦 → (1)
𝑝
𝑎𝑛 ≠ 0, and 𝑎0 ≠ 0, be a polynomial with integer coefficients. If with 𝑠𝑖𝑛2 𝑥 − 5 sin 𝑥 + 4 = 0 ⟹ 𝑦 2 − 5𝑦 + 4 = 0
𝑞
(𝑝, 𝑞) = 1, is a root of the polynomial, then p is a factor of 𝑎0 and q is a ⟹ (𝑦 − 1)(𝑦 − 4) = 0
factor of 𝑎𝑛 . ⟹ 𝑦 = 1,4
 RECIPROCAL POLYNOMIAL: A polynomial 𝑃(𝑥) of degree n is said to be a Sub. 𝑦 = 1,4 in (1),
reciprocal polynomial if one of the following conditions is true: (𝑖)𝑃(𝑥) = sin 𝑥 = 1 sin 𝑥 = 4 is not possible.
1 1 𝜋
𝑥 𝑛 𝑃 ( ) (𝑖𝑖)𝑃(𝑥) = −𝑥 𝑛 𝑃 ( ) ⟹ sin 𝑥 = sin
𝑥 𝑥 4
𝜋
𝑛
 RECIPROCAL EQUATION: A polynomial equation 𝑎𝑛 𝑥 + 𝑎𝑛−1 𝑥 𝑛−1
+ ⋯+ ⟹ 𝑥 = 2𝑛𝜋 + , 𝑛 ∈ 𝑧
2
𝑎1 𝑥 + 𝑎0 = 0, (𝑎𝑛 ≠ 0) is a reciprocal equation if and only if one of the (𝑖𝑖)12𝑥 3 + 8𝑥 = 29𝑥 2 − 4 ⟹ 12𝑥 3 − 29𝑥 2 + 8𝑥 + 4 = 0
following two statements is true: (𝑖)𝑎𝑛 = 𝑎0 , 𝑎𝑛−1 = 𝑎1 , 𝑎𝑛−2 = Let 𝑃(𝑥) = 12𝑥 3 − 29𝑥 2 + 8𝑥 + 4
𝑎2 , … … . (𝑖𝑖)𝑎𝑛 = −𝑎0 , 𝑎𝑛−1 = −𝑎1 , 𝑎𝑛−2 = −𝑎2 , … …. 𝑃(2) = 12(2)3 − 29(2)2 + 8(2) + 4 = 96 − 116 + 16 + 4 = 116 − 116 = 0
 A reciprocal equation cannot have 0 as a solution. ∴ (𝑥 − 2) is a factor.
 The coefficients and the solutions are not restricted to be real. 12 −29 8 4
1 2 0 24 −10 −4
 If a polynomial equation 𝑃(𝑥) = 0 has 𝛼 as a root and if is also a
𝛼 12 −5 −2 0
root then the polynomial 𝑃(𝑥) = 0 must be a reciprocal equation is Quotient = 12𝑥 2 − 5𝑥 − 2
not true. = 12𝑥 2 + 3𝑥 − 8𝑥 − 2
 For an odd degree reciprocal equation satisfying = 3𝑥(4𝑥 + 1) − 2(4𝑥 + 1)
𝑎𝑛 = 𝑎0 , 𝑎𝑛−1 = 𝑎1 , 𝑎𝑛−2 = 𝑎2 , … …., 𝑥 = −1 must be a solution. = (4𝑥 + 1)(3𝑥 − 2)
 For an even degree reciprocal equation satisfying (4𝑥 + 1)(3𝑥 − 2) = 0 ⟹ 𝑥 = − ,
1 2
4 3
𝑎𝑛 = −𝑎0 , 𝑎𝑛−1 = −𝑎1 , 𝑎𝑛−2 = −𝑎2 , … … ., 𝑥 = 1 must be a 1 2
Thus the roots are 2, − , .
solution. 4 3

V.GNANAMURUGAN, P,G,T, G.H.S.S, S.S.KOTTAI, SIVAGANGAI DT – 94874 43870 ; 82489 56766 , www.tnppgta.com, www.ednnet.in Page 55
3 3 3 1
𝐸𝑋𝐴𝑀𝑃𝐿𝐸 3.25 Solve the equation 𝑥 3 − 5𝑥 2 − 4𝑥 + 20 = 0. 𝑦 = 𝑥 2𝑛 ⟹ 𝑥 2𝑛 = 8 ⟹ 𝑥 2𝑛 = 23 ⟹ 𝑥 2𝑛 = 2 ⟹ 𝑥 = 22𝑛 ⟹ 𝑥 = 4𝑛
2.Examine for the rational roots of (𝑖)2𝑥 3 − 𝑥 2 − 1 = 0 (𝑖𝑖)𝑥 8 − 3𝑥 + 1 = 0 𝑥 𝑎 𝑏 6𝑎
(𝑖)2𝑥 3 − 𝑥 2 − 1 = 0 4.Solve: 2√ + 3√ = + .
𝑎 𝑥 𝑎 𝑏
Sum of all coefficients = 2 − 1 − 1 = 0 𝑥 𝑎 𝑏 6𝑎
∴ (𝑥 − 1) is a factor. 2√ + 3√ = +
𝑎 𝑥 𝑎 𝑏
2 −1 0 −1 2
𝑥 𝑎 𝑏 6𝑎 2
1 0 2 1 1 Squaring on both sides, (2√ + 3√ ) = ( + )
𝑎 𝑥 𝑎 𝑏
2 1 1 0
Quotient = 2𝑥 2 + 𝑥 + 1 𝑥
⟹ 4 + 9 + 12√ . √ =
𝑎 𝑥 𝑎 𝑏2
+
36𝑎2 𝑏 6𝑎
+ 2. .
𝑎 𝑥 𝑎 𝑥 𝑎 2 𝑏2 𝑎 𝑏
−𝑏±√𝑏2 −4𝑎𝑐 −(1)±√(1)2 −4(2)(1) −1±√−7 −1±√7𝑖
⟹𝑥= = = = , which are 𝑥 𝑎 𝑏2 36𝑎2
2𝑎 2(2) 4 4
⟹ 4 + 9 + 12 = + + 12
imaginary. 𝑎 𝑥 𝑎2 𝑏2
𝑥 𝑎 𝑏2 36𝑎2
∴ 𝑥 = 1 is a rational root. ⟹4 +9 = +
𝑎 𝑥 𝑎2 𝑏2
(𝑖𝑖)𝑥 8 − 3𝑥 + 1 = 0 4𝑥 2 +9𝑎2 𝑏4 +36𝑎4
⟹ = 2 2
Let 𝑃(𝑥) = 𝑥 8 − 3𝑥 + 1. 𝑎𝑥 𝑎 𝑏
𝑝 ⟹ (4𝑥 2 + 9𝑎2 )𝑎𝑏 2 = (𝑏 4 + 36𝑎4 )𝑥
𝑎𝑛 = 1 ; 𝑎0 = 1 ⟹ 𝑝 = ±1, 𝑞 = ±1 ⟹ = ±1
𝑞 ⟹ 4𝑎𝑏 2 𝑥 2 + 9𝑎3 𝑏 2 = (𝑏 4 + 36𝑎4 )𝑥
Sum of all co.eff.= 1 − 3 + 1 ≠ 0 ⟹ 1 is not a root. ⟹ 4𝑎𝑏 2 𝑥 2 − (𝑏4 + 36𝑎4 )𝑥 + 9𝑎3 𝑏 2 = 0
Sum of even term co.eff.≠ Sum of odd term co.eff.⟹ −1 is not a root. ⟹ 4𝑎𝑏 2 𝑥 2 − 𝑏 4 𝑥 − 36𝑎4 𝑥 + 9𝑎3 𝑏 2 = 0
∴ 𝑃(𝑥) has no rational roots. ⟹ 𝑏 2 𝑥(4𝑎𝑥 − 𝑏 2 ) − 9𝑎3 (4𝑎𝑥 − 𝑏 2 ) = 0
3 −3
3.Solve: 8𝑥 2𝑛 − 8𝑥 2𝑛 = 63 ⟹ (4𝑎𝑥 − 𝑏 2 )(𝑏 2 𝑥 − 9𝑎3 ) = 0
3 𝑏2 9𝑎3
Let 𝑦 = 𝑥 .
2𝑛
⟹𝑥= , 2
4𝑎 𝑏
3 −3 3
8 5.Solve the equations: (𝑖)6𝑥 − 35𝑥 3 + 62𝑥 2 − 35𝑥 + 6 = 0
4
8𝑥 2𝑛 − 8𝑥 2𝑛 = 63 ⟹ 8𝑥 2𝑛 − 3 = 63
𝑥 2𝑛 (𝑖𝑖)𝑥 4 + 3𝑥 3 − 3𝑥 − 1 = 0
8 (𝑖)6𝑥 4 − 35𝑥 3 + 62𝑥 2 − 35𝑥 + 6 = 0
⟹ 8𝑦 − = 63
𝑦
8𝑦 2 −8
The given polynomial equation is an even degree reciprocal equation of Type-I.
⟹ = 63 Dividing the given equation by 𝑥 2 , we get
𝑦
2 35 6 1 1
⟹ 8𝑦 − 8 = 63𝑦 6𝑥 2 − 35𝑥 + 62 − + 2 = 0 ⟹ 6 (𝑥 2 + 2) − 35 (𝑥 + ) + 62 = 0 → (1)
𝑥 𝑥 𝑥 𝑥
⟹ 8𝑦 2 − 63𝑦 − 8 = 0 1 1 2 1 1
⟹ 8𝑦 2 − 64𝑦 + 𝑦 − 8 = 0 𝐿𝑒𝑡 𝑦 = 𝑥 + ⟹ 𝑦 2 = (𝑥 + ) ⟹ 𝑦 2 = 𝑥 2 + 2 + 2 ⟹ 𝑥 2 + 2 = 𝑦 2 − 2
𝑥 𝑥 𝑥 𝑥
⟹ 8𝑦(𝑦 − 8) + 1(𝑦 − 8) = 0 2
(1) ⟹ 6(𝑦 − 2 ) − 35𝑦 + 62 = 0
⟹ (𝑦 − 8)(8𝑦 + 1) = 0 ⟹ 6𝑦 2 − 12 − 35𝑦 + 62 = 0
1
⟹ 𝑦 = 8 (𝑜𝑟) 𝑦 = − (Not a solution)
8
⟹ 6𝑦 2 − 35𝑦 + 50 = 0

V.GNANAMURUGAN, P,G,T, G.H.S.S, S.S.KOTTAI, SIVAGANGAI DT – 94874 43870 ; 82489 56766 , www.tnppgta.com, www.ednnet.in Page 56
⟹ 6𝑦 2 − 15𝑦 − 20𝑦 + 50 = 0 ⟹ (𝑦 − 4)(𝑦 − 8) = 0
⟹ 3𝑦(2𝑦 − 5) − 10(2𝑦 − 5) = 0 ⟹ 𝑦 = 4,8
⟹ (2𝑦 − 5)(3𝑦 − 10) = 0 ⟹ 𝑦 =
5
;𝑦 =
10 Substituting the values of y in 2𝑥 = 𝑦
2
1
3 When 𝑦 = 4 ⟹ 2𝑥 = 4 ⟹ 2𝑥 = 22 ⟹ 𝑥 = 2
Substituting the values of y in 𝑦 = 𝑥 + , When 𝑦 = 8 ⟹ 2𝑥 = 8 ⟹ 2𝑥 = 23 ⟹ 𝑥 = 3
𝑥
5 1 5 10 1 10 Thus the roots are 2 and 3.
When 𝑦 = ⟹ 𝑥 + = When 𝑦 = ⟹𝑥+ =
2 𝑥 2 3 𝑥 3 1
𝑥 2 +1 5 𝑥 2 +1 10 7. Solve the equation 6𝑥 4 − 5𝑥 3 − 38𝑥 2 − 5𝑥 + 6 = 0 if it is known that is a
⟹ = ⟹ = 3
𝑥 2 𝑥 3 solution.
2 2
⟹ 2𝑥 + 2 = 5𝑥 ⟹ 3𝑥 + 3 = 10𝑥
The given polynomial equation is an even degree reciprocal equation of Type-I.
⟹ 2𝑥 2 − 5𝑥 + 2 = 0 ⟹ 3𝑥 2 − 10𝑥 + 3 = 0 1 1
⟹ 2𝑥 2 − 4𝑥 − 𝑥 + 2 = 0 ⟹ 3𝑥 2 − 9𝑥 − 𝑥 + 3 = 0 Since is a root, then 3 is also a root. Let the other roots are 𝛼 and .
3 𝛼
⟹ 2𝑥(𝑥 − 2) − 1(𝑥 − 2) = 0 ⟹ 3𝑥(𝑥 − 3) − 1(𝑥 − 3) = 0 1 1 5 10 𝛼 2 +1 5
∑1 = 3 + + 𝛼 + = ⟹ + =
⟹ (𝑥 − 3)(3𝑥 − 1) = 0 3 𝛼 6 3 𝛼 6
⟹ (𝑥 − 2)(2𝑥 − 1) = 0 𝛼 2 +1 5 10
⟹ 𝑥 = 2,
1
⟹ 𝑥 = 3,
1 ⟹ = −
𝛼 6 3
2 3 𝛼 2 +1 5−20
1 1 ⟹ =
Thus the roots are 2, , 3, . 𝛼 6
2 3
𝛼 2 +1 5
(𝑖𝑖)𝑥 4 + 3𝑥 3 − 3𝑥 − 1 = 0 ⟹ =−
𝛼 2
The given polynomial equation is an even degree reciprocal equation of Type-II. 2
⟹ 2𝛼 + 2 = −5𝛼
1
So 1 and -1 are the roots of the equation. Hence the other two roots are 𝛼 and .
𝛼
⟹ 2𝛼 2 + 5𝛼 + 2 = 0
1 𝛼 2 +1 ⟹ 2𝛼 2 + 𝛼 + 4𝛼 + 2 = 0
∑ 1 = −1 + 1 + 𝛼 + = −3 ⟹ = −3 ⟹ 𝛼 2 + 1 = −3𝛼
𝛼 𝛼 ⟹ 2𝛼(𝛼 + 2) + 1(𝛼 + 2) = 0
⟹ 𝛼 2 + 3𝛼 + 1 = 0 ⟹ (𝛼 + 2)(2𝛼 + 1) = 0
−𝑏±√𝑏2 −4𝑎𝑐 −(3)±√(3)2 −4(1)(1) −3±√9−4 −3±√5 1
⟹𝛼= = = = ⟹ 𝛼 = −2, −
2𝑎 2(1) 2 2 2
−3+√5 −3−√5 1 1
Thus the roots are −1,1, , Thus the roots are −2, − , 3, .
2 2 2 3
6.Find all the real numbers satisfying 4 − 3(2 𝑥 𝑥+2 ) 5
+ 2 = 0. 𝐸𝑋𝐴𝑀𝑃𝐿𝐸 3.28 Solve the equation 𝑥 4 − 10𝑥 3 + 26𝑥 2 − 10𝑥 + 1 = 0
4𝑥 − 3(2𝑥+2 ) + 25 = 0 𝐸𝑋𝐴𝑀𝑃𝐿𝐸 3.26 Find the roots of 2𝑥 3 + 3𝑥 2 + 2𝑥 + 3
⟹ (2𝑥 )2 − 3(2𝑥 22 ) + 32 = 0 Let 𝑃(𝑥) = 2𝑥 3 + 3𝑥 2 + 2𝑥 + 3.
3 2
⟹ (2𝑥 )2 − 12(2𝑥 ) + 32 = 0
Let 2𝑥 = 𝑦 𝑃 (− 3⁄2) = 2 (− 3⁄2) + 3 (− 3⁄2) + 2 (− 3⁄2) + 3
⟹ (𝑦)2 − 12(𝑦) + 32 = 0 27 9
⟹ 𝑦 2 − 4𝑦 − 8𝑦 + 32 = 0 = 2 (− ) + 3 ( ) − 3 + 3
8 4
⟹ 𝑦(𝑦 − 4) − 8(𝑦 − 4) = 0 27 27
=− +
4 4
V.GNANAMURUGAN, P,G,T, G.H.S.S, S.S.KOTTAI, SIVAGANGAI DT – 94874 43870 ; 82489 56766 , www.tnppgta.com, www.ednnet.in Page 57
=0 1 1
When 𝑦 = ⟹ cos 𝑥 = When 𝑦 = 4,
3 2 2
∴ − is one of the root. 𝜋 ⟹ cos 𝑥 = 4 is not possible
2 ⟹ cos 𝑥 = co
3 3
To find other roots divide 2𝑥 3 + 3𝑥 2 + 2𝑥 + 3 by − 𝜋
⟹ 𝑥 = 2𝑛𝜋 ± , 𝑛 ∈ 𝑧
2 3
2 3 2 3
DESCARTES RULE
− 3⁄2 0 −3 0 −3
Descartes Rule: If 𝑝 is the number of positive zero of a polynomial 𝑃(𝑥) with real
2 0 2 0
1
Quotient= [2𝑥 2 + 2] = 𝑥 2 + 1 coefficients and 𝑠 is the number of sign changes in coefficients of 𝑃(𝑥), then 𝑠 − 𝑝
2 is a non-negative even integer.
−𝑏±√𝑏2 −4𝑎𝑐 −(0)±√(0)2 −4(1)(1) ±√−4 ±2𝑖
⟹𝑥= = = = = ±𝑖 The number negative zeros of the polynomial 𝑃(𝑥) cannot be more than the
2𝑎 2(1) 2 2
3 number of sign changes in coefficients of 𝑃(−𝑥) and the difference between the
Thus the roots are − , 𝑖, −𝑖.
2 number of sign changes in coefficients of 𝑃(−𝑥) and the number of negative zeros
𝐸𝑋𝐴𝑀𝑃𝐿𝐸 3.27 Solve the equation 7𝑥 3 − 43𝑥 2 − 43𝑥 + 7 = 0 of the polynomial 𝑃(𝑥) is even.
The given polynomial equation is an odd degree reciprocal equation of Type-I. EXERCISE 3.6
Thus -1 is one root. 1.Discuss the maximum possible number of positive and negative roots of the
To find the other roots divide 7𝑥 3 − 43𝑥 2 − 43𝑥 + 7 by -1. polynomial equation 9𝑥 9 − 4𝑥 8 + 4𝑥 7 − 3𝑥 6 + 2𝑥 5 + 𝑥 3 + 7𝑥 2 + 7𝑥 + 2 = 0
7 −43 −43 7 Let 𝑃(𝑥) = 9𝑥 9 − 4𝑥 8 + 4𝑥 7 − 3𝑥 6 + 2𝑥 5 + 𝑥 3 + 7𝑥 2 + 7𝑥 + 2
−1 0 −7 50 −7
𝑃(−𝑥) = 9(−𝑥)9 − 4(−𝑥)8 + 4(−𝑥)7 − 3(−𝑥)6 + 2(−𝑥)5 + (−𝑥)3 + 7(−𝑥)2 +
7 −50 7 0
Quotient = 7𝑥 2 − 50𝑥 + 7 7(−𝑥) + 2
= 7𝑥 2 − 49𝑥 − 𝑥 + 7 = −9𝑥 9 − 4𝑥 8 − 4𝑥 7 − 3𝑥 6 − 2𝑥 5 − 𝑥 3 + 7𝑥 2 − 7𝑥 + 2
= 7𝑥(𝑥 − 7) − 1(𝑥 − 7) The number of sign changes for 𝑃(𝑥) and 𝑃(−𝑥) are 4 and 2 respectively. Hence
= (𝑥 − 7)(7𝑥 − 1) it has at most 4 positive real roots and 2 negative real roots.
1 2. Discuss the maximum possible number of positive and negative roots of the
⟹ 𝑥 = 7,
7
1
polynomial equation 𝑥 2 − 5𝑥 + 6 and 𝑥 2 − 5𝑥 + 16 . Also draw rough sketch of
Thus the roots are −1,7, . the graph.
7
𝐸𝑋𝐴𝑀𝑃𝐿𝐸 3.29 Find the solution, if any of the equation 2𝑐𝑜𝑠 2 𝑥 − 9 cos 𝑥 + 4 = 𝑃(𝑥) = (𝑥 2 − 5𝑥 + 6 ); 𝑃(−𝑥) = (𝑥 2 + 5𝑥 + 6)
0 The number of sign changes for 𝑃(𝑥) and 𝑃(−𝑥) are 2 and 0 respectively. Hence
Let cos 𝑥 = 𝑦 → (1) it has at most 2 positive real roots and no negative real roots.
2𝑐𝑜𝑠 2 𝑥 − 9 cos 𝑥 + 4 = 0 ⟹ 2𝑦 2 − 9𝑦 + 4 = 0 𝑄(𝑥) = 𝑥 2 − 5𝑥 + 16 ; 𝑄(−𝑥) = 𝑥 2 + 5𝑥 + 16
⟹ 2𝑦 2 − 𝑦 − 8𝑦 + 4 = 0 The number of sign changes for 𝑄(𝑥) and 𝑄(−𝑥) are 2 and 0 respectively. Hence
⟹ 𝑦(2𝑦 − 1) − 4(2𝑦 − 1) = 0 it has at most 2 positive real roots and no negative real roots.
⟹ (𝑦 − 4)(2𝑦 − 1) = 0 𝑦 = 𝑥 2 − 5𝑥 + 6
1 x -1 0 1 2 3 4
⟹ 𝑦 = 4,
2
y 12 6 2 0 0 2
V.GNANAMURUGAN, P,G,T, G.H.S.S, S.S.KOTTAI, SIVAGANGAI DT – 94874 43870 ; 82489 56766 , www.tnppgta.com, www.ednnet.in Page 58
𝑦 = 𝑥 2 − 5𝑥 + 16 = −𝑥 9 − 9𝑥 7 − 7𝑥 5 − 5𝑥 3 − 3𝑥
x -1 0 1 2 3 4 The number of sign changes for 𝑃(𝑥) and 𝑃(−𝑥) are 0 and 0 respectively. Hence
y 22 16 12 10 10 12 it has no positive real roots and no negative real roots. 0 is one root of the
equation. Hence the given equation has only one real root and 8 imaginary roots.
𝐸𝑋𝐴𝑀𝑃𝐿𝐸 3.31 Discuss the nature of the roots of the following polynomials:
(𝑖)𝑥 2018 + 1947𝑥 1950 + 15𝑥 8 + 26𝑥 6 + 2019 (𝑖𝑖)𝑥 5 − 19𝑥 4 + 2𝑥 3 + 5𝑥 2 + 11
4.INVERSE TRIGONOMETRIC FUNCTIONS
DOMAIN AND RANGE OF TRIGONOMETRIC FUNCTIONS:
DOMAIN RANGE
−𝟏
𝐬𝐢𝐧 𝒙 [−𝟏, 𝟏] 𝜋 𝜋
[− , ]
2 2
−𝟏 [−𝟏, 𝟏] [0, 𝜋]
𝐜𝐨𝐬 𝒙
3. Show that the equation 𝑥 9 − 5𝑥 5 + 4𝑥 4 + 2𝑥 2 + 1 = 0 has at least 6 imaginary −𝟏
𝐭𝐚𝐧 𝒙 𝑹 𝜋 𝜋
(− , )
solutions. 2 2
−𝟏
𝐜𝐨𝐬𝐞𝐜 𝒙 (−∞, −𝟏] ∪ [𝟏, ∞) 𝜋 𝜋
𝑃(𝑥) = 𝑥 9 − 5𝑥 5 + 4𝑥 4 + 2𝑥 2 + 1 [− , ] − {0}
𝑃(−𝑥) = (−𝑥)9 − 5(−𝑥)5 + 4(−𝑥)4 + 2(−𝑥)2 + 1 2 2
−𝟏
𝐬𝐞𝐜 𝒙 (−∞, −𝟏] ∪ [𝟏, ∞) 𝜋
= −𝑥 9 + 5𝑥 5 + 4𝑥 4 + 2𝑥 2 + 1 [0, 𝜋] − { }
2
The number of sign changes for 𝑃(𝑥) and 𝑃(−𝑥) are 2 and 1 respectively. Hence −𝟏
𝐜𝐨𝐭 𝒙 𝑹 (0, 𝜋)
it has at most 2 positive real root and 1 negative real root. PROPERTIES OF TRIGONOMETRIC FUNCTIONS:
Number of imaginary roots = 9 − (2 + 1) = 9 − 3 = 6. PROPERTY 𝐈 ∶ −
Hence it has at least 6 imaginary solutions. 𝜋 𝜋
1) 𝑠𝑖𝑛−1 (sin 𝜃) = 𝜃, 𝑖𝑓 𝜃 ∈ [− , ]
𝐸𝑋𝐴𝑀𝑃𝐿𝐸 3.30 Show that the polynomial 9𝑥 9 + 2𝑥 5 − 𝑥 4 − 7𝑥 2 + 2 has at 2 2
least 6 imaginary roots. 2) 𝑐𝑜𝑠 −1 (cos 𝜃) = 𝜃, 𝑖𝑓 𝜃 ∈ [0, 𝜋]
𝜋 𝜋
4. Determine the number of positive and negative roots of the equation 3) 𝑡𝑎𝑛−1 (tan 𝜃) = 𝜃, 𝑖𝑓 𝜃 ∈ (− , )
2 2
𝑥 9 − 5𝑥 8 − 14𝑥 7 = 0 4) 𝑐𝑜𝑠𝑒𝑐 −1 (cosec 𝜋 𝜋
𝜃) = 𝜃, 𝑖𝑓 𝜃 ∈ [− , ] − {0}
𝑃(𝑥) = 𝑥 9 − 5𝑥 8 − 14𝑥 7 2 2
𝜋
𝑃(−𝑥) = (−𝑥)9 − 5(−𝑥)8 − 14(−𝑥)7 = −𝑥 9 + 5𝑥 8 + 14𝑥 7 5) 𝑠𝑒𝑐 −1 (sec 𝜃) = 𝜃, 𝑖𝑓 𝜃 ∈ [0, 𝜋] − { }
2
The number of sign changes for 𝑃(𝑥) and 𝑃(−𝑥) are 1 and 1 respectively. Hence 6) 𝑐𝑜𝑡 −1 (cot 𝜃) = 𝜃, 𝑖𝑓 𝜃 ∈ (0, 𝜋);
it has at most 1 positive real root and 1 negative real root. PROPERTY 𝐈𝐈 ∶ −
5. Find the exact number of real and imaginary roots of the equation 𝑥 9 + 9𝑥 7 + 1) sin(𝑠𝑖𝑛−1 𝑥) = 𝑥, 𝑖𝑓 𝑥 ∈ [−1,1]
7𝑥 5 + 5𝑥 3 + 3𝑥 = 0. 2) cos(𝑐𝑜𝑠 −1 𝑥) = 𝑥, 𝑖𝑓 𝑥 ∈ [−1,1]
𝑃(𝑥) = 𝑥 9 + 9𝑥 7 + 7𝑥 5 + 5𝑥 3 + 3𝑥. 3) tan(𝑡𝑎𝑛−1 𝑥) = 𝑥, 𝑖𝑓 𝑥 ∈ 𝑅;
𝑃(−𝑥) = (−𝑥)9 + 9(−𝑥)7 + 7(−𝑥)5 + 5(−𝑥)3 + 3(−𝑥) 4) cosec(𝑐𝑜𝑠𝑒𝑐 −1 𝑥) = 𝑥, 𝑖𝑓 𝑥 ∈ 𝑅\(−1,1)

V.GNANAMURUGAN, P,G,T, G.H.S.S, S.S.KOTTAI, SIVAGANGAI DT – 94874 43870 ; 82489 56766 , www.tnppgta.com, www.ednnet.in Page 59
5) sec(𝑠𝑒𝑐 −1 𝑥) = 𝑥, 𝑖𝑓 𝑥 ∈ 𝑅\(−1,1) 1−𝑥 2
2) 2𝑡𝑎𝑛−1 𝑥 = 𝑐𝑜 −1 ( ),𝑥 ≥ 0
6) cot(𝑐𝑜𝑡 −1 𝑥) = 𝑥, 𝑖𝑓 𝑥 ∈ 𝑅 1+𝑥 2
2𝑥
PROPERTY 𝐈𝐈𝐈 ∶ −RECIPROCAL INVERSE IDENTITIES 3) 2𝑡𝑎𝑛−1 𝑥 = 𝑠𝑖𝑛−1 ( 2) , |𝑥|
≤1
1+𝑥
1
1) 𝑠𝑖𝑛−1 ( ) = 𝑐𝑜𝑠𝑒𝑐𝑥, 𝑖𝑓 𝑥 ∈ 𝑅\(−1,1) PROPERTY 𝐕𝐈𝐈𝐈 ∶ −
𝑥 1 1 1
1 1) 𝑠𝑖𝑛−1 (2𝑥√1 − 𝑥 2 ) = 2𝑠𝑖𝑛−1 𝑥, 𝑖𝑓|𝑥| ≤ 𝑜𝑟 − ≤𝑥≤
2) 𝑐𝑜𝑠 −1 ( ) −1
= 𝑠𝑒𝑐𝑥 𝑥, 𝑖𝑓 𝑥 ∈ 𝑅\(−1,1) √2 √2 √2
𝑥 −1 −1 1
𝑐𝑜𝑡 −1 𝑥 , 𝑥 > 0 2) 𝑠𝑖𝑛 (2𝑥√1 − 𝑥 2) = 2𝑐𝑜𝑠 𝑥, 𝑖𝑓 ≤𝑥≤1
−1 1 √2
3) 𝑡𝑎𝑛 ( ) = {
𝑥 −𝜋 + 𝑐𝑜𝑡 −1 𝑥 , 𝑥 < 0 PROPERTY 𝐈𝐗 ∶ −
PROPERTY 𝐈𝐕 ∶ −REFLECTION IDENTITIES 1) 𝑠𝑖𝑛−1 (𝑥) = 𝑐𝑜𝑠 −1 √1 − 𝑥 2 , 𝑖𝑓 0 ≤ 𝑥 ≤ 1
1) 𝑠𝑖𝑛−1 (−𝑥) = −𝑠𝑖𝑛−1 𝑥, 𝑖𝑓 𝑥 ∈ [−1,1] 2) 𝑠𝑖𝑛−1 (𝑥) = −𝑐𝑜𝑠 −1 √1 − 𝑥 2 , 𝑖𝑓 − 1 ≤ 𝑥 < 0
2) 𝑡𝑎𝑛−1 (−𝑥) = −𝑡𝑎𝑛−1 𝑥, 𝑖𝑓 𝑥 ∈ 𝑅 𝑥
3) 𝑠𝑖𝑛−1 (𝑥) = 𝑡𝑎𝑛−1 ( ) , 𝑖𝑓 − 1 < 𝑥 < 1
3) 𝑐𝑜𝑠𝑒𝑐 −1 (−𝑥) = −𝑐𝑜𝑠𝑒𝑐 −1 𝑥, 𝑖𝑓 |𝑥| ≥ 1 𝑜𝑟 𝑥 ∈ 𝑅\(−1,1) 2 √1−𝑥
−1 (𝑥) −1
4) 𝑐𝑜𝑠 −1 (−𝑥) = 𝜋 − 𝑐𝑜𝑠 −1 𝑥, 𝑖𝑓 𝑥 ∈ [−1,1] 4) 𝑐𝑜𝑠 = 𝑠𝑖𝑛 √1 − 𝑥 2 , 𝑖𝑓 0 ≤ 𝑥 ≤ 1
5) 𝑠𝑒𝑐 −1 (−𝑥) = 𝜋 − 𝑠𝑒𝑐 −1 𝑥|𝑥| ≥ 1 𝑜𝑟 𝑥 ∈ 𝑅\(−1,1) 5) 𝑐𝑜𝑠 −1 (𝑥) = 𝜋 − 𝑠𝑖𝑛−1 √1 − 𝑥 2 , 𝑖𝑓 − 1 ≤ 𝑥 < 0
6) 𝑐𝑜𝑡 −1 (−𝑥) = −𝑐𝑜𝑡 −1 𝑥, 𝑖𝑓 𝑥 ∈ 𝑅 6) 𝑡𝑎𝑛−1 𝑥 = 𝑡𝑎𝑛−1 (
𝑥
) = 𝑐𝑜𝑠 −1 (
1
) , 𝑖𝑓 𝑥 > 0
2 √1+𝑥 2
PROPERTY 𝐕 ∶ −COFUNCTION INVERSE IDENTITIES √1+𝑥
𝜋 PROPERTY 𝐗 ∶ −
1) 𝑠𝑖𝑛−1 𝑥 + 𝑐𝑜𝑠 −1 𝑥 = , 𝑥 ∈ [−1,1] 𝜋 𝜋
2
−1 −1 𝜋 1) 3𝑠𝑖𝑛−1 𝑥 = 𝑠𝑖𝑛−1 (3𝑥 − 4𝑥 3 ), 𝑥 ∈ [− , ]
2) 𝑡𝑎𝑛 𝑥 + 𝑐𝑜𝑡 𝑥 = ,𝑥 ∈ 𝑅 2 2
2 −1 −1 (4𝑥 3 1
𝜋
3) 𝑐𝑜𝑠𝑒𝑐 −1 𝑥 + 𝑠𝑒𝑐 −1 𝑥 = , 𝑥 ∈ 𝑅\(−1,1) 𝑜𝑟 |𝑥| ≥ 1 2) 3𝑐𝑜𝑠 𝑥 = 𝑐𝑜𝑠 − 3𝑥), 𝑥 ∈ [ , 1]
2
2
PROPERTY 𝐕𝐈 ∶ − EXERCISE 4.1
1) 𝑠𝑖𝑛−1 𝑥 + 𝑠𝑖𝑛−1 𝑦 = 𝑠𝑖𝑛−1 (𝑥√1 − 𝑦 2 + 𝑦√1 − 𝑥 2 ), where either 𝑥 2 + 𝑦 2 ≤ 1 1.Find all the values of 𝑥 such that
or 𝑥𝑦 < 0
(𝑖) − 10𝜋 ≤ 𝑥 ≤ 10𝜋 and sin 𝑥 = 0 (𝑖𝑖) − 8𝜋 ≤ 𝑥 ≤ 8𝜋 and sin 𝑥 = −1
(𝑖) sin 𝑥 = 0 ⟹ 𝑥 = 𝑛𝜋, 𝑛 ∈ 𝑧.
2) 𝑠𝑖𝑛−1 𝑥 − 𝑠𝑖𝑛−1 𝑦 = 𝑠𝑖𝑛−1 (𝑥√1 − 𝑦 2 − 𝑦√1 − 𝑥 2 ), where either 𝑥 2 + 𝑦 2 ≤ 1
−10𝜋 ≤ 𝑥 ≤ 10𝜋 ⟹ 𝑥 = 𝑛𝜋, 𝑛 = 0, ±1, ±2, ….
or 𝑥𝑦 > 0
(𝑖𝑖) sin 𝑥 = −1
3) 𝑐𝑜𝑠 −1 𝑥 + 𝑐𝑜𝑠 −1 𝑦 = 𝑐𝑜𝑠 −1 [𝑥𝑦 − √1 − 𝑥 2 √1 − 𝑦 2 ], 𝑖𝑓 𝑥 + 𝑦 ≥ 0 𝜋
⟹ 𝑠𝑖𝑛𝑥 = −𝑠𝑖𝑛 ( )
4) 𝑐𝑜𝑠 −1 𝑥 − 𝑐𝑜𝑠 −1 𝑦 = 𝑐𝑜𝑠 −1 [𝑥𝑦 + √1 − 𝑥 2 √1 − 𝑦 2 ], 𝑖𝑓 𝑥 ≤ 𝑦 2
𝜋
−1 −1 −1 𝑥+𝑦 ⟹ 𝑥 = 𝑛𝜋 + (−1)𝑛 (− ) , 𝑛 ∈ 𝑧
5) 𝑡𝑎𝑛 𝑥 + 𝑡𝑎𝑛 𝑦 = 𝑡𝑎𝑛 (1−𝑥𝑦) , 𝑥𝑦 < 1 2
𝜋
−1 −1 −1 𝑥−𝑦 ⟹ 𝑥 = 𝑛𝜋 − (−1)𝑛+1 ( ) , 𝑛 ∈ 𝐼
6) 𝑡𝑎𝑛 𝑥 − 𝑡𝑎𝑛 𝑦 = 𝑡𝑎𝑛 (1+𝑥𝑦) , 𝑥𝑦 > −1 𝜋
2
𝜋 𝜋 𝜋 𝜋 𝜋 𝜋 𝜋
⟹ 𝑥 = − , 𝜋 + , −𝜋 + , 2𝜋 − , −2𝜋 − , 3𝜋 + , −3𝜋 + , 4𝜋 − ,
PROPERTY 𝐕𝐈𝐈 ∶ − 2
𝜋
2
𝜋
2 2
𝜋 𝜋
2 2
𝜋 𝜋
2 2
𝜋
2𝑥 −4𝜋 − , 5𝜋 + , −5𝜋 + , 6𝜋 − , −6𝜋 − , 7𝜋 + , −7𝜋 + ,
1) 2𝑡𝑎𝑛−1 𝑥 = 𝑡𝑎𝑛−1 ( ) , |𝑥| < 1 2 2 2 2 2 2 2
1−𝑥 2

V.GNANAMURUGAN, P,G,T, G.H.S.S, S.S.KOTTAI, SIVAGANGAI DT – 94874 43870 ; 82489 56766 , www.tnppgta.com, www.ednnet.in Page 60
𝜋 𝜋
8𝜋 − , −8𝜋 − .
2 2
𝜋 3𝜋 5𝜋 7𝜋 9𝜋 11𝜋 13𝜋 15𝜋 17𝜋
⟹𝑥=− , ,− , ,− , ,− , (− ∉ [−8𝜋, 8𝜋])
2 2 2 2 2 2 2 2 2
13𝜋 9𝜋 5𝜋 𝜋 3𝜋 7𝜋 11𝜋 15𝜋
⟹𝑥=− ,− ,− ,− , , , ,
2 2 2 2 2 2 2 2
𝜋
⟹ (4𝑛 − 1) , 𝑛 = 0, ±1, ±2, ±3,4
2
1
2.Find the period and amplitude of (𝑖)𝑦 = sin 7𝑥 (𝑖𝑖)𝑦 = − sin ( 𝑥) (𝑖𝑖𝑖)𝑦 =
3
4 sin(−2𝑥)
2𝜋
Period and amplitude of 𝑦 = 𝑎 sin 𝑏𝑥 are |𝑏| 𝑎𝑛𝑑 |𝑎| 2𝜋 5𝜋
4.Find the value of (𝑖)𝑠𝑖𝑛−1 (sin ( )) (𝑖𝑖) 𝑠𝑖𝑛−1 (sin ( ))
(𝑖)𝑦 = sin 7𝑥 3 4
2𝜋
𝑎 = 1, 𝑏 = 7 (𝑖)𝑠𝑖𝑛−1 (sin ( ))
3
2𝜋 2𝜋 𝜋 𝜋 𝜋 𝜋 𝜋
Period = |7| = ; Amplitude = |𝑎| = |1| = 1 = 𝑠𝑖𝑛−1 (sin (𝜋 − )) = 𝑠𝑖𝑛−1 (sin ( )) = ∈ [− , ]
7 3 3 3 2 2
1 −1 5𝜋
(𝑖𝑖)𝑦 = − sin ( 𝑥) (𝑖𝑖) 𝑠𝑖𝑛 (sin ( 4 ))
3
1 𝜋 𝜋 𝜋 𝜋
𝑎 = −1, 𝑏 = = 𝑠𝑖𝑛−1 (sin (𝜋 + )) = 𝑠𝑖𝑛−1 (− sin ( )) = 𝑠𝑖𝑛−1 (sin (− )) = − ∈
3 4 4 4 4
2𝜋 2𝜋 𝜋 𝜋
Period = 1 = 1 = 6𝜋 ; Amplitude = |𝑎| = |−1| = 1 [− , ]
| | 3 2 2
3
(𝑖𝑖𝑖)𝑦 = 4 sin(−2𝑥) 5.For what value of 𝑥 does sin 𝑥 = 𝑠𝑖𝑛−1 𝑥?
𝑎 = 4, 𝑏 = −2 sin 𝑥 = 0 and 𝑠𝑖𝑛−1 𝑥 = 0
2𝜋 2𝜋 ⟹ 𝑥 = 0 is the solution.
Period = |−2| = = 𝜋 ; Amplitude = |𝑎| = |4| = 4
2
1 6.Find the domain of the following.
3.Sketch the graph of 𝑦 = sin ( 𝑥) 𝑓𝑜𝑟 0 ≤ 𝑥 ≤ 6𝜋 . 𝑥 2 +1 𝜋
3 (𝑖)𝑓(𝑥) = 𝑠𝑖𝑛−1 ( ) (𝑖𝑖)𝑔(𝑥) = 2𝑠𝑖𝑛−1 (2𝑥 − 1) − 4
1 2𝑥
𝑦 = sin ( 𝑥) Domain of 𝑠𝑖𝑛−1 𝑖𝑠 [−1,1]
3
1 𝑥 2 +1
𝑎 = 1, 𝑏 = So, −1 ≤ ≤1
3 2𝑥
2𝜋 2𝜋
Period = 1 = 1 = 6𝜋 ; Amplitude= |𝑎| = |1| = 1 ⟹ −2𝑥 ≤ 𝑥 2 + 1 ≤ 2𝑥
| |
3 3
⟹ −2𝑥 ≤ 𝑥 2 + 1 ⟹ 𝑥 2 + 1 ≤ 2𝑥
𝑥 0 𝜋 2𝜋 3𝜋 4𝜋 5𝜋 6𝜋 ⟹ 0 ≤ 𝑥 2 + 1 + 2𝑥 ⟹ 𝑥 2 + 1 − 2𝑥 ≤ 0
𝑦 0 1⁄ √3⁄ 0 0
2 2 − √3⁄2 − √3⁄2 ⟹ 0 ≤ (𝑥 + 1)2 ⟹ (𝑥 − 1)2 ≤ 0
⟹ (𝑥 + 1)2 ≥ 0 ⟹𝑥−1≤0
⟹𝑥+1≥0 ⟹ 𝑥 ≤ 1 → (2)
⟹ 𝑥 ≥ −1 → (1)
From (1) & (2) ⟹ −1 ≤ 𝑥 ≤ 1
V.GNANAMURUGAN, P,G,T, G.H.S.S, S.S.KOTTAI, SIVAGANGAI DT – 94874 43870 ; 82489 56766 , www.tnppgta.com, www.ednnet.in Page 61
𝑥 2 +1 𝜋 𝜋 𝜋 𝜋
∴ 𝐷𝑜𝑚𝑎𝑖𝑛 𝑜𝑓𝑓(𝑥) = 𝑠𝑖𝑛−1 ( ) 𝑖𝑠 [−1,1]. (𝑖𝑖)𝑠𝑖𝑛−1 (sin (− )) = − ∈ [− , ]
2𝑥 3 3 2 2
(𝑖𝑖) Domain of 𝑠𝑖𝑛−1 𝑖𝑠 [−1,1] 5𝜋 𝜋
(𝑖𝑖𝑖)𝑦 = 𝑠𝑖𝑛−1 (sin ( )) = 𝑠𝑖𝑛−1 (sin (𝜋 − ))
6 6
So,−1 ≤ 2𝑥 − 1 ≤ 1 𝜋
⟹ −1 ≤ 2𝑥 − 1 ⟹ 2𝑥 − 1 ≤ 1 = 𝑠𝑖𝑛−1 (sin ( ))
6
𝜋 𝜋 𝜋
⟹ 0 ≤ 2𝑥 − 1 + 1 ⟹ 2𝑥 ≤ 2 = ∈ [− , ]
6 2 2
⟹ 0 ≤ 2𝑥 ⟹ 𝑥 ≤ 1 → (2)
𝐸𝑋𝐴𝑀𝑃𝐿𝐸 4.4 Find the domain of 𝑠𝑖𝑛−1 (2 − 3𝑥 2 ).
⟹ 𝑥 ≥ 0 → (1)
Domain of 𝑠𝑖𝑛−1 𝑖𝑠 [−1,1]. So, −1 ≤ 2 − 3𝑥 2 ≤ 1.
From (1) & (2) ⟹ 0 ≤ 𝑥 ≤ 1
𝜋 −1 ≤ 2 − 3𝑥 2 2 − 3𝑥 2 ≤ 1
∴ 𝐷𝑜𝑚𝑎𝑖𝑛 𝑜𝑓𝑔(𝑥) = 2𝑠𝑖𝑛−1 (2𝑥 − 1) − 𝑖𝑠 [0,1]
4 ⟹ −3 ≤ −3𝑥 2 ⟹ −3𝑥 2 ≤ −1
5𝜋 𝜋 5𝜋 𝜋
7.Find the value of 𝑠𝑖𝑛−1 (sin cos + cos sin ) ⟹ 3𝑥 2 ≤ 3 ⟹ 3𝑥 2 ≥ 1
9 9 9 9 1
−1 5𝜋 𝜋 5𝜋 𝜋 ⟹ 𝑥 2 ≤ 1 → (1) ⟹ 𝑥 2 ≥ → (2)
𝑠𝑖𝑛 (sin cos + cos sin ) 3
9 9 9 9
5𝜋 𝜋 1 1 1 1
= 𝑠𝑖𝑛−1 (sin ( + )) (∵ sin(𝐴 + 𝐵) = 𝑠𝑖𝑛𝐴𝑐𝑜𝑠𝐵 + 𝑐𝑜𝑠𝐴𝑠𝑖𝑛𝐵) From(1) & (2), ≤ 𝑥 2 ≤ 1 ⟹ ≤ |𝑥| ≤ 1 ⟹ 𝑥 ∈ [−1, − ] ∪ [ , 1].
9 9 3 √3 √3 √3
6𝜋
= 𝑠𝑖𝑛−1 (sin ( )) (∵ 𝐼𝑓 𝑎 ≤ |𝑥| ≤ 𝑏, 𝑡ℎ𝑒𝑛 𝑥 ∈ [−𝑏, −𝑎] ∪ [𝑎, 𝑏])
9
2𝜋 EXERCISE 4.2
= 𝑠𝑖𝑛−1 (sin ( ))
3 1.Find all value of 𝑥 such that
𝜋
= 𝑠𝑖𝑛−1 (sin (𝜋 − )) (𝑖) − 6𝜋 ≤ 𝑥 ≤ 6𝜋 and cos 𝑥 = 0 (𝑖𝑖) − 5𝜋 ≤ 𝑥 ≤ 5𝜋 and cos 𝑥 = 1
3 𝜋
𝜋 (𝑖) cos 𝑥 = 0 ⟹ 𝑥 = (2𝑛 + 1) , 𝑛 ∈ 𝑧
= 𝑠𝑖𝑛−1 (sin ( )) 𝜋
2
3
𝜋 𝜋 𝜋 cos 𝑥 = (2𝑛 + 1) , 𝑛 = 0, ±1, ±2, ±3, ±4, ±5.
= ∈ [− , ] 2
3 2 2 (𝑖𝑖) cos 𝑥 = 1 ⟹ cos 𝑥 = cos 0 ⟹ 𝛼 = 0
1
𝐸𝑋𝐴𝑀𝑃𝐿𝐸 4.1 Find the principal value of 𝑠𝑖𝑛−1 (− ) (in radians and degrees). ⟹ 𝑥 = 2𝑛𝜋 ± 𝛼 = 2𝑛𝜋, 𝑛 = 0, ±1, ±2
2
𝜋 𝜋
1 𝜋
𝑦 = 𝑠𝑖𝑛−1 (− ) = 𝑠𝑖𝑛−1 (−𝑠𝑖𝑛 ( )) = 𝑠𝑖𝑛−1 (𝑠𝑖𝑛 (− )) = −
𝜋 𝜋 2.State the reason for 𝑐𝑜𝑠 −1 (cos (− )) ≠ − .
6 6
2 6 6 6 𝜋 𝜋
𝜋 𝜋 𝜋 𝜋 0 𝑐𝑜𝑠 −1 (cos (− )) = − ∉ [0, 𝜋] (∵ 𝑐𝑜𝑠 −1 (cos(𝑥)) = 𝑥)
So,𝑦 = − ∈ [− , ] . 𝑦 = − = −30 . 6 6
6 2 2 6 𝜋 𝜋
𝐸𝑋𝐴𝑀𝑃𝐿𝐸 4.2 Find the principal value of 𝑠𝑖𝑛 if it exists. −1 (2), So, 𝑐𝑜𝑠 −1 (cos (− )) ≠ − .
6 6
−1 −1 −1 (−𝑥) −1 (𝑥)
Domain of 𝑠𝑖𝑛 𝑖𝑠 [−1,1]. But 2 ∉ [−1,1]. So 𝑠𝑖𝑛 (2) does not exists. 3.Is 𝑐𝑜𝑠 = 𝜋 − 𝑐𝑜𝑠 true? Justify your answer.
−1 (−𝑥) −1 (𝑥)
𝐸𝑋𝐴𝑀𝑃𝐿𝐸 4.3 Find the principal value of 𝑐𝑜𝑠 = 𝜋 − 𝑐𝑜𝑠 is true.
−1 (−𝑥)
1 𝜋 5𝜋
(𝑖)𝑠𝑖𝑛−1 ( ) (𝑖𝑖)𝑠𝑖𝑛−1 (sin (− )) (𝑖𝑖𝑖)𝑠𝑖𝑛−1 (sin ( )) 𝑐𝑜𝑠 =𝛼
√2 3 6 ⟹ −𝑥 = cos 𝛼
1 𝜋 𝜋 𝜋 𝜋
(𝑖)𝑦 = 𝑠𝑖𝑛−1 ( ) = 𝑠𝑖𝑛−1 (𝑠𝑖𝑛 ( )) = ∈ [− , ] ⟹ 𝑥 = − cos 𝛼
2 √ 4 4 2 2
⟹ 𝑥 = cos(𝜋 − 𝛼)
V.GNANAMURUGAN, P,G,T, G.H.S.S, S.S.KOTTAI, SIVAGANGAI DT – 94874 43870 ; 82489 56766 , www.tnppgta.com, www.ednnet.in Page 62
⟹ 𝑐𝑜𝑠 −1 𝑥 = 𝜋 − 𝛼 −1 ≤
|𝑥|−2
≤1 −1 ≤
1−|𝑥|
≤1
⟹ 𝛼 = 𝜋 − 𝑐𝑜𝑠 −1 𝑥 3 4
⟹ −3 ≤ |𝑥| − 2 ≤ 3 ⟹ −4 ≤ 1 − |𝑥| ≤ 4
⟹ 𝑐𝑜𝑠 −1 (−𝑥) = 𝜋 − 𝑐𝑜𝑠 −1 (𝑥)
1 ⟹ −1 ≤ |𝑥| ≤ 5 ⟹ −5 ≤ −|𝑥| ≤ 3
4.Find the principal value of 𝑐𝑜𝑠 −1 ( ). ⟹ |𝑥| ≤ 5 ⟹ −3 ≤ |𝑥| ≤ 5
2
1 𝜋 𝜋 ⟹ |𝑥| ≤ 5
𝑐𝑜𝑠 −1 ( ) = 𝑐𝑜𝑠 −1 (𝑐𝑜𝑠 ( )) = ∈ [0, 𝜋]
2 3 3 ∴𝑥 ∈ [−5,5]
1 𝜋
∴ The principal value of 𝑐𝑜𝑠 −1 ( ) is . (𝑖𝑖) The domain of 𝑠𝑖𝑛−1 𝑥 and 𝑐𝑜𝑠 −1 𝑥 is[−1,1].
2 3
1 1 Hence the domain of the given function is [−1,1].
5.Find the value of (𝑖)2𝑐𝑜𝑠 −1 ( ) + 𝑠𝑖𝑛−1 ( ) 𝜋
2 2 7.For what value of 𝑥, the inequality < 𝑐𝑜𝑠 −1 (3𝑥 − 1) < 𝜋 holds?
−1 1 −1 −1 𝜋 𝜋 𝜋 𝜋 2
(𝑖𝑖)𝑐𝑜𝑠 ( ) + 𝑠𝑖𝑛 (−1) (𝑖𝑖𝑖)𝑐𝑜𝑠 (cos cos − sin sin ) 𝜋
< 𝑐𝑜𝑠 −1 (3𝑥
− 1) < 𝜋
2 7 17 7 17
1 1 2
−1
(𝑖)2𝑐𝑜𝑠 ( ) + 𝑠𝑖𝑛 ( ) −1
2 2
𝑐𝑜𝑠 −1 𝑥 lies in the second quadrant, if −1 < 𝑥 < 0.
𝜋 𝜋 1
= 2( ) + ( ) −1 < 3𝑥 − 1 < 0 ⟹ 0 < 3𝑥 < 1 ⟹ 0 < 𝑥 <
3 6 3
4𝜋+𝜋 5 1
= = ∴𝑥 ∈ (0, )
6 6 3
1 4 4
(𝑖𝑖)𝑐𝑜𝑠 ( ) + 𝑠𝑖𝑛−1 (−1)
−1
8. Find the value of (𝑖) cos ( 𝑐𝑜𝑠 −1 ( ) + 𝑠𝑖𝑛−1 ( ))
2 5 5
−1 1 4𝜋 5𝜋
= 𝑐𝑜𝑠 ( ) − 𝑠𝑖𝑛−1 (1) (𝑖𝑖)𝑐𝑜𝑠 −1 (cos ( ) + 𝑐𝑜𝑠 −1 (cos ( )))
2 3 4
𝜋 𝜋
= − 4 4
3 2 (𝑖) cos ( 𝑐𝑜𝑠 −1 ( ) + 𝑠𝑖𝑛−1 ( ))
2𝜋−3𝜋 𝜋 5 5
= =− 4
6
𝜋
6
𝜋 𝜋 𝜋
Let 𝑠𝑖𝑛−1 ( ) = 𝛼.
5
(𝑖𝑖𝑖)𝑐𝑜𝑠 −1 (cos 7 cos 17 − sin 7 sin 17) 4
𝜋 𝜋
⟹ sin 𝛼 =
5
= 𝑐𝑜𝑠 −1 (cos ( + )) 𝜋 4
7 17
17𝜋+7𝜋
⟹ cos ( − 𝛼) =
2 5
= 𝑐𝑜𝑠 −1 (cos ( )) 𝜋 −1 4
119
24𝜋
⟹ − 𝛼 = 𝑐𝑜𝑠 ( )
−1 2 5
= 𝑐𝑜𝑠 (cos ( 119 )) 𝜋 −1 4
24𝜋
⟹ = 𝑐𝑜𝑠 ( ) + 𝛼
2 5
= 4 4 𝜋
119
|𝑥|−2 1−|𝑥|
⟹ 𝑐𝑜𝑠 −1 ( ) + 𝑠𝑖𝑛−1 ( ) =
5 5 2
6.Find the domain of (𝑖)𝑓(𝑥) = 𝑠𝑖𝑛−1 ( ) + 𝑐𝑜𝑠 −1 ( 4 )
3 −1 4 −1 4 𝜋
∴ cos ( 𝑐𝑜𝑠 (5) + 𝑠𝑖𝑛 (5)) = cos 2 = 0
(𝑖𝑖)𝑔(𝑥) = 𝑠𝑖𝑛−1 𝑥 + 𝑐𝑜𝑠 −1 (𝑥)
The domain of 𝑠𝑖𝑛−1 𝑥 is [−1,1] and the domain of 𝑐𝑜𝑠 −1 𝑥 is [−1,1]. 4𝜋
(𝑖𝑖)𝑐𝑜𝑠 −1 (cos ( ) + 𝑐𝑜𝑠 −1 (cos ( )))
5𝜋
|𝑥|−2 1−|𝑥| 3 4
So, −1 ≤ ≤ 1 ; −1 ≤ ≤1 𝜋
= 𝑐𝑜𝑠 −1 (cos (𝜋 + ) + 𝑐𝑜𝑠 −1 (cos (𝜋 + )))
𝜋
3 4
3 4
V.GNANAMURUGAN, P,G,T, G.H.S.S, S.S.KOTTAI, SIVAGANGAI DT – 94874 43870 ; 82489 56766 , www.tnppgta.com, www.ednnet.in Page 63
𝜋 𝜋 5𝜋
= 𝑐𝑜𝑠 −1 (− cos ( ) + 𝑐𝑜𝑠 −1 (−cos ( ))) =
6
∈ [0, 𝜋]
3 4
𝜋 𝜋 2+sin 𝑥
= 𝑐𝑜𝑠 −1 (cos (𝜋 − ) + 𝑐𝑜𝑠 −1 (cos (𝜋 − ))) 𝐸𝑋𝐴𝑀𝑃𝐿𝐸 4.7 Find the domain of 𝑐𝑜𝑠 −1 ( )
3 4 3
2𝜋 3𝜋 2+sin 𝑥
= 𝑐𝑜𝑠 −1 (cos ( ) + 𝑐𝑜𝑠 −1 (cos ( ))) The domain of 𝑐𝑜𝑠 −1 𝑥 is [−1,1]. So −1 ≤ ≤1
3 4 3
2𝜋 3𝜋 ⟹ −3 ≤ 2 + sin 𝑥 ≤ 3
= +
3 4 ⟹ −5 ≤ 𝑠𝑖𝑛𝑥 ≤ 1
8𝜋+9𝜋
= ⟹ −1 ≤ 𝑠𝑖𝑛𝑥 ≤ 1
12
17𝜋
= ⟹ −𝑠𝑖𝑛−1 (1) ≤ 𝑥 ≤ 𝑠𝑖𝑛−1 (1)
12 𝜋 𝜋
√3 ⟹− ≤𝑥≤
𝐸𝑋𝐴𝑀𝑃𝐿𝐸 4.5 Find the principal value of 𝑐𝑜𝑠 −1 ( ) 2 2
2+sin 𝑥 𝜋 𝜋
2
−1 (𝑥) ∴ The domain of 𝑐𝑜𝑠 −1 ( ) 𝑖𝑠 [− 2 , 2 ].
The range of the principal values of 𝑐𝑜𝑠 𝑖𝑠[0, 𝜋]. 3
√3 𝜋 𝜋 EXERCISE 4.3
𝑦 = 𝑐𝑜𝑠 −1 ( ) = 𝑐𝑜𝑠 −1 (𝑐𝑜𝑠 ( )) = ∈ [0, 𝜋]
2 6 6 1. Find the domain of the following functions :
√3 𝜋 1 𝜋
∴ The principal value of𝑐𝑜𝑠 −1 ( ) is . (𝑖)𝑡𝑎𝑛−1 (√9 − 𝑥 2 ) (𝑖𝑖) 𝑡𝑎𝑛−1 (1 − 𝑥 2 ) −
2 6 2 4
1 𝜋 −1
(𝑖)The domain of 𝑡𝑎𝑛 𝑥 𝑖𝑠 (−∞, ∞).
𝐸𝑋𝐴𝑀𝑃𝐿𝐸 4.6 Find (𝑖)𝑐𝑜𝑠 −1 (− ) (𝑖𝑖)𝑐𝑜𝑠 −1 (cos (− 3 ))
√ 2
7𝜋 If √9 − 𝑥 2 is real, then √9 − 𝑥 2 ≥ 0
(𝑖𝑖𝑖)𝑐𝑜𝑠 −1 (cos ( 6 )) ⟹ 9 − 𝑥2 ≥ 0
Inverse cosine function 𝑐𝑜𝑠 −1 : [−1,1] → [0, 𝜋]. ⟹ 9 ≥ 𝑥2
(𝑖)𝑐𝑜𝑠 −1 (−
1 𝜋 ⟹ 𝑥2 ≤ 9
) = 𝑐𝑜𝑠 −1 (−𝑐𝑜𝑠 ( 4 ))
√ 2 ⟹ −3 ≤ 𝑥 ≤ 3
𝜋
= 𝑐𝑜𝑠 −1 (cos (𝜋 − )) So, The domain of 𝑡𝑎𝑛−1 (√9 − 𝑥 2 ) 𝑖𝑠 [−3,3].
4
=𝜋−
𝜋 (𝑖𝑖)The domain of 𝑡𝑎𝑛−1 𝑥 𝑖𝑠 (−∞, ∞).
4
3𝜋 For all values of 𝑥, 1 − 𝑥 2 is real.
= ∈ [0, 𝜋] 1 𝜋
4 So,the domain of 𝑡𝑎𝑛−1 (1 − 𝑥 2 ) − 𝑖𝑠 𝑅.
𝜋 𝜋 2 4
(𝑖𝑖)𝑐𝑜𝑠 −1 (cos (− )) = 𝑐𝑜𝑠 −1 (cos ( )) 5𝜋 𝜋
3 3
𝜋 2.Find the value of (𝑖)𝑡𝑎𝑛−1 (tan ) (𝑖𝑖)𝑡𝑎𝑛−1 (tan (− 6 ))
4
= ∈ [0, 𝜋] 5𝜋 𝜋 𝜋 𝜋 𝜋 𝜋
7𝜋
3
𝜋
(𝑖)𝑡𝑎𝑛−1 (tan ) = 𝑡𝑎𝑛−1 (tan (𝜋 + 4 )) = 𝑡𝑎𝑛−1 (tan ( 4 )) = ∈ (− , )
4 4 2 2
(𝑖𝑖𝑖)𝑐𝑜𝑠 −1 (cos ( )) = 𝑐𝑜𝑠 −1 (cos (𝜋 + )) 𝜋 𝜋 𝜋 𝜋
6 6 (𝑖𝑖)𝑡𝑎𝑛−1
𝜋 (tan (− 6 )) = − 6 ∈ (− 2 , 2 )
−1
= 𝑐𝑜𝑠 (− cos ( 6 )) 7𝜋
𝜋 3. Find the value of (𝑖) tan (𝑡𝑎𝑛−1 ( )) (𝑖𝑖) tan(𝑡𝑎𝑛−1 (1947))
= 𝑐𝑜𝑠 −1 (cos (𝜋 − )) 4
6
𝜋 (𝑖𝑖𝑖) tan(𝑡𝑎𝑛−1 (−0.2021))
=𝜋−
6

V.GNANAMURUGAN, P,G,T, G.H.S.S, S.S.KOTTAI, SIVAGANGAI DT – 94874 43870 ; 82489 56766 , www.tnppgta.com, www.ednnet.in Page 64
7𝜋 7𝜋 2
(𝑖) tan (𝑡𝑎𝑛−1 ( )) = =−
4 4 5√5
2 √5
(𝑖𝑖) tan(𝑡𝑎𝑛−1 (1947)) = 1947 = − .
5√5 √5
(𝑖𝑖𝑖) tan(𝑡𝑎𝑛−1 (−0.2021)) = −0.2021 2√5
=−
1 1 25
4. Find the value of (𝑖) tan (𝑐𝑜𝑠 −1 ( ) − 𝑠𝑖𝑛 −1
(− 2)) 4 3
2 (𝑖𝑖𝑖) cos (𝑠𝑖𝑛−1 ( ) − 𝑡𝑎𝑛−1 ( ))
1 4 4 3 5 4
(𝑖𝑖) sin (𝑡𝑎𝑛−1 ( ) − 𝑐𝑜𝑠 −1
(5)) (𝑖𝑖𝑖) cos (𝑠𝑖𝑛−1 ( ) − 𝑡𝑎𝑛−1 ( )) 4 3
2 5 4 𝑐𝑜𝑠(𝑥 − 𝑦) = 𝑐𝑜𝑠𝑥𝑐𝑜𝑠𝑦 + 𝑠𝑖𝑛𝑥𝑠𝑖𝑛𝑦 . Let 𝑠𝑖𝑛−1 ( ) = 𝑥 ; 𝑡𝑎𝑛−1 ( ) = 𝑦 .
5 4
−1 1 1
(𝑖) tan (𝑐𝑜𝑠 (2) − 𝑠𝑖𝑛−1 (− 2)) 4
𝑠𝑖𝑛−1 ( ) = 𝑥 ⟹ 𝑠𝑖𝑛𝑥 =
4
5 5
𝜋 𝜋 16 9 3
= tan ( − (− )) 𝑐𝑜𝑠𝑥 = √1 − 𝑠𝑖𝑛2 𝑥 = √1 − =√ =
3 6 25 25 5
𝜋 𝜋
= tan ( + ) 3
𝑡𝑎𝑛−1 ( ) = 𝑦 ⟹ 𝑡𝑎𝑛𝑦 =
3
3 6
𝜋 4 4
= tan ( ) 2 2
𝑠𝑒𝑐 𝑦 = 1 + 𝑡𝑎𝑛 𝑦 = 1 +
9
=
25
⟹ 𝑠𝑒𝑐𝑦 = ⟹ 𝑐𝑜𝑠𝑦 =
5 4
2
16 16 4 5
=∞ 16 9 3
1 4 𝑠𝑖𝑛𝑦 = √1 − 𝑐𝑜𝑠 2 𝑦 = √1 − =√ =
(𝑖𝑖) sin (𝑡𝑎𝑛−1 ( ) − 𝑐𝑜𝑠 −1 ( )) 25 25 5
2 5
4 3
𝑠𝑖𝑛(𝑥 − 𝑦) = 𝑠𝑖𝑛𝑥𝑐𝑜𝑠𝑦 − 𝑐𝑜𝑠𝑥𝑠𝑖𝑛𝑦. Let 𝑡𝑎𝑛−1 ( ) = 𝑥 ; 𝑐𝑜𝑠 −1 ( ) = 𝑦.
1 4 cos (𝑠𝑖𝑛−1 ( ) − 𝑡𝑎𝑛−1 ( )) = 𝑐𝑜𝑠(𝑥 − 𝑦)
2 5 5 4
1
𝑡𝑎𝑛−1 ( ) = 𝑥 ⟹ tan 𝑥 =
1 = 𝑐𝑜𝑠𝑥𝑐𝑜𝑠𝑦 + 𝑠𝑖𝑛𝑥𝑠𝑖𝑛𝑦
2 2 3 4 4 3
1 5 √5 2
= . + .
5 5 5 5
𝑠𝑒𝑐 2 𝑥 = 1 + 𝑡𝑎𝑛2 𝑥 = 1 + = ⟹ 𝑠𝑒𝑐𝑥 = ⟹ 𝑐𝑜𝑠𝑥 = 12+12
4 4 2 √5 =
25
4 1 1 25
𝑠𝑖𝑛𝑥 = √1 − 𝑐𝑜𝑠 2 𝑥 = √1 − = √ = =
5 5 √5 25
4
𝑐𝑜𝑠 −1 ( ) = 𝑦 ⟹ 𝑐𝑜𝑠𝑦 =
4 𝐸𝑋𝐴𝑀𝑃𝐿𝐸 4.8 Find the principal value of 𝑡𝑎𝑛−1 (√3).
5 5
𝜋 𝜋 𝜋 𝜋
16 9 3 𝑦 = 𝑡𝑎𝑛−1 (√3) = 𝑡𝑎𝑛−1 (𝑡𝑎𝑛 ( )) ⟹ 𝑦 = ∈ (− , )
𝑠𝑖𝑛𝑦 = √1 − 𝑐𝑜𝑠 2 𝑦 = √1 − =√ = 3 3 2 2
25 25 5 3𝜋
1 4 𝐸𝑋𝐴𝑀𝑃𝐿𝐸 4.9 Find (𝑖)𝑡𝑎𝑛−1 (−√3) (𝑖𝑖)𝑡𝑎𝑛−1 (tan )
5
sin (𝑡𝑎𝑛−1 ( ) − 𝑐𝑜𝑠 −1 ( )) = 𝑠𝑖𝑛(𝑥 − 𝑦)
2 5 (𝑖𝑖𝑖) tan(𝑡𝑎𝑛−1 (2019))
= 𝑠𝑖𝑛𝑥𝑐𝑜𝑠𝑦 − 𝑐𝑜𝑠𝑥𝑠𝑖𝑛𝑦 𝜋 𝜋 𝜋 𝜋
1 4 2 3 (𝑖 )𝑡𝑎𝑛−1 (−√3) = 𝑡𝑎𝑛−1 (𝑡𝑎𝑛 (− )) = − ∈ (− , )
= . − . 3 3 2 2
√5 5 √5 5 3𝜋 3𝜋
=
4−6 (𝑖𝑖)𝑡𝑎𝑛−1 (tan ( )) = 𝑡𝑎𝑛−1 (tan ( − 𝜋))
5 5
5√5 2𝜋
= 𝑡𝑎𝑛−1 (tan (− ))
5
V.GNANAMURUGAN, P,G,T, G.H.S.S, S.S.KOTTAI, SIVAGANGAI DT – 94874 43870 ; 82489 56766 , www.tnppgta.com, www.ednnet.in Page 65
2𝜋 𝜋 𝜋 𝜋
=− ∈ (− , ) = ∈ [0, 𝜋]
5 2 2 6
(𝑖𝑖𝑖) tan(𝑡𝑎𝑛−1 (2019)) = 2019 (𝑖𝑖)𝑦 = 𝑐𝑜𝑡 −1 (√3) ⟹ 𝑐𝑜𝑡𝑦 = √3
1 1 1
𝐸𝑋𝐴𝑀𝑃𝐿𝐸 4.10 Find the value of 𝑡𝑎𝑛−1 (−1) + 𝑐𝑜𝑠 −1 ( ) + 𝑠𝑖𝑛−1 (− ) ⟹ = √3
2 2 𝑡𝑎𝑛𝑦
𝜋 𝜋 𝜋 𝜋 𝜋 1
𝑡𝑎𝑛−1 (−1) = 𝑡𝑎𝑛−1 (−𝑡𝑎𝑛 ) = 𝑡𝑎𝑛−1 (𝑡𝑎𝑛 (− )) = − ∈ (− , ) ⟹ 𝑡𝑎𝑛𝑦 = ( )
4 4 4 2 2 3 √
−1 1
1 𝜋 𝜋 ⟹ 𝑦 = 𝑡𝑎𝑛 ( 3)
𝑐𝑜𝑠 −1 ( ) = 𝑐𝑜𝑠 −1
(𝑐𝑜𝑠 ( 3 )) = ∈ [0, 𝜋] √
2 3 𝜋
−1 1 𝜋 𝜋 𝜋 𝜋 𝜋 = 𝑡𝑎𝑛−1 (𝑡𝑎𝑛 ( ))
𝑠𝑖𝑛 (− 2) = 𝑠𝑖𝑛−1 (−𝑠𝑖𝑛 6 ) = 𝑠𝑖𝑛−1 (𝑠𝑖𝑛 (− 6 )) = − 6 ∈ [− 2 , 2 ] 6
𝜋 𝜋 𝜋
1 1 𝜋 𝜋 𝜋 𝜋 = ∈ (− , )
𝑡𝑎𝑛−1 (−1) + 𝑐𝑜𝑠 −1 ( ) + 𝑠𝑖𝑛−1 (− ) = − + − = − 6 2 2
2 2 4 3 6 12 (𝑖𝑖𝑖)𝑦 = 𝑐𝑜𝑠𝑒𝑐 −1
𝑥 (−√2) ⟹ 𝑐𝑜𝑠𝑒𝑐𝑦 = −√2
𝐸𝑋𝐴𝑀𝑃𝐿𝐸 4.11 Prove that tan(𝑠𝑖𝑛−1 (𝑥)) = , −1 < 𝑥 < 1. 1
√1−𝑥 2 ⟹ = −√2
𝐼𝑓 𝑥 = 0 then both sides are equal to 0 → (1) 𝑠𝑖𝑛𝑦
1
𝜋 ⟹ 𝑠𝑖𝑛𝑦 = −
𝐴𝑠𝑠𝑢𝑚𝑒 𝑡ℎ𝑎𝑡 0 < 𝑥 < 1.Let 𝜃 = 𝑠𝑖𝑛−1 𝑥.Then 0 < 𝜃 < . Now √2
2
𝑥 1
𝜃 = 𝑠𝑖𝑛−1 𝑥 ⟹ sin 𝜃 = ⟹ Opposie side = 𝑥, Hypotenuse = 1 ⟹ 𝑦 = 𝑠𝑖𝑛 (−
√2
)
1
Adjacent side = √1 − 𝑥 2. = 𝑠𝑖𝑛 (−𝑠𝑖𝑛 ( ))
𝜋
𝑥 𝑥 4
tan 𝜃 = 2
⟹ tan(𝑠𝑖𝑛−1 (𝑥)) = → (2)
√1−𝑥 √1−𝑥 2 𝜋
−1 𝜋 = 𝑠𝑖𝑛 (𝑠𝑖𝑛 (− ))
𝐿𝑒𝑡 − 1 < 𝑥 < 0. 𝑇ℎ𝑒𝑛 𝜃 = 𝑠𝑖𝑛 𝑥 gives − < 𝜃 < 0. 4
2 𝜋 𝜋 𝜋
𝑥 𝑥 𝑥
−1
𝜃 = 𝑠𝑖𝑛 𝑥 ⟹ sin 𝜃 = ⟹ tan 𝜃 = ⟹ tan(𝑠𝑖𝑛−1 (𝑥)) = → (3) = − ∈ [− , ]
1 √1−𝑥 2 √1−𝑥 2 4 2 2
𝑥
𝐹𝑟𝑜𝑚 (1), (2), (3), tan(𝑠𝑖𝑛−1 (𝑥)) = , −1 < 𝑥 < 1 2.Find the value of (𝑖)𝑡𝑎𝑛−1 (√3) − 𝑠𝑒𝑐 −1 (−2)
√1−𝑥 2 1
EXERCISE 4.4 (𝑖𝑖)𝑠𝑖𝑛−1 (−1) + 𝑐𝑜𝑠 −1 ( ) + 𝑐𝑜𝑡 −1 (2)
2
2
1.Find the principal value of (𝑖)𝑠𝑒𝑐 −1 ( ) (𝑖𝑖)𝑐𝑜𝑡 −1 (√3) (𝑖𝑖𝑖)𝑐𝑜𝑠𝑒𝑐 −1 (−√2) (𝑖𝑖𝑖)𝑐𝑜𝑡 −1 (1) + 𝑠𝑖𝑛−1 (−
√3
) − 𝑠𝑒𝑐 −1 (−√2)
3 √ 2
2 2
(𝑖)𝑦 = 𝑠𝑒𝑐 −1 ( ) ⟹ 𝑠𝑒𝑐𝑦 = (𝑖) 𝑡𝑎𝑛−1 (√3) − 𝑠𝑒𝑐 −1 (−2)
3
√ √3
1 2 𝜋 1
⟹ = = 𝑡𝑎𝑛−1 (tan ( )) − 𝑐𝑜𝑠 (− )
𝑐𝑜𝑠𝑦 √3 3 2
√3 𝜋 𝜋
⟹ 𝑐𝑜𝑠𝑦 = ( ) = 𝑡𝑎𝑛−1 (tan ( )) − cos (−𝑐𝑜𝑠 ( ))
2 3 3
√3 𝜋 𝜋
⟹ 𝑦 = 𝑐𝑜𝑠 −1 ( ) = 𝑡𝑎𝑛−1 (tan ( )) − cos (𝑐𝑜𝑠 (𝜋 − ))
2 3 3
𝜋 𝜋 𝜋
= 𝑐𝑜𝑠 −1 (𝑐𝑜𝑠 ) = − (𝜋 − )
6 3 3

V.GNANAMURUGAN, P,G,T, G.H.S.S, S.S.KOTTAI, SIVAGANGAI DT – 94874 43870 ; 82489 56766 , www.tnppgta.com, www.ednnet.in Page 66
𝜋 2𝜋 1
= − 𝐸𝑋𝐴𝑀𝑃𝐿𝐸 4.15 Show that 𝑐𝑜𝑡 −1 ( ) = 𝑠𝑒𝑐 −1 𝑥, |𝑥| > 1
3 3 √𝑥 2 −1
𝜋 1
=− 𝐿𝑒𝑡 𝑐𝑜𝑡 −1 ( ) = 𝑦 → (1)
3 √𝑥 2 −1
−1 1
(𝑖𝑖)𝑠𝑖𝑛 (−1) + 𝑐𝑜𝑠 −1 ( ) + 𝑐𝑜𝑡 −1 (2) ⟹ 𝑐𝑜𝑡𝑦 =
1
2 √𝑥 2 −1
−1 𝜋 −1 𝜋 −1 (2) √𝑥 2 −1
= 𝑠𝑖𝑛 (𝑠𝑖𝑛 (− 2 )) + 𝑐𝑜𝑠 (cos ( 3 )) + 𝑐𝑜𝑡 ⟹ 𝑡𝑎𝑛𝑦 = ; 𝑂𝑝𝑝𝑜𝑠𝑖𝑡𝑒 𝑠𝑖𝑑𝑒 = √𝑥 2 − 1 ; 𝑎𝑑𝑗𝑎𝑐𝑒𝑛𝑡 𝑠𝑖𝑑𝑒 = 1
1
𝜋 𝜋
= − + + 𝑐𝑜𝑡 −1 (2) 2
2
𝜋
3 𝐻𝑦𝑝𝑜𝑡𝑒𝑛𝑢𝑠𝑒 = √(√𝑥 2 − 1) + (1)2 = √𝑥 2 − 1 + 1 = √𝑥 2 = 𝑥
= − + 𝑐𝑜𝑡 −1 (2) 𝑥
6
√3
𝑠𝑒𝑐𝑦 = ⟹ 𝑠𝑒𝑐 −1 𝑥 = 𝑦 → (2)
1
(𝑖𝑖𝑖)𝑐𝑜𝑡 −1 (1)+ 𝑠𝑖𝑛−1 (− ) − 𝑠𝑒𝑐 −1 (−√2) 1
2 (1), (2) ⟹ 𝑐𝑜𝑡 −1 ( ) = 𝑠𝑒𝑐 −1 𝑥
√3 1 √𝑥 2 −1
= tan(1) + 𝑠𝑖𝑛−1 (− sin ( )) − cos (− ) EXERCISE 4.5
2 √2
𝜋 𝜋 𝜋
= −1
tan ( ) + 𝑠𝑖𝑛 (sin (− )) − cos (𝜋 − ) 1.Find the value, if it exists. If not, give the reason for non-existence
4 3 4 5𝜋
𝜋
= − −
𝜋 3𝜋 (𝑖)𝑠𝑖𝑛−1 (cos 𝜋) (𝑖𝑖)𝑡𝑎𝑛−1 (sin (− )) (𝑖𝑖𝑖)𝑠𝑖𝑛−1 (sin 5)
2
4 3 4
2𝜋 𝜋 𝜋 𝜋 𝜋
=− − (𝑖)𝑠𝑖𝑛−1 (cos 𝜋) = 𝑠𝑖𝑛−1 (−1) = 𝑠𝑖𝑛−1 (−𝑠𝑖𝑛 ( )) = 𝑠𝑖𝑛−1 (𝑠𝑖𝑛 (− )) = −
4 3 2 2 2
𝜋 𝜋 5𝜋 5𝜋
=− − (𝑖𝑖)𝑡𝑎𝑛−1 (sin (− )) = 𝑡𝑎𝑛−1 (− sin ( 2 ))
2 3 2
5𝜋 𝜋
=− = 𝑡𝑎𝑛−1 (− sin (2𝜋 + ))
6 2
𝐸𝑋𝐴𝑀𝑃𝐿𝐸 4.12 Find the principal value of (𝑖)𝑐𝑜𝑠𝑒𝑐 −1 (−1) (𝑖𝑖)𝑠𝑒𝑐 −1 (−2) 𝜋
𝜋 𝜋 𝜋
= 𝑡𝑎𝑛−1 (− sin ( ))
2
(𝑖)𝑐𝑜𝑠𝑒𝑐 −1 (−1) = 𝑠𝑖𝑛(−1) = −𝑠𝑖𝑛(1) ⟹ − ∈ [− , ] −1 (−1)
2 2 2 = 𝑡𝑎𝑛
1 𝜋 2𝜋 𝜋
(𝑖𝑖)𝑠𝑒𝑐 −1 (−2) = 𝑐𝑜𝑠 (− ) ⟹ 𝜋 − = ∈ [0, 𝜋] − { } = 𝑡𝑎𝑛−1 (−𝑡𝑎𝑛 ( ))
𝜋
2 3 3 2
4
2√ 3
𝐸𝑋𝐴𝑀𝑃𝐿𝐸 4.13 Find the value of 𝑠𝑒𝑐 −1 (− ) 𝜋
3 = 𝑡𝑎𝑛−1 (𝑡𝑎𝑛 (− ))
2√3 2 √3 𝜋 5𝜋 𝜋 4
𝑠𝑒𝑐 −1 (− ) = 𝑠𝑒𝑐 −1 (− ) = 𝑐𝑜𝑠 (− ) ⟹𝜋− = ∈ [0, 𝜋] − { } =−
𝜋
3 √3 2 6 6 2
4
1
𝐸𝑋𝐴𝑀𝑃𝐿𝐸 4.14 If 𝑐𝑜𝑡 −1 ( ) = 𝜃, Find the value of cos 𝜃. (𝑖𝑖𝑖)𝑠𝑖𝑛−1 (sin 5) = 𝑠𝑖𝑛−1 [𝑠𝑖𝑛(2𝜋 − (2𝜋 − 5))]
7
1
𝑐𝑜𝑡 −1 ( ) = 𝜃 = 𝑠𝑖𝑛−1 [−𝑠𝑖𝑛(2𝜋 − 5)]
7
1 7
= −𝑠𝑖𝑛−1 [𝑠𝑖𝑛(2𝜋 − 5)]
⟹ cot 𝜃 = ⟹ tan 𝜃 = ⟹ 𝑂𝑝𝑝𝑜𝑠𝑖𝑡𝑒 𝑠𝑖𝑑𝑒 = 7 ; 𝑎𝑑𝑗𝑎𝑐𝑒𝑛𝑡 𝑠𝑖𝑑𝑒 = 1 = −(2𝜋 − 5)
7 1
𝐻𝑦𝑝𝑜𝑡𝑒𝑛𝑢𝑠𝑒 = √72 + 2
1 = √50 = √25 × 2 = 5√2 = 5 − 2𝜋
So, cos 𝜃 =
1 2.Find the value of the expression in terms of 𝑥, with the help of a reference
5√2 triangle.
V.GNANAMURUGAN, P,G,T, G.H.S.S, S.S.KOTTAI, SIVAGANGAI DT – 94874 43870 ; 82489 56766 , www.tnppgta.com, www.ednnet.in Page 67
1 4−4𝑥 2 −4𝑥−1
(𝑖) sin(𝑐𝑜𝑠 −1 (1 − 𝑥)) (𝑖𝑖) cos(𝑡𝑎𝑛−1 (3𝑥 − 1)) (𝑖𝑖𝑖) tan (𝑠𝑖𝑛−1 (𝑥 + )) =√
2 4
(𝑖) sin(𝑐𝑜𝑠 −1 (1 − 𝑥)) √3−4𝑥 2 −4𝑥
=
𝐿𝑒𝑡 𝑦 = 𝑐𝑜𝑠 −1 (1 − 𝑥) → (1) 1
2
⟹ 𝑐𝑜𝑠𝑦 = 1 − 𝑥 𝑠𝑖𝑛𝑦 𝑥+ (2𝑥+1)⁄2 2𝑥+1
2
𝑡𝑎𝑛𝑦 = = = =
𝑐𝑜𝑠𝑦 √3−4𝑥2 −4𝑥 (√3−4𝑥 2 −4𝑥)⁄2 √3−4𝑥 2 −4𝑥
⟹ 𝑠𝑖𝑛𝑦 = √1 − 𝑐𝑜𝑠 2 𝑦 2
= √1 − (1 − 𝑥)2 1 2𝑥+1
⟹ tan (𝑠𝑖𝑛−1 (𝑥 + )) = ( 𝐹𝑟𝑜𝑚 (1) )
2 √3−4𝑥 2 −4𝑥
= √1 − (1 − 2𝑥 + 𝑥 2 )
√3
= √1 − 1 + 2𝑥 − 𝑥 2 3.Find the value of (𝑖)𝑠𝑖𝑛−1 (cos (𝑠𝑖𝑛−1 ( )))
2
= √2𝑥 − 𝑥 2 3 4 3 3
(𝑖𝑖) cot (𝑠𝑖𝑛−1 + 𝑠𝑖𝑛−1 ) (𝑖𝑖𝑖) tan (𝑠𝑖𝑛−1 + 𝑐𝑜𝑡 −1 )
⟹ sin(𝑐𝑜𝑠 −1 (1 − 𝑥)) = √2𝑥 − 𝑥 2 (𝐹𝑟𝑜𝑚 (1) ) 5 5 5 2
(𝑖𝑖) cos(𝑡𝑎𝑛−1 (3𝑥 − 1)) √3
(𝑖)𝑠𝑖𝑛−1 (cos (𝑠𝑖𝑛−1 ( ))) = 𝑠𝑖𝑛−1 (cos (𝑠𝑖𝑛−1 (𝑠𝑖𝑛 )))
𝜋
2 3
𝐿𝑒𝑡 𝑦 = 𝑡𝑎𝑛−1 (3𝑥 − 1) → (1).
𝜋
⟹ 𝑡𝑎𝑛𝑦 = 3𝑥 − 1 = 𝑠𝑖𝑛−1 (𝑐𝑜𝑠 )
3
⟹ 𝑠𝑒𝑐𝑦 = √1 + 𝑡𝑎𝑛2 𝑦 = 𝑠𝑖𝑛−1 ( )
1
2
= √1 + (3𝑥 − 1)2 𝜋
= 𝑠𝑖𝑛−1 (𝑠𝑖𝑛 )
= √1 + (9𝑥 2 − 6𝑥 + 1) 𝜋
6

= √1 + 9𝑥 2 − 6𝑥 + 1 =
6
3 4
= √9𝑥 2 − 6𝑥 + 2 (𝑖𝑖) cot (𝑠𝑖𝑛−1 + 𝑠𝑖𝑛−1 )
1 5 5
⟹ 𝑐𝑜𝑠𝑦 = 2 𝑠𝑖𝑛−1 𝑥 + 𝑠𝑖𝑛−1 𝑦 = 𝑠𝑖𝑛−1 [𝑥√1 − 𝑦 2 + 𝑦√1 − 𝑥 2 ]
√9𝑥 −6𝑥+2
−1 (3𝑥 1
⟹ cos(𝑡𝑎𝑛 − 1)) = ( 𝐹𝑟𝑜𝑚 (1) ) 3 4 3 4 4 3 2 2
√9𝑥 2 −6𝑥+2 𝑠𝑖𝑛−1 + 𝑠𝑖𝑛−1 = 𝑠𝑖𝑛−1 [ √1 − ( ) + √1 − ( ) ]
1 5 5 5 5 5 5
(𝑖𝑖𝑖) tan (𝑠𝑖𝑛−1 (𝑥 + ))
2 3 9 4 16
−1 1 = 𝑠𝑖𝑛−1 [ √ + √ ]
𝐿𝑒𝑡 𝑦 = 𝑠𝑖𝑛 (𝑥 + ) → (1).
2
5 25 5 25
1 3 3 4 4
⟹ 𝑠𝑖𝑛𝑦 = 𝑥 + ⟹ 𝑐𝑜𝑠𝑦 = √1 − 𝑠𝑖𝑛2 𝑦 = 𝑠𝑖𝑛−1 [ . + . ]
2 5 5 5 5
−1 9 16
1 2 = 𝑠𝑖𝑛 [ + ]
= √1 − (𝑥 + ) 25 25
2
−1 25
1
= 𝑠𝑖𝑛 [ ]
25
= √1 − 𝑥2 −𝑥− = 𝑠𝑖𝑛 −1 [1]
4

V.GNANAMURUGAN, P,G,T, G.H.S.S, S.S.KOTTAI, SIVAGANGAI DT – 94874 43870 ; 82489 56766 , www.tnppgta.com, www.ednnet.in Page 68
𝜋
= 𝑠𝑖𝑛−1 (𝑠𝑖𝑛 ) 3 5 2 5
√1 − (3) )
2
𝜋
2 = 𝑠𝑖𝑛−1 ( √1 − ( ) −
5 13 13 5
=
2
3 169−25 5 25−9
−1 3 −1 4 𝜋
= 𝑠𝑖𝑛−1 ( √ − √ )
∴ cot (𝑠𝑖𝑛 + 𝑠𝑖𝑛 )= cot ( ) = 0 5 169 13 25
5 5 2
3 3
(𝑖𝑖𝑖) tan (𝑠𝑖𝑛−1 + 𝑐𝑜𝑡 −1 ) 3 144 5 16
5 2 = 𝑠𝑖𝑛−1 ( √ − √ )
3 3 9 16 4 5 169 13 25
𝑠𝑖𝑛−1 = 𝑥 ⟹ sin 𝑥 = ⟹ 𝑐𝑜𝑠𝑥 = √1 − 𝑠𝑖𝑛2 𝑥 = √1 − =√ = 3 12 5 4
5 5 25 25 5 = 𝑠𝑖𝑛−1 ( . − . )
3⁄ 5 13 13 5
sin 𝑥 3
tan 𝑥 = =45= = −1 36−20
𝑠𝑖𝑛 ( )
cos 𝑥 ⁄5 4 65
3 3 2 16
𝑐𝑜𝑡 −1 = 𝑦 ⟹ cot 𝑦 = ⟹ tan 𝑦 = = 𝑠𝑖𝑛−1
2 2 3 65
3 2 𝑥+𝑦+𝑧−𝑥𝑦𝑧
+ 17⁄
tan(𝑥 + 𝑦) =
tan 𝑥+tan 𝑦
= 4 3
= 12
=
17 5.Prove that 𝑡𝑎𝑛 𝑥 + 𝑡𝑎𝑛−1 𝑦 + 𝑡𝑎𝑛−1 𝑧 = 𝑡𝑎𝑛−1 [
−1
]
1−tan 𝑥 tan 𝑦 3 2 6⁄ 6 1−𝑥𝑦−𝑦𝑧−𝑧𝑥
1−(4)(3) 12 𝑥+𝑦
2
7 1 3 12 16 𝑡𝑎𝑛−1 𝑥 + 𝑡𝑎𝑛−1 𝑦 = 𝑡𝑎𝑛−1 ( )
4.Prove that (𝑖)𝑡𝑎𝑛−1
+ 𝑡𝑎𝑛−1 = 𝑡𝑎𝑛−1 (𝑖𝑖)𝑠𝑖𝑛−1 − 𝑐𝑜𝑠 −1 = 𝑠𝑖𝑛−1 1−𝑥𝑦
11
24 2 5 13 65 𝑥+𝑦
−1 −1
−1
(𝑖)𝑡𝑎𝑛 𝑥 + 𝑡𝑎𝑛 𝑦 = 𝑡𝑎𝑛 (
𝑥+𝑦 𝑡𝑎𝑛−1 𝑥 + 𝑡𝑎𝑛−1 𝑦 + 𝑡𝑎𝑛−1 𝑧 = 𝑡𝑎𝑛−1 ( ) + 𝑡𝑎𝑛−1 𝑧
1−𝑥𝑦
) 1−𝑥𝑦
𝑥+𝑦
+𝑧
2⁄ +7⁄ −1 1−𝑥𝑦
2 7
𝑡𝑎𝑛−1 + 𝑡𝑎𝑛−1 = 𝑡𝑎𝑛−1 ( 112 24 = 𝑡𝑎𝑛 ( )
7 )
𝑥+𝑦
11 24 1−(1−𝑥𝑦)𝑧
1−(11)(24)
𝑥+𝑦+𝑧−𝑥𝑦
48+77 ⁄1−𝑥𝑦
−1
= 𝑡𝑎𝑛−1 ( 264
) = 𝑡𝑎𝑛 (1−𝑥𝑦−𝑥𝑧−𝑦𝑧 )
14 ⁄1−𝑥𝑦
1−
264
125⁄ −1 𝑥+𝑦+𝑧−𝑥𝑦𝑧
= 𝑡𝑎𝑛 [ ]
= 𝑡𝑎𝑛−1 (250 264) 1−𝑥𝑦−𝑦𝑧−𝑧𝑥
⁄264
125 264
6.𝐼𝑓 𝑡𝑎𝑛−1 𝑥 + 𝑡𝑎𝑛−1 𝑦 + 𝑡𝑎𝑛−1 𝑧 = 𝜋, 𝑠ℎ𝑜𝑤 𝑡ℎ𝑎𝑡 𝑥 + 𝑦 + 𝑧 = 𝑥𝑦𝑧.
= 𝑡𝑎𝑛−1 ( × ) 𝑥+𝑦
264 250 𝑡𝑎𝑛−1 𝑥 + 𝑡𝑎𝑛−1 𝑦 = 𝑡𝑎𝑛−1 ( ) 1−𝑥𝑦
−1 1
= 𝑡𝑎𝑛 ( ) 𝑥+𝑦
2 𝑡𝑎𝑛 𝑥 + 𝑡𝑎𝑛 𝑦 + 𝑡𝑎𝑛 𝑧 = 𝑡𝑎𝑛−1 (
−1 −1 −1
) + 𝑡𝑎𝑛−1 𝑧
1−𝑥𝑦
(𝑖𝑖)𝑠𝑖𝑛−1 𝑥 − −1 −1
𝑠𝑖𝑛 𝑦 = 𝑠𝑖𝑛 (𝑥√1 − 𝑦 2 − 𝑦√1 − 𝑥 2 ) 𝑥+𝑦
+𝑧
12 12 1−𝑥𝑦
𝑦 = 𝑐𝑜𝑠 −1 ⟹ 𝑐𝑜𝑠𝑦 = = 𝑡𝑎𝑛−1 ( 𝑥+𝑦 )
13 13 1−(1−𝑥𝑦)𝑧
12 2 169−144 25 5 5 𝑥+𝑦+𝑧−𝑥𝑦
⁄1−𝑥𝑦
𝑠𝑖𝑛𝑦 = √1 − 𝑐𝑜𝑠 2 𝑦 = √1 − ( ) = √ =√ = ⟹ 𝑦 = 𝑠𝑖𝑛−1 ( ) = 𝑡𝑎𝑛 −1
(1−𝑥𝑦−𝑥𝑧−𝑦𝑧 )
13 169 169 13 13
⁄1−𝑥𝑦
12 5
∴𝑐𝑜𝑠 −1 = 𝑠𝑖𝑛−1 ( ) 𝑥 + 𝑦 + 𝑧 − 𝑥𝑦𝑧
13
3
13
12 3 5
= 𝑡𝑎𝑛−1 [ ]
𝑠𝑖𝑛−1 − 𝑐𝑜𝑠 −1 = 𝑠𝑖𝑛−1 − 𝑠𝑖𝑛−1 ( ) 1 − 𝑥𝑦 − 𝑦𝑧 − 𝑧𝑥
5 13 5 13

V.GNANAMURUGAN, P,G,T, G.H.S.S, S.S.KOTTAI, SIVAGANGAI DT – 94874 43870 ; 82489 56766 , www.tnppgta.com, www.ednnet.in Page 69
𝑥+𝑦+𝑧−𝑥𝑦𝑧
𝑡𝑎𝑛−1 𝑥 + 𝑡𝑎𝑛−1 𝑦 + 𝑡𝑎𝑛−1 𝑧 = 𝜋 ⟹ 𝑡𝑎𝑛−1 [ ]=𝜋 𝑥 𝑦
1−𝑥𝑦−𝑦𝑧−𝑧𝑥 = 𝑡𝑎𝑛−1 − [𝑡𝑎𝑛−1 (1) − 𝑡𝑎𝑛−1 ( ⁄𝑥 )]
𝑥+𝑦+𝑧−𝑥𝑦𝑧 𝑦
⟹ = 𝑡𝑎𝑛𝜋 = 0
1−𝑥𝑦−𝑦𝑧−𝑧𝑥 𝑥 𝜋
⟹ 𝑥 + 𝑦 + 𝑧 − 𝑥𝑦𝑧 = 0 = 𝑡𝑎𝑛−1 − [ − 𝑐𝑜𝑡 −1 (𝑥⁄𝑦)]
𝑦 4
2𝑥 3𝑥−𝑥 3 1 −1 𝑥 𝜋 𝜋
7.𝑃𝑟𝑜𝑣𝑒 𝑡ℎ𝑎𝑡 𝑡𝑎𝑛 −1
𝑥 + 𝑡𝑎𝑛−1
1−𝑥 2
= 𝑡𝑎𝑛−1
1−3𝑥 2
, |𝑥| <
√3
. = 𝑡𝑎𝑛 + 𝑐𝑜𝑡 −1 (𝑥⁄𝑦) − (∵ 𝑡𝑎𝑛−1 𝑥 + 𝑐𝑜𝑡 −1 𝑥 = 2 )
𝑦 4
−1 −1 𝑥+𝑦 𝜋 𝜋
𝑡𝑎𝑛 𝑥 + 𝑡𝑎𝑛 𝑦 = 𝑡𝑎𝑛−1 ( ) = −
2 4
1−𝑥𝑦
2𝑥 𝜋
2𝑥 𝑥+
1−𝑥2
=
𝑡𝑎𝑛−1 𝑥 + 𝑡𝑎𝑛−1 = 𝑡𝑎𝑛−1 ( 2𝑥 ) 5
4
1−𝑥 2 −1 12 𝜋
1−𝑥(
1−𝑥2
) 9.Solve (𝑖)𝑠𝑖𝑛−1 + 𝑠𝑖𝑛 =
3
𝑥−𝑥 +2𝑥 𝑥 𝑥 2
⁄ 1−𝑎 2 2
= 𝑡𝑎𝑛−1 (1−𝑥 2 −2𝑥 2 1−𝑥 )
2
(𝑖𝑖)2𝑡𝑎𝑛−1 −1 1−𝑏
𝑥 = 𝑐𝑜𝑠 −1 − 𝑐𝑜𝑠 , 𝑎 > 0, 𝑏 > 0
⁄ 1+𝑎2 1+𝑏2
1−𝑥 2 𝜋
3𝑥−𝑥 3 (𝑖𝑖𝑖)2𝑡𝑎𝑛−1 (cos −1 (2𝑐𝑜𝑠𝑒𝑐𝑥) (𝑖𝑣)𝑐𝑜𝑡 −1
𝑥) = 𝑡𝑎𝑛 𝑥 − 𝑐𝑜𝑡 −1 (𝑥 + 2) = ,𝑥 > 0
= 𝑡𝑎𝑛−1 12
1−3𝑥 2 5 12 𝜋
𝑥 𝑥−𝑦 (𝑖)𝑠𝑖𝑛−1 + 𝑠𝑖𝑛−1 =
8.Simplify: 𝑡𝑎𝑛−1 − 𝑡𝑎𝑛−1 . 𝑥 𝑥 2
𝑦 𝑥+𝑦 −1 5 𝜋 −1 12
𝑥−𝑦 ⟹ 𝑠𝑖𝑛 = − 𝑠𝑖𝑛
𝑡𝑎𝑛−1 𝑥 − 𝑡𝑎𝑛−1 𝑦 = 𝑡𝑎𝑛−1 ( ) 𝑥 2 𝑥
1+𝑥𝑦 −1 5 −1 12 𝜋
𝑥 𝑥−𝑦 ⟹ 𝑠𝑖𝑛
𝑥
= 𝑐𝑜𝑠
𝑥
(∵ 𝑠𝑖𝑛−1 𝑥 + 𝑐𝑜𝑠 −1 𝑥 = 2 )

−1 𝑥 −1 𝑥−𝑦 −1 𝑦 𝑥+𝑦
𝑡𝑎𝑛 − 𝑡𝑎𝑛 = 𝑡𝑎𝑛 ( 𝑥 𝑥−𝑦 ) 5 12 2
𝑦 𝑥+𝑦 1+(𝑦)(𝑥+𝑦) ⟹ 𝑠𝑖𝑛−1 = 𝑠𝑖𝑛−1 √1 − ( ) (∵ 𝑐𝑜𝑠 −1 𝑥 = 𝑠𝑖𝑛−1 √1 − 𝑥 2 )
𝑥(𝑥+𝑦)−𝑦(𝑥−𝑦) 𝑥 𝑥
⁄𝑦(𝑥+𝑦)
5 144
−1 ⟹ = √1 −
= 𝑡𝑎𝑛 𝑥 𝑥2
𝑦(𝑥+𝑦)+𝑥(𝑥−𝑦)
⁄𝑦(𝑥+𝑦)
5 𝑥 2 −144
( ) ⟹ =√
𝑥 𝑥2
𝑥 2 +𝑥𝑦−𝑥𝑦+𝑦 2
= 𝑡𝑎𝑛−1 ( ) 5 √𝑥 2 −144
𝑥𝑦+𝑦 2 +𝑥 2 −𝑥𝑦 ⟹ =
2 2 𝑥 𝑥
−1 𝑥 +𝑦
= 𝑡𝑎𝑛 (𝑥 2+𝑦2) ⟹5= √𝑥 2− 144
= 𝑡𝑎𝑛−1 (1) Squaring on both sides,
𝜋 ⟹ 25 = 𝑥 2 − 144
= 𝑡𝑎𝑛−1 (𝑡𝑎𝑛 )
𝜋
4 ⟹ 𝑥 2 = 169
= ⟹ 𝑥 = 13
4
(OR) (𝑖𝑖)𝐿𝑒𝑡 𝑎 = tan 𝛼 , 𝑏 = tan 𝛽 ⟹ 𝛼 = 𝑡𝑎𝑛−1 𝑎, 𝛽 = 𝑡𝑎𝑛−1 𝑏
𝑦
1− ⁄ 1−𝑎2 1−𝑏2
: 𝑡𝑎𝑛−1
𝑥
− 𝑡𝑎𝑛−1
𝑥−𝑦
= 𝑡𝑎𝑛−1
𝑥
− 𝑡𝑎𝑛−1 𝑦 𝑋 2𝑡𝑎𝑛−1 𝑥 = 𝑐𝑜𝑠 −1 2
− 𝑐𝑜𝑠 −1
𝑦 𝑥+𝑦 𝑦 1+ ⁄𝑋 1+𝑎 1+𝑏2

V.GNANAMURUGAN, P,G,T, G.H.S.S, S.S.KOTTAI, SIVAGANGAI DT – 94874 43870 ; 82489 56766 , www.tnppgta.com, www.ednnet.in Page 70
1−𝑡𝑎𝑛2 𝛼 1−𝑡𝑎𝑛2 𝛽 √3−1 √3−1
= 𝑐𝑜𝑠 −1 − 𝑐𝑜𝑠 −1 = ×
1+𝑡𝑎𝑛2 𝛼 1+𝑡𝑎𝑛2 𝛽 √ 3+1 √3−1
= 𝑐𝑜𝑠 −1 (𝑐𝑜𝑠2𝛼) − 𝑐𝑜𝑠 −1 (𝑐𝑜𝑠2𝛽)
=
3−2√3+1
2𝑡𝑎𝑛−1 𝑥 = 2𝛼 − 2𝛽 3−1
4−2√3
⟹ 𝑡𝑎𝑛−1 𝑥 = 𝛼 − 𝛽 =
2
⟹ 𝑡𝑎𝑛−1 𝑥 = 𝑡𝑎𝑛−1 𝑎 − 𝑡𝑎𝑛−1 𝑏 = 2 − √3
𝑎−𝑏
⟹ 𝑡𝑎𝑛−1 𝑥 = 𝑡𝑎𝑛−1 (1) ⟹
2
= 2 − √3 ⟹ (𝑥 + 1)2 =
2
1+𝑎𝑏 (𝑥+1)2 2−√3
𝑎−𝑏
⟹𝑥= 2 2+√3
1+𝑎𝑏 ⟹ (𝑥 + 1)2 = ×
(𝑖𝑖𝑖)2𝑡𝑎𝑛−1 (cos 𝑥) = 𝑡𝑎𝑛−1 (2𝑐𝑜𝑠𝑒𝑐𝑥) 2−√3 2+√3
2 4+2√3
2𝑐𝑜𝑠𝑥 2𝑥 ⟹ (𝑥 + 1) =
⟹ 𝑡𝑎𝑛−1 ( ) = 𝑡𝑎𝑛−1 (2𝑐𝑜𝑠𝑒𝑐𝑥) (∵ 2𝑡𝑎𝑛−1 𝑥 = 𝑡𝑎𝑛−1 (1−𝑥 2)) 4−3
1−𝑐𝑜𝑠 2 𝑥
2𝑐𝑜𝑠𝑥 ⟹ (𝑥 + 1)2 = 4 + 2√3
⟹ = 2𝑐𝑜𝑠𝑒𝑐𝑥
1−𝑐𝑜𝑠 2 𝑥 ⟹ (𝑥 + 1)2 = 3 + 2√3 + 1
2𝑐𝑜𝑠𝑥 2 2
⟹ = ⟹ (𝑥 + 1)2 = √3 + 2√3 + 12
1−𝑐𝑜𝑠 2 𝑥 𝑠𝑖𝑛𝑥
cos 𝑥 1 2
⟹ = ⟹ (𝑥 + 1)2 = (√3 + 1)
𝑠𝑖𝑛2 𝑥 𝑠𝑖𝑛𝑥
sin 𝑥
⟹ =1 ⟹ 𝑥 + 1 = √3 + 1
cos 𝑥
⟹ tan 𝑥 = 1 ⟹ 𝑥 = √3
𝜋
⟹ 𝑥 = 𝑛𝜋 + , 𝑛 ∈ 𝑧 10.Find the number of solution of the equation 𝑡𝑎𝑛−1 (𝑥 − 1) + 𝑡𝑎𝑛−1 𝑥 +
4
𝜋 𝑡𝑎𝑛−1 (𝑥 + 1) = 𝑡𝑎𝑛−1 (3𝑥)
(𝑖𝑣)𝑐𝑜𝑡 −1 𝑥 − 𝑐𝑜𝑡 −1 (𝑥
+ 2) =
12 ⟹ 𝑡𝑎𝑛−1 (𝑥 − 1) + 𝑡𝑎𝑛−1 (𝑥 + 1) = 𝑡𝑎𝑛−1 (3𝑥) − 𝑡𝑎𝑛−1 𝑥
1 1 𝜋 𝑥−1+𝑥+1 3𝑥−𝑥
⟹ 𝑡𝑎𝑛−1 ( ) − 𝑡𝑎𝑛 −1
(𝑥+2) = 12 ⟹ 𝑡𝑎𝑛−1 ( ) = 𝑡𝑎𝑛−1 ( )
𝑥 1−(𝑥−1)(𝑥+1) 1+(3𝑥)(𝑥)
1 1
− 𝜋 2𝑥 2𝑥
⟹ 𝑡𝑎𝑛 −1
( 𝑥 𝑥+2
) = 12 ⟹ 𝑡𝑎𝑛−1 ( ) = 𝑡𝑎𝑛−1 (1+3𝑥 2)
1 1
1+(𝑥)(𝑥+2) 1−(𝑥 2 −1)
2𝑥 2𝑥
−1 𝑥+2−𝑥 𝜋 ⟹ 𝑡𝑎𝑛−1 ( ) = 𝑡𝑎𝑛−1 (1+3𝑥 2)
⟹ 𝑡𝑎𝑛 (𝑥 2+2𝑥+1) = 12 2−𝑥 2
2𝑥 2𝑥
2 𝜋 ⟹ =
⟹ (𝑥+1)2 = tan 2−𝑥 2 1+3𝑥 2
12
2 ⟹ 2𝑥(1 + 3𝑥 2 ) = 2𝑥(2 − 𝑥 2 )
⟹ (𝑥+1)2 = 𝑡𝑎𝑛15 → (1) ⟹ 2𝑥 + 6𝑥 3 = 4𝑥 − 2𝑥 3
𝑡𝑎𝑛15 = tan(45 − 30) =
tan 45−tan 30 ⟹ 8𝑥 3 − 2𝑥 = 0
1+tan 45 tan 30
1 ⟹ 4𝑥 3 − 𝑥 = 0, is a cubic equation and so it has three solutions.
1− √3−1 𝜋 3𝜋
√3
=
1+
1 =
√3+1
𝐸𝑋𝐴𝑀𝑃𝐿𝐸 4.16 𝑃𝑟𝑜𝑣𝑒 𝑡ℎ𝑎𝑡 ≤ 𝑠𝑖𝑛−1 𝑥 + 2𝑐𝑜𝑠 −1 𝑥 ≤ .
2 2
√3 𝜋
−1 −1 −1 −1 −1
𝑠𝑖𝑛 𝑥 + 2𝑐𝑜𝑠 𝑥 = 𝑠𝑖𝑛 𝑥 + 𝑐𝑜𝑠 𝑥 + 𝑐𝑜𝑠 𝑥 = + 𝑐𝑜𝑠 −1 𝑥
2

V.GNANAMURUGAN, P,G,T, G.H.S.S, S.S.KOTTAI, SIVAGANGAI DT – 94874 43870 ; 82489 56766 , www.tnppgta.com, www.ednnet.in Page 71
𝜋 𝜋 𝜋 𝜃 9
0 ≤ 𝑐𝑜𝑠 −1 𝑥 ≤ 𝜋 ⟹ + 0 ≤ + 𝑐𝑜𝑠 −1 𝑥 ≤ + 𝜋 ⟹ 𝑐𝑜𝑠 2 ( ) =
2 2 2 2 16
𝜋 3𝜋
⟹ ≤ 𝑠𝑖𝑛 𝑥 + 𝑐𝑜𝑠 −1 𝑥 + 𝑐𝑜𝑠 −1 𝑥 ≤
−1
⟹ cos ( ) =
𝜃 3
2 2 2 4
𝜋 3𝜋
⟹ ≤ 𝑠𝑖𝑛−1 𝑥 + 2𝑐𝑜𝑠 −1 𝑥 ≤ 1
⟹ cos [ 𝑐𝑜𝑠 −1 ( )] =
1 3
(𝐹𝑟𝑜𝑚 (1))
2 2
13𝜋 3𝜋 2 8 4
𝐸𝑋𝐴𝑀𝑃𝐿𝐸 4.17 Simplify: (𝑖)𝑐𝑜𝑠 −1 (cos ( )) 𝑖𝑖)𝑡𝑎𝑛 −1
(tan ( 4 )) 1
(𝑖𝑖𝑖) tan [ 𝑠𝑖𝑛−1 ( 2) + 𝑐𝑜𝑠 −1 ( 2)]
2𝑎 1 1−𝑎2
3
5𝜋 2 1+𝑎 2 1+𝑎
(𝑖𝑖𝑖)𝑠𝑒𝑐 −1 (sec ( 3 )) (𝑖𝑣)𝑠𝑖𝑛−1 (𝑠𝑖𝑛10) 𝐿𝑒𝑡 𝑎 = tan 𝜃.
13𝜋 𝜋 𝜋 𝜋 1 2𝑎 1 1−𝑎2
(𝑖)𝑐𝑜𝑠 −1 (cos ( )) = 𝑐𝑜𝑠 −1 (cos (4𝜋 + 3 )) = 𝑐𝑜𝑠 −1 (cos ( 3 )) = ∈ [0, 𝜋] tan [ 𝑠𝑖𝑛−1 ( ) + 2 𝑐𝑜𝑠 −1 (1+𝑎2)]
3 3 2 1+𝑎2
−1 3𝜋 −1 𝜋 −1 𝜋 1 2 tan 𝜃 1 1−𝑡𝑎𝑛2 𝜃
𝑖𝑖)𝑡𝑎𝑛 (tan ( 4 )) = 𝑡𝑎𝑛 (tan (𝜋 − 4 )) = 𝑡𝑎𝑛 (tan (− 4 )) = tan [ 𝑠𝑖𝑛−1 ( ) + 2 𝑐𝑜𝑠 −1 (1+𝑡𝑎𝑛2𝜃)]
2 1+𝑡𝑎𝑛2 𝜃
𝜋 𝜋 𝜋 1 1 2 tan 𝜃 1−𝑡𝑎𝑛2 𝜃
= − ∈ (− , )
4 2 2
= tan [ 𝑠𝑖𝑛−1 (sin 2𝜃) + 𝑐𝑜𝑠 −1 (cos 2𝜃)] (∵ 1+𝑡𝑎𝑛2𝜃 = sin 2𝜃 ; 1+𝑡𝑎𝑛2𝜃 = cos 2𝜃 )
2 2
5𝜋 𝜋 𝜋 1 1
(𝑖𝑖𝑖)𝑠𝑒𝑐 −1 (sec ( 3 )) = 𝑠𝑒𝑐 −1
(sec (2𝜋 − 3 )) = 𝑠𝑒𝑐 −1
(sec ( 3 )) = tan [ (2𝜃) + (2𝜃)]
2 2
𝜋 𝜋 = tan[𝜃 + 𝜃]
= ∈ [0, 𝜋] − { }
3 2
= tan[2𝜃]
(𝑖𝑣)𝑠𝑖𝑛−1 (𝑠𝑖𝑛10) −1
= 𝑠𝑖𝑛 [𝑠𝑖𝑛(3𝜋 + (10 − 3𝜋))] 2 tan 𝜃 2 tan 𝐴
= (∵ tan[2𝐴] = 1−𝑡𝑎𝑛2 𝐴)
= 𝑠𝑖𝑛−1 [−𝑠𝑖𝑛((10 − 3𝜋))] 2
1−𝑡𝑎𝑛 𝜃
2𝑎
= −𝑠𝑖𝑛−1 [𝑠𝑖𝑛((10 − 3𝜋))] =
1−𝑎2
𝑥
= −(10 − 3𝜋) 𝐸𝑋𝐴𝑀𝑃𝐿𝐸 4.19 Prove that tan(𝑠𝑖𝑛−1 𝑥) = for |𝑥| < 1.
𝜋 𝜋 √1−𝑥 2
= 3𝜋 − 10 ∈ [− , ] 𝑠𝑖𝑛−1 𝑥 = 𝜃 ⟹ 𝑥 = sin 𝜃 , −1 ≤ 𝑥 ≤ 1.
2 2
sin 𝜃 sin 𝜃 𝑥
𝐸𝑋𝐴𝑀𝑃𝐿𝐸 4.18 Find the value of tan(𝑠𝑖𝑛−1 𝑥) = tan 𝜃 = = 2
= , |𝑥| < 1
cos 𝜃 √1−𝑠𝑖𝑛 𝜃 √1−𝑥 2
𝜋 1 1 1
(𝑖) sin [ − 𝑠𝑖𝑛−1 (− )] (𝑖𝑖) cos [ 𝑐𝑜𝑠 −1 ( )] 𝐸𝑋𝐴𝑀𝑃𝐿𝐸 4.20 Evaluate
3 5
sin [𝑠𝑖𝑛−1 ( ) + 𝑠𝑒𝑐 −1 ( )]
3 2 2 8 5 4
1 2𝑎 1 1−𝑎2
(𝑖𝑖𝑖) tan [ 𝑠𝑖𝑛−1 ( ) + 2 𝑐𝑜𝑠 −1 (1+𝑎2)] 5
𝑠𝑒𝑐 −1 ( ) = 𝜃 ⟹ sec 𝜃 = ⟹ cos 𝜃 =
5 4
2 1+𝑎2 4 4 5
𝜋 1 𝜋 𝜋 𝜋 𝜋 𝜋
(𝑖) sin [ − 𝑠𝑖𝑛−1 (− )] = sin [ − (− )] = sin [ + ] = sin [ ] = 1 4 2 9 3 3
3
1 1
2 3 6 3 6 2 ⟹ sin 𝜃 = √1 − 𝑐𝑜𝑠 2 𝜃 = √1 − ( ) = √ = ⟹ 𝜃 = 𝑠𝑖𝑛−1 ( )
−1 5 25 5 5
(𝑖𝑖) cos [ 𝑐𝑜𝑠 ( )] 5 3
2 8
−1 1
∴ 𝑠𝑒𝑐 −1 ( ) = 𝑠𝑖𝑛−1 ( )
𝑐𝑜𝑠 ( ) = 𝜃 → (1) 4 5
3 5
8
1 sin [𝑠𝑖𝑛−1 ( ) + 𝑠𝑒𝑐 −1 ( )]
⟹ cos 𝜃 = 5 4
8 3 3
2 𝜃 1 = sin [𝑠𝑖𝑛−1 ( ) + 𝑠𝑖𝑛−1 ( )]
⟹ 2𝑐𝑜𝑠 ( ) −1= 5 5
2 8 3
𝜃 9 = sin [2𝑠𝑖𝑛−1 ( )]
⟹ 2𝑐𝑜𝑠 2 ( ) = 5
2 8

V.GNANAMURUGAN, P,G,T, G.H.S.S, S.S.KOTTAI, SIVAGANGAI DT – 94874 43870 ; 82489 56766 , www.tnppgta.com, www.ednnet.in Page 72
4 1
3 3 2 1 = 𝑡𝑎𝑛−1 + 𝑡𝑎𝑛−1
= sin [𝑠𝑖𝑛 −1
(2 × 5 √1 − (5) )] (∵ 𝐼𝑓 |𝑥| ≤ , 𝑠𝑖𝑛 −1
(2𝑥√1 − 𝑥2 ) −1
= 2𝑠𝑖𝑛 𝑥) 3
4 1
7
√2 + 𝑥+𝑦
= 𝑡𝑎𝑛−1 ( 3 7
4 1 ) (∵ 𝑡𝑎𝑛−1 𝑥 + 𝑡𝑎𝑛−1 𝑦 = 𝑡𝑎𝑛−1 (1−𝑥𝑦) , 𝑥𝑦 < 1)
6 16 1−(3)(7)
= sin [𝑠𝑖𝑛−1 ( √ )]
5 25 31⁄
6 4 = 𝑡𝑎𝑛−1 (17 21)
= sin [𝑠𝑖𝑛−1 ( × )] ⁄21
5 5 31
24 = 𝑡𝑎𝑛−1
= sin [𝑠𝑖𝑛−1 ( )] 17
25
24 𝐸𝑋𝐴𝑀𝑃𝐿𝐸 4.22 𝐼𝑓 𝑐𝑜𝑠 −1 𝑥 + 𝑐𝑜𝑠 −1 𝑦 + 𝑐𝑜𝑠 −1 𝑧 = 𝜋 and 0 < 𝑥, 𝑦, 𝑧 < 1, show
= ∈ [−1,1] that 𝑥 2 + 𝑦 2 + 𝑧 2 + 2𝑥𝑦𝑧 = 1.
25
(OR) 𝑐𝑜𝑠 −1 𝑥 = 𝛼 ⟹ 𝑥 = cos 𝛼 ⟹ sin 𝛼 = √1 − 𝑐𝑜𝑠 2 𝛼 = √1 − 𝑥 2
5
𝑠𝑒𝑐 −1 ( ) = 𝜃 → (1) ⟹ sec 𝜃 =
5
⟹ cos 𝜃 =
4 } → (1)
4 4 5 𝑐𝑜𝑠 −1 𝑦 = 𝛽 ⟹ 𝑦 = cos 𝛽 ⟹ sin 𝛽 = √1 − 𝑐𝑜𝑠 2 𝛽 = √1 − 𝑦 2
4 2 9 3 3 𝑐𝑜𝑠 −1 𝑥 + 𝑐𝑜𝑠 −1 𝑦 + 𝑐𝑜𝑠 −1 𝑧 = 𝜋 ⟹ 𝛼 + 𝛽 + 𝑐𝑜𝑠 −1 𝑧 = 𝜋
⟹ sin 𝜃 = √1 − 𝑐𝑜𝑠 2 𝜃 = √1 − ( ) = √ = ⟹ 𝜃 = 𝑠𝑖𝑛−1 ( ) → (2) ⟹ 𝛼 + 𝛽 = 𝜋 − 𝑐𝑜𝑠 −1 𝑧 → (2)
5 25 5 5
3
sin [𝑠𝑖𝑛−1 ( ) + 𝑠𝑒𝑐 −1 ( )]
5 cos(𝛼 + 𝛽) = cos 𝛼 cos 𝛽 − sin 𝛼 sin 𝛽
5 4 𝐹𝑟𝑜𝑚 (1), (2),
= sin[𝜃 + 𝜃] 𝐹𝑟𝑜𝑚 (1), (2)
⟹ cos(𝜋 − 𝑐𝑜𝑠 −1 𝑧) = 𝑥𝑦 − √1 − 𝑥 2 √1 − 𝑦 2
= sin[2𝜃]
= 2 sin 𝜃 cos 𝜃 ⟹ −𝑐𝑜𝑠(𝑐𝑜𝑠 −1 𝑧) = 𝑥𝑦 − √1 − 𝑥 2 √1 − 𝑦 2 (∵ cos(𝜋 − 𝜃) = − cos 𝜃)
3 4
=2× × ⟹ −𝑧 = 𝑥𝑦 − √1 − 𝑥 2 √1 − 𝑦 2
5 5
=
24
∈ [−1,1] ⟹ −𝑥𝑦 − 𝑧 = −√1 − 𝑥 2 √1 − 𝑦 2
25 Squaring on both sides,
1 1 𝜋
𝐸𝑋𝐴𝑀𝑃𝐿𝐸 4.21 Prove that (𝑖)𝑡𝑎𝑛−1 + 𝑡𝑎𝑛−1 = ⟹ (𝑥𝑦 + 𝑧)2 = (1 − 𝑥 2 )(1 − 𝑦 2 )
2 3 4
1 1 31
(𝑖𝑖)2𝑡𝑎𝑛−1 + 𝑡𝑎𝑛−1 = 𝑡𝑎𝑛−1 ⟹ 𝑥 2 𝑦 2 + 𝑧 2 + 2𝑥𝑦𝑧 = 1 − 𝑦 2 − 𝑥 2 + 𝑥 2 𝑦 2
2 7 17
−1 1 −1 1 𝑥+𝑦 ⟹ 𝑥 2 + 𝑦 2 + 𝑧 2 + 2𝑥𝑦𝑧 = 1
(𝑖)𝑡𝑎𝑛 + 𝑡𝑎𝑛 (∵ 𝑡𝑎𝑛−1 𝑥 + 𝑡𝑎𝑛−1 𝑦 = 𝑡𝑎𝑛−1 ( ) , 𝑥𝑦 < 1)
2 3 1−𝑥𝑦 𝐸𝑋𝐴𝑀𝑃𝐿𝐸 4.23 𝐼𝑓 𝑎1 , 𝑎2 , 𝑎3 , … . , 𝑎𝑛 is an arithmetic progression with common
1 1 5⁄
+ 𝜋 difference d, prove that
= 𝑡𝑎𝑛 −1
( 2 3
1 1 ) = 𝑡𝑎𝑛−1 (5 6) = 𝑡𝑎𝑛−1 (1) = 4 𝑑 𝑑 𝑑 𝑎 −𝑎1
⁄6
1−(2)(3) 𝑡𝑎𝑛 [𝑡𝑎𝑛−1 ( ) + 𝑡𝑎𝑛−1 (1+𝑎 ) + ⋯ . +𝑡𝑎𝑛−1 (1+𝑎 𝑛
)] = 1+𝑎
1 1 1+𝑎1 𝑎2 2 𝑎3 𝑛 𝑎𝑛−1 1 𝑎𝑛
(𝑖𝑖)2𝑡𝑎𝑛−1 + 𝑡𝑎𝑛−1 𝑎1 , 𝑎2 , 𝑎3 , … . , 𝑎𝑛 is an A.P.
2 7
1 𝑑 𝑎 −𝑎1
2( )
= 𝑡𝑎𝑛−1 ( 2
) + 𝑡𝑎𝑛−1
1
(∵ 2𝑡𝑎𝑛−1 𝑥 = 𝑡𝑎𝑛−1 (
2𝑥
, |𝑥| < 1) 𝑡𝑎𝑛−1 ( ) = 𝑡𝑎𝑛−1 (1+𝑎
2
) = 𝑡𝑎𝑛−1 𝑎2 − 𝑡𝑎𝑛−1 𝑎1
1 2 7 1−𝑥 2) 1+𝑎1 𝑎2 1 𝑎2
1−( ) 𝑑 𝑎3 −𝑎2
1
2
1
𝑡𝑎𝑛−1 ( ) = 𝑡𝑎𝑛−1 ( ) = 𝑡𝑎𝑛−1 𝑎3 − 𝑡𝑎𝑛−1 𝑎2
−1 1+𝑎2 𝑎3 1+𝑎2 𝑎3
= 𝑡𝑎𝑛 (3⁄ ) + 𝑡𝑎𝑛−1 .
4 7

V.GNANAMURUGAN, P,G,T, G.H.S.S, S.S.KOTTAI, SIVAGANGAI DT – 94874 43870 ; 82489 56766 , www.tnppgta.com, www.ednnet.in Page 73
. 𝑥
𝑠𝑖𝑛−1 𝑥 = 𝜃 ⟹ sin 𝜃 = ⟹ cos 𝜃 = √1 − 𝑠𝑖𝑛2 𝜃 = √1 − 𝑥 2
. 1
𝑑 𝑎𝑛 −𝑎𝑛−1 cos 𝜃 √1−𝑥 2 √1−𝑥 2
𝑡𝑎𝑛−1 ( ) = 𝑡𝑎𝑛−1 (1+𝑎 ) = 𝑡𝑎𝑛−1 𝑎𝑛 − 𝑡𝑎𝑛−1 𝑎𝑛−1 cot(𝜃) = = ⟹ cot(𝑠𝑖𝑛−1 𝑥) =
1+𝑎𝑛 𝑎𝑛−1 𝑛 𝑎𝑛−1
sin 𝜃 𝑥 𝑥
𝜋
𝑡𝑎𝑛 [𝑡𝑎𝑛−1 (
𝑑
) + 𝑡𝑎𝑛−1 (
𝑑
) + ⋯ . +𝑡𝑎𝑛−1 (
𝑑
)] 𝐸𝑋𝐴𝑀𝑃𝐿𝐸 4.27 𝑆𝑜𝑙𝑣𝑒 𝑡𝑎𝑛 2𝑥 + 𝑡𝑎𝑛 3𝑥 = , 𝑖𝑓 6𝑥 2 < 1.−1 −1
4
1+𝑎1 𝑎2 1+𝑎2 𝑎3 1+𝑎𝑛 𝑎𝑛−1 −1 −1 𝜋
= 𝑡𝑎𝑛[𝑡𝑎𝑛−1 𝑎2 − 𝑡𝑎𝑛−1 𝑎1 + 𝑡𝑎𝑛−1 𝑎3 − 𝑡𝑎𝑛−1 𝑎2 + ⋯ . +𝑡𝑎𝑛−1 𝑎𝑛 − 𝑡𝑎𝑛 2𝑥 + 𝑡𝑎𝑛 3𝑥 =
4
𝑡𝑎𝑛−1 𝑎𝑛−1 ] 2𝑥+3𝑥 𝜋 𝑥+𝑦
⟹ 𝑡𝑎𝑛−1 [ ]= (∵ 𝑡𝑎𝑛−1 𝑥 + 𝑡𝑎𝑛−1 𝑦 = 𝑡𝑎𝑛−1 (1−𝑥𝑦))
= 𝑡𝑎𝑛[𝑡𝑎𝑛−1 𝑎𝑛 − 𝑡𝑎𝑛−1 𝑎1 ] 1−(2𝑥)(3𝑥) 4
𝑎 −𝑎 5𝑥 𝜋
= 𝑡𝑎𝑛−1 ( 𝑛 1 ) ⟹ 𝑡𝑎𝑛−1 [ ]=
1+𝑎𝑛 𝑎1 1−6𝑥 2 4
𝑎𝑛 −𝑎1 5𝑥 𝜋
= ⟹ = tan
1+𝑎1 𝑎𝑛 1−6𝑥 2 4
1−𝑥 1 5𝑥
𝐸𝑋𝐴𝑀𝑃𝐿𝐸 4.24 Solve 𝑡𝑎𝑛−1 ( ) = 𝑡𝑎𝑛−1 𝑥, for 𝑥 > 0. ⟹
1−6𝑥 2
=1
1+𝑥 2
−1 1−𝑥 1 −1 −1 1
𝑡𝑎𝑛 ( ) = 𝑡𝑎𝑛 𝑥 ⟹ 𝑡𝑎𝑛 1 − 𝑡𝑎𝑛−1 𝑥 = 𝑡𝑎𝑛−1 𝑥 ⟹ 5𝑥 = 1 − 6𝑥 2
1+𝑥 2
1
2 ⟹ 6𝑥 2 + 5𝑥 − 1 = 0
⟹ 𝑡𝑎𝑛 1 = 𝑡𝑎𝑛 𝑥 + 𝑡𝑎𝑛−1 𝑥
−1 −1 1
⟹ (6𝑥 − 1)(𝑥 + 1) = 0 ⟹ 𝑥 = ; 𝑥 = −1 (Not a soln.)
2
𝜋 3 6
−1
⟹ = 𝑡𝑎𝑛 𝑥 −1 𝑥−1 𝑥+1 𝜋
4 2 𝐸𝑋𝐴𝑀𝑃𝐿𝐸 4.28 Solve 𝑡𝑎𝑛 ( ) + 𝑡𝑎𝑛−1 ( )=
−1 𝜋 2 𝑥−2 𝑥+2 4
⟹ 𝑡𝑎𝑛 𝑥 = × 𝑥−1 −1 𝑥+1 𝜋
4
𝜋
3 𝑡𝑎𝑛−1 ( ) + 𝑡𝑎𝑛 (𝑥+2) =
−1 𝑥−2 4
⟹ 𝑡𝑎𝑛 𝑥 = 𝑥−1 𝑥+1
6 + 𝜋 𝑥+𝑦
⟹ 𝑥 = tan
𝜋 ⟹ 𝑡𝑎𝑛−1 [ 𝑥−2 𝑥+2
𝑥−1 𝑥+1 ]= (∵ 𝑡𝑎𝑛−1 𝑥 + 𝑡𝑎𝑛−1 𝑦 = 𝑡𝑎𝑛−1 (1−𝑥𝑦))
6 1−(𝑥−2)(𝑥+2) 4
1 (𝑥−1)(𝑥+2)+(𝑥+1)(𝑥−2)
⟹𝑥= ⁄(𝑥−2)(𝑥+2)
√3
𝜋
𝐸𝑋𝐴𝑀𝑃𝐿𝐸 4.25 Solve 𝑠𝑖𝑛−1 𝑥 > 𝑐𝑜𝑠 −1 𝑥. ⟹ (𝑥−2)(𝑥+2)−(𝑥−1)(𝑥+1) = tan
4
𝑠𝑖𝑛−1 𝑥 > 𝑐𝑜𝑠 −1 𝑥 ⟹ 𝑠𝑖𝑛−1 𝑥 + 𝑠𝑖𝑛−1 𝑥 > 𝑠𝑖𝑛−1 𝑥 + 𝑐𝑜𝑠 −1 𝑥 ⁄(𝑥−2)(𝑥+2)

⟹ 2𝑠𝑖𝑛−1 𝑥 >
𝜋 [ ]
2 𝑥 2 +𝑥−2+𝑥 2 −𝑥−2
⟹ 𝑠𝑖𝑛−1 𝑥 >
𝜋 =[ ]=1
𝑥 2 −4−𝑥 2 +1
4 2𝑥 2 −4
𝜋
⟹ 𝑥 > sin ⟹ =1
4 −3
⟹𝑥>
1 ⟹ 2𝑥 2 − 4 = −3
√2
1
⟹ 2𝑥 2 = 1
⟹𝑥=( , 1] ⟹ 𝑥2 =
1
√2
2
−1 √1−𝑥 2 1
𝐸𝑋𝐴𝑀𝑃𝐿𝐸 4.26 𝑆ℎ𝑜𝑤 𝑡ℎ𝑎𝑡 cot(𝑠𝑖𝑛 𝑥) = , −1 ≤ 𝑥 ≤ 1 and 𝑥 ≠ 0 ⟹𝑥=±
𝑥 √2

V.GNANAMURUGAN, P,G,T, G.H.S.S, S.S.KOTTAI, SIVAGANGAI DT – 94874 43870 ; 82489 56766 , www.tnppgta.com, www.ednnet.in Page 74
𝑥 3
𝐸𝑋𝐴𝑀𝑃𝐿𝐸 4.29 Solve cos (𝑠𝑖𝑛−1 ( )) = sin {𝑐𝑜𝑡 −1 (4)}
√1+𝑥 2
𝑥 𝑥
𝑠𝑖𝑛−1 ( ) = 𝛼 ⟹ sin 𝛼 =
√1+𝑥 2 √1+𝑥 2
𝑥 2 𝑥2 1 1
cos 𝛼 = √1 − 𝑠𝑖𝑛2 𝛼 = √1 − ( 2
) = √1 − 1+𝑥 2 = √1+𝑥 2 =
√1+𝑥 √1+𝑥 2
1
𝛼 = 𝑐𝑜𝑠 −1 ( )
√1+𝑥 2
𝑥 𝑥 1
𝑠𝑖𝑛−1 ( ) = 𝛼 ⟹ 𝑠𝑖𝑛−1 ( ) = 𝑐𝑜𝑠 −1 ( )
√1+𝑥 2 √1+𝑥 2 √1+𝑥 2
𝑥 1 1
cos (𝑠𝑖𝑛−1 ( )) = cos (𝑐𝑜𝑠 −1 ( ))= ⟹ (1)
√1+𝑥 2 √1+𝑥 2 √1+𝑥 2
3 3
𝑐𝑜𝑡 −1 ( ) = 𝛽 ⟹ cot 𝛽 =
4 4
𝑜𝑝𝑝𝑜𝑠𝑖𝑡𝑒 𝑠𝑖𝑑𝑒 = 4, 𝑎𝑑𝑗𝑎𝑐𝑒𝑛𝑡 𝑠𝑖𝑑𝑒 = 3
𝐻𝑦𝑝𝑜𝑡𝑒𝑛𝑢𝑠𝑒 = √42 + 32 = √25 = 5
4 4
sin 𝛽 = ⟹ 𝛽 = 𝑠𝑖𝑛−1 ( )
5 5
3 3 4
𝑐𝑜𝑡 −1 ( ) = 𝛽 ⟹ 𝑐𝑜𝑡 −1 ( ) = 𝑠𝑖𝑛−1 ( )
4 4 5
3 4 4
sin {𝑐𝑜𝑡 −1 ( )} = sin {𝑠𝑖𝑛−1 ( ) } = → (2)
4 5 5
𝐹𝑟𝑜𝑚 (1), (2),
1 4
2
=
√1+𝑥 5
5
⟹ √1 + 𝑥 2 =
4
Squaring on both sides,
25
⟹ 1 + 𝑥2 =
16
2 25
⟹𝑥 = −1
16
9
⟹ 𝑥2 =
16
3
⟹𝑥=±
4

V.GNANAMURUGAN, P,G,T, G.H.S.S, S.S.KOTTAI, SIVAGANGAI DT – 94874 43870 ; 82489 56766 , www.tnppgta.com, www.ednnet.in Page 75

You might also like